Sei sulla pagina 1di 120

1

SOLIVEN VS MAKASIAR Sound policy dictates this procedure, otherwise judges would be unduly laden with the
In these consolidated cases, three principal issues were raised: (1) whether or not petitioners preliminary examination and investigation of criminal complaints instead of concentrating on
were denied due process when informations for libel were filed against them although the hearing and deciding cases filed before their courts.
finding of the existence of a prima faciecase was still under review by the Secretary of Justice On June 30, 1987, the Supreme Court unanimously adopted Circular No. 12, setting down
and, subsequently, by the President; (2) whether or not the constitutional rights of Beltran were guidelines for the issuance of warrants of arrest. The procedure therein provided is reiterated
violated when respondent RTC judge issued a warrant for his arrest without personally and clarified in this resolution.
examining the complainant and the witnesses, if any, to determine probable cause; and (3) It has not been shown that respondent judge has deviated from the prescribed procedure. Thus,
whether or not the President of the Philippines, under the Constitution, may initiate criminal with regard to the issuance of the warrants of arrest, a finding of grave abuse of discretion
proceedings against the petitioners through the filing of a complaint-affidavit. amounting to lack or excess of jurisdiction cannot be sustained.
Subsequent events have rendered the first issue moot and academic. On March 30, 1988, the Anent the third issue, petitioner Beltran argues that "the reasons which necessitate presidential
Secretary of Justice denied petitioners' motion for reconsideration and upheld the resolution of immunity from suit impose a correlative disability to file suit." He contends that if criminal
the Undersecretary of Justice sustaining the City Fiscal's finding of a prima facie case against proceedings ensue by virtue of the President's filing of her complaint-affidavit, she may
petitioners. A second motion for reconsideration filed by petitioner Beltran was denied by the subsequently have to be a witness for the prosecution, bringing her under the trial court's
Secretary of Justice on April 7, 1988. On appeal, the President, through the Executive Secretary, jurisdiction. This, continues Beltran, would in an indirect way defeat her privilege of immunity
affirmed the resolution of the Secretary of Justice on May 2, 1988. The motion for from suit, as by testifying on the witness stand, she would be exposing herself to possible
reconsideration was denied by the Executive Secretary on May 16, 1988. With these contempt of court or perjury.
developments, petitioners' contention that they have been denied the administrative remedies The rationale for the grant to the President of the privilege of immunity from suit is to assure the
available under the law has lost factual support. exercise of Presidential duties and functions free from any hindrance or distraction, considering
It may also be added that with respect to petitioner Beltran, the allegation of denial of due that being the Chief Executive of the Government is a job that, aside from requiring all of the
process of law in the preliminary investigation is negated by the fact that instead of submitting office holder's time, also demands undivided attention.
his counter- affidavits, he filed a "Motion to Declare Proceedings Closed," in effect waiving his But this privilege of immunity from suit, pertains to the President by virtue of the office and may
right to refute the complaint by filing counter-affidavits. Due process of law does not require be invoked only by the holder of the office; not by any other person in the President's behalf.
that the respondent in a criminal case actually file his counter-affidavits before the preliminary Thus, an accused in a criminal case in which the President is complainant cannot raise the
investigation is deemed completed. All that is required is that the respondent be given the presidential privilege as a defense to prevent the case from proceeding against such accused.
opportunity to submit counter-affidavits if he is so minded. Moreover, there is nothing in our laws that would prevent the President from waiving the
The second issue, raised by petitioner Beltran, calls for an interpretation of the constitutional privilege. Thus, if so minded the President may shed the protection afforded by the privilege and
provision on the issuance of warrants of arrest. The pertinent provision reads: submit to the court's jurisdiction. The choice of whether to exercise the privilege or to waive it is
Art. III, Sec. 2. The right of the people to be secure in their persons, houses, solely the President's prerogative. It is a decision that cannot be assumed and imposed by any
papers and effects against unreasonable searches and seizures of whatever other person.
nature and for any purpose shall be inviolable, and no search warrant or As regards the contention of petitioner Beltran that he could not be held liable for libel because
warrant of arrest shall issue except upon probable cause to be determined of the privileged character or the publication, the Court reiterates that it is not a trier of facts
personally by the judge after examination nder oath or affirmation of the and that such a defense is best left to the trial court to appreciate after receiving the evidence of
complainant and the witnesses he may produce, and particularly describing the parties.
the place to be searched and the persons or things to be seized. As to petitioner Beltran's claim that to allow the libel case to proceed would produce a "chilling
The addition of the word "personally" after the word "determined" and the deletion of the grant effect" on press freedom, the Court finds no basis at this stage to rule on the point.
of authority by the 1973 Constitution to issue warrants to "other responsible officers as may be The petitions fail to establish that public respondents, through their separate acts, gravely
authorized by law," has apparently convinced petitioner Beltran that the Constitution now abused their discretion as to amount to lack of jurisdiction. Hence, the writs of certiorari and
requires the judge to personally examine the complainant and his witnesses in his determination prohibition prayed for cannot issue.
of probable cause for the issuance of warrants of arrest. This is not an accurate interpretation. WHEREFORE, finding no grave abuse of discretion amounting to excess or lack of jurisdiction on
What the Constitution underscores is the exclusive and personal responsibility of the issuing the part of the public respondents, the Court Resolved to DISMISS the petitions in G. R. Nos.
judge to satisfy himself of the existence of probable cause. In satisfying himself of the existence 82585, 82827 and 83979. The Order to maintain the status quo contained in the Resolution of
of probable cause for the issuance of a warrant of arrest, the judge is not required to personally the Court en banc dated April 7, 1988 and reiterated in the Resolution dated April 26, 1988 is
examine the complainant and his witnesses. Following established doctrine and procedure, he LIFTED.
shall: (1) personally evaluate the report and the supporting documents submitted by the fiscal
regarding the existence of probable cause and, on the basis thereof, issue a warrant of arrest; or
(2) if on the basis thereof he finds no probable cause, he may disregard the fiscal's report and
require the submission of supporting affidavits of witnesses to aid him in arriving at a conclusion
as to the existence of probable cause.
2

CRUZ VS PEOPLE 8. Petitioner thus filed with respondent Sandiganbayan (First Division) an Omnibus
The present original action for certiorari, prohibition and mandamus seeks the reversal of the Motion to Quash the Information, dated September 17, 1992, wherein he prayed ". . .
Orders issued by respondent Sandiganbayan in Criminal Case No. 14252, dated February 17, for the production of (the) record of the preliminary investigation), and that the
information be quashed outright or the disapproval of the Ombudsman set aside, or in
1993 1 and May 12, 1993, 2 denying petitioner’s Omnibus Motion and Motion for
the alternative, that the Office of the Ombudsman be ordered to conduct further
Reconsideration, respectively. proceedings, particularly the handwriting analysis prayed for by the petitioner which
The facts are summarized in the Memorandum of public respondents as follows: would establish who committed the alleged falsification. . . .
1. The Government Service Insurance System (the GSIS, for short) filed two separate On February 17, 1993, respondent Sandiganbayan promulgated a Resolution dated
criminal complaints against petitioner Roman A. Cruz, Jr., a former public official who February 15, 1993, the dispositive portion of which reads:
used to be the President and General Manager of the GSIS and, also, the President of WHEREFORE, the Omnibus Motion of accused Roman A. Cruz, Jr.
the Manila Hotel, for violation of Section 3(e) of Republic Act No. 3019, as amended. is DENIED for lack of merit. . . .
The first complaint against petitioner was filed with the Office of the Special 10. A Motion for Reconsideration, dated April 12, 1993, of the aforequoted Resolution
Prosecutor (the OSP, for short) and docketed as OSP-88-02028 while the second, which was filed by petitioner . . . .
involved the same set of facts, was filed with the Presidential Commission on Good 11. On May 12, 1993, respondent Sandiganbayan promulgated a Resolution, the
Government (the PCGG, for short) but which was later endorsed to the Office of the dispositive portion of which reads:
Ombudsman and docketed as OMB-0-91-0986. . . . WHEREFORE, the Motion for Reconsideration of accused Roman A. Cruz, Jr. of this
2. A preliminary investigation was conducted by the PCGG where petitioner duly Court’s Resolution dated February 17, 1993 is DENIED for lack of merit. . . .
submitted his counter-affidavit. As a consequence’ of said investigation, an 12. Hence, petitioner filed the instant petition. 3
Information was filed with the first Division of the Sandiganbayan, docketed as Petitioner contends that respondent Sandiganbayan committed a grave abuse of discretion:
Criminal Case No. 14134, charging petitioner with violation of Section 3(e) of Republic
1. In not dismissing the information considering that the Ombudsman’s
Act No. 3019. . . .
3. During the proceedings before the OSP, petitioner moved to dismiss the complaint. approval of the order dismissing the complaint did not state the factual or
The OSP, however, denied the motion and filed with the Third Division of the legal basis therefor;
Sandiganbayan an Information charging petitioner with Estafa through Falsification of 2. In not requiring the production of the record of the preliminary
Public Documents (Articles 171 and 315 of the Revised Penal Code), docketed as investigation in wanton disregard of petitioner’s right to due process;
Criminal Case No. 14252. Petitioner was deemed by the OSP to have waived his right 3. In not dismissing the information considering that, as found by the
to submit a counter-affidavit and supporting evidence. . . . investigating prosecutor, the money received by petitioner was a cash
4. As a result of the filing of two informations with respondent Sandiganbayan advance; and
involving the same accused (herein petitioner) and the same set of facts, Criminal Case
4. In not requiring the Office of the Ombudsman to conduct further
No. 14252 was consolidated with Criminal Case No. 14134 which was pending before
the First Division of respondent Sandiganbayan. . . . proceedings.
5. Respondent Sandiganbayan, however, remanded the consolidated cases against We do not find the instant petition to be impressed with merit as to warrant the extraordinary
petitioner to the Office of the Ombudsman for reinvestigation inasmuch as: writs prayed for.
a) the Information in Criminal Case No. 14134 was ordered dismissed in compliance The information filed against herein petitioner charging him with estafa through falsification of
with the ruling of the Supreme Court in Cojuangco, Jr. vs. PCGG, et al., G.R. Nos. 92319- public documents and for which he stands to be tried before respondent court alleges:
20, October 2, 1990, which declared null and void the preliminary investigations That on or about or during the period from March 26, 1984 to May 11, 1984,
conducted by the PCGG in all criminal cases involving matters which were the subject or sometime prior or subsequent thereto, at the City of Manila, Philippines,
matter of civil cases earlier filed; and
and within the jurisdiction of this Honorable Court, Roman Cruz, Jr., then
b) the Information in Criminal Case No. 14252 was correctly assailed by petitioner as
having been filed without the proper preliminary investigation. . . . President and General Manager of the Government Service Insurance System
6. During the preliminary investigation conducted anew by the Office of the (GSIS) and likewise President of the Manila Hotel, hence a public official
Ombudsman, petitioner submitted his counter-affidavit and supporting documents. having been duly appointed/elected and qualified as such, taking advantage
After the completion of said investigation, Prosecutor Leonardo P. Tamayo of the of his position, by means of deceit, committing an offense in relation to his
Office of the Ombudsman prepared a Resolution dated February 11, 1992, which office, did then and there wilfully, unlawfully and feloniously falsify Manila
recommended the withdrawal of the Information in Criminal Case No. 14252. . . . Hotel Invoices, Transportation, Charge, Cash, Budget for Food and Drinks
7. Respondent Ombudsman, however, despite the above recommendation of the vouchers in the aggregate amount of P350,000.00 and then make it appear
investigating prosecutor ordered the prosecution to proceed under the existing
that the GSIS management and staff had a five-day coordination meeting at
Information in Criminal Case No. 14252 on his observation, viz:
Let us not do the defending for the accused. The explanations the Manila Hotel from March 23 to 30, 1984 at the cost of P350,000.00, for
offered are too strained to be believed. At best they are matters which reason the GSIS paid/issued its check with No. 039511 dated May 11,
of defense for the accused to prove at the trial. 1984 in the amount of P350,000.00 which check was deposited to the
The alleged character of the funds involved being confidential account of the Manila Hotel, and thereafter cause the Manila Hotel to issue
and requires no auditing is totally immaterial. It could even its check with No. 007272 dated May 11, 1984 in the amount of P350,000.00
explain why this anomaly was committed. . . . payable to Roman Cruz, Jr. or himself, when in truth and in fact, as the
3

accused well knew that there was no such five-day GSIS management and In the present case, petitioner asserts that his right to due process was violated in that
staff coordination meeting conducted/held at the Manila Hotel; and further respondent Ombudsman failed to assess and consider the evidence presented by petitioner in
thereafter convert and appropriate to his own personal use and disapproving the recommendation for dismissal of the case by the investigating prosecutor, and
benefit/deposit the said check to his own personal account with the Far East his ruling is not supported by the evidence on record. The argument is specious.
Bank and Trust Co. the said check/amount of P350,000.00 to the damage and His submission that he was deprived of his right to due process hinges on the erroneous
prejudice of the GSIS and/or Manila Hotel and/or the government in the said assumption that the order of the Ombudsman for the filing of the necessary information is
amount of P350,000.00. 4 lacking in any factual or legal basis. Such a conclusion, however, stems from the fact that said
I. Petitioner initially submits that respondent Sandiganbayan acted with grave abuse of order did not entail a discussion of the rationale for the Ombudsman’s action.
discretion in not dismissing the information considering that the Ombudsman’s disapproval of It may seem that the ratio decidendi for the Ombudsman’s order may be wanting but this is not a
the order dismissing the complaint did not state the factual or legal basis therefor, in violation of case of a total absence of factual and legal bases nor a failure to appreciate the evidence
the cardinal rules set forth in Ang Tibay, et al. vs. CIR, et al. 5 The submission is premised on the presented. What is actually involved here is merely a review of the conclusion arrived at by the
theory that said rules apply to a preliminary investigation which is to be considered quasi-judicial investigating prosecutor as a result of his study and analysis of the complaint, counter-affidavits,
in nature. Petitioner avers that it is the duty of the Ombudsman to assess the evidence and and the evidence submitted by the parties during the preliminary investigation. The Ombudsman
defenses of the respondent in deciding a case, a failure wherein constitutes a violation of one’s here is not conducting anew another investigation but is merely determining the propriety and
right to due process of law. He further claims that "while the duty to deliberate does not impose correctness of the recommendation given by the investigating prosecutor, that is, whether
the obligation to decide right, it does imply a necessity which cannot be disregarded, namely, probable cause actually exists or not, on the basis of the findings of fact of the latter. Verily, it is
that of having something to support the decision. The Ombudsman in this case not only failed to discretionary upon the Ombudsman if he will rely mainly on the findings of fact of the
decide right but has nothing at all to support his decision." 6 investigating prosecutor in making a review of the latter’s report and recommendation, as the
Respondents, on the other hand, aver that the Office of the Ombudsman is not exercising quasi- Ombudsman can very well make his own findings of fact. There is nothing to prevent him from
judicial or quasi-legislative powers because "it does not act as a court" when it conducts acting one way or the other. As a matter of fact, Section 4, Rule 112 of the Rules of Court
preliminary investigation of cases falling under its jurisdiction. provides that "where the investigating assistant fiscal recommends the dismissal of the case but
It is settled that the conduct of a preliminary investigation, which is defined as "an inquiry or his findings are reversed by the provincial or city fiscal or the chief state prosecutor on the
proceeding for the purpose of determining whether there is sufficient ground to engender a ground that a probable cause exists, the latter may, by himself, file the corresponding
well-founded belief that a crime cognizable by the Regional Trial Court has been committed and information against the respondent or direct any other assistant fiscal or state prosecutor to do
that the respondent is probably guilty thereof, and should be held for trial," 7 is, like court so, without conducting another preliminary investigation. 9
proceedings, subject to the requirements of both substantive and procedural due process. This is With more reason may the Ombudsman not be faulted in arriving at a conclusion different from
because, a preliminary investigation is considered as a judicial proceeding wherein the that of the investigating prosecutor on the basis of the same set of facts. It cannot be said that
prosecutor or investigating officer, by the nature of his functions, acts as a quasi-judicial officer. the Ombudsman committed a grave abuse of discretion simply because he opines contrarily to
As we held in Cojuangco, Jr. vs. PCGG, et al.: 8 the prosecutor that, under the facts obtaining in the case, there is probable cause to believe that
. . . It must be undertaken in accordance with the procedure provided in Section 3, herein petitioner is guilty of the offense charged.
Rule 112 of the 1985 Rules of Criminal Procedure. This procedure is to be observed in As aptly pointed out by respondent court in its resolution denying petitioner’s motion for
order to assure that a person undergoing such preliminary investigation will be reconsideration, "to the Ombudsman, the narration of facts by Prosecutor Tamayo, . . .
afforded due process.
demonstrated adequate cause to prosecute the accused Cruz." 10 Furthermore, public
As correctly pointed out by petitioner, an indispensable requisite of due process is that
the person who presides and decides over a proceeding, including a preliminary respondents, in their Memorandum, correctly observed that "(f)rom the tenor of respondent
investigation, must possess the cold neutrality of an impartial judge. Ombudsman’s statement, it is clear that he agreed with the findings of facts of the investigating
Although such a preliminary investigation is not a trial and is not intended to usurp the prosecutor butdisagreed with the latter’s conclusion on the import and significance of said
function of the trial court, it is not a casual affair. The officer conducting the same findings. On the basis of the findings of facts of the investigating prosecutor, which were not
investigates or inquires into the facts concerning the commission of the crime with the disputed by petitioner, respondent Ombudsman believed that there was sufficient ground to
end in view of determining whether or not an information may be prepared against engender a well-founded belief that a crime had been committed and that petitioner is probably
the accused. Indeed, a preliminary investigation is in effect a realistic judicial appraisal guilty thereof." 11
of the merits of the case. Sufficient proof of the guilt of the accused must be adduced
Petitioner argues that the indication of disapproval by the Ombudsman which consists merely of
so that when the case is tried, the trial court may not be bound as a matter of law to
order an acquittal. A preliminary investigation has then been called a judicial inquiry. It two paragraphs fails to point out the issues and relevant facts and is consequently whimsical,
is a judicial proceeding. An act becomes judicial when there is opportunity to be heard capricious and arbitrary. Such proposition is fallacious. The mere fact that the order to file an
and for the production and weighing of evidence, and a decision is rendered thereon. information against petitioner consists only of two paragraphs is not sufficient to impute
The authority of a prosecutor or investigating officer duly empowered to preside or to arbitariness or caprice on the part of the Ombudsman, absent a clear showing that he gravely
conduct a preliminary investigation is no less than that of a municipal judge or even a abused his discretion in disapproving the recommendation of the investigating prosecutor.
regional trial court judge. While the investigating officer, strictly speaking is not a Neither is it tainted with vindictiveness or oppression. He disapproved the recommendation of
"judge," by the nature of his functions he is and must be considered to be a quasi- the special prosecutor because he sincerely believed that there is sufficient evidence to indict
judicial officer.
4

the accused. This is an exercise of the Ombudsman’s power based upon constitutional mandate, above indicate that the elements of Article 315 of the Revised Penal Code as well as of
and the courts should not interfere in such exercise. Articles 171 and 172 thereof may probably be established.
The rule is based not only upon the investigatory and prosecutory powers granted by the It is true that the Manila Hotel eventually treated the P350,000.00 as a "cash advance"
to him. Accused Cruz, however, does not claim that there were cash advances made by
Constitution to the Office of the Ombudsman but upon practicality as well. Otherwise, the
him as a consequence of which he received this sum. Nor has accused Roman Cruz said
functions of the courts will be grievously hampered by innumerable petitions assailing the that he had obtained a loan or cash advance from the Manila Hotel for a particular
dismissal of investigatory proceedings conducted by the Office of the Ombudsman with regard to purpose for which he was expected to subsequently render an accounting. All that
complaints filed before it, in much the same way that the courts would be extremely swamped if Manila Hotel’s subsequent description of this amount as a "cash advance," in fact, says
they could be compelled to review the exercise of discretion on the part of the prosecuting is that when it turned out that P350,000.00 could not be properly accounted for, it had
attorneys each time they decide to file an information in court or dismiss a complaint by a to be treated as an amount which accused Cruz had to pay back; thus, accountingwise,
private complaint. 12 a cash advance.
II. Petitioner next avers that the error of respondent court in not requiring the production of the For accused to have received such a large amount from a company of which he was
the President required him to sign a receipt which would specify clearly what he was
record of the preliminary investigation is two-fold. First, it was in violation of the constitutional
receiving it for. If he received the sum as a cash advance for some future expense, the
right against arbitrary arrests because probable cause was not "personally determined by the Manila Hotel documents would clearly so demonstrate. If he received it as a cash
judge," considering that the records of the preliminary investigation were not elevated to the advance (against his salaries or other benefits), it would appear as a loan in Manila
judge for examination. Second, it was in violation of petitioner’s right to due process of law since Hotel’s books. Accused Cruz, however, has said no such thing in any of his pleadings
he was deprived of the opportunity to examine the evidence against him and prepare his nor apparently has he so stated during the preliminary investigation.
defense. In other words, accused Cruz as President of the Manila Hotel — and, therefore, in a
On the first issue, petitioner relies on the ruling in Lim, Sr., et al. vs. Felix, et al. 13 which held that position of great fiduciary nature — received P350,000.00 in 1984 either for a non-
— existent reason or for a false reason.
He may have an explanation. As of this time, however, if the evidence on record is
If a judge relies entirely on the certification of the prosecutor as in this case
actually presented at trial, enough evidence would exist to put accused Roman A. Cruz,
where all the records of the investigation are in Masbate, he or she has not Jr. at peril of his liberty and would require him to explain his side of the matter.
personally determined probable cause. The determination is made by the A case has, therefore, been demonstrated in the record and in the averment of accused
Provincial Prosecutor. The constitutional requirement has not been satisfied. Cruz himself that the crime charged has probably been committed and that the accused
The judge commits a grave abuse of discretion. is probably guilty thereof.(Emphasis supplied.) 16
The conduct of a preliminary investigation should be distinguished as to whether it is an Petitioner would have respondent court order the production of the records of the preliminary
investigation for the determination of a sufficient ground for the filing of the information or one investigation in its determination of the existence of probable cause for the issuance of the
for the determination of a probable cause for the issuance of a warrant of arrest. The first aspect warrant of arrest. First and foremost, as hereinabove stated, in a preliminary examination for the
of preliminary investigation is executive in nature. It is part of the prosecution’s job. The second issuance of a warrant of arrest, the court is not tasked to review in detail the evidence submitted
kind of preliminary investigation, which is more properly called preliminary examination, is during the preliminary investigation. It is sufficient that the judge personally evaluates the report
judicial in nature and is lodged with the judge. 14 and supporting documents submitted by the prosecution in determining probable cause. 17This is
For the latter, in the exercise of the exclusive and personal responsibility of the issuing judge to precisely what respondent court did. In resolving the issue of probable cause, respondent court
satisfy himself of the existence of probable cause for the issuance of a warrant of arrest, the made an in-depth analysis of the findings of fact of Prosecutor Tamayo, as well as the Omnibus
judge is not required to personally examine the complainant and his witnesses. Following Motion submitted by petitioner. The correctness of these facts was not even questioned by
established doctrine and procedure, he shall: (1) personally evaluate the report and the herein petitioner but, on the contrary was expressly affirmed in the latter’s Omnibus Motion
supporting documents submitted by the fiscal regarding the existence of probable cause and, on dated September 17, 1992 wherein it was stated that "(t)he Order issued by the investigating
the basis thereof, issue a warrant of arrest; or (2) if on the basis thereof he finds no probable prosecutor . . . contains a lucid narration of the relevant facts."
cause, he may disregard the fiscal’s report and require the submission of supporting affidavits of The case of Lim cited by petitioner is not applicable to the present case because, in the former, a
witnesses to aid him in arriving at a conclusion as to the existence of probable cause. 15 warrant of arrest was issued by the respondent judge therein without conducting his own
Coming now to the case at bar, contrary to petitioner’s thesis, respondent court, in its resolution personal evaluation of the case even if only on the basis of the report submitted by the fiscal.
promulgated on February 17, 1993 denying petitioner’s motion to quash the information, found Instead, the respondent therein simply declared: "Considering that both the two competent
the existence of probable cause after making a deliberate and exhaustive review of the facts officers to whom such duty was entrusted by law have declared the existence of probable cause,
obtaining in the case. Thus: each information is complete in form and substance, and there is no visible defect on its face,
All of the above logical process, which is supported both by the finding of fact in the this Court finds it just and proper to rely on the prosecutor’s certification in each information . . .
Resolution and by admissions in the Motion of the accused, lead to the conclusion that . This is far from what actually transpired before the Sandiganbayan as reflected by the records
probable cause exists against accused Roman Cruz, Jr., for acts described in the in this case. Hence, the ruling in Lim cannot be properly invoked.
Information in the instant case.
As to the second issue, petitioner relies on the provisions of Section 8, Rule 112 of the 1985
The narration of facts culled from the record (as affirmed by both parties) support the
narration of facts in the Information. The superficial analysis of the admissions made Rules on Criminal Procedure, to wit:
Sec. 8. Record of preliminary investigation. — The record of the preliminary
investigation whether conducted by a judge or a fiscal, shall not form part of the
5

record of the case in the Regional Trial Court. However, the said court, on its own Also, it is argued that petitioner was denied due process when respondent court failed to
initiative or that of any party, may order the production of the record or any part remand the case to the Ombudsman for further proceedings for the purpose of determining the
thereof whenever the same shall be necessary in the resolution of the case or any persons who actually forged the questioned documents by conducting a handwriting analysis.
incident therein, or shall be introduced as evidence by the party requesting for its
This would have secured him from hasty and malicious prosecution, and would even have led to
production.
the discovery of the true culprit, if indeed documents had been fabricated.
Petitioner’s prayer for the production of the record is intended not only for proper observance of
It must here be stressed that a preliminary investigation is merely inquisitorial, and it is often the
the constitutional requirement that probable cause be determined personally by the judge, but
only means of discovering the persons who may be reasonably charged with a crime, to enable
also to enable him to examine the evidence and prepare his defenses and for trial.
the prosecutor to prepare his complaint or information. It is not a trial of the case on the merits
Public respondents contend that the production of the record of the preliminary examination is
and has no purpose except that of determining whether a crime has been committed and
not necessary since petitioner can always resort to any of the modes of discovery available to an
whether there is probable cause to believe that the accused is guilty thereof, and it does not
accused under the Rules of Court, specifically citing Section 11 of Rule 116, which provides:
Sec. 11. Production or inspection of material evidence in possession of prosecution. — place the persons against whom it is taken in jeopardy. 20
On motion of the accused showing good cause and with notice to all parties, the court, The established rule is that a preliminary investigation is not the occasion for the full and
in order to prevent surprise, suppression, or alteration, may order the prosecution to exhaustive display of the parties’ evidence; it is for the presentation of such evidence only as
produce and permit the inspection and copying or photographing, of any written may engender a well-grounded belief that an offense has been committed and that the accused
statements given by the complainant and other witnesses in any investigation of the is probably guilty thereof. 21
offense conducted by the prosecution or any other investigating officers, as well as of Conformably therewith, the arguments raised by herein petitioner that the cash advance is
any designated documents, papers, books, accounts, letters, photographs, objects or actually in the form of a loan and therefore no criminal liability attaches, and that respondent
tangible things, not otherwise privileged, which constitute or contain evidence
court should have remanded the case for further investigation to determine the true identity of
material to any matter involved in the case, and which are in the possession or under
the control of the prosecution, the police, or any other law investigating agencies. the forgers, are all matters of defense which are best presented during the trial before
This rule refers to the right of the accused to move for production or inspection of material respondent court for its consideration.
evidence in the possession of the prosecution. It authorizes the defense to inspect, copy or The main function of the government prosecutor during the preliminary investigation is merely
photograph any evidence of the prosecution in its possession after obtaining the permission of to determine the existence of probable cause, and to file the corresponding information if he
the court. A motion showing good reasons for the granting of the permission must be filed by finds it to be so. And, probable cause has been defined as the existence of such facts and
the defense for this purpose, with notice to all parties. 18 circumstances as would excite the belief, in a reasonable mind, acting on the facts within the
It will be noted at the outset that precisely, as suggested by public respondents, herein knowledge of the prosecutor, that the person charged was guilty of the crime for which he was
petitioner, in asking for the production of the records of the preliminary investigation in order to prosecuted. 22
enable him to prepare for his defense and for trial, is actually trying to avail of this mode of In the case at bar, the Ombudsman found that there was sufficient ground to believe that
discovery. There was good cause shown for the motion to produce the records, that is, so that petitioner is guilty of the crime charged on the basis of the factual findings of Prosecutor Tamayo
they may be introduced as evidence by the party requesting for their production, which is one of in the latter’s Order dated February 11, 1992 which were arrived at after taking into
the grounds provided for under Section 8, Rule 112 of the Rules of Court. consideration the evidence presented by the parties. A cursory perusal of the records of this case
It is true that the granting of permission lies within the discretion of the court. However, will show that the findings of fact by the Office of the Ombudsman are supported by substantial
respondent court in this case has failed to sufficiently justify its refusal to have the records of the evidence, hence the same should be considered conclusive. 23
preliminary investigation produced before it so that petitioner may use them for his defense, Furthermore, the Ombudsman’s findings are essentially factual in nature. Accordingly, in
either in its resolutions denying petitioner’s Omnibus Motion and Motion for Reconsideration, or assailing said findings on the contention that the Ombudsman committed a grave abuse of
in the pleadings and Memorandum filed by herein respondents before this Court. Consequently, discretion in holding that petitioner is liable for estafa through falsification of public documents,
we find no reason to deny petitioner the right to avail of such mode of discovery. If only for the petitioner is clearly raising questions of fact here. 24 His arguments are anchored on the
reason that petitioner should be given the opportunity to inspect the evidence presented during propriety of or error in the Ombudsman’s appreciation of facts. Petitioner cannot be unaware
the preliminary investigation solely for the purpose of enabling him to prepare for his defense that the Supreme Court is not a trier of facts, more so in the consideration of the extraordinary
and for trial, this questioned resolution of respondent Sandiganbayan should be modified. writ of certiorari where neither questions of fact nor even of law are entertained, but only
III. It is likewise contended that respondent court abused its discretion in not dismissing the questions of lack or excess of jurisdiction or grave abuse of discretion. 25 Insofar as this third
information considering that, as found by the investigating prosecutor, the money received by issue is concerned, therefore, we find that no grave abuse of discretion has been committed by
petitioner was a cash advance for which he can only be held civilly liable, but which civil liability respondents which would warrant the granting of the writ of certiorari.
has already been extinguished. Citing the case ofYong Chan Kim vs. People, et al., 19 which held WHEREFORE, the resolutions appealed from are hereby AFFIRMED, with the modification that
that a cash advance is in the form of a loan and, therefore, there can be no estafa committed, respondent Ombudsman is DIRECTED to produce the pertinent records of the preliminary
petitioner argues that he only incurred civil liability for the cash advance he obtained from the investigation before the Sandiganbayan at the proper juncture of the proceedings therein and on
Manila Hotel. However, he contends that such liability had allegedly been extinguished when his sufficient justification therefor.
leave credits and other benefits were withheld, the total of which was more than sufficient to SO ORDERED.
liquidate the advance made.
6

MENDOZA VS PEOPLE in excess of its jurisdiction "in supplanting the public prosecutor’s findings of probable cause
While the determination of probable cause to charge a person of a crime is the sole function of with her own findings of insufficiency of evidence and lack of probable cause."20
the. prosecutor, the trial court may, in the protection of one's fundamental right to liberty, Aggrieved, Alfredo filed a petition for review under Rule 45 before this court. In essence, he
dismiss the case if, upon a personal assessment of the evidence, it finds that the evidence does argued that the trial court was correct in finding that there was no probable cause as shown by
not establish probable cause. the evidence on record. He argued that "judicial determination of probable cause is broader than
This is a petition for review on certiorari1 assailing the Court of Appeals' decision2 dated January [the] executive determination of probable cause"21and that "[i]t is not correct to say that the
14, 2011, which reversed the Regional Trial Court's dismissal of the complaint against petitioner determination of probable cause is exclusively vested on the prosecutor x x x."22
Alfredo C. Mendoza for qualified theft and estafa. In its comment,23 Juno Cars argued that Alfredo presented questions, issues, and arguments that
This case stems from a complaint-affidavit filed by Juno Cars, Inc. through its representative, Raul were a mere rehash of those already considered and passed upon by the appellate court.
C. Evangelista, on January 8, 2008 for qualified theft and estafa against Alfredo.3 The Office of the Solicitor General, arguing for public respondent, stated in its comment24 that
In the complaint-affidavit, Juno Cars alleged that on June 2, 2007, it hired Alfredo as Trade- the appellate court correctly sustained the public prosecutor in his findings of probable cause
In/Used Car Supervisor. On November 19, 2007, its Dealer/Operator, Rolando Garcia, conducted against Alfredo. Since there was no showing of grave abuse of discretion on the part of
a partial audit of the used cars and discovered that five (5) cars had been sold and released by Prosecutor Rey F. Delgado, the trial court should respect his determination of probable cause.
Alfredo without Rolando’s or the finance manager’s permission.4 In his reply,25 Alfredo reiterated that "judicial determination of probable cause[,] while not a
The partial audit showed that the buyers of the five cars made payments, but Alfredo failed to superior faculty[,] covers a broader encompassing perspective in the disposition of the issue on
remit the payments totalling P886,000.00. It was further alleged that while there were 20 cars the existence of probable cause."26He argued that the findings of the trial court should be
under Alfredo’s custody, only 18 were accounted for. Further investigation revealed that Alfredo accorded greater weight than the appellate court’s. It merely reviewed the findings of the trial
failed to turn over the files of a 2001 Hyundai Starex and a Honda City 1.5 LXI. Juno Cars alleged court.
that taking into account the unremitted amounts and the acquisition cost of the Honda City, The primordial issue is whether the trial court may dismiss an information filed by the prosecutor
Alfredo pilfered a total amount of P1,046,000.00 to its prejudice and damage.5 on the basis of its own independent finding of lack of probable cause.
In his counter-affidavit, Alfredo raised, among others, Juno Cars’ supposed failure to prove Time and again, this court has been confronted with the issue of the difference between the
ownership over the five (5) cars or its right to possess them with the purported unremitted determination of probable cause by the prosecutor on one hand and the determination of
payments. Hence, it could not have suffered damage.6 probable cause by the judge on the other. We examine these two concepts again.
On March 4, 2008, Provincial Prosecutor Rey F. Delgado issued a Resolution7 finding probable Juno Cars filed a complaint against Alfredo for qualified theft27 and estafa under Article 315,
cause and recommending the filing of an information against Alfredo for qualified theft and fourth paragraph, no. 3(c)28 of the Revised Penal Code. Since qualified theft is punishable by
estafa. reclusion perpetua, a preliminary investigation must first be conducted "to determine whether
Alfredo moved for reconsideration, but the motion was denied. 8 He then filed a petition for there is sufficient ground to engender a well-founded belief that a crime has been committed
review with the Department of Justice on May 16, 2008.9 and the respondent is probably guilty thereof, and should be held for trial," in accordance with
While Alfredo’s motion for reconsideration was still pending before the Office of the City Rule 112, Section 1 of the Rules on Criminal Procedure.
Prosecutor of Mandaluyong, two informations for qualified theft10 and estafa11 were filed before At this stage, the conduct of the preliminary investigation and the subsequent determination of
the Regional Trial Court, Branch 212, Mandaluyong City. On March 31, 2008, Alfredo filed a the existence of probable cause lie solely within the discretion of the public prosecutor.29 If upon
motion for determination of probable cause12 before the trial court. On April 28, 2008, he also evaluation of the evidence, the prosecutor finds sufficient basis to find probable cause, he or she
filed a motion to defer arraignment. shall then cause the filing of the information with the court.
Several clarificatory hearings were scheduled but were not conducted.13 On February 4, 2009, Once the information has been filed, the judge shall then "personally evaluate the resolution of
the parties agreed to submit all pending incidents, including the clarificatory hearing, for the prosecutor and its supporting evidence"30 to determine whether there is probable cause to
resolution.14 issue a warrant of arrest. At this stage, a judicial determination of probable cause exists.
On March 3, 2009, the trial court, through Presiding Judge Rizalina Capco-Umali, issued an In People v. Castillo and Mejia,31 this court has stated:
order15 dismissing the complaint, stating that: There are two kinds of determination of probable cause: executive and judicial. The executive
After conducting an independent assessment of the evidence on record which includes the determination of probable cause is one made during preliminary investigation. It is a function
assailed Resolution dated 04 March 2008, the court holds that the evidence adduced does not that properly pertains to the public prosecutor who is given a broad discretion to determine
support a finding of probable cause for the offenses of qualified theft and estafa. x x x.16 whether probable cause exists and to charge those whom he believes to have committed the
Juno Cars filed a motion for reconsideration, which the trial court denied on July 3, 2009.17 crime as defined by law and thus should be held for trial. Otherwise stated, such official has the
Juno Cars then filed a petition for certiorari with the Court of Appeals, arguing that the trial court quasi-judicial authority to determine whether or not a criminal case must be filed in court.
acted without or in excess of its jurisdiction and with grave abuse of discretion when it dismissed Whether or not that function has been correctly discharged by the public prosecutor, i.e.,
the complaint. It argued that "the determination of probable cause and the decision whether or whether or not he has made a correct ascertainment of the existence of probable cause in a
not to file a criminal case in court, rightfully belongs to the public prosecutor."18 case, is a matter that the trial court itself does not and may not be compelled to pass upon.
On January 14, 2011, the Court of Appeals rendered a decision,19 reversed the trial court, and The judicial determination of probable cause, on the other hand, is one made by the judge to
reinstated the case. In its decision, the appellate court ruled that the trial court acted without or ascertain whether a warrant of arrest should be issued against the accused. The judge must
7

satisfy himself that based on the evidence submitted, there is necessity for placing the accused for the deferment or suspension of the proceedings until such appeal is resolved.36 (Emphasis
under custody in order not to frustrate the ends of justice. If the judge finds no probable cause, supplied)
the judge cannot be forced to issue the arrest warrant.32 In this case, the resolution dated March 4, 2008 of Prosecutor Rey F. Delgado found that the
The difference is clear: The executive determination of probable cause concerns itself with facts and evidence were "sufficient to warrant the indictment of [petitioner] x x x." 37 There was
whether there is enough evidence to support an Information being filed. The judicial nothing in his resolution which showed that he issued it beyond the discretion granted to him by
determination of probable cause, on the other hand, determines whether a warrant of arrest law and jurisprudence.
should be issued. In People v. Inting:33 While the information filed by Prosecutor Delgado was valid, Judge Capco-Umali still had the
x x x Judges and Prosecutors alike should distinguish the preliminary inquiry which determines discretion to make her own finding of whether probable cause existed to order the arrest of the
probable cause for the issuance of a warrant of arrest from the preliminary investigation proper accused and proceed with trial.
which ascertains whether the offender should be held for trial or released. Even if the two Jurisdiction over an accused is acquired when the warrant of arrest is served. Absent this, the
inquiries are conducted in the course of one and the same proceeding, there should be no court cannot hold the accused for arraignment and trial.
confusion about the objectives. The determination of probable cause for the warrant of arrest is Article III, Section 2 of the Constitution states:
made by the Judge. The preliminary investigation proper—whether or not there is reasonable The right of the people to be secure in their persons, houses, papers, and effects against
ground to believe that the accused is guilty of the offense charged and, therefore, whether or unreasonable searches and seizures of whatever nature and for any purpose shall be inviolable,
not he should be subjected to the expense, rigors and embarrassment of trial—is the function of and no search warrant or warrant of arrest shall issue except upon probable cause to be
the Prosecutor.34 (Emphasis supplied) determined personally by the judge after examination under oath or affirmation of the
While it is within the trial court’s discretion to make an independent assessment of the evidence complainant and the witnesses he may produce, and particularly describing the place to be
on hand, it is only for the purpose of determining whether a warrant of arrest should be issued. searched and the persons or things to be seized.
The judge does not act as an appellate court of the prosecutor and has no capacity to review the The Constitution prohibits the issuance of search warrants or warrants of arrest where the judge
prosecutor’s determination of probable cause; rather, the judge makes a determination of has not personally determined the existence of probable cause. The phrase "upon probable
probable cause independent of the prosecutor’s finding. cause to be determined personally by the judge after examination under oath or affirmation of
People v. Court of Appeals and Jonathan Cerbo35 discussed the rationale. In that case, Jonathan the complainant and the witnesses he may produce" allows a determination of probable cause
Cerbo allegedly shot Rosalinda Dy in the presence of his father, Billy Cerbo. An information for by the judge ex parte.
murder was filed against Jonathan Cerbo. The daughter of Rosalinda Dy, as private complainant, For this reason, Section 6, paragraph (a) of Rule 112 of the Rules on Criminal Procedure
executed a complaint-affidavit charging Billy Cerbo with conspiracy. The prosecutor then filed a mandates the judge to "immediately dismiss the case if the evidence on record fails to establish
motion to amend the information, which was granted by the court. The information was then probable cause." Section 6, paragraph (a) of Rule 112 reads:
amended to include Billy Cerbo as one of the accused, and a warrant of arrest was issued against Section 6. When warrant of arrest may issue. — (a) By the Regional Trial Court. — Within ten
him. (10) days from the filing of the complaint or information, the judge shall personally evaluate the
Billy Cerbo filed a motion to quash the warrant arguing that it was issued without probable resolution of the prosecutor and its supporting evidence. He may immediately dismiss the case if
cause. The trial court granted this motion, recalled the warrant, and dismissed the case against the evidence on record clearly fails to establish probable cause. If he finds probable cause, he
him. The Court of Appeals affirmed this dismissal. This court, however, reversed the Court of shall issue a warrant of arrest, or a commitment order if the accused has already been arrested
Appeals and ordered the reinstatement of the amended information against Billy Cerbo, stating pursuant to a warrant issued by the judge who conducted the preliminary investigation or when
that: the complaint or information was filed pursuant to section 7 of this Rule. In case of doubt on the
In granting this petition, we are not prejudging the criminal case or the guilt or innocence of existence of probable cause, the judge may order the prosecutor to present additional evidence
Private Respondent Billy Cerbo. We are simply saying that, as a general rule, if the information is within five (5) days from notice and the issue must be resolved by the court within thirty (30)
valid on its face and there is no showing of manifest error, grave abuse of discretion or prejudice days from the filing of the complaint of information.
on the part of the public prosecutor, courts should not dismiss it for ‘want of evidence,’ because In People v. Hon. Yadao:38
evidentiary matters should be presented and heard during the trial. The functions and duties of Section 6, Rule 112 of the Rules of Court gives the trial court three options upon the filing of the
both the trial court and the public prosecutor in "the proper scheme of things" in our criminal criminal information: (1) dismiss the case if the evidence on record clearly failed to establish
justice system should be clearly understood. probable cause; (2) issue a warrant of arrest if it finds probable cause; and (3) order the
The rights of the people from what could sometimes be an "oppressive" exercise of government prosecutor to present additional evidence within five days from notice in case of doubt as to the
prosecutorial powers do need to be protected when circumstances so require. But just as we existence of probable cause.
recognize this need, we also acknowledge that the State must likewise be accorded due process. But the option to order the prosecutor to present additional evidence is not
Thus, when there is no showing of nefarious irregularity or manifest error in the performance of mandatory.1âwphi1 The court’s first option under the above is for it to "immediately dismiss the
a public prosecutor’s duties, courts ought to refrain from interfering with such lawfully and case if the evidence on record clearly fails to establish probable cause." That is the situation
judicially mandated duties. here: the evidence on record clearly fails to establish probable cause against the
In any case, if there was palpable error or grave abuse of discretion in the public prosecutor’s respondents.39 (Emphasis supplied)
finding of probable cause, the accused can appeal such finding to the justice secretary and move
8

It is also settled that "once a complaint or information is filed in court, any disposition of the
case, whether as to its dismissal or the conviction or the acquittal of the accused, rests in the
sound discretion of the court."40
In this case, Judge Capco-Umali made an independent assessment of the evidence on record and
concluded that "the evidence adduced does not support a finding of probable cause for the
offenses of qualified theft and estafa."41 Specifically, she found that Juno Cars "failed to prove by
competent evidence"42 that the vehicles alleged to have been pilfered by Alfredo were lawfully
possessed or owned by them, or that these vehicles were received by Alfredo, to be able to
substantiate the charge of qualified theft. She also found that the complaint "[did] not state with
particularity the exact value of the alleged office files or their valuation purportedly have been
removed, concealed or destroyed by the accused,"43 which she found crucial to the prosecution
of the crime of estafa under Article 315, fourth paragraph, no. 3(c) of the Revised Penal Code.
She also noted that:
x x x As a matter of fact, this court had even ordered that this case be set for clarificatory hearing
to clear out essential matters pertinent to the offense charged and even directed the private
complainant to bring documents relative to the same/payment as well as affidavit of
witnesses/buyers with the end view of satisfying itself that indeed probable cause exists to
commit the present case which private complainant failed to do.44
Accordingly, with the present laws and jurisprudence on the matter, Judge Capco-Umali correctly
dismissed the case against Alfredo.
Although jurisprudence and procedural rules allow it, a judge must always proceed with caution
in dismissing cases due to lack of probable cause, considering the preliminary nature of the
evidence before it. It is only when he or she finds that the evidence on hand absolutely fails to
support a finding of probable cause that he or she can dismiss the case. On the other hand, if a
judge finds probable cause, he or she must not hesitate to proceed with arraignment and trial in
order that justice may be served.
WHEREFORE, the petition is GRANTED. The decision dated January 14, 2011 of the Court of
Appeals in CA-G.R. SP. No. 110774 is REVERSED and SET ASIDE. Criminal Case Nos. MC08-11604-
05 against Alfredo C. Mendoza are DISMISSED.
SO ORDERED.
9

LIM VS FELIX 194 SCRA 292 On November 21, 1989, petitioners Vicente Lim, Sr. and Susana Lim filed with us a verified
May a Judge without ascertaining the facts through his own personal determination and relying petition for change of venue. (Case No. A.M. No. 89-11-1270-MTC, formerly, G.R. Nos. 90587-90)
solely on the certification or recommendation of a prosecutor that a probable cause exists issue On December 14, 1989, we issued an en banc Resolution authorizing the change of venue from
a warrant of arrest? the Regional Trial Court of Masbate to the Regional Trial Court of Makati to avoid a miscarriage
On March 17, 1989, at about 7:30 o'clock in the morning, at the vicinity of the airport road of the of justice, to wit:
Masbate Domestic Airport, located at the municipality of Masbate province of Masbate, Acting on the petition for change of venue of the trial of Criminal Cases Nos. 5811,
Congressman Moises Espinosa, Sr. and his security escorts, namely Provincial Guards Antonio 5812, 5813, and 5814 from the Regional Trial Court, Masbate, Masbate to any of the
Cortes, Gaspar Amaro, and Artemio Fuentes were attacked and killed by a lone assassin. Dante Regional Trial Courts at Quezon City or Makati, the Court Resolved to (a) GRANT the
Siblante another security escort of Congressman Espinosa, Sr. survived the assassination plot, aforesaid petition for transfer of venue in order to avoid miscarriage of justice (Article
although, he himself suffered a gunshot wound. VIII, Section 5(4) of the Philippine Constitution); (b) DIRECT the Clerk of Court, Regional
An investigation of the incident then followed. Trial Court, Masbate, Masbate to transmit the records of the aforesaid cases to the
Thereafter, and for the purpose of preliminary investigation, the designated investigator, Harry Executive Judge, Regional Trial Court, Makati, for raffling among the other branches of
O. Tantiado, TSg, of the PC Criminal Investigation Service at Camp Bagong Ibalon Legazpi City the court; and (c) ORDER the Regional Trial Court of Masbate, Masbate to desist from
filed an amended complaint with the Municipal Trial Court of Masbate accusing, among others, further taking cognizance of the said cases until such time that the petition is finally
Vicente Lim, Sr., Mayor Susana Lim of Masbate (petitioners in G.R. Nos. 9405457), Jolly T. resolved.
Fernandez, Florencio T. Fernandez, Jr., Nonilon A. Bagalihog, Mayor Nestor C. Lim and Mayor The cases were raffled to Branch 56 presided by respondent Judge Nemesio S. Felix.
Antonio Kho (petitioners in G.R. Nos. 94266-69) of the crime of multiple murder and frustrated Petitioners Vicente Lim, Sr. and Susana Lim filed with the respondent court several motions and
murder in connection with the airport incident. The case was docketed as Criminal Case No. manifestations which in substance prayed for the following:
9211. 1. An order be issued requiring the transmittal of the initial records of the preliminary
After conducting the preliminary investigation, the court issued an order dated July 31, 1989 inquiry or investigation conducted by the Municipal Judge Barsaga of Masbate for the
stating therein that: best enlightenment of this Honorable Court in its personal determination of the
. . . after weighing the affidavits and answers given by the witnesses for the existence of a probable cause or prima facie evidence as well as its determination of
prosecution during the preliminary examination in searching questions and answers, the existence of guilt, pursuant to the mandatory mandate of the constitution that no
concludes that a probable cause has been established for the issuance of a warrant of warrant shall issue unless the issuing magistrate shall have himself been personally
arrest of named accused in the amended complaint, namely, Jimmy Cabarles, Ronnie convinced of such probable cause.
Fernandez, Nonilon Bagalihog, Jolly Fernandez, Florencio Fernandez, Jr., Vicente Lim, 2. Movants be given ample opportunity to file their motion for preliminary
Sr., Susana Lim, Nestor Lim, Antonio Kho, Jaime Liwanag, Zaldy Dumalag and Rene investigation as a matter of right; and
Tualla aliasTidoy. (Rollo, p. 58, G.R. Nos. 94054-57) 3. In the event that this court may later be convinced of the existence of a probable
xxx xxx xxx cause, to be allowed to file a motion for reduction of bail or for admission of bail. (p.
In the same Order, the court ordered the arrest of the petitioners and recommended the 17, Rollo, G.R. Nos. 94054-57)
amount of P200,000.00 as bail for the provisional liberty of each of the accused. In another manifestation, the Lims reiterated that the court conduct a hearing to determine if
Petitioners Jolly Fernandez and Nonilon Bagalihog filed a motion for the reduction of bail which there really exists aprima facie case against them in the light of documents which are
was granted by the court and they were allowed to post bail in the amount of P150,000.00 each. recantations of some witnesses in the preliminary investigation. The motions and manifestations
Except for Jimmy Cabarles, all the rest of the accused posted bail at P200,000.00 each. were opposed by the prosecution.
On August 29, 1989, the entire records of the case consisting of two hundred sixty one (261) On July 5, 1990, the respondent court issued an order denying for lack of merit the motions and
pages were transmitted to the Provincial Prosecutor of Masbate. Respondent Acting Fiscal manifestations and issued warrants of arrest against the accused including the petitioners
Antonio C. Alfane was designated to review the case. herein. The respondent Judge said:
On September 22, 1989, Fiscal Alfane issued a Resolution which affirmed the finding of a prima In the instant cases, the preliminary investigation was conducted by the Municipal Trial
facie case against the petitioners but differed in the designation of the crime in that the ruled Court of Masbate, Masbate which found the existence of probable cause that the
that ". . . all of the accused should not only be charged with Multiple Murder With Frustrated offense of multiple murder was committed and that all the accused are probably guilty
Murder" but for a case of MURDER for each of the killing of the four victims and a physical thereof, which was affirmed upon review by the Provincial Prosecutor who properly
injuries case for inflicting gunshot wound on the buttocks of Dante Siblante." (Annex "H", filed with the Regional Trial Court four separate informations for murder. Considering
Comment of Fiscal Alfane, p. 186, Rollo, G.R. Nos. 94054-57) A motion to reconsider the that both the two competent officers to whom such duty was entrusted by law have
Resolution filed by petitioners Vicente Lim, Sr. and Mayor Susana Lim was denied. declared the existence of probable cause, each information is complete in form and
On October 30, 1989, Fiscal Alfane filed with the Regional Trial Court of Masbate, four (4) substance, and there is no visible defect on its face, this Court finds it just and proper to
separate informations of murder against the twelve (12) accused with a recommendation of no rely on the prosecutor's certification in each information which reads: (pp. 19-20, Rollo,
bail. G.R Nos. 94054-57; Emphasis supplied)
xxx xxx xxx
10

The petitioners then filed these consolidated petitions questioning the July 5, 1990 Order. probable cause. For as the ensuing events would show, after petitioners had submitted
In a Resolution dated July 17, 1990 in G.R. Nos. 94054-57, we issued ". . . a TEMPORARY the required affidavits, respondent wasted no time in issuing the warrants of arrest in
RESTRAINING ORDER, effective immediately and continuing until further orders from this Court, the case where he was satisfied that probable cause existed.
ordering the respondent judge or his duly authorized representatives or agents to CEASE and The case of Soliven v. Makasiar (167 SCRA 393 [19881) was decided after the effectivity of the
DESIST from enforcing or implementing the warrant of arrest without bail issued against the 1987 Constitution. We stated:
petitioners in his Order dated July 5, 1990 in Criminal Cases Nos. 5811-14. The second issue, raised by petitioner Beltran, calls for an interpretation of the
In another Resolution dated July 31, 1990 in G.R. Nos. 94266-69, we resolved: constitutional provision on the issuance of warrants of arrest. The pertinent provision
xxx xxx xxx reads:
. . . To ISSUE writs of (1) PRELIMINARY MANDATORY INJUNCTION, ordering and Art. III, Sec. 2. The right of the people to be secure in their persons, houses, papers and
directing the respondent judge to recall/set aside and/or annul the legal effects of the effects against unreasonable searches and seizures of whatever nature and for any
warrants of arrest without bail issued against and served upon herein petitioners Jolly purpose shall be inviolable, and no search warrant or warrant of arrest shall issue
T. Fernandez, Florencio T. Fernandez, Jr. and Nonilon Bagalihog and release them from except upon probable cause to be determined personally by the judge after
confinement at PC-CIS Detention Center, Camp Crame, Quezon City; and (2) examination under oath or affirmation of the complainant and the witnesses he may
TEMPORARY RESTRAINING ORDER, effective immediately and continuing until further produce, and particularly describing the place to be searched and the persons or things
orders from this Court, ordering the respondent judge or his duly authorized to be seized.
representatives or agents, to CEASE AND DESIST from enforcing or implementing the The addition of the word "personally" after the word "determined" and the deletion of
warrants of arrest without bail issued against petitioners Mayors Nestor C. Lim and the grant of authority by the 1973 Constitution to issue warrants to "other respondent
Antonio T. Kho. officers as may be authorized by law", has apparently convinced petitioner Beltran that
The primary issue in these consolidated petitions centers on whether or not a judge may issue a the Constitution now requires the judge to personally examine the complainant and his
warrant of arrest without bail by simply relying on the prosecution's certification and witnesses in his determination of probable cause for the issuance of arrest. This is not
recommendation that a probable cause exists. an accurate interpretation.
This is not a novel question. In the case of Placer v. Villanueva (126 SCRA 463 [1983]), we ruled What the Constitution underscores is the exclusive and personal responsibility of the
that a judge may rely upon the fiscal's certification of the existence of probable cause and, on issuing judge to satisfy himself of the existence of probable cause. In satisfying himself
the basis thereof, issue a warrant of arrest. However, the certification does not bind the judge to of the existence of probable cause for the issuance of a warrant of arrest, the judge is
come out with the warrant of arrest. This decision interpreted the "search and seizure" provision not required to personally examine the complainant and his witnesses. Following
of the 1973 Constitution which provides: established doctrine and procedures, he shall: (1) personally evaluate the report and
. . . no search warrant or warrant of arrest shall issue except upon probable cause to be the supporting documents submitted by the fiscal regarding the existence of probable
determined by the judge, or such other responsible officer as may be authorized by cause and, on the basis thereof, issue a warrant of arrest; or (2) if on the basis thereof
law, after examination under oath or affirmation of the complainant and the witnesses he finds no probable cause, he may disregard the fiscal's report and require the
he may produce . . . submission of supporting affidavits of witnesses to aid him in arriving at a conclusion as
We ruled: to the existence of probable cause.
. . . The issuance of a warrant is not a mere ministerial function; it calls for the exercise Sound policy dictates this procedure, otherwise judges would be unduly laden with the
of judicial discretion on the part of the issuing magistrate. This is clear from the preliminary examinations and investigation of criminal complaints instead of
following provisions of Section 6, Rule 112 of the Rules of Court. concentrating on hearing and deciding cases filed before their courts.
Warrant of arrest, when issued. — If the judge be satisfied from the preliminary The decision in People v. Honorable Enrique B. Inting, et al. (G.R. No. 88919, July 25, 1990),
examination conducted by him or by the investigating officer that the offense reiterated the above interpretation of "personal" determination by the Judge:
complained of has been committed and that there is reasonable ground to believe that We emphasize important features of the constitutional mandate that ". . . no search
the accused has committed it, he must issue a warrant or order for his arrest. warrant or warrant of arrest shall issue except upon probable cause to be determined
Under this section, the judge must satisfy himself of the existence of probable cause personally by the judge . . ." (Article III, Section 2, Constitution)
before issuing a warrant or order of arrest. If on the face of the information the judge First, the determination of probable cause is a function of the Judge. It is not for the
finds no probable cause, he may disregard the fiscal's certification and require the Provincial Fiscal or Prosecutor nor for the Election Supervisor to ascertain. Only the
submission of the affidavits of witnesses to aid him in arriving at a conclusion as to the Judge and the Judge alone makes this determination.
existence of a probable cause. This has been the rule since U.S. v. Ocampo (18 Phil. 1) Second, the preliminary inquiry made by a Prosecutor does not bind the Judge. It
and Amarga v. Abbas (98 Phil. 739). And this evidently is the reason for the issuance by merely assists him to make the determination of probable cause. The Judge does not
respondent of the questioned orders of April 13, 15, 16, 19, 1982 and July 13, 1982. have to follow what the Prosecutor presents to him. By itself, the Prosecutor's
Without the affidavits of the prosecution witnesses and other evidence which, as a certification of probable cause is ineffectual. It is the report, the affidavits, the
matter of long-standing practice had been attached to the information filed in his sala, transcripts of stenographic notes (if any), and all other supporting documents behind
respondent found the informations inadequate bases for the determination of
11

the Prosecutor's certification which are material in assisting the Judge to make his of a warrant of arrest. It might be added that this distinction accords, rather
determination. than conflicts, with the rationale of Salta because both law and rule, in
And third, Judges and Prosecutors alike should distinguish the preliminary inquiry restricting to judges the authority to order arrest, recognize the function to
which determines probable cause for the issuance of a warrant of arrest from the be judicial in nature.
preliminary investigation proper which ascertains whether the offender should be held We reiterate that preliminary investigation should be distinguished as to whether it is
for trial or released. Even if the two inquiries are conducted in the course of one and an investigation for the determination of a sufficient ground for the filing of the
the same proceeding, there should be no confusion about the objectives. The information or it is an investigation for the determination of a probable cause for the
determination of probable cause for the warrant of arrest is made by the Judge. The issuance of a warrant of arrest. The first kind of preliminary investigation is executive in
preliminary investigation proper –– whether or not there is reasonable ground to nature. It is part of the prosecution's job. The second kind of preliminary investigation
believe that the accused is guilty of the offense charged and, therefore, whether or not which is more properly called preliminary examination is judicial in nature and is
he should be subjected to the expense, rigors and embarrassment of trial –– is the lodged with the Judge. . . .
function of the Prosecutor. Finally in the recent case of People v. Delgado, et al. (G.R. Nos. 93419-32, September 18, 1990)
The Court made this clear in the case of Castillo v. Villaluz (171 SCRA 39 [19891): there is a statement that the judge may rely on the resolution of COMELEC to file the
Judges of Regional Trial Courts (formerly Courts of First Instance) no longer information by the same token that it may rely on the certification made by the prosecutor who
have authority to conduct preliminary investigations. That authority, at one conducted the preliminary investigation in the issuance of the warrant of arrest. We, however,
time reposed in them under Sections 13, 14 and 16, Rule 112 of the Rules of also reiterated that ". . . the court may require that the record of the preliminary investigation be
Court of 1964, (See Sec. 4, Rule 108, Rules of Court of 1940; People v. Solon, submitted to it to satisfy itself that there is probable cause which will warrant the issuance of a
47 Phil. 443, cited in Moran, Comments on the Rules, 1980 ed., Vol. 4, pp. warrant of arrest." (Section 2, Article III, Constitution). Reliance on the COMELEC resolution or
115-116) was removed from them by the 1985 Rules on Criminal Procedure, the Prosecutor's certification presupposes that the records of either the COMELEC or the
effective on January 1, 1985, (Promulgated on November 11, 1984) which Prosecutor have been submitted to the Judge and he relies on the certification or resolution
deleted all provisions granting that power to said Judges. We had occasion to because the records of the investigation sustain the recommendation. The warrant issues not on
point tills out in Salta v. Court of Appeals, 143 SCRA 228, and to stress as well the strength of the certification standing alone but because of the records which sustain it.
certain other basic propositions, namely: (1) that the conduct of a It is obvious from the present petition that notwithstanding the above decisions, some Judges
preliminary investigation is "not a judicial function . . . (but) part of the are still bound by the inertia of decisions and practice under the 1935 and 1973 Constitutions
prosecution's job, a function of the executive," (2) that whenever "there are and are sadly confused or hesitant. Prosecutors are also interested in a clear cut ruling. We will,
enough his or prosecutors to conduct preliminary investigations, courts are therefore, restate the rule in greater detail and hopefully clearer terms.
counseled to leave this job which is essentially executive to them," and the There is no problem with search warrants which are relatively fewer and far between and where
fact "that a certain power is granted does not necessary mean that it should there is no duplication of work between the Judge and the Prosecutor. The problem lies with
be indiscriminately exercised. warrants of arrest especially in metropolitan or highly urban areas. If a Judge has to personally
The 1988 Amendments to the 1985 Rules on Criminal Procedure, declared question each complainant and witness or go over the records of the Prosecutor's investigation
effective on October 1, 1988, (The 1988 Amendments were published in the page by page and word for word before he acts on each of a big pile of applications for arrest
issue of Bulletin Today of October 29, 1988) did not restore that authority to warrants on his desk, he or she may have no more time for his or her more important judicial
Judges of Regional Trial Courts; said amendments did not in fact deal at all functions.
with the officers or courts having authority to conduct preliminary At the same time, the Judge cannot ignore the clear words of the 1987 Constitution which
investigations. requires ". . . probable cause to be personally determined by the judge . . .", not by any other
This is not to say, however, that somewhere along the line RTC Judges also officer or person.
lost the power to make a preliminary examination for the purpose of If a Judge relies solely on the certification of the Prosecutor as in this case where all the records
determining whether probable cause exists to justify the issuance of a of the investigation are in Masbate, he or she has not personally determined probable cause. The
warrant of arrest (or search warrant). Such a power –– indeed, it is as much a determination is made by the Provincial Prosecutor. The constitutional requirement has not
duty as it is a power –– has been and remains vested in every judge by the been satisfied. The Judge commits a grave abuse of discretion.
provisions in the Bill of Rights in the 1935, the 1973 and the present [1987] The records of the preliminary investigation conducted by the Municipal Court of Masbate and
Constitutions securing the people against unreasonable searches and reviewed by the respondent Fiscal were still in Masbate when the respondent Fiscal issued the
seizures, thereby placing it beyond the competence of mere Court Rule or warrants of arrest against the petitioners. There was no basis for the respondent Judge to make
Statute to revoke. The distinction must, therefore, be made clear while an his own personal determination regarding the existence of a probable cause for the issuance of a
RTC Judge may no longer conduct preliminary investigations to ascertain warrant of arrest as mandated by the Constitution. He could not possibly have known what
whether there is sufficient ground for the filing of a criminal complaint or transpired in Masbate as he had nothing but a certification. Significantly, the respondent Judge
information, he retains the authority, when such a pleading is filed with his denied the petitioners' motion for the transmittal of the records on the ground that the mere
court, to determine whether there is probable cause justifying the issuance
12

certification and recommendation of the respondent Fiscal that a probable cause exists is WHEREFORE, the instant petitions are hereby GRANTED. The questioned Order of respondent
sufficient for him to issue a warrant of arrest. Judge Nemesio S. Felix of Branch 56, Regional Trial Court of Makati dated July 5, 1990 is declared
We reiterate the ruling in Soliven v. Makasiar that the Judge does not have to personally NULL and VOID and SET ASIDE. The Temporary Restraining Orders and Preliminary Mandatory
examine the complainant and his witnesses. The Prosecutor can perform the same functions as a Injunction issued in the instant Petitions are made PERMANENT.
commissioner for the taking of the evidence. However, there should be a report and necessary SO ORDERED.
documents supporting the Fiscal's bare certification. All of these should be before the Judge.
The extent of the Judge's personal examination of the report and its annexes depends on the
circumstances of each case.1âwphi1 We cannot determine beforehand how cursory or
exhaustive the Judge's examination should be. The Judge has to exercise sound discretion for,
after all, the personal determination is vested in the Judge by the Constitution. It can be as brief
or as detailed as the circumstances of each case require. To be sure, the Judge must go beyond
the Prosecutor's certification and investigation report whenever necessary. He should call for the
complainant and witnesses themselves to answer the court's probing questions when the
circumstances of the case so require.
It is worthy to note that petitioners Vicente Lim, Sr. and Susana Lim presented to the respondent
Judge documents of recantation of witnesses whose testimonies were used to establish a prima
facie case against them. Although, the general rule is that recantations are not given much
weight in the determination of a case and in the granting of a new trial (Tan Ang Bun v. Court of
Appeals, et al. G.R. No. L-47747, February 15, 1990, People v. Lao Wan Sing, 46 SCRA 298 [1972])
the respondent Judge before issuing his own warrants of arrest should, at the very least, have
gone over the records of the preliminary examination conducted earlier in the light of the
evidence now presented by the concerned witnesses in view of the "political undertones"
prevailing in the cases. Even the Solicitor General recognized the significance of the recantations
of some witnesses when he recommends a reinvestigation of the cases, to wit:
It must be pointed out, however, that among the documents attached to this Petition
are affidavits of recantation subsequently executed by Jimmy Cabarles and Danilo
Lozano and an affidavit executed by one, Camilo Sanano, father of the complainant's
witnesses, Renato and Romeo Sanano. It was precisely on the strength of these earlier
written statements of these witnesses that the Municipal Trial Court of Masbate found
the existence of a prima facie case against petitioners and accordingly recommended
the filing of a Criminal Information. Evidently, the same written statements were also
the very basis of the "Fiscal's Certification", since the attached affidavits of recantation
were not yet then available. Since the credibility of the prosecution witnesses is now
assailed and put in issue and, since the petitioners have not yet been arraigned, it
would be to the broader interest of justice and fair play if a reinvestigation of this case
be had to secure the petitioners against hasty prosecution and to protect them from
an open and public accusation of crime, from the trouble, expense and anxiety of a
public trial, and also to protect the State from useless and expensive trials (Salonga v.
Paño G.R. No. 59524, February 18,1985). (Rollo of G.R. Nos. 94054-56, pp. 200-201)
We reiterate that in making the required personal determination, a Judge is not precluded from
relying on the evidence earlier gathered by responsible officers. The extent of the reliance
depends on the circumstances of each case and is subject to the Judge's sound discretion.
However, the Judge abuses that discretion when having no evidence before him, he issues a
warrant of arrest.
Indubitably, the respondent Judge committed a grave error when he relied solely on the
Prosecutor's certification and issued the questioned Order dated July 5, 1990 without having
before him any other basis for his personal determination of the existence of a probable cause.
13

BACHE VS RUIZ 1970, an order dismissing the petition for dissolution of the search warrant. In the meantime, or
This is an original action of certiorari, prohibition and mandamus, with prayer for a writ of on April 16, 1970, the Bureau of Internal Revenue made tax assessments on petitioner
preliminary mandatory and prohibitory injunction. In their petition Bache & Co. (Phil.), Inc., a corporation in the total sum of P2,594,729.97, partly, if not entirely, based on the documents
corporation duly organized and existing under the laws of the Philippines, and its President, thus seized. Petitioners came to this Court.
Frederick E. Seggerman, pray this Court to declare null and void Search Warrant No. 2-M-70
issued by respondent Judge on February 25, 1970; to order respondents to desist from enforcing The petition should be granted for the following reasons:chanrob1es virtual 1aw library
the same and/or keeping the documents, papers and effects seized by virtue thereof, as well as
from enforcing the tax assessments on petitioner corporation alleged by petitioners to have 1. Respondent Judge failed to personally examine the complainant and his witness.
been made on the basis of the said documents, papers and effects, and to order the return of
the latter to petitioners. We gave due course to the petition but did not issue the writ of The pertinent provisions of the Constitution of the Philippines and of the Revised Rules of Court
preliminary injunction prayed for therein. are:jgc:chanrobles.com.ph

The pertinent facts of this case, as gathered from record, are as follows:chanrob1es virtual 1aw "(3) The right of the people to be secure in their persons, houses, papers and effects against
library unreasonable searches and seizures shall not be violated, and no warrants shall issue but upon
probable cause, to be determined by the judge after examination under oath or affirmation of
On February 24, 1970, respondent Misael P. Vera, Commissioner of Internal Revenue, wrote a the complainant and the witnesses he may produce, and particularly describing the place to be
letter addressed to respondent Judge Vivencio M. Ruiz requesting the issuance of a search searched, and the persons or things to be seized." (Art. III, Sec. 1, Constitution.)
warrant against petitioners for violation of Section 46(a) of the National Internal Revenue Code,
in relation to all other pertinent provisions thereof, particularly Sections 53, 72, 73, 208 and 209, "SEC. 3. Requisites for issuing search warrant. — A search warrant shall not issue but upon
and authorizing Revenue Examiner Rodolfo de Leon, one of herein respondents, to make and file probable cause in connection with one specific offense to be determined by the judge or justice
the application for search warrant which was attached to the letter. of the peace after examination under oath or affirmation of the complainant and the witnesses
In the afternoon of the following day, February 25, 1970, respondent De Leon and his witness, he may produce, and particularly describing the place to be searched and the persons or things
respondent Arturo Logronio, went to the Court of First Instance of Rizal. They brought with them to be seized.
the following papers: respondent Vera’s aforesaid letter-request; an application for search
warrant already filled up but still unsigned by respondent De Leon; an affidavit of respondent "No search warrant shall issue for more than one specific offense.
Logronio subscribed before respondent De Leon; a deposition in printed form of respondent
Logronio already accomplished and signed by him but not yet subscribed; and a search warrant "SEC. 4. Examination of the applicant. — The judge or justice of the peace must, before issuing
already accomplished but still unsigned by respondent Judge. the warrant, personally examine on oath or affirmation the complainant and any witnesses he
At that time respondent Judge was hearing a certain case; so, by means of a note, he instructed may produce and take their depositions in writing, and attach them to the record, in addition to
his Deputy Clerk of Court to take the depositions of respondents De Leon and Logronio. After the any affidavits presented to him." (Rule 126, Revised Rules of Court.)
session had adjourned, respondent Judge was informed that the depositions had already been
taken. The stenographer, upon request of respondent Judge, read to him her stenographic The examination of the complainant and the witnesses he may produce, required by Art. III, Sec.
notes; and thereafter, respondent Judge asked respondent Logronio to take the oath and 1, par. 3, of the Constitution, and by Secs. 3 and 4, Rule 126 of the Revised Rules of Court, should
warned him that if his deposition was found to be false and without legal basis, he could be be conducted by the judge himself and not by others. The phrase "which shall be determined by
charged for perjury. Respondent Judge signed respondent de Leon’s application for search the judge after examination under oath or affirmation of the complainant and the witnesses he
warrant and respondent Logronio’s deposition, Search Warrant No. 2-M-70 was then sign by may produce," appearing in the said constitutional provision, was introduced by Delegate
respondent Judge and accordingly issued. Francisco as an amendment to the draft submitted by the Sub-Committee of Seven. The
Three days later, or on February 28, 1970, which was a Saturday, the BIR agents served the following discussion in the Constitutional Convention (Laurel, Proceedings of the Philippine
search warrant petitioners at the offices of petitioner corporation on Ayala Avenue, Makati, Constitutional Convention, Vol. III, pp. 755-757) is enlightening:jgc:chanrobles.com.ph
Rizal. Petitioners’ lawyers protested the search on the ground that no formal complaint or
transcript of testimony was attached to the warrant. The agents nevertheless proceeded with "SR. ORENSE. Vamos a dejar compañero los piropos y vamos al grano.
their search which yielded six boxes of documents.
On March 3, 1970, petitioners filed a petition with the Court of First Instance of Rizal praying that En los casos de una necesidad de actuar inmediatamente para que no se frusten los fines de la
the search warrant be quashed, dissolved or recalled, that preliminary prohibitory and justicia mediante el registro inmediato y la incautacion del cuerpo del delito, no cree Su Señoria
mandatory writs of injunction be issued, that the search warrant be declared null and void, and que causaria cierta demora el procedimiento apuntado en su enmienda en tal forma que podria
that the respondents be ordered to pay petitioners, jointly and severally, damages and frustrar los fines de la justicia o si Su Señoria encuentra un remedio para esto casos con el fin de
attorney’s fees. On March 18, 1970, the respondents, thru the Solicitor General, filed an answer compaginar los fines de la justicia con los derechos del individuo en su persona, bienes etcetera,
to the petition. After hearing, the court, presided over by respondent Judge, issued on July 29, etcetera.
14

Clerk Gonzales, stenographer Gaspar, complainant De Leon and witness Logronio went to
"SR. FRANCISCO. No puedo ver en la practica el caso hipottico que Su Señoria pregunta por la respondent Judge’s chamber and informed the Judge that they had finished the depositions.
siguiente razon: el que solicita un mandamiento de registro tiene que hacerlo por escrito y ese Respondent Judge then requested the stenographer to read to him her stenographic notes.
escrito no aparecer en la Mesa del Juez sin que alguien vaya el juez a presentar ese escrito o Special Deputy Clerk Gonzales testified as follows:jgc:chanrobles.com.ph
peticion de sucuestro. Esa persona que presenta el registro puede ser el mismo denunciante o
alguna persona que solicita dicho mandamiento de registro. Ahora toda la enmienda en esos "A And after finishing reading the stenographic notes, the Honorable Judge requested or
casos consiste en que haya peticion de registro y el juez no se atendra solamente a sea peticion instructed them, requested Mr. Logronio to raise his hand and warned him if his deposition will
sino que el juez examiner a ese denunciante y si tiene testigos tambin examiner a los testigos. be found to be false and without legal basis, he can be charged criminally for perjury. The
Honorable Court told Mr. Logronio whether he affirms the facts contained in his deposition and
"SR. ORENSE. No cree Su Señoria que el tomar le declaracion de ese denunciante por escrito the affidavit executed before Mr. Rodolfo de Leon.
siempre requeriria algun tiempo?.
"Q And thereafter?
"SR. FRANCISCO. Seria cuestio de un par de horas, pero por otro lado minimizamos en todo lo "A And thereafter, he signed the deposition of Mr. Logronio.
posible las vejaciones injustas con la expedicion arbitraria de los mandamientos de registro. Creo "Q Who is this he?
que entre dos males debemos escoger. el menor. "A The Honorable Judge.
"Q The deposition or the affidavit?
x x x "A The affidavit, Your Honor."cralaw virtua1aw library

"MR. LAUREL. . . . The reason why we are in favor of this amendment is because we are Thereafter, respondent Judge signed the search warrant.
incorporating in our constitution something of a fundamental character. Now, before a judge
could issue a search warrant, he must be under the obligation to examine personally under oath The participation of respondent Judge in the proceedings which led to the issuance of Search
the complainant and if he has any witness, the witnesses that he may produce . . ."cralaw Warrant No. 2-M-70 was thus limited to listening to the stenographer’s readings of her notes, to
virtua1aw library a few words of warning against the commission of perjury, and to administering the oath to the
complainant and his witness. This cannot be consider a personal examination. If there was an
The implementing rule in the Revised Rules of Court, Sec. 4, Rule 126, is more emphatic and examination at all of the complainant and his witness, it was the one conducted by the Deputy
candid, for it requires the judge, before issuing a search warrant, to "personally examine on oath Clerk of Court. But, as stated, the Constitution and the rules require a personal examination by
or affirmation the complainant and any witnesses he may produce . . ."cralaw virtua1aw library the judge. It was precisely on account of the intention of the delegates to the Constitutional
Convention to make it a duty of the issuing judge to personally examine the complainant and his
Personal examination by the judge of the complainant and his witnesses is necessary to enable witnesses that the question of how much time would be consumed by the judge in examining
him to determine the existence or non-existence of a probable cause, pursuant to Art. III, Sec. 1, them came up before the Convention, as can be seen from the record of the proceedings quoted
par. 3, of the Constitution, and Sec. 3, Rule 126 of the Revised Rules of Court, both of which above. The reading of the stenographic notes to respondent Judge did not constitute sufficient
prohibit the issuance of warrants except "upon probable cause." The determination of whether compliance with the constitutional mandate and the rule; for by that manner respondent Judge
or not a probable cause exists calls for the exercise of judgment after a judicial appraisal of facts did not have the opportunity to observe the demeanor of the complainant and his witness, and
and should not be allowed to be delegated in the absence of any rule to the contrary. to propound initial and follow-up questions which the judicial mind, on account of its training,
was in the best position to conceive. These were important in arriving at a sound inference on
In the case at bar, no personal examination at all was conducted by respondent Judge of the the all-important question of whether or not there was probable cause.
complainant (respondent De Leon) and his witness (respondent Logronio). While it is true that
the complainant’s application for search warrant and the witness’ printed-form deposition were 2. The search warrant was issued for more than one specific offense.
subscribed and sworn to before respondent Judge, the latter did not ask either of the two any
question the answer to which could possibly be the basis for determining whether or not there Search Warrant No. 2-M-70 was issued for" [v]iolation of Sec. 46(a) of the National Internal
was probable cause against herein petitioners. Indeed, the participants seem to have attached Revenue Code in relation to all other pertinent provisions thereof particularly Secs. 53, 72, 73,
so little significance to the matter that notes of the proceedings before respondent Judge were 208 and 209." The question is: Was the said search warrant issued "in connection with one
not even taken. At this juncture it may be well to recall the salient facts. The transcript of specific offense," as required by Sec. 3, Rule 126?
stenographic notes (pp. 61-76, April 1, 1970, Annex J-2 of the Petition) taken at the hearing of
this case in the court below shows that per instruction of respondent Judge, Mr. Eleodoro V. To arrive at the correct answer it is essential to examine closely the provisions of the Tax Code
Gonzales, Special Deputy Clerk of Court, took the depositions of the complainant and his witness, referred to above. Thus we find the following:chanrob1es virtual 1aw library
and that stenographic notes thereof were taken by Mrs. Gaspar. At that time respondent Judge
was at the sala hearing a case. After respondent Judge was through with the hearing, Deputy Sec. 46(a) requires the filing of income tax returns by corporations.
15

M-70 in this manner:jgc:chanrobles.com.ph


Sec. 53 requires the withholding of income taxes at source.
"Unregistered and private books of accounts (ledgers, journals, columnars, receipts and
Sec. 72 imposes surcharges for failure to render income tax returns and for rendering false and disbursements books, customers ledgers); receipts for payments received; certificates of stocks
fraudulent returns. and securities; contracts, promissory notes and deeds of sale; telex and coded messages;
business communications, accounting and business records; checks and check stubs; records of
Sec. 73 provides the penalty for failure to pay the income tax, to make a return or to supply the bank deposits and withdrawals; and records of foreign remittances, covering the years 1966 to
information required under the Tax Code. 1970."cralaw virtua1aw library

Sec. 208 penalizes" [a]ny person who distills, rectifies, repacks, compounds, or manufactures any The description does not meet the requirement in Art III, Sec. 1, of the Constitution, and of Sec.
article subject to a specific tax, without having paid the privilege tax therefore, or who aids or 3, Rule 126 of the Revised Rules of Court, that the warrant should particularly describe the things
abets in the conduct of illicit distilling, rectifying, compounding, or illicit manufacture of any to be seized.
article subject to specific tax . . .," and provides that in the case of a corporation, partnership, or
association, the official and/or employee who caused the violation shall be responsible. In Stonehill, this Court, speaking thru Mr. Chief Justice Roberto Concepcion,
said:jgc:chanrobles.com.ph
Sec. 209 penalizes the failure to make a return of receipts, sales, business, or gross value of
output removed, or to pay the tax due thereon. "The grave violation of the Constitution made in the application for the contested search
warrants was compounded by the description therein made of the effects to be searched for and
The search warrant in question was issued for at least four distinct offenses under the Tax Code. seized, to wit:chanrob1es virtual 1aw library
The first is the violation of Sec. 46(a), Sec. 72 and Sec. 73 (the filing of income tax returns), which
are interrelated. The second is the violation of Sec. 53 (withholding of income taxes at source). ‘Books of accounts, financial records, vouchers, journals, correspondence, receipts, ledgers,
The third is the violation of Sec. 208 (unlawful pursuit of business or occupation); and the fourth portfolios, credit journals, typewriters, and other documents and/or paper showing all business
is the violation of Sec. 209 (failure to make a return of receipts, sales, business or gross value of transactions including disbursement receipts, balance sheets and related profit and loss
output actually removed or to pay the tax due thereon). Even in their classification the six above- statements.’
mentioned provisions are embraced in two different titles: Secs. 46(a), 53, 72 and 73 are under
Title II (Income Tax); while Secs. 208 and 209 are under Title V (Privilege Tax on Business and "Thus, the warrants authorized the search for and seizure of records pertaining to all business
Occupation). transactions of petitioners herein, regardless of whether the transactions were legal or illegal.
The warrants sanctioned the seizure of all records of the petitioners and the aforementioned
Respondents argue that Stonehill, Et. Al. v. Diokno, Et Al., L-19550, June 19, 1967 (20 SCRA 383), corporations, whatever their nature, thus openly contravening the explicit command of our Bill
is not applicable, because there the search warrants were issued for "violation of Central Bank of Rights — that the things to be seized be particularly described — as well as tending to defeat
Laws, Internal Revenue (Code) and Revised Penal Code;" whereas, here Search Warrant No 2-M- its major objective: the elimination of general warrants."cralaw virtua1aw library
70 was issued for violation of only one code, i.e., the National Internal Revenue Code. The
distinction more apparent than real, because it was precisely on account of the Stonehill While the term "all business transactions" does not appear in Search Warrant No. 2-M-70, the
incident, which occurred sometime before the present Rules of Court took effect on January 1, said warrant nevertheless tends to defeat the major objective of the Bill of Rights, i.e., the
1964, that this Court amended the former rule by inserting therein the phrase "in connection elimination of general warrants, for the language used therein is so all-embracing as to include
with one specific offense," and adding the sentence "No search warrant shall issue for more than all conceivable records of petitioner corporation, which, if seized, could possibly render its
one specific offense," in what is now Sec. 3, Rule 126. Thus we said in business inoperative.
Stonehill:jgc:chanrobles.com.ph
In Uy Kheytin, Et. Al. v. Villareal, etc., Et Al., 42 Phil. 886, 896, this Court had occasion to explain
"Such is the seriousness of the irregularities committed in connection with the disputed search the purpose of the requirement that the warrant should particularly describe the place to be
warrants, that this Court deemed it fit to amend Section 3 of Rule 122 of the former Rules of searched and the things to be seized, to wit:jgc:chanrobles.com.ph
Court that ‘a search warrant shall not issue but upon probable cause in connection with one
specific offense.’ Not satisfied with this qualification, the Court added thereto a paragraph, ". . . Both the Jones Law (sec. 3) and General Orders No. 58 (sec. 97) specifically require that a
directing that ‘no search warrant shall issue for more than one specific offense.’" search warrant should particularly describe the place to be searched and the things to be seized.
The evident purpose and intent of this requirement is to limit the things to be seized to those,
3. The search warrant does not particularly describe the things to be seized. and only those, particularly described in the search warrant — to leave the officers of the law
with no discretion regarding what articles they shall seize, to the end that ‘unreasonable
The documents, papers and effects sought to be seized are described in Search Warrant No. 2- searches and seizures’ may not be made, — that abuses may not be committed. That this is the
16

correct interpretation of this constitutional provision is borne out by American appropriate to such body. Its property cannot be taken without compensation. It can only be
authorities."cralaw virtua1aw library proceeded against by due process of law, and is protected, under the 14th Amendment, against
unlawful discrimination . . ." (Hale v. Henkel, 201 U.S. 43, 50 L. ed. 652.)
The purpose as thus explained could, surely and effectively, be defeated under the search
warrant issued in this case. "In Linn v. United States, 163 C.C.A. 470, 251 Fed. 476, 480, it was thought that a different rule
applied to a corporation, the ground that it was not privileged from producing its books and
A search warrant may be said to particularly describe the things to be seized when the papers. But the rights of a corporation against unlawful search and seizure are to be protected
description therein is as specific as the circumstances will ordinarily allow (People v. Rubio; 57 even if the same result might have been achieved in a lawful way." (Silverthorne Lumber
Phil. 384); or when the description expresses a conclusion of fact — not of law — by which the Company, Et. Al. v. United States of America, 251 U.S. 385, 64 L. ed. 319.)
warrant officer may be guided in making the search and seizure (idem., dissent of Abad
Santos, J.,); or when the things described are limited to those which bear direct relation to the In Stonehill, Et. Al. v. Diokno, Et Al., supra, this Court impliedly recognized the right of a
offense for which the warrant is being issued (Sec. 2, Rule 126, Revised Rules of Court). The corporation to object against unreasonable searches and seizures, thus:jgc:chanrobles.com.ph
herein search warrant does not conform to any of the foregoing tests. If the articles desired to
be seized have any direct relation to an offense committed, the applicant must necessarily have "As regards the first group, we hold that petitioners herein have no cause of action to assail the
some evidence, other than those articles, to prove the said offense; and the articles subject of legality of the contested warrants and of the seizures made in pursuance thereof, for the simple
search and seizure should come in handy merely to strengthen such evidence. In this event, the reason that said corporations have their respective personalities, separate and distinct from the
description contained in the herein disputed warrant should have mentioned, at least, the dates, personality of herein petitioners, regardless of the amount of shares of stock or the interest of
amounts, persons, and other pertinent data regarding the receipts of payments, certificates of each of them in said corporations, whatever, the offices they hold therein may be. Indeed, it is
stocks and securities, contracts, promissory notes, deeds of sale, messages and communications, well settled that the legality of a seizure can be contested only by the party whose rights have
checks, bank deposits and withdrawals, records of foreign remittances, among others, been impaired thereby, and that the objection to an unlawful search and seizure is purely
enumerated in the warrant. personal and cannot be availed of by third parties. Consequently, petitioners herein may not
validly object to the use in evidence against them of the documents, papers and things seized
Respondents contend that certiorari does not lie because petitioners failed to file a motion for from the offices and premises of the corporations adverted to above, since the right to object to
reconsideration of respondent Judge’s order of July 29, 1970. The contention is without merit. In the admission of said papers in evidence belongs exclusively to the corporations, to whom the
the first place, when the questions raised before this Court are the same as those which were seized effects belong, and may not be invoked by the corporate officers in proceedings against
squarely raised in and passed upon by the court below, the filing of a motion for reconsideration them in their individual capacity . . ."cralaw virtua1aw library
in said court before certiorari can be instituted in this Court is no longer a prerequisite. (Pajo,
etc., Et. Al. v. Ago, Et Al., 108 Phil., 905). In the second place, the rule requiring the filing of a In the Stonehill case only the officers of the various corporations in whose offices documents,
motion for reconsideration before an application for a writ of certiorari can be entertained was papers and effects were searched and seized were the petitioners. In the case at bar, the
never intended to be applied without considering the circumstances. (Matutina v. Buslon, Et Al., corporation to whom the seized documents belong, and whose rights have thereby been
109 Phil., 140.) In the case at bar time is of the essence in view of the tax assessments sought to impaired, is itself a petitioner. On that score, petitioner corporation here stands on a different
be enforced by respondent officers of the Bureau of Internal Revenue against petitioner footing from the corporations in Stonehill.
corporation, On account of which immediate and more direct action becomes necessary.
(Matute v. Court of Appeals, Et Al., 26 SCRA 768.) Lastly, the rule does not apply where, as in this The tax assessments referred to earlier in this opinion were, if not entirely — as claimed by
case, the deprivation of petitioners’ fundamental right to due process taints the proceeding petitioners — at least partly — as in effect admitted by respondents — based on the documents
against them in the court below not only with irregularity but also with nullity. (Matute v. Court seized by virtue of Search Warrant No. 2-M-70. Furthermore, the fact that the assessments were
of Appeals, Et Al., supra.) made some one and one-half months after the search and seizure on February 25, 1970, is a
strong indication that the documents thus seized served as basis for the assessments. Those
It is next contended by respondents that a corporation is not entitled to protection against assessments should therefore not be enforced.
unreasonable search and seizures. Again, we find no merit in the contention.
PREMISES CONSIDERED, the petition is granted. Accordingly, Search Warrant No. 2-M-70 issued
"Although, for the reasons above stated, we are of the opinion that an officer of a corporation by respondent Judge is declared null and void; respondents are permanently enjoined from
which is charged with a violation of a statute of the state of its creation, or of an act of Congress enforcing the said search warrant; the documents, papers and effects seized thereunder are
passed in the exercise of its constitutional powers, cannot refuse to produce the books and ordered to be returned to petitioners; and respondent officials the Bureau of Internal Revenue
papers of such corporation, we do not wish to be understood as holding that a corporation is not and their representatives are permanently enjoined from enforcing the assessments mentioned
entitled to immunity, under the 4th Amendment, against unreasonable searches and seizures. A in Annex "G" of the present petition, as well as other assessments based on the documents,
corporation is, after all, but an association of individuals under an assumed name and with a papers and effects seized under the search warrant herein nullified, and from using the same
distinct legal entity. In organizing itself as a collective body it waives no constitutional immunities against petitioners in any criminal or other proceeding. No pronouncement as to costs.
17

ROAN VS GONZALES the guaranty against unreasonable searches and seizures. Although the condition did not appear
Once again we are asked to annul a search warrant on the ground that it violates the in the corresponding provision of the federa Constitution of the United States which served as
Constitution. As we can do no less if we are to be true to the mandate of the fundamental law, our model it was then already embodied in the Code of Criminal Procedure. Nevertheless,
we do annul. Delegate Jose P. Laurel, Chairman of the Committee on the Bill of Rights of that body, readily
One of the most precious rights of the citizen in a free society is the right to be left alone in the accepted the proposal and it was thereafter, following a brief debate, approved by the
privacy of his own house. That right has ancient roots, dating back through the mists of history to Convention. 8
the mighty English kings in their fortresses of power. Even then, the lowly subject had his own Implementing this requirement, the Rules of Court provided in what was then Rule 126:
castle where he was monarch of all he surveyed. This was his humble cottage from which he SEC. 4. Examination of the applicant. — The municipal or city judge must,
could bar his sovereign lord and all the forces of the Crown. before issuing the warrant, personally examine on oath or affirmation the
That right has endured through the ages albeit only in a few libertarian regimes. Their number, complainant and any witnesses he may produce and take their depositions in
regrettably, continues to dwindle against the onslaughts of authoritarianism. We are among the writing, and attach them to the record, in addition to any affidavits
fortunate few, able again to enjoy this right after the ordeal of the past despotism. We must presented to him.
cherish and protect it all the more now because it is like a prodigal son returning. The petitioner claims that no depositions were taken by the respondent judge in accordance
That right is guaranteed in the following provisions of Article IV of the 1973 Constitution: with the above rule, but this is not entirely true. As a matter of fact, depositions were taken of
SEC. 3. The right of the people to be secure in their persons, houses, papers the complainant's two witnesses in addition to the affidavit executed by them. 9 It is correct to
and effects against unreasonable searches and seizures of whatever nature say, however, that the complainant himself was not subjected to a similar interrogation.
and for any purpose shall not be violated, and no search warrant or warrant Commenting on this matter, the respondent judge declared:
of arrest shall issue except upon probable cause to be determined by the The truth is that when PC Capt. Mauro P. Quinosa personally filed his
judge, or such other responsible officer as may be authorized by law, after application for a search warrant on May 10, 1984, he appeared before me in
examination under oath or affirmation of the complainant and the witnesses the company of his two (2) witnesses, Esmael Morada and Jesus Tohilida,
he may produce, and particularly describing the place to be searched, and both of whom likewise presented to me their respective affidavits taken by
the persons or things to be seized. Pat. Josue V. Lining, a police investigator assigned to the PC-INP command at
SEC. 4. (1) The privacy of communication and cor- respondence shag be Camp Col. Maximo Abad. As the application was not yet subscribed and
inviolable except upon lawful order of the court, or when public safety and sworn to, I proceeded to examine Captain Quillosa on the contents thereof
order require otherwise. to ascertain, among others, if he knew and understood the same.
(2) Any evidence obtained in violation of this or the preceding section shall Afterwards, he subscribed and swore to the same before me. 10
be inadmissible for any purpose in any proceeding. By his own account, an he did was question Captain Quillosa on the contents of his affidavit only
Invoking these provisions, the petitioner claims he was the victim of an illegal search and seizure "to ascertain, among others, if he knew and understood the same," and only because "the
conducted by the military authorities. The articles seized from him are sought to be used as application was not yet subscribed and swom to." The suggestion is that he would not have
evidence in his prosecution for illegal possession of firearms. He asks that their admission be asked any questions at all if the affidavit had already been completed when it was submitted to
temporarily restrained (which we have) 1 and thereafter permanently enjoined. him. In any case, he did not ask his own searching questions. He limited himself to the contents
The challenged search warrant was issued by the respondent judge on May 10, 1984. 2 The of the affidavit. He did not take the applicant's deposition in writing and attach them to the
petitioner's house was searched two days later but none of the articles listed in the warrant was record, together with the affidavit presented to him.
discovered. 3 However, the officers conducting the search found in the premises one Colt As this Court held in Mata v. Bayona: 11
Magnum revolver and eighteen live bullets which they confiscated. They are now the bases of Mere affidavits of the complainant and his witnesses are thus not sufficient. The
the charge against the petitioner. 4 examining Judge has to take depositions in writing of the complainant and the
To be valid, a search warrant must be supported by probable cause to be determined by the witnesses he niay produce and attach them to the record. Such written deposition is
necessary in order that the Judge may be able to properly determine the existence or
judge or some other authorized officer after examining the complainant and the witnesses he
non-existence of the probable cause, to hold liable for perjury the person giving it if it
may produce. No less important, there must be a specific description of the place to be searched wifl be found later that his declarations are false.
and the things to be seized, to prevent arbitrary and indiscriminate use of the warrant. 5 We, therefore, hold that the search warrant is tainted with illegality by the failure of
Probable cause was described by Justice Escolin in Burgos v. Chief of Staff 6 as referring to "such the Judge to conform with the essential requisites of taking the depositions in writing
facts and circumstances which would lead a reasonably discreet and prudent man to believe that and attaching them to the record, rendering the search warrant invalid.
an offense has been committed and that the objects sought in connection with the offense are in The respondent judge also declared that he "saw no need to have applicant Quillosa's deposition
the place sought to be searched." As held in a long line of decisions, the probable cause must taken considering that he was applying for a search warrant on the basis of the information
refer to only one specific offense. 7 provided by the aforenamed witnesses whose depositions as aforementioned had already been
The inclusion of the requirement for the "examination under oath or affirmation of the taken by the undersigned." 12
complainant and the witnesses he may produce" was a refinement proposed by Delegate In other words, the applicant was asking for the issuance of the search warrant on the basis of
Vicente J. Francisco in the1934 Constitutional Convention. His purpose was the strengthening of mere hearsay and not of information personally known to him, as required by settled
18

jurisprudence." 13 The rationale of the requirement, of course, is to provide a ground for a Prohibited articles may be seized but only as long as the search is valid. In this case, it was not
prosecution for perjury in case the applicant's declarations are found to be false. His application, because: 1) there was no valid search warrant; and 2) absent such a warrant, the right thereto
standing alone, was insufficient to justify the issuance of the warrant sought. It was therefore was not validly waived by the petitioner. In short, the military officers who entered the
necessary for the witnesses themselves, by their own personal information, to establish the petitioner's premises had no right to be there and therefore had no right either to seize the
apphcant's claims. 14 pistol and bullets.
Even assuming then that it would have sufficed to take the depositions only of the witnesses and It does not follow that because an offense is malum prohibitum, the subject thereof is
not of the applicant himself, there is still the question of the sufficiency of their depositions. necessarily illegal per se.Motive is immaterial in mala prohibita, but the subjects of this kind of
It is axiomatic that the examination must be probing and exhaustive, not merely routinary offense may not be summarily seized simply because they are prohibited. A search warrant is
or pro-forma, if the claimed probable cause is to be established. The examining magistrate must still necessary. If the rule were otherwise, then the military authorities could have just entered
not simply rehash the contents of the affidavit but must make his own inquiry on the intent and the premises and looked for the guns reportedly kept by the petitioner without bothering to first
justification of the application. 15 secure a search warrant. The fact that they did bother to do so indicates that they themselves
A study of the depositions taken from witnesses Esmael Morada and Jesus Tohilida, who both recognized the necessity of such a warrant for the seizure of the weapons the petitioner was
claimed to be "intelligence informers," shows that they were in the main a mere restatement of suspected of possessing.
their allegations in their affidavits, except that they were made in the form of answers to the It is true that there are certain instances when a search may be validly made without warrant
questions put to them by the respondent judge. Significantly, the meaningful remark made by and articles may be taken validly as a result of that search. For example, a warrantless search
Tohilida that they were suspicious of the petitioner because he was a follower of the opposition may be made incidental to a lawful arrest, 22 as when the person being arrested is frished for
candidate in the forthcoming election (a "Lecarista") 16 did not excite the respondent judge's weapons he may otherwise be able to use against the arresting officer. Motor cars may be
own suspicions. This should have put him on guard as to the motivations of the witnesses and inspected at borders to prevent smuggling of aliens and contraband 23 and even in the interior
alerted him to possible misrepresentations from them. upon a showing of probable cause. 24 Vessels and aircraft are also traditionally removed from the
The respondent judge almost unquestioningly received the witnesses' statement that they saw operation of the rule because of their mobility and their relative ease in fleeing the state's
eight men deliver arms to the petitioner in his house on May 2, 1984. 17 This was supposedly jurisdiction. 25 The individual may knowingly agree to be searched or waive objections to an
done overtly, and Tohilida said he saw everything through an open window of the house while illegal search. 26 And it has also been held that prohibited articles may be taken without warrant
he was near the gate. 18 He could even positively say that six of the weapons were.45 caliber if they are open to eye and hand and the peace officer comes upon them inadvertently. 27
pistols and two were.38 caliber revolvers. 19 Clearly, though, the instant case does not come under any of the accepted exceptions. The
One may well wonder why it did not occur to the respondent judge to ask how the witness could respondents cannot even claim that they stumbled upon the pistol and bullets for the fact is that
be so certain even as to the caliber of the guns, or how far he was from the window, or whether these things were deliberately sought and were not in plain view when they were taken. Hence,
it was on the first floor or a second floor, or why his presence was not noticed at all, or if the acts the rule having been violated and no exception being applicable, the conclusion is that the
related were really done openly, in the full view of the witnesses, considering that these acts petitioner's pistol and bullets were confiscated illegally and therefore are protected by the
were against the law. These would have been judicious questions but they were injudiciously exclusionary principle.
omitted. Instead, the declarations of the witnesses were readily accepted and the search Stonehill v. Diokno established this rule which was later expressly affirmed in the 1973
warrant sought was issued forthwith. Constitution. While conceding that there may be occasions when the criminal might be allowed
The above-discussed defects have rendered the search warrant invalid. Nonetheless, the to go free because "the constable has blundered," Chief Justice Concepcion observed that the
Solicitor General argues that whatever defect there was, was waived when the petitioner exclusionary rule was nonetheless "the only practical means of enforcing the constitutional
voluntarily submitted to the search and manifested his conformity in writing. 20 injunction" against abuse. The decision cited Judge Learned Hand's justification that "only in case
We do not agree. What we see here is pressure exerted by the military authorities, who the prosecution which itself controls the seizing officials, know that it cannot profit by their
practically coerced the petitioner to sign the supposed waiver as a guaranty against a possible wrong, will the wrong be repressed. "
challenge later to the validity of the search they were conducting. Confronted with the armed The pistol and bullets cannot, of course, be used as evidence against the petitioner in the
presence of the military and the presumptive authority of a judicial writ, the petitioner had no criminal action against him for illegal possession of firearms. Pending resolution of that case,
choice but to submit. This was not, as we held in a previous case, 21 the manifestation merely of however, the said articles must remain incustodia legis.
our traditional Filipino hospitality and respect for authority. Given the repressive atmosphere of Finally, it is true that the petitioner should have, before coming to this Court, filed a motion for
the Marcos regime, there was here, as we see it, an intimidation that the petitioner could not the quashal of the search warrant by the respondent judge in accordance with the normal
resist. procedure. But as we said and did in Burgos, "this procedural flaw notwithstanding, we take
The respondents also argue that the Colt Magnum pistol and the eighteen have bullets seized cognizance of this petition in view of the seriousness and urgency of the constitutional issues
from the petitioner were illegal per se and therefore could have been taken by the military raised. 28
authorities even without a warrant. Possession of the said articles, it is urged, was violative of WHEREFORE, Search Warrant No. 1-84 issued by the respondent judge on May 10, 1984, is
P.D. 1866 and considered malum prohibitum. Hence, the Wegal articles could be taken even hereby declared null and void and accordingly set aside. Our restraining order of August 6,1985,
without a warrant. is made permanent. No costs.
SO ORDERED.
19

ALVAREZ vs CFI oath of all the documents seized by him, to return the search warrant together with the affidavit
The petitioner asks that the warrant of June 3, 1936, issued by the Court of First Instance of it presented in support thereof, or to present the report of the proceedings taken by him; and
Tayabas, ordering the search of his house and the seizure, at any time of the day or night, of prayed that said agent be directed to filed the documents in question immediately. On the 25th
certain accounting books, documents and papers belonging to him in his residence situated in of said month the court issued an order requiring agent Emilio L. Siongco forthwith to file the
Infanta, Province of Tayabas, as well as the order of a later date, authorizing the agents of the search warrant and the affidavit in the court, together with the proceedings taken by him, and to
Anti-Usury Board to retain the articles seized, be declared illegal and set aside, and prays that all present an inventory duly verified by oath of all the articles seized. On July 2d of said year, the
the articles in question be returned to him. attorney for the petitioner filed another petition alleging that the search warrant issue was
On the date above-mentioned, the chief of the secret service of the Anti-Usury Board, of the illegal and that it had nit yet been returned to date together with the proceedings taken in
Department of Justice, presented to Judge Eduardo Gutierrez David then presiding over the connection therewith, and praying that said warrant be cancelled, that an order be issued
Court of First Instance of Tayabas, an affidavit alleging that according to reliable information, the directing the return of all the articles seized to the petitioner, that the agent who seized them be
petitioner kept in his house in Infanta, Tayabas, books, documents, receipts, lists, chits and other declared guilty of contempt of court, and that charges be filed against him for abuse of authority.
papers used by him in connection with his activities as a money-lender charging usurious rates of On September 10, 1936, the court issued an order holding: that the search warrant was obtained
interest in violation of the law. In his oath at the and of the affidavit, the chief of the secret and issued in accordance with the law, that it had been duly complied with and, consequently,
service stated that his answers to the questions were correct to the best of his knowledge and should not be cancelled, and that agent Emilio L. Siongco did not commit any contempt of court
belief. He did not swear to the truth of his statements upon his own knowledge of the facts but and must, therefore, be exonerated, and ordering the chief of the Anti-Usury Board in Manila to
upon the information received by him from a reliable person. Upon the affidavit in question the show case, if any, within the unextendible period of two (2) days from the date of notice of said
Judge, on said date, issued the warrant which is the subject matter of the petition, ordering the order, why all the articles seized appearing in the inventory, Exhibit 1, should not be returned to
search of the petitioner's house at nay time of the day or night, the seizure of the books and the petitioner. The assistant chief of the Anti-Usury Board of the Department of Justice filed a
documents above-mentioned and the immediate delivery thereof to him to be disposed of in motion praying, for the reasons stated therein, that the articles seized be ordered retained for
accordance with the law. With said warrant, several agents of the Anti-Usury Board entered the the purpose of conducting an investigation of the violation of the Anti-Usury Law committed by
petitioner's store and residence at seven o'clock on the night of June 4, 1936, and seized and the petitioner. In view of the opposition of the attorney for the petitioner, the court, on
took possession of the following articles: internal revenue licenses for the years 1933 to 1936, September 25th, issued an order requiring the Anti-Usury Board to specify the time needed by it
one ledger, two journals, two cashbooks, nine order books, four notebooks, four checks stubs, to examine the documents and papers seized and which of them should be retained, granting it a
two memorandums, three bankbooks, two contracts, four stubs, forty-eight stubs of purchases period of five (5) days for said purpose. On the 30th of said month the assistant chief of the Anti-
of copra, two inventories, two bundles of bills of lading, one bundle of credit receipts, one Usury Board filed a motion praying that he be granted ten (10) days to comply with the order of
bundle of stubs of purchases of copra, two packages of correspondence, one receipt book September 25th and that the clerk of court be ordered to return to him all the documents and
belonging to Luis Fernandez, fourteen bundles of invoices and other papers many documents papers together with the inventory thereof. The court, in an order of October 2d of said year,
and loan contracts with security and promissory notes, 504 chits, promissory notes and stubs of granted him the additional period of ten(10) days and ordered the clerk of court to send him a
used checks of the Hongkong & Shanghai Banking Corporation. The search for and a seizure of copy of the inventory. On October 10th, said official again filed another motion alleging that he
said articles were made with the opposition of the petitioner who stated his protest below the needed sixty (60) days to examine the documents and papers seized, which are designated on
inventories on the ground that the agents seized even the originals of the documents. As the pages 1 to 4 of the inventory by Nos. 5, 1016, 23, 25, 26, 27, 30, 31, 34, 36, 37, 38, 39, 40, 41, 42,
articles had not been brought immediately to the judge who issued the search warrant, the 43 and 45, and praying that he be granted said period of sixty (60) days. In an order of October
petitioner, through his attorney, filed a motion on June 8, 1936, praying that the agent Emilio L. 16th, the court granted him the period of sixty (60) days to investigate said nineteen (19)
Siongco, or any other agent, be ordered immediately to deposit all the seized articles in the documents. The petitioner alleges, and it is not denied by the respondents, that these nineteen
office of the clerk of court and that said agent be declared guilty of contempt for having (19)documents continue in the possession of the court, the rest having been returned to said
disobeyed the order of the court. On said date the court issued an order directing Emilio L. petitioner.
Siongco to deposit all the articles seized within twenty-four hours from the receipt of notice I. A search warrant is an order in writing, issued in the name of the People of the Philippine
thereof and giving him a period of five (5) days within which to show cause why he should not be Islands, signed by a judge or a justice of the peace, and directed to a peace officer, commanding
punished for contempt of court. On June 10th, Attorney Arsenio Rodriguez, representing the him to search for personal property and bring it before the court (section 95, General Orders. No.
58, as amended by section 6 of Act No. 2886). Of all the rights of a citizen, few are of greater
Anti-Usury Board, filed a motion praying that the order of the 8th of said month be set aside and
importance or more essential to his peace and happiness than the right of personal security, and
that the Anti-Usury Board be authorized to retain the articles seized for a period of thirty (30) that involves the exemption of his private affairs, books, and papers from the inspection and
days for the necessary investigation. The attorney for the petitioner, on June 20th, filed another scrutiny of others (In re Pacific Railways Commission, 32 Fed., 241; Interstate Commerce
motion alleging that, notwithstanding the order of the 8th of said month, the officials of the Commission vs Brimson, 38 Law. ed., 1047; Broyd vs. U. S., 29 Law. ed., 746; Caroll vs. U. S., 69
Anti-Usury Board had failed to deposit the articles seized by them and praying that a search Law. ed., 543, 549). While the power to search and seize is necessary to the public welfare, still it
warrant be issued, that the sheriff be ordered to take all the articles into his custody and deposit must be exercised and the law enforced without transgressing the constitutional rights or citizen,
of the Anti-Usury Board be punished for contempt of court. Said attorney, on June 24th, filed an for the enforcement of no statue is of sufficient importance to justify indifference to the basis
ex parte petition alleging that while agent Emilio L. Siongco had deposited some documents and principles of government (People vs. Elias, 147 N. E., 472).
II. As the protection of the citizen and the maintenance of his constitutional right is one of the
papers in the office of the clerk of court, he had so far failed to file an inventory duly verified by
highest duties and privileges of the court, these constitutional guaranties should be given a liberal
20

construction or a strict construction in favor of the individual, to prevent stealthy encroachment In view of the foregoing and under the above-cited authorities, it appears that the affidavit, which
upon, or gradual depreciation on, the rights secured by them(State vs. Custer County, 198 Pac., served as the exclusive basis of the search warrant, is insufficient and fatally defective by reason
362; State vs. McDaniel, 231 Pac., 965; 237 Pac., 373). Since the proceeding is a drastic one, it is of the manner in which the oath was made, and therefore, it is hereby held that the search
the general rule that statutes authorizing searches and seizure or search warrants must be strictly warrant in question and the subsequent seizure of the books, documents and other papers are
construed (Rose vs. St. Clair, 28 Fed., [2d], 189; Leonardvs. U. S., 6 Fed. [2d], 353; Perry vs. U. S. 14 illegal and do not in any way warrant the deprivation to which the petitioner was subjected.
Fed. [2d],88; Cofer vs. State, 118 So., 613). IV. Another ground alleged by the petitioner in asking that the search warrant be declared illegal
III. The petitioner claims that the search warrant issued by the court is illegal because it has been and cancelled is that it was not supported by other affidavits aside from that made by the
based upon the affidavit of agent Mariano G. Almeda in whose oath he declared that he had no applicant. In other words, it is contended that the search warrant cannot be issued unless it be
personal knowledge of the facts which were to serve as a basis for the issuance of the warrant but supported by affidavits made by the applicant and the witnesses to be presented necessity by
that he had knowledge thereof through mere information secured from a person whom he him. Section 1, paragraph 3, of Article III of the Constitution provides that no warrants shall issue
considered reliable. To the question "What are your reason for applying for this search warrant", but upon probable cause, to be determined by the judge after examination under oath or
appearing in the affidavit, the agent answered: "It has been reported to me by a person whom I affirmation of the complainant and the witnesses he may produce. Section 98 of General Orders,
consider to be reliable that there are being kept in said premises, books, documents, receipts, No. 58 provides that the judge or justice must, before issuing the warrant, examine under oath
lists, chits, and other papers used by him in connection with his activities as a money-lender, the complainant and any witnesses he may produce and take their depositions in writing. It is the
charging a usurious rate of interest, in violation of the law" and in attesting the truth of his practice in this jurisdiction to attach the affidavit of at least the applicant or complainant to the
statements contained in the affidavit, the said agent states that he found them to be correct and application. It is admitted that the judge who issued the search warrant in this case, relied
true to the best of his knowledge and belief. exclusively upon the affidavit made by agent Mariano G. Almeda and that he did not require nor
Section 1, paragraph 3, of Article III of the Constitution, relative to the bill of rights, provides that take the deposition of any other witness. Neither the Constitution nor General Orders. No. 58
"The right of the people to be secure in their persons, houses, papers, and effects against provides that it is of imperative necessity to take the deposition of the witnesses to be presented
unreasonable searches and seizures shall not be violated, and no warrants shall issue but upon by the applicant or complainant in addition to the affidavit of the latter. The purpose of both in
probable cause, to be determined by the judge after examination under oath or affirmation of the requiring the presentation of depositions is nothing more than to satisfy the committing
complainant and the witnesses he may produce, and particularly describing the place top be magistrate of the existence of probable cause. Therefore, if the affidavit of the applicant or
searched, and the persons or things to be seized." Section 97 of General Orders, No. 58 provides complainant is sufficient, the judge may dispense with that of other witnesses. Inasmuch as the
that "A search warrant shall not issue except for probable cause and upon application supported affidavit of the agent in this case was insufficient because his knowledge of the facts was not
by oath particularly describing the place to be searched and the person or thing to be seized." It personal but merely hearsay, it is the duty of the judge to require the affidavit of one or more
will be noted that both provisions require that there be not only probable cause before the witnesses for the purpose of determining the existence of probable cause to warrant the issuance
issuance of a search warrant but that the search warrant must be based upon an application of the search warrant. When the affidavit of the applicant of the complaint contains sufficient
supported by oath of the applicant ands the witnesses he may produce. In its broadest sense, an facts within his personal and direct knowledge, it is sufficient if the judge is satisfied that there
oath includes any form of attestation by which a party signifies that he is bound in conscience to exist probable cause; when the applicant's knowledge of the facts is mere hearsay, the affidavit of
perform an act faithfully and truthfully; and it is sometimes defined asan outward pledge given by one or more witnesses having a personal knowledge of the fact is necessary. We conclude,
the person taking it that his attestation or promise is made under an immediate sense of his therefore, that the warrant issued is likewise illegal because it was based only on the affidavit of
responsibility to God (Bouvier's Law Dictionary; State vs. Jackson, 137 N. W., 1034; In re Sage, 24 the agent who had no personal knowledge of the facts.
Oh. Cir. Ct. [N. S.], 7; Pumphery vs.State, 122 N. W., 19; Priest vs. State, 6 N. W., 468; V. The petitioner alleged as another ground for the declaration of the illegality of the search
State vs. Jones, 154 Pac., 378; Atwood vs. State, 111 So., 865). The oath required must refer to the warrant and the cancellation thereof, the fact that it authorized its execution at night. Section 101
truth of the facts within the personal knowledge of the petitioner or his witnesses, because the of General Orders, No. 58 authorizes that the search be made at night when it is positively
purpose thereof is to convince the committing magistrate, not the individual making the affidavit asserted in the affidavits that the property is on the person or in the place ordered to be
and seeking the issuance of the warrant, of the existence of probable cause (U. S. vs. Tureaud, 20 searched. As we have declared the affidavits insufficient and the warrant issued exclusively upon
Fed., 621; U. S. vs. Michalski, 265 Fed., 8349; U. S. vs. Pitotto, 267 Fed., 603; U. S.vs. Lai Chew, 298 it illegal, our conclusion is that the contention is equally well founded and that the search could
Fed., 652). The true test of sufficiency of an affidavit to warrant issuance of a search warrant is not legally be made at night.
whether it has been drawn in such a manner that perjury could be charged thereon and affiant be VI. One of the grounds alleged by the petitioner in support of his contention that the warrant was
held liable for damages caused (State vs. Roosevelt Country 20th Jud. Dis. Ct., 244 Pac., 280; issued illegally is the lack of an adequate description of the books and documents to be seized.
State vs.Quartier, 236 Pac., 746). Section 1, paragraphs 3, of Article III of the Constitution, and section 97 of General Orders, No. 58
It will likewise be noted that section 1, paragraph 3, of Article III of the Constitution prohibits provide that the affidavit to be presented, which shall serve as the basis for determining whether
unreasonable searches and seizure. Unreasonable searches and seizures are a menace against probable cause exist and whether the warrant should be issued, must contain a particular
which the constitutional guarantee afford full protection. The term "unreasonable search and description of the place to be searched and the person or thing to be seized. These provisions are
seizure" is not defined in the Constitution or in General Orders No. 58, and it is said to have no mandatory and must be strictly complied with (Munch vs. U. S., 24 Fed. [2d], 518; U. S. vs. Boyd, 1
fixed, absolute or unchangeable meaning, although the term has been defined in general Fed. [2d], 1019; U. S. vs. Carlson, 292 Fed., 463; U. S. vs. Borkowski, 268 Fed., 408; In re Tri-State
language. All illegal searches and seizure are unreasonable while lawful ones are reasonable. Coal & Coke Co., 253 Fed., 605; People vs. Mayen, 188 Cal., 237; People vs. Kahn, 256 Ill. App.,
What constitutes a reasonable or unreasonable search or seizure in any particular case is purely a 4125); but where, by the nature of the goods to be seized, their description must be rather
judicial question, determinable from a consideration of the circumstances involved, including the generally, it is not required that a technical description be given, as this would mean that no
purpose of the search, the presence or absence or probable cause, the manner in which the warrant could issue (People vs. Rubio, 57 Phil., 284; People vs. Kahn, supra). The only description
search and seizure was made, the place or thing searched, and the character of the articles of the articles given in the affidavit presented to the judge was as follows: "that there are being
procured (Go-Bart Importing Co. vs. U. S. 75 Law. ed., 374; Peru vs. U. S., 4 Fed., [2d], 881;U. kept in said premises books, documents, receipts, lists, chits and other papers used by him in
S. vs. Vatune, 292 Fed., 497; Angelo vs. U. S. 70 Law, ed., 145; Lambert vs. U. S. 282 Fed., 413; U. connection with his activities as money-lender, charging a usurious rate of interest, in violation of
S. vs.Bateman, 278 Fed., 231; Mason vs. Rollins, 16 Fed. Cas. [No. 9252], 2 Biss., 99). the law." Taking into consideration the nature of the article so described, it is clear that no other
21

more adequate and detailed description could have been given, particularly because it is difficult 4. That as the warrant had been issued unreasonably, and as it does not appear positively in the
to give a particular description of the contents thereof. The description so made substantially affidavit that the articles were in the possession of the petitioner and in the place indicated,
complies with the legal provisions because the officer of the law who executed the warrant was neither could the search and seizure be made at night;
thereby placed in a position enabling him to identify the articles, which he did. 5. That although it is not mandatory to present affidavits of witnesses to corroborate the
VII. The last ground alleged by the petitioner, in support of his claim that the search warrant was applicant or a complainant in cases where the latter has personal knowledge of the facts, when
obtained illegally, is that the articles were seized in order that the Anti-Usury Board might provide the applicant's or complainant's knowledge of the facts is merely hearsay, it is the duty of the
itself with evidence to be used by it in the criminal case or cases which might be filed against him judge to require affidavits of other witnesses so that he may determine whether probable cause
for violation of the Anti-usury Law. At the hearing of the incidents of the case raised before the exists;
court it clearly appeared that the books and documents had really been seized to enable the Anti- 6. That a detailed description of the person and place to be searched and the articles to be seized
Usury Board to conduct an investigation and later use all or some of the articles in question as is necessary, but whereby, by the nature of the articles to be seized, their description must be
evidence against the petitioner in the criminal cases that may be filed against him. The seizure of rather general, but is not required that a technical description be given, as this would mean that
books and documents by means of a search warrant, for the purpose of using them as evidence in no warrant could issue;
a criminal case against the person in whose possession they were found, is unconstitutional 7. That the petitioner did not waive his constitutional rights because the offer of compromise or
because it makes the warrant unreasonable, and it is equivalent to a violation of the settlement attributed to him, does not mean, if so made, that he voluntarily tolerated the search
constitutional provision prohibiting the compulsion of an accused to testify against himself (Uy and seizure; and
Kheytin vs.Villareal, 42 Phil,, 886; Brady vs. U. S., 266 U. S., 620; Temperani vs. U. S., 299 Fed., 365; 8. That an appeal from the orders questioned by the petitioner, if taken by him, would not be an
U. S. vs.Madden, 297 Fed., 679; Boyd vs. U. S.,116 U. S., 116; Caroll vs. U. S., 267 U. S., 132). effective, speedy or adequate remedy in the ordinary course of law, and, consequently, the
Therefore, it appearing that at least nineteen of the documents in question were seized for the petition for mandamusfiled by him, lies.
purpose of using them as evidence against the petitioner in the criminal proceeding or For the foregoing considerations, the search warrant and the seizure of June 3, 1936, and the
proceedings for violation against him, we hold that the search warrant issued is illegal and that orders of the respondent court authorizing the relation of the books and documents, are
the documents should be returned to him. declared illegal and are set aside, and it is ordered that the judge presiding over the Court of First
The Anti-Usury Board insinuates in its answer that the petitioner cannot now question the Instance of Tayabas direct the immediate return to the petitioner of the nineteen (19)
validity of the search warrant or the proceedings had subsequent to the issuance thereof, documents designated on pages 1 to 4 of the inventory by Nos. 5, 10, 16, 23, 25,26, 27, 30, 31,
because he has waived his constitutional rights in proposing a compromise whereby he agreed 34, 36, 37, 38, 39, 40, 41, 42, 43 and 45, without special pronouncement as to costs. So ordered.
to pay a fine of P200 for the purpose of evading the criminal proceeding or proceedings. We are
of the opinion that there was no such waiver, first, because the petitioner has emphatically
denied the offer of compromise and, second, because if there was a compromise it reffered but
to the institution of criminal proceedings fro violation of the Anti-Usury Law. The waiver would
have been a good defense for the respondents had the petitioner voluntarily consented to the
search and seizure of the articles in question, but such was not the case because the petitioner
protested from the beginning and stated his protest in writing in the insufficient inventory
furnished him by the agents.
Said board alleges as another defense that the remedy sought by the petitioner does not lie
because he can appeal from the orders which prejudiced him and are the subject matter of his
petition. Section 222 of the Code of Civil Procedure in fact provides that mandamus will not issue
when there is another plain, speedy and adequate remedy in the ordinary course of law. We are
of the opinion, however, that an appeal from said orders would have to lapse before he recovers
possession of the documents and before the rights, of which he has been unlawfully deprived,
are restored to him (Fajardo vs. Llorente, 6 Phil., 426; Manotoc vs. McMicking and Trinidad, 10
Phil., 119; Cruz Herrera de Lukban vs. McMicking, 14 Phil., 641; Lamb vs. Phipps, 22 Phil., 456).
Summarizing the foregoing conclusions, we hold:
1. That the provisions of the Constitution and General Orders, No. 58, relative to search and
seizure, should be given a liberal construction in favor of the individual in order to maintain the
constitutional guaranties whole and in their full force;
2. That since the provisions in question are drastic in their form and fundamentally restrict the
enjoyment of the ownership, possession and use of the personal property of the individual, they
should be strictly construed;
3. That the search and seizure made are illegal for the following reasons: (a) Because the warrant
was based solely upon the affidavit of the petitioner who had no personal knowledge of the facts
of probable cause, and (b) because the warrant was issued for the sole purpose of seizing
evidence which would later be used in the criminal proceedings that might be instituted against
the petitioner, for violation of the Anti-Usury Law;
22

MICROSOFT 438 SCRA 224 presented in this petition involve questions of law. Petitioners point out that the facts are not in
The Case dispute.
This petition for review on certiorari[1] seeks to reverse the Court of Appeals A petition for review under Rule 45 of the Rules of Court should cover questions of
Decision[2] dated 23 December 1998 and its Resolution dated 29 November 1999 in CA-G.R. SP law.[6] Questions of fact are not reviewable. As a rule, the findings of fact of the Court of Appeals
No. 44777. The Court of Appeals reversed the Order [3] of the Regional Trial Court, Branch 23, are final and conclusive and this Court will not review them on appeal,[7] subject to exceptions as
Manila (RTC), denying respondent Maxicorp, Inc.s (Maxicorp) motion to quash the search when the findings of the appellate court conflict with the findings of the trial court.[8]
warrant that the RTC issued against Maxicorp. Petitioners are the private complainants against The distinction between questions of law and questions of fact is settled. A question of law
Maxicorp for copyright infringement under Section 29 of Presidential Decree No. 49 (Section 29 exists when the doubt or difference centers on what the law is on a certain state of facts. A
of PD 49)[4]and for unfair competition under Article 189 of the Revised Penal Code (RPC).[5] question of fact exists if the doubt centers on the truth or falsity of the alleged facts. Though this
Antecedent Facts delineation seems simple, determining the true nature and extent of the distinction is
On 25 July 1996, National Bureau of Investigation (NBI) Agent Dominador Samiano, Jr. (NBI sometimes problematic. For example, it is incorrect to presume that all cases where the facts are
Agent Samiano) filed several applications for search warrants in the RTC against Maxicorp for not in dispute automatically involve purely questions of law.
alleged violation of Section 29 of PD 49 and Article 189 of the RPC. After conducting a There is a question of law if the issue raised is capable of being resolved without need of
preliminary examination of the applicant and his witnesses, Judge William M. Bayhon issued reviewing the probative value of the evidence.[9] The resolution of the issue must rest solely on
Search Warrants Nos. 96-451, 96-452, 96-453 and 96-454, all dated 25 July 1996, against what the law provides on the given set of circumstances. Once it is clear that the issue invites a
Maxicorp. review of the evidence presented, the question posed is one of fact.[10] If the query requires a re-
Armed with the search warrants, NBI agents conducted on 25 July 1996 a search of evaluation of the credibility of witnesses, or the existence or relevance of surrounding
Maxicorps premises and seized property fitting the description stated in the search warrants. circumstances and their relation to each other, the issue in that query is factual.[11] Our ruling
On 2 September 1996, Maxicorp filed a motion to quash the search warrants alleging that inPaterno v. Paterno[12] is illustrative on this point:
there was no probable cause for their issuance and that the warrants are in the form of general Such questions as whether certain items of evidence should be accorded probative value or
warrants. The RTC denied Maxicorps motion on 22 January 1997. The RTC also denied Maxicorps weight, or rejected as feeble or spurious, or whether or not the proofs on one side or the other
motion for reconsideration. are clear and convincing and adequate to establish a proposition in issue, are without doubt
The RTC found probable cause to issue the search warrants after examining NBI Agent questions of fact. Whether or not the body of proofs presented by a party, weighed and analyzed
Samiano, John Benedict Sacriz (Sacriz), and computer technician Felixberto Pante (Pante). The in relation to contrary evidence submitted by adverse party, may be said to be strong, clear and
three testified on what they discovered during their respective visits to Maxicorp. NBI Agent convincing; whether or not certain documents presented by one side should be accorded full
Samiano also presented certifications from petitioners that they have not authorized Maxicorp faith and credit in the face of protests as to their spurious character by the other side; whether
to perform the witnessed activities using petitioners products. or not inconsistencies in the body of proofs of a party are of such gravity as to justify refusing to
On 24 July 1997, Maxicorp filed a petition for certiorari with the Court of Appeals seeking give said proofs weight all these are issues of fact.
to set aside the RTCs order. On 23 December 1998, the Court of Appeals reversed the RTCs order It is true that Maxicorp did not contest the facts alleged by petitioners. But this situation
denying Maxicorps motion to quash the search warrants. Petitioners moved for reconsideration. does not automatically transform all issues raised in the petition into questions of law. The
The Court of Appeals denied petitioners motion on 29 November 1999. issues must meet the tests outlined in Paterno.
The Court of Appeals held that NBI Agent Samiano failed to present during the preliminary Of the three main issues raised in this petition the legal personality of the petitioners, the
examination conclusive evidence that Maxicorp produced or sold the counterfeit products. The nature of the warrants issued and the presence of probable cause only the first two qualify as
Court of Appeals pointed out that the sales receipt NBI Agent Samiano presented as evidence questions of law. The pivotal issue of whether there was probable cause to issue the search
that he bought the products from Maxicorp was in the name of a certain Joel Diaz. warrants is a question of fact. At first glance, this issue appears to involve a question of law since
Hence, this petition. it does not concern itself with the truth or falsity of certain facts. Still, the resolution of this issue
The Issues would require this Court to inquire into the probative value of the evidence presented before the
Petitioners seek a reversal and raise the following issues for resolution: RTC. For a question to be one of law, it must not involve an examination of the probative value
1. WHETHER THE PETITION RAISES QUESTIONS OF LAW; of the evidence presented by the litigants or any of them.[13]
2. WHETHER PETITIONERS HAVE LEGAL PERSONALITY TO FILE THE PETITION; Yet, this is precisely what the petitioners ask us to do by raising arguments requiring an
3. WHETHER THERE WAS PROBABLE CAUSE TO ISSUE THE SEARCH WARRANTS; examination of the TSNs and the documentary evidence presented during the search warrant
4. WHETHER THE SEARCH WARRANTS ARE GENERAL WARRANTS. proceedings. In short, petitioners would have us substitute our own judgment to that of the RTC
The Ruling of the Court and the Court of Appeals by conducting our own evaluation of the evidence. This is exactly the
The petition has merit. situation which Section 1, Rule 45 of the Rules of Court prohibits by requiring the petition to
On Whether the Petition Raises Questions of Law raise only questions of law. This Court is not a trier of facts. It is not the function of this court to
Maxicorp assails this petition as defective since it failed to raise questions of law. Maxicorp analyze or weigh evidence.[14] When we give due course to such situations, it is solely by way of
insists that the arguments petitioners presented are questions of fact, which this Court should exception. Such exceptions apply only in the presence of extremely meritorious
not consider in a Rule 45 petition for review. Petitioners counter that all the issues they circumstances.[15]
23

Indeed, this case falls under one of the exceptions because the findings of the Court of of counterfeit products is but one of these acts. Both NBI Agent Samiano and Sacriz related to
Appeals conflict with the findings of the RTC.[16] Since petitioners properly raised the conflicting the RTC how they personally saw Maxicorp commit acts of infringement and unfair competition.
findings of the lower courts, it is proper for this Court to resolve such contradiction. During the preliminary examination, the RTC subjected the testimonies of the witnesses to
On Whether Petitioners have the Legal Personality to File this Petition the requisite examination. NBI Agent Samiano testified that he saw Maxicorp display and offer
Maxicorp argues that petitioners have no legal personality to file this petition since the for sale counterfeit software in its premises. He also saw how the counterfeit software were
proper party to do so in a criminal case is the Office of the Solicitor General as representative of produced and packaged within Maxicorps premises. NBI Agent Samiano categorically stated that
the People of the Philippines. Maxicorp states the general rule but the exception governs this he was certain the products were counterfeit because Maxicorp sold them to its customers
case.[17] We ruled in Columbia Pictures Entertainment, Inc. v. Court of Appeals [18] that the without giving the accompanying ownership manuals, license agreements and certificates of
petitioner-complainant in a petition for review under Rule 45 could argue its case before this authenticity.
Court in lieu of the Solicitor General if there is grave error committed by the lower court or lack Sacriz testified that during his visits to Maxicorp, he witnessed several instances when
of due process. This avoids a situation where a complainant who actively participated in the Maxicorp installed petitioners software into computers it had assembled. Sacriz also testified
prosecution of a case would suddenly find itself powerless to pursue a remedy due to that he saw the sale of petitioners software within Maxicorps premises. Petitioners never
circumstances beyond its control. The circumstances in Columbia Pictures Entertainment are authorized Maxicorp to install or sell their software.
sufficiently similar to the present case to warrant the application of this doctrine. The testimonies of these two witnesses, coupled with the object and documentary
On Whether there was Probable Cause to Issue the Search Warrants evidence they presented, are sufficient to establish the existence of probable cause. From what
Petitioners argue that the Court of Appeals erred in reversing the RTC based on the fact they have witnessed, there is reason to believe that Maxicorp engaged in copyright infringement
that the sales receipt was not in the name of NBI Agent Samiano. Petitioners point out that the and unfair competition to the prejudice of petitioners. Both NBI Agent Samiano and Sacriz were
Court of Appeals disregarded the overwhelming evidence that the RTC considered in clear and insistent that the counterfeit software were not only displayed and sold within
determining the existence of probable cause. Maxicorp counters that the Court of Appeals did Maxicorps premises, they were also produced, packaged and in some cases, installed there.
not err in reversing the RTC. Maxicorp maintains that the entire preliminary examination that the The determination of probable cause does not call for the application of rules and
RTC conducted was defective. standards of proof that a judgment of conviction requires after trial on the merits. As implied by
The Court of Appeals based its reversal on two factual findings of the RTC. First, the fact the words themselves, probable cause is concerned with probability, not absolute or even moral
that the sales receipt presented by NBI Agent Samiano as proof that he bought counterfeit goods certainty. The prosecution need not present at this stage proof beyond reasonable doubt. The
from Maxicorp was in the name of a certain Joel Diaz. Second, the fact that petitioners other standards of judgment are those of a reasonably prudent man,[24] not the exacting calibrations of
witness, John Benedict Sacriz, admitted that he did not buy counterfeit goods from Maxicorp. a judge after a full-blown trial.
We rule that the Court of Appeals erred in reversing the RTCs findings. No law or rule states that probable cause requires a specific kind of evidence. No formula
Probable cause means such reasons, supported by facts and circumstances as will warrant or fixed rule for its determination exists.[25] Probable cause is determined in the light of
a cautious man in the belief that his action and the means taken in prosecuting it are legally just conditions obtaining in a given situation.[26] Thus, it was improper for the Court of Appeals to
and proper.[19] Thus, probable cause for a search warrant requires such facts and circumstances reverse the RTCs findings simply because the sales receipt evidencing NBI Agent Samianos
that would lead a reasonably prudent man to believe that an offense has been committed and purchase of counterfeit goods is not in his name.
the objects sought in connection with that offense are in the place to be searched.[20] For purposes of determining probable cause, the sales receipt is not the only proof that the
The judge determining probable cause must do so only after personally examining under sale of petitioners software occurred. During the search warrant application proceedings, NBI
oath the complainant and his witnesses. The oath required must refer to the truth of the facts Agent Samiano presented to the judge the computer unit that he purchased from Maxicorp, in
within the personal knowledge of the petitioner or his witnesses, because the purpose thereof is which computer unit Maxicorp had pre-installed petitioners software.[27] Sacriz, who was present
to convince the committing magistrate, not the individual making the affidavit and seeking the when NBI Agent Samiano purchased the computer unit, affirmed that NBI Agent Samiano
issuance of the warrant, of the existence of probable cause. [21] The applicant must have personal purchased the computer unit.[28] Pante, the computer technician, demonstrated to the judge the
knowledge of the circumstances. Reliable information is insufficient.[22] Mere affidavits are not presence of petitioners software on the same computer unit.[29] There was a comparison
enough, and the judge must depose in writing the complainant and his witnesses. [23] between petitioners genuine software and Maxicorps software pre-installed in the computer
The Court of Appeals reversal of the findings of the RTC centers on the fact that the two unit that NBI Agent Sambiano purchased.[30] Even if we disregard the sales receipt issued in the
witnesses for petitioners during the preliminary examination failed to prove conclusively that name of Joel Diaz, which petitioners explained was the alias NBI Agent Samiano used in the
they bought counterfeit software from Maxicorp. The Court of Appeals ruled that this amounted operation, there still remains more than sufficient evidence to establish probable cause for the
to a failure to prove the existence of a connection between the offense charged and the place issuance of the search warrants.
searched. This also applies to the Court of Appeals ruling on Sacrizs testimony. The fact that Sacriz did
The offense charged against Maxicorp is copyright infringement under Section 29 of PD 49 not actually purchase counterfeit software from Maxicorp does not eliminate the existence of
and unfair competition under Article 189 of the RPC. To support these charges, petitioners probable cause. Copyright infringement and unfair competition are not limited to the act of
presented the testimonies of NBI Agent Samiano, computer technician Pante, and Sacriz, a selling counterfeit goods. They cover a whole range of acts, from copying, assembling, packaging
civilian. The offenses that petitioners charged Maxicorp contemplate several overt acts. The sale to marketing, including the mere offering for sale of the counterfeit goods. The clear and firm
testimonies of petitioners witnesses on such other acts stand untarnished. The Constitution and
24

the Rules of Court only require that the judge examine personally and thoroughly the applicant connection with fake or counterfeit products bearing the Microsoft copyrights
for the warrant and his witnesses to determine probable cause. The RTC complied adequately and/or trademarks owned by MICROSOFT CORPORATION;
with the requirement of the Constitution and the Rules of Court. e) Computer hardware, including central processing units including hard disks, CD-ROM
Probable cause is dependent largely on the opinion and findings of the judge who drives, keyboards, monitor screens and diskettes, photocopying machines and
conducted the examination and who had the opportunity to question the applicant and his other equipment or paraphernalia used or intended to be used in the illegal and
witnesses.[31] For this reason, the findings of the judge deserve great weight. The reviewing court unauthorized copying or reproduction of Microsoft software and their manuals,
should overturn such findings only upon proof that the judge disregarded the facts before him or or which contain, display or otherwise exhibit, without the authority of
ignored the clear dictates of reason.[32] Nothing in the records of the preliminary examination MICROSOFT CORPORATION, any and all Microsoft trademarks and copyrights;
proceedings reveal any impropriety on the part of the judge in this case. As one can readily see, and
here the judge examined thoroughly the applicant and his witnesses. To demand a higher degree f) Documents relating to any passwords or protocols in order to access all computer hard
of proof is unnecessary and untimely. The prosecution would be placed in a compromising drives, data bases and other information storage devices containing unauthorized
situation if it were required to present all its evidence at such preliminary stage. Proof beyond Microsoft software.[37](Emphasis supplied)
reasonable doubt is best left for trial. It is only required that a search warrant be specific as far as the circumstances will
On Whether the Search Warrants are in the Nature of General Warrants ordinarily allow.[38] The description of the property to be seized need not be technically accurate
A search warrant must state particularly the place to be searched and the objects to be or precise. The nature of the description should vary according to whether the identity of the
seized. The evident purpose for this requirement is to limit the articles to be seized only to those property or its character is a matter of concern.[39] Measured against this standard we find that
particularly described in the search warrant. This is a protection against potential abuse. It is paragraph (e) is not a general warrant. The articles to be seized were not only sufficiently
necessary to leave the officers of the law with no discretion regarding what articles they shall identified physically, they were also specifically identified by stating their relation to the offense
seize, to the end that no unreasonable searches and seizures be committed.[33] charged. Paragraph (e) specifically refers to those articles used or intended for use in the illegal
In addition, under Section 4, Rule 126 of the Rules of Criminal Procedure, a search warrant and unauthorized copying of petitioners software. This language meets the test of specificity.[40]
shall issue in connection with one specific offense. The articles described must bear a direct The cases cited by the Court of Appeals are inapplicable. In those cases, the Court found
relation to the offense for which the warrant is issued.[34] Thus, this rule requires that the the warrants too broad because of particular circumstances, not because of the mere use of the
warrant must state that the articles subject of the search and seizure are used or intended for phrase used or intended to be used. In Columbia Pictures, Inc. v. Flores, the warrants ordering
use in the commission of a specific offense. the seizure of television sets, video cassette recorders, rewinders and tape cleaners x x x were
Maxicorp argues that the warrants issued against it are too broad in scope and lack the found too broad since the defendant there was a licensed distributor of video tapes. [41] The mere
specificity required with respect to the objects to be seized. After examining the wording of the presence of counterfeit video tapes in the defendants store does not mean that the machines
warrants issued, the Court of Appeals ruled in favor of Maxicorp and reversed the RTCs Order were used to produce the counterfeit tapes. The situation in this case is different. Maxicorp is
thus: not a licensed distributor of petitioners. In Bache & Co. (Phil.), Inc., et al. v. Judge Ruiz, et al.,
Under the foregoing language, almost any item in the petitioners store can be seized on the the Court voided the warrants because they authorized the seizure of records pertaining to all
ground that it is used or intended to be used in the illegal or unauthorized copying or business transactions of the defendant.[42] And in 20th Century Fox Film Corp. v. Court of
reproduction of the private respondents software and their manuals.[35] Appeals, the Court quashed the warrant because it merely gave a list of articles to be seized,
The Court of Appeals based its reversal on its perceived infirmity of paragraph (e) of the search aggravated by the fact that such appliances are generally connected with the legitimate business
warrants the RTC issued. The appellate court found that similarly worded warrants, all of which of renting out betamax tapes.[43]
noticeably employ the phrase used or intended to be used, were previously held void by this However, we find paragraph (c) of the search warrants lacking in particularity. Paragraph
Court.[36] The disputed text of the search warrants in this case states: (c) states:
a) Complete or partially complete reproductions or copies of Microsoft software bearing c) Sundry items such as labels, boxes, prints, packages, wrappers, receptacles,
the Microsoft copyrights and/or trademarks owned by MICROSOFT CORPORATION advertisements and other paraphernalia bearing the copyrights and/or trademarks
contained in CD-ROMs, diskettes and hard disks; owned by MICROSOFT CORPORATION;
b) Complete or partially complete reproductions or copies of Microsoft instruction The scope of this description is all-embracing since it covers property used for personal or other
manuals and/or literature bearing the Microsoft copyrights and/or trademarks purposes not related to copyright infringement or unfair competition. Moreover, the description
owned by MICROSOFT CORPORATION; covers property that Maxicorp may have bought legitimately from Microsoft or its licensed
c) Sundry items such as labels, boxes, prints, packages, wrappers, receptacles, distributors. Paragraph (c) simply calls for the seizure of all items bearing the Microsoft logo,
advertisements and other paraphernalia bearing the copyrights and/or trademarks whether legitimately possessed or not. Neither does it limit the seizure to products used in
owned by MICROSOFT CORPORATION; copyright infringement or unfair competition.
d) Sales invoices, delivery receipts, official receipts, ledgers, journals, purchase orders and Still, no provision of law exists which requires that a warrant, partially defective in
all other books of accounts and documents used in the recording of the specifying some items sought to be seized yet particular with respect to the other items, should
reproduction and/or assembly, distribution and sales, and other transactions in be nullified as a whole. A partially defective warrant remains valid as to the items specifically
described in the warrant.[44] A search warrant is severable, the items not sufficiently described
25

may be cut off without destroying the whole warrant.[45] The exclusionary rule found in Section
3(2) of Article III of the Constitution renders inadmissible in any proceeding all evidence obtained
through unreasonable searches and seizure. Thus, all items seized under paragraph (c) of the
search warrants, not falling under paragraphs a, b, d, e or f, should be returned to Maxicorp.
WHEREFORE, we PARTIALLY GRANT the instant petition. The Decision of the Court of
Appeals dated 23 December 1998 and its Resolution dated 29 November 1999 in CA-G.R. SP No.
44777 are REVERSED and SET ASIDE except with respect to articles seized under paragraph (c) of
Search Warrants Nos. 96-451, 96-452, 96-453 and 96-454. All articles seized under paragraph (c)
of the search warrants, not falling under paragraphs a, b, d, e or f, are ordered returned to
Maxicorp, Inc. immediately.
SO ORDERED.
26

PEOPLE vs TEE all having a grand total weight of 928.74 kilograms, a prohibited drug, without the authority of
For automatic review is the consolidated judgment[1] of the Regional Trial Court (RTC) of law to possess, in violation of the above-cited provision of law.
Baguio City, Branch 6, dated September 17, 1999, in Criminal Cases Nos. 15800-R and 15822-R, CONTRARY TO LAW.[4]
involving violations of Section 8, Article II, of the Dangerous Drugs Law.[2] Since appellant was On August 7, 1998, the prosecution moved to amend the foregoing charge sheet
acquitted in the second case, we focus on the first case, where appellant has been found guilty considering that subject marijuana were seized in two (2) different places.[5]
and sentenced to death and fined one million pesos. As a result, the information in Criminal Case No. 15800-R was amended to read as follows:
The decretal portion of the trial courts decision reads: That on or about the 1st day of July, 1998, in the City of Baguio, Philippines, and within the
WHEREFORE, judgment is hereby rendered, as follows: jurisdiction of this Honorable Court, the above-named accused, did then and there willfully,
1. In Crim. Case No. 15800-R, the Court finds the accused Modesto Tee guilty beyond reasonable unlawfully, feloniously and knowingly have in his possession the following, to wit:
doubt of the offense of illegal possession of marijuana of about 591.81 kilos in violation of - Six hundred two (602) bricks of dried flowering tops separately contained in twenty-six (26)
Section 8, Article II of RA 6425 as amended by Section 13 of RA 7659 as charged in the boxes and a yellow sack, weighing 591.81 kilograms
Information, seized by virtue of a search warrant and sentences him to the supreme penalty of a prohibited drug, without the authority of law to possess, in violation of the above-cited
death and to pay a fine of 1 million pesos without subsidiary imprisonment in case of insolvency. provision of law.
The 591.81 kilos of marijuana contained in 26 boxes and one yellow sack (Exhibits U-1 to U-27) CONTRARY TO LAW.[6]
are ordered forfeited in favor of the State to be destroyed immediately in accordance with law. A separate amended information docketed as Criminal Case No. 15822-R was likewise
2. In Crim. Case No. 15822-R, the Court finds that the prosecution failed to prove the guilt of filed, the accusatory portion of which reads:
accused Modesto Tee beyond reasonable doubt and hereby acquits him of the charge of illegal That on or about the 1st day of July, 1998 in the City of Baguio, Philippines, and within the
possession of marijuana in violation of Section 8, Art. 2 of RA 6425 as amended by Section 13 of jurisdiction of this Honorable Court, the above-named accused, did then and there willfully,
RA 7659 as charged in the Information since the marijuana confiscated have to be excluded in unlawfully, feloniously and knowingly have in his possession the following, to wit:
evidence as a product of unreasonable search and seizure. 1. Ninety-two (92) bricks of dried flowering tops separately contained in four (4)
The 336.93 kilos of marijuana contained in 13 sacks and four boxes (Exh. B to S and their boxes; and
component parts) although excluded in evidence as the product(s) of unreasonable search and 2. hundred fifty-eight (158) bricks, twenty-one (21) blocks, and twenty-three (23)
seizure, are nevertheless ordered forfeited in favor of the State to be destroyed immediately in bags of dried flowering tops separately contained in thirteen (13) sacks, with a
accordance with law considering that they are prohibited articles. total weight of 336.93 kilograms;
The City Jail Warden is, therefore, directed to release the accused Modesto Tee in connection a prohibited drug, without the authority of law to possess, in violation of the above-cited
with Crim. Case No. 15822-R unless held on other charges. provision of law.
COST(S) DE OFFICIO. CONTRARY TO LAW.[7]
SO ORDERED.[3] On September 4, 1998, the trial court denied the motion to quash the search warrant and
Appellant is a Chinese national in his forties, a businessman, and a resident of Baguio City. ordered appellants arraignment.
A raid conducted by operatives of the National Bureau of Investigation (NBI) and Philippine When arraigned in Criminal Cases Nos. 15800-R and 15822-R, appellant refused to enter a
National Police Narcotics Command (PNP NARCOM) at premises allegedly leased by appellant plea. The trial court entered a plea of not guilty for him.[8] Trial on the merits then ensued.
and at his residence yielded huge quantities of marijuana. The facts of this case, as gleaned from the records, are as follows:
On July 20, 1998, appellant moved to quash the search warrant on the ground that it was Prosecution witness Danilo Abratique, a Baguio-based taxi driver, and the appellant
too general and that the NBI had not complied with the requirements for the issuance of a valid Modesto Tee are well acquainted with each other, since Abratiques wife is the sister of Tees
search warrant. The pendency of said motion, however, did not stop the filing of the appropriate sister-in-law.[9]
charges against appellant. In an information dated July 24, 1998, docketed as Criminal Case No. Sometime in late June 1998, appellant asked Abratique to find him a place for the storage
15800-R, the City Prosecutor of Baguio City charged Modesto Tee, alias Estoy Tee, with illegal of smuggled cigarettes.[10] Abratique brought appellant to his friend, Albert Ballesteros, who had
possession of marijuana, allegedly committed as follows: a house for rent in Bakakeng, Baguio City. After negotiating the terms and conditions, Ballesteros
That on or about the 1st day of July, 1998 in the City of Baguio, Philippines, and within the agreed to rent out his place to appellant. Appellant then brought several boxes of purported
jurisdiction of this Honorable Court, the above-named accused, did then and there willfully, blue seal cigarettes to the leased premises.
unlawfully, feloniously and knowingly have in his possession the following, to wit: Shortly thereafter, however, Ballesteros learned that the boxes stored in his place were
1. Ninety-two (92) bricks of dried flowering tops separately contained in four (4) boxes; and not blue seal cigarettes but marijuana. Fearful of being involved, Ballesteros informed Abratique.
2. One hundred fifty-eight (158) bricks, twenty-one (21) blocks, and twenty-three (23) bags of Both later prevailed upon appellant to remove them from the premises.[11]
dried flowering tops separately contained in thirteen (13) sacks, with a total weight of 336.93 Appellant then hired Abratiques taxi and transported the boxes of cannabis from the
kilograms; and Ballesteros place to appellants residence at Km. 6, Dontogan, Green Valley, Sto. Tomas, Baguio
3 Six hundred two (602) bricks of dried flowering tops separately contained in twenty-six (boxes) City.[12]
and a yellow sack, weighing 591.81 kilograms, On June 30, 1998, appellant hired Abratique to drive him to La Trinidad, Benguet on the
pretext of buying and transporting strawberries. Upon reaching La Trinidad, however, appellant
27

directed Abratique to proceed to Sablan, Benguet, where appellant proceeded to load several evidence against appellant. Appellant was accordingly acquitted of the charge. However, the trial
sacks of marijuana in Abratiques taxi. He then asked Abratique to find him a place where he court found that the prosecutions evidence was more than ample to prove appellants guilt in
could store the contraband.[13] Criminal Case No. 15800-R and as earlier stated, duly convicted him of illegal possession of
Abratique brought appellant to his grandmothers house at No. 27 Dr. Cario St., QM marijuana and sentenced him to death.
Subdivision, Baguio City, which was being managed by Abratiques aunt, Nazarea Abreau. Hence, this automatic review.
Nazarea agreed to rent a room to appellant. Abratique and appellant unloaded and stored there Before us, appellant submits that the trial court erred in:
the sacks of marijuana brought from Sablan.[14] Abratique was aware that they were transporting 1UPHOLDING THE LEGALITY OF THE SEARCH WARRANT DESPITE LACK OF
marijuana as some of the articles in the sacks became exposed in the process of loading.[15] COMPLIANCE OF (sic) SEVERAL REQUIREMENTS BEFORE IT SHOULD HAVE BEEN
Eventually, Abratique and Nazarea were bothered by the nature of the goods stored in the ISSUED AND ITBEING A GENERAL WARRANT;
rented room. She confided to her daughter, Alice Abreau Fianza, about their predicament. As 2.GRAVELY ABUSED ITS DISCRETION IN REOPENING THE CASE AND ALLOWING
Alice Fianzas brother-in-law, Edwin Fianza, was an NBI agent, Alice and Abratique phoned him ABRITIQUE TO TESTIFY AGAINST APPELLANT;
and disclosed what had transpired.[16] 3GIVING CREDENCE TO THE TESTIMONY OF ABRITIQUE;
On the morning of July 1, 1998, alerted by information that appellant would retrieve the 4. NOT ACQUITTING THE ACCUSED IN BOTH CASES AND SENTENCING HIM TO DEATH
sacks of prohibited drugs that day, Edwin Fianza and other NBI operatives conducted a stake out DESPITE THE ILLEGALLY OBTAINED EVIDENCE AS FOUND IN THE FIRST CASE.[28]
at No. 27, Dr. Cario St. While the NBI agents were conducting their surveillance, they noticed that We find that the pertinent issues for resolution concern the following: (1) the validity of
several PNP NARCOM personnel were also watching the place.[17] The NBI then learned that the the search conducted at the appellants residence; (2) the alleged prejudice caused by the
PNP NARCOM had received a tip from one of their informers regarding the presence of a huge reopening of the case and absences of the prosecution witness, on appellants right to speedy
amount of drugs in that place. The NBI and PNP NARCOM agreed to have a joint operation. trial; (3) the sufficiency of the prosecutions evidence to sustain a finding of guilt with moral
As the day wore on and appellant did not show up, the NBI agents became apprehensive certainty; and (4) the propriety of the penalty imposed.
that the whole operation could be jeopardized. They sought the permission of Nazarea Abreau 1. On the Validity of the Search Warrant; Its Obtention and Execution
to enter the room rented by appellant. She acceded and allowed them entry. The NBI team then Appellant initially contends that the warrant, which directed the peace officers to search
searched the rented premises and found four (4) boxes and thirteen (13) sacks of marijuana, for and seize an undetermined amount of marijuana, was too general and hence, void for
totaling 336.93 kilograms.[18] vagueness. He insists that Abratique could already estimate the amount of marijuana supposed
Later that evening, NBI Special Agent Darwin Lising, with Abratique as his witness, applied to be found at appellants residence since Abratique helped to transport the same.
for a search warrant from RTC Judge Antonio Reyes at his residence. [19] Judge Reyes ordered the For the appellee, the Office of the Solicitor General (OSG) counters that a search warrant is
NBI agents to fetch the Branch Clerk of Court, Atty. Delilah Muoz, so the proceedings could be issued if a judge finds probable cause that the place to be searched contains prohibited drugs,
properly recorded. After Atty. Muoz arrived, Judge Reyes questioned Lising and and not that he believes the place contains a specific amount of it. The OSG points out that, as
Abratique. Thereafter, the judge issued a warrant directing the NBI to search appellants the trial court observed, it is impossible beforehand to determine the exact amount of
residence at Km. 6, Dontogan, Green Valley, Baguio City for marijuana.[20] prohibited drugs that a person has on himself.
The NBI operatives, with some PNP NARCOM personnel in tow, proceeded to appellants Appellant avers that the phrase an undetermined amount of marijuana as used in the
residence where they served the warrant upon appellant himself. [21] The search was witnessed search warrant fails to satisfy the requirement of Article III, Section 2[29] of the Constitution that
by appellant, members of his family, barangay officials, and members of the the things to be seized must be particularly described. Appellants contention, in our view, has no
media.[22] Photographs were taken during the actual search. [23] The law enforcers found 26 boxes leg to stand on. The constitutional requirement of reasonable particularity of description of the
and a sack of dried marijuana[24] in the water tank, garage, and storeroom of appellants things to be seized is primarily meant to enable the law enforcers serving the warrant to: (1)
residence.[25] The total weight of the haul was 591.81 kilograms.[26] Appellant was arrested for readily identify the properties to be seized and thus prevent them from seizing the wrong
illegal possession of marijuana. items;[30] and (2) leave said peace officers with no discretion regarding the articles to be seized
The seized items were then submitted to the NBI laboratory for testing. NBI Forensic and thus prevent unreasonable searches and seizures.[31] What the Constitution seeks to avoid
Chemist Maria Carina Madrigal conducted the tests. Detailed microscopic and chromatographic are search warrants of broad or general characterization or sweeping descriptions, which will
examinations of the items taken from appellants rented room at No. 27, Dr. Cario St., as well as authorize police officers to undertake a fishing expedition to seize and confiscate any and all
those from his residence at Green Valley, showed these to be marijuana.[27] kinds of evidence or articles relating to an offense. [32] However, it is not required that technical
In his defense, appellant contended that the physical evidence of the prosecution was precision of description be required, [33] particularly, where by the nature of the goods to be
illegally obtained, being the products of an unlawful search, hence inadmissible. Appellant seized, their description must be rather general, since the requirement of a technical description
insisted that the search warrant was too general and the process by which said warrant was would mean that no warrant could issue.[34]
acquired did not satisfy the constitutional requirements for the issuance of a valid search Thus, it has been held that term narcotics paraphernalia is not so wanting in particularity
warrant. Moreover, Abratiques testimony, which was heavily relied upon by the judge who as to create a general warrant.[35] Nor is the description any and all narcotics and all implements,
issued the warrant, was hearsay. paraphernalia, articles, papers and records pertaining to the use, possession, or sale of narcotics
In Criminal Case No. 15822-R, the trial court agreed with appellant that the taking of the or dangerous drugs so broad as to be unconstitutional.[36] A search warrant commanding peace
336.93 kilograms of marijuana was the result of an illegal search and hence, inadmissible in officers to seize a quantity of loose heroin has been held sufficiently particular. [37]
28

Tested against the foregoing precedents, the description an undetermined amount of In this case, witness Abratique personally saw and handled the marijuana. Hence, the NBI did not
marijuana must be held to satisfy the requirement for particularity in a search warrant. rely on hearsay information in applying for a search warrant but on personal knowledge of the
Noteworthy, what is to be seized in the instant case is property of a specified character, i.e., witness, Abratique.
marijuana, an illicit drug. By reason of its character and the circumstances under which it would Before a valid search warrant is issued, both the Constitution[45] and the 2000 Revised
be found, said article is illegal. A further description would be unnecessary and ordinarily Rules of Criminal Procedure[46] require that the judge must personally examine the complainant
impossible, except as to such character, the place, and the circumstances. [38] Thus, this Court has and his witnesses under oath or affirmation. The personal examination must not be merely
held that the description illegally in possession of undetermined quantity/amount of dried routinary or pro forma, but must be probing and exhaustive.[47] In the instant case, it is not
marijuana leaves and Methamphetamine Hydrochloride (Shabu) and sets of paraphernalia disputed that Judge Antonio Reyes personally examined NBI Special Investigator III Darwin A.
particularizes the things to be seized.[39] Lising, the applicant for the search warrant as well as his witness, Danilo G. Abratique. Notes of
The search warrant in the present case, given its nearly similar wording, undetermined the proceedings were taken by Atty. Delilah Muoz, Clerk of Court, RTC of Baguio City, Branch 61,
amount of marijuana or Indian hemp, in our view, has satisfied the Constitutions requirements whom Judge Reyes had ordered to be summoned. In the letter of transmittal of the Clerk of
on particularity of description. The description therein is: (1) as specific as the circumstances will Court of the RTC of Baguio City, Branch 61 to Branch 6 of said court, mention is made of notes at
ordinarily allow; (2) expresses a conclusion of fact not of law by which the peace officers may be pages 7-11.[48] We have thoroughly perused the records of Search Warrant No. 415 (7-98) and
guided in making the search and seizure; and (3) limits the things to be seized to those which nowhere find said notes. The depositions of Lising and Abratique were not attached to Search
bear direct relation to the offense for which the warrant is being issued. [40] Said warrant imposes Warrant No. 415 (7-98) as required by the Rules of Court. We must stress, however, that the
a meaningful restriction upon the objects to be seized by the officers serving the warrant. Thus, purpose of the Rules in requiring depositions to be taken is to satisfy the examining magistrate
it prevents exploratory searches, which might be violative of the Bill of Rights. as to the existence of probable cause.[49] The Bill of Rights does not make it an imperative
Appellant next assails the warrant for merely stating that he should be searched, as he necessity that depositions be attached to the records of an application for a search warrant.
could be guilty of violation of Republic Act No. 6425. Appellant claims that this is a sweeping Hence, said omission is not necessarily fatal, for as long as there is evidence on the record
statement as said statute lists a number of offenses with respect to illegal drugs. Hence, he showing what testimony was presented.[50] In the testimony of witness Abratique, Judge Reyes
contends, said warrant is a general warrant and is thus unconstitutional. required Abratique to confirm the contents of his affidavit;[51] there were instances when Judge
For the appellee, the OSG points out that the warrant clearly states that appellant has in Reyes questioned him extensively.[52] It is presumed that a judicial function has been regularly
his possession and control marijuana or Indian hemp, in violation of Section 8 of Republic Act No. performed,[53] absent a showing to the contrary. A magistrates determination of probable cause
6425. for the issuance of a search warrant is paid great deference by a reviewing court, [54] as long as
We have carefully scrutinized Search Warrant No. 415 (7-98),[41] and we find that it is there was substantial basis for that determination.[55] Substantial basis means that the questions
captioned For Violation of R.A. 6425, as amended.[42] It is clearly stated in the body of the of the examining judge brought out such facts and circumstances as would lead a reasonably
warrant that there is probable cause to believe that a case for violation of R.A. 6425, as discreet and prudent man to believe that an offense has been committed, and the objects in
amended, otherwise known as the Dangerous Drugs Act of 1972, as further amended by R.A. connection with the offense sought to be seized are in the place sought to be searched.
7659 has been and is being committed by one MODESTO TEE a.k.a. ESTOY TEE of Km. 6, On record, appellant never raised the want of adequate depositions to support Warrant
Dontogan Bgy., Green Valley, Sto. Tomas, Baguio City by having in his possession and control an No. 415 (7-98) in his motion to quash before the trial court. Instead, his motion contained vague
UNDETERMINED AMOUNT OF MARIJUANA or INDIAN HEMP in violation of the aforementioned generalities that Judge Reyes failed to ask searching questions of the applicant and his witness.
law.[43] In an earlier case, we held that though the specific section of the Dangerous Drugs Law is Belatedly, however, he now claims that Judge Reyes perfunctorily examined said witness. [56] But
not pinpointed, there is no question at all of the specific offense alleged to have been committed it is settled that when a motion to quash a warrant is filed, all grounds and objections then
as a basis for the finding of probable cause.[44] Appellants averment is, therefore, available, existent or known, should be raised in the original or subsequent proceedings for the
baseless. Search Warrant No. 415 (7-98) appears clearly issued for one offense, namely, illegal quashal of the warrant, otherwise they are deemed waived.[57]
possession of marijuana. In this case, NBI Special Investigator Lisings knowledge of the illicit drugs stored in
Appellant next faults the Judge who issued Search Warrant No. 415 (7-98) for his failure to appellants house was indeed hearsay. But he had a witness, Danilo Abratique, who had personal
exhaustively examine the applicant and his witness. Appellant points out that said magistrate knowledge about said drugs and their particular location. Abratiques statements to the NBI and
should not have swallowed all of Abratiques statements hook, line, and sinker. He points out to Judge Reyes contained credible and reliable details. As the NBIs witness, Abratique was a
that since Abratique consented to assist in the transport of the marijuana, the examining judge person on whose statements Judge Reyes could rely. His detailed description of appellants
should have elicited from Abratique his participation in the crime and his motive for squealing on activities with respect to the seized drugs was substantial. In relying on witness Abratique, Judge
appellant. Appellant further points out that the evidence of the NBI operative who applied for Reyes was not depending on casual rumor circulating in the underworld, but on personal
the warrant is merely hearsay and should not have been given credit at all by Judge Reyes. knowledge Abratique possessed.
Again, the lack of factual basis for appellants contention is apparent. The OSG points out In Alvarez vs. Court of First Instance of Tayabas, 64 Phil. 33, 44 (1937), we held that:
that Abratique personally assisted appellant in loading and transporting the marijuana to the The true test of sufficiency of a deposition or affidavit to warrant issuance of a search warrant is
latters house and to appellants rented room at No. 27 Dr. Cario St., Baguio City. Definitely, this whether it has been drawn in such a manner that perjury could be charged thereon and affiant
indicates personal knowledge on Abratiques part. Law enforcers cannot themselves be be held liable for damages caused.[58]
eyewitnesses to every crime; they are allowed to present witnesses before an examining judge.
29

Appellant argues that the address indicated in the search warrant did not clearly indicate beyond a reasonable period of time, as in this instance, for more than a year, the accused is
the place to be searched. The OSG points out that the address stated in the warrant is as specific entitled to relief by a proceeding in mandamus to compel a dismissal of the information, or if he
as can be. The NBI even submitted a detailed sketch of the premises prepared by Abratique, thus be restrained of his liberty, by habeas corpus to obtain his freedom.
ensuring that there would be no mistake. The concept of speedy trial is necessarily relative. A determination as to whether the right
A description of the place to be searched is sufficient if the officer serving the warrant can, has been violated involves the weighing of several factors such as the length of the delay, the
with reasonable effort, ascertain and identify the place intended [59] and distinguish it from other reason for the delay, the conduct of the prosecution and the accused, and the efforts exerted by
places in the community.[60] A designation or description that points out the place to be searched the defendant to assert his right, as well as the prejudice and damage caused to the accused.[74]
to the exclusion of all others, and on inquiry unerringly leads the peace officers to it, satisfies the The Speedy Trial Act of 1998, provides that the trial period for criminal cases in general
constitutional requirement of definiteness. shall be one hundred eighty (180) days.[75] However, in determining the right of an accused to
Appellant finally harps on the use of unnecessary force during the execution of the search speedy trial, courts should do more than a mathematical computation of the number of
warrant. Appellant fails, however, to point to any evidentiary matter in the record to support his postponements of the scheduled hearings of the case.[76] The right to a speedy trial is deemed
contention. Defense witness Cipriana Tee, appellants mother, testified on the search conducted violated only when: (1) the proceedings are attended by vexatious, capricious, and oppressive
but she said nothing that indicated the use of force on the part of the NBI operatives who delays;[77] or (2) when unjustified postponements are asked for and secured;[78] or (3) when
conducted the search and seizure.[61] What the record discloses is that the warrant was served without cause or justifiable motive a long period of time is allowed to elapse without the party
on appellant,[62] who was given time to read it,[63] and the search was witnessed by the barangay having his case tried.[79]
officials, police operatives, members of the media, and appellants kith and kin. [64] No breakage or In the present case, although the absences of prosecution witness Abratique totaled
other damage to the place searched is shown. No injuries sustained by appellant, or any witness, twenty (20) hearing days, there is no showing whatsoever that prosecution capriciously caused
appears on record. The execution of the warrant, in our view, has been orderly and peaceably Abratiques absences so as to vex or oppress appellant and deny him his rights. On record, after
performed. Abratique repeatedly failed to show up for the taking of his testimony, the prosecution went to
2. On The Alleged Violation of Appellants Substantive Rights the extent of praying that the trial court order the arrest of Abratique to compel his attendance
Appellant insists that the prosecutions unjustified and willful delay in presenting witness at trial. The prosecution likewise tried to get the NBI to produce Abratique as the latter was in
Abratique unduly delayed the resolution of his case. He points out that a total of eight (8) the Bureaus custody, but to no avail. Eventually, the trial court ordered the prosecution to waive
scheduled hearings had to be reset due to the failure or willful refusal of Abratique to testify its right to present Abratique and rest its case on the evidence already offered.[80]
against him. Appellant insists that said lapse on the prosecutions part violated Supreme Court Nor do we find a delay of twenty (20) hearing days to be an unreasonable length of time.
Circular No. 38-98.[65] Appellant now alleges that the prosecution deliberately resorted to Delay of less than two months has been found, in fact, to be not an unreasonably lengthy period
delaying the case to cause him untold miseries. of time.[81]
For the appellee, the OSG points out that the two-month delay in the trial is not such a Moreover, nothing on record shows that appellant Modesto Tee objected to the inability
great length of time as to amount to a violation of appellants right to a speedy trial. A trial is of the prosecution to produce its witness. Under the Rules, appellant could have moved the trial
always subject to reasonable delays or postponements, but absent any showing that these court to require that witness Abratique post bail to ensure that the latter would testify when
delays are capricious and oppressive, the State should not be deprived of a reasonable required.[82] Appellant could have moved to have Abratique found in contempt and duly
opportunity to prosecute the criminal action. sanctioned. Appellant did neither. It is a bit too late in the day for appellant to invoke now his
On record, the trial court found that prosecution witness Danilo G. Abratique failed to right to speedy trial.
appear in no less than eighteen (18) hearings, namely those set for February 1, 2, 3, 4, 8, 9, 10, No persuasive reason supports appellants claim that his constitutional right to speedy trial
and 24; March 9, 15, 22, and 23; April 6, 7, 8, 16, and 19, all in 1999. [66] No less than four (4) was violated. One must take into account that a trial is always subject to postponements and
warrants of arrest were issued against him to compel him to testify.[67] The NBI agent who other causes of delay. But in the absence of a showing that delays were unreasonable and
supposedly had him in custody was found guilty of contempt of court for failing to produce capricious, the State should not be deprived of a reasonable opportunity of prosecuting an
Abratique at said hearings and sanctioned. [68] The prosecution had to write the NBI Regional accused.[83]
Director in Baguio City and NBI Director in Manila regarding the failure of the Bureaus agents to Appellant next contends that the trial court gravely abused its discretion, and exhibited
bring Abratique to court.[69] Nothing on record discloses the reason for Abratiques aforecited partiality, when it allowed the reopening of the case after the prosecution had failed to present
absences. On the scheduled hearing of June 7, 1999, he was again absent thus causing the trial Abratique on several occasions and had been directed to rest its case. Appellant stresses that the
court to again order his arrest for the fifth time.[70] He also failed to show up at the hearing of lower courts order to reopen the case to receive Abratiques further testimony is an indication
June 8, 1999.[71] that the trial court favored the prosecution and unduly prejudiced appellant.
Appellant now stresses that the failure of Abratique to appear and testify on twenty (20) On appellees behalf, the Solicitor General points out that the trial courts order was in the
hearing dates violated appellants constitutional[72] and statutory right to a speedy trial. interest of substantial justice and hence, cannot be termed as an abuse of discretion. The OSG
A speedy trial means a trial conducted according to the law of criminal procedure and the points out that the prosecution had not formally rested its case and had yet to present its formal
rules and regulations, free from vexatious, capricious, and oppressive delays. [73] In Conde v. offer of evidence, hence, the submission of additional testimony by the same witness cannot be
Rivera and Unson, 45 Phil. 650, 652 (1924), the Court held that where a prosecuting officer, prejudicial to the accused, it being but the mere continuation of an uncompleted testimony.
without good cause, secures postponements of the trial of a defendant against his protest Furthermore, appellant did not properly oppose the prosecutions motion to reopen the case.
30

At the time Criminal Cases Nos. 15800-R and 15822-R were being tried, the 1985 Rules of The OSG contends that Abratiques testimony, taken as a whole, is credible. It points out
Criminal Procedure were in effect. There was no specific provision at that time governing that Abratique testified in a straightforward manner as to his knowledge of the huge cache of
motions to reopen.[84] Nonetheless, long and established usage has led to the recognition and prohibited drugs stashed by appellant in two different places. His testimony, said the OSG, when
acceptance of a motion to reopen. In view of the absence of a specific procedural rule, the only fused with the physical evidence consisting of 591.81 kilograms of marijuana found by law
controlling guideline governing a motion to reopen was the paramount interests of justice. As a enforcers at appellants residence, inexorably leads to the inculpation of appellant.
rule, the matter of reopening of a case for reception of further evidence after either prosecution It is the bounden duty of the courts to test the prosecution evidence rigorously, so that no
or defense has rested its case is within the discretion of the trial court. [85] However, a concession innocent person is made to suffer the unusually severe penalties meted out for drug
to a reopening must not prejudice the accused or deny him the opportunity to introduce counter offenses.[93] Though we scrutinized minutely the testimony of Abratique, we find no cogent
evidence.[86] reason to disbelieve him. From his account, Abratique might appear aware treading the thin line
Strictly speaking, however, there was no reopening of the cases in the proceedings below. between innocence and feeling guilty, with certain portions of his story tending to be self-
A motion to reopen may properly be presented only after either or both parties have formally exculpatory. However, his whole testimony could not be discredited. The established rule is that
offered and closed their evidence, but before judgment.[87] In the instant case, the records show testimony of a witness may be believed in part and disbelieved in other parts, depending on the
that on April 19, 1999, the prosecution was directed to close its evidence and given 15 days to corroborative evidence and the probabilities and improbabilities of the case. But it is accepted,
make its formal offer of evidence.[88] This order apparently arose from the manifestation of the as a matter of common sense, that if certain parts of a witness testimony are found true, his
prosecution on April 16, 1999 that should they fail to produce witness Abratique on the next testimony cannot be disregarded entirely.[94]
scheduled hearing the prosecution would rest its case.[89] On April 19, 1999, which was the next Abratique testified in open court that appellant rented the taxicab he was driving, and he
scheduled hearing after April 16, 1999, Abratique was absent notwithstanding notices, orders, helped appellant transport huge amounts of marijuana to appellants rented room at No. 27 Dr.
and warrants of arrest. However, on April 27, 1999, or before the prosecution had formally Cario St., Baguio City and to appellants residence at Km. 6, Dontogan, Green Valley, Sto. Tomas,
offered its evidence, Abratique was brought to the trial court by the NBI. In its order of said date, Baguio City. He also declared on the witness stand that out of fear of being involved, he decided
the trial court pointed out that the prosecution could move to reopen the case for the taking of to divulge his knowledge of appellants possession of large caches of marijuana to the NBI. When
Abratiques testimony.[90] On May 7, 1999, the prosecution so moved, stressing that it had not yet the places referred to by Abratique were searched by the authorities, marijuana in staggering
formally offered its evidence and that the substantial rights of the accused would not be quantities was found and seized by the law enforcers. Stated plainly, the physical evidence in this
prejudiced inasmuch as the latter had yet to present his evidence. Appellant filed no opposition case corroborated Abratiques testimony on material points.
to the motion. The trial court granted the motion six days later. Plainly, there was nothing to Appellant imputes questionable motives to Abratique in an effort to discredit him. He
reopen, as the prosecution had not formally rested its case. Moreover, the taking of Abratiques demands that Abratique should likewise be prosecuted. However, by no means is the possible
testimony was not for the purpose of presenting additional evidence, but more properly for guilt of Abratique a tenable defense for appellant. Nor would Abratiques prosecution mean
the completion of his unfinished testimony. In U.S. vs. Base,[91] we held that a trial court is not in appellants absolution.
error, if it opts to reopen the proceedings of a case, even after both sides had rested and the In a prosecution for illegal possession of dangerous drugs, the following facts must be
case submitted for decision, by the calling of additional witnesses or recalling of witnesses so as proven with moral certainty: (1) that the accused is in possession of the object identified as
to satisfy the judges mind with reference to particular facts involved in the case. A judge cannot prohibited or regulated drug; (2) that such possession is not authorized by law; and (3) that the
be faulted should he require a material witness to complete his testimony, which is what accused freely and consciously possessed the said drug.[95]
happened in this case. It is but proper that the judges mind be satisfied on any and all questions We find the foregoing elements proven in Criminal Case No. 15800-R beyond reasonable
presented during the trial, in order to serve the cause of justice. doubt.
Appellants claim that the trial courts concession to reopen the case unduly prejudiced him In said case, the testimony of Abratique and the recovery of 591.81 kilograms of marijuana
is not well taken. We note that appellant had every opportunity to present his evidence to from appellants residence served to prove appellants possession of a prohibited drug. Tests
support his case or to refute the prosecutions evidence point-by-point, after the prosecution had conducted by the NBI forensic chemist proved the seized articles to be marijuana. These articles
rested its case. In short, appellant was never deprived of his day in court. A day in court is the were seized pursuant to a valid search warrant and hence, fully admissible in evidence.
touchstone of the right to due process in criminal justice. [92] Thus, we are unable to hold that a In People v. de los Reyes, 239 SCRA 439 (1994), we held that the Dangerous Drugs Act
grave abuse of discretion was committed by the trial court when it ordered the so-called applies generally to all persons and proscribes the sale of dangerous drugs by any person, and no
reopening in order to complete the testimony of a prosecution witness. person is authorized to sell such drugs. Said doctrine is equally applicable with respect to
3. On the Sufficiency of the Prosecutions Evidence possession of prohibited drugs. Republic Act No. 6425, which penalizes the possession of
In bidding for acquittal, appellant assails the credibility of Abratique as a witness. Appellant prohibited drugs, applies equally to all persons in this jurisdiction and no person is authorized to
insists that Abratiques testimony is profuse with lies, contrary to human nature, hence possess said articles, without authority of law.
incredible. According to appellant, Abratique was evasive from the outset with respect to certain Anent the third element, we have held that to warrant conviction, possession of illegal
questions of the trial court. He adds that it appeared the court entertained in particular the drugs must be with knowledge of the accused or that animus possidendi existed together with
suspicion that witness Abratique had conspired with appellant in committing the crime charged. the possession or control of said articles.[96] Nonetheless, this dictum must be read in
Appellant questions Abratiques motive in informing the NBI about his activities related to the consonance with our ruling that possession of a prohibited drug per se constitutes prima
marijuana taking, transfer, and warehousing. facie evidence of knowledge or animus possidendi sufficient to convict an accused absent a
31

satisfactory explanation of such possession.[97] In effect, the onus probandi is shifted to accused (P1,000,000.00) PESOS imposed on him is sustained. Appellant is likewise directed to pay the
to explain the absence of knowledge or animus possidendi[98] in this situation. costs of suit.
Appellant Modesto Tee opted not to testify in his defense. Instead, he presented his SO ORDERED.
mother as his lone witness, who testified on matters totally irrelevant to his case. We can only
conclude that, failing to discharge the burden of the evidence on the possession of prohibited
drug, appellants guilt in Criminal Case No. 15800-R was established beyond reasonable doubt.
3. On The Proper Penalty
Under Republic Act No. 6425 as amended by Republic Act No. 7659, the penalty
of reclusion perpetua to death and a fine ranging from five hundred thousand pesos
(P500,000.00) to ten million pesos (P10,000,000.00)[99] shall be imposed if the quantity of
marijuana involved in a conviction for possession of marijuana or Indian hemp shall be 750
grams or more.[100]
In the present case, the quantity of marijuana involved has been shown by the prosecution
to be far in excess of 750 grams, as stressed by the trial court:
The volume is rather staggering. It is almost one whole house or one whole room. In fact, when
they were first brought to the court, it took hours to load them on the truck and hours also to
unload them prompting the court to direct that the boxes and sack of marijuana be instead kept
at the NBI office in Baguio. And the identification of said marijuana during the trial was made in
the NBI premises itself by the witnesses since it was physically cumbersome and inconvenient to
keep bringing them to the court during every trial.[101]
In sentencing appellant to death, the trial court noted not only the huge quantity of
marijuana bales involved, but also the acts of accused of hiding them in different placesand
transferring them from place to place and making them appear as boxes of cigarettes to avoid
and evade apprehension and detection. They showed his being a big supplier, said the trial court,
[whose] criminal perversity and craft that deserve the supreme penalty of death.[102]
We are unable to agree, however, with the penalty imposed by the trial court. The
legislature never intended that where the quantity involved exceeds those stated in Section 20
of Republic Act No. 6425 the maximum penalty of death shall automatically be imposed. [103] The
statute prescribes two indivisible penalties: reclusion perpetua and death. Hence, the penalty to
be imposed must conform with Article 63[104] of the Revised Penal Code. As already held, the
death penalty law, Republic Act No. 7659 did not amend Article 63 of the Revised Penal
Code.[105] The rules in Article 63 apply although the prohibited drugs involved are in excess of the
quantities provided for in Section 20 of Republic Act No. 6425.[106] Thus, finding neither
mitigating nor aggravating circumstances in the present case, appellants possession of 591.81
kilograms of marijuana in Criminal Case No. 15800-R, does not merit capital punishment but only
the lesser penalty of reclusion perpetua.
The trial court imposed a fine on appellant in the sum of One Million Pesos
(P1,000,000.00), without subsidiary imprisonment in case of insolvency. The imposition of a fine
is mandatory in cases of conviction of possession of illegal drugs. This being within the limits
allowed by the law, the amount of the fine must be sustained. All these sanctions might not
remedy all the havoc wrought by prohibited drugs on the moral fiber of our society, especially
the youth.[107] But these penalties should warn peddlers of prohibited drugs that they cannot ply
their trade in our streets with impunity.
WHEREFORE, the decision of the Regional Trial Court of Baguio City, Branch 6, in Criminal
Case No. 15800-R, convicting appellant MODESTO TEE alias ESTOY TEE of violation of Section 8 of
Republic Act No. 6425, as amended, is AFFIRMED with the MODIFICATION that appellant is
hereby sentenced to suffer the penalty of reclusion perpetua. The fine of ONE MILLION
32

TAMBASEN VS PEOPLE 246 SCRA 184 therefore, should the money be found as having been earmarked for subversive activities, it
This is a petition for certiorari and prohibition under Rule 65 of the Revised Rules of Court to set should be confiscated pursuant to Article 45 of the Revised Penal Code.
aside the Order dated July 20, 1989 of the Regional Trial Court (RTC), Branch 44, Bacolod City in On July 20, 1989, RTC, Branch 44 issued an order granting the petition for certiorari and directing
Civil Case No. 5331, which nullified the order earlier issued by the Municipal Trial Circuit Court the clerk of court to return to the MTCC the money pending the resolution of the preliminary
(MTCC) of the City of Bacolod. The MTCC Order directed the return to petitioner of the amount investigation being conducted by the city prosecutor on the criminal complaint. In said order, the
of P14,000.00 which had been seized by the police. RTC held:
I The Court observed that private respondent Leon Tambasen never
On August 31, 1988, P/Sgt. Flumar Natuel applied for the issuance of a search warrant from the questioned the validity of the search warrant issued by respondent Judge
MTCC, alleging that he received information that petitioner had in his possession at his house at Demosthenes L. Magallanes. A perusal of private respondent's "Motion to
the North Capitol Road, Bacolod City, "M-16 Armalite Rifles (Mags & Ammos), Hand Grenades, Declare Search and Seizure Illegal and to Return Seized Properties" dated
.45 Cal. Pistols (Mags & Ammos), Dynamite Sticks and Subversive Documents," which articles October 7, 1988 shows that respondent Tambasen questions not the validity
were "used or intended to be used" for illegal purposes (Rollo, p. 14). On the same day, the of the search warrant issued by respondent Judge Demosthenes Magallanes,
application was granted by the MTCC with the issuance of Search Warrant No. 365, which but rather, the execution or implementation of the said warrant principally
allowed the seizure of the items specified in the application (Rollo, p. 15). on the ground that the articles seized are not allegedly mentioned in the
At around 6:30 P.M. of September 9, 1988, a police team searched the house of petitioner and search warrant. However, the question thus raised involves matters
seized the following articles: determinative of the admissibility in evidence and the legality of the articles
(1) Two (2) envelopes containing cash in the total amount of P14,000.00 (one seized. These matters, it is submitted, go beyond the immediate and limited
envelope P10,000.00 and another P4,000.00); jurisdiction of the respondent Judge to inquire into the validity of the search
(2) one (1) AR 280 handset w/antenae (sic) SN-00485; warrant he issued. These issues which relate exclusively or principally with
(3) one (1) YAESU FM Transceiver FT 23R w/Antenae (sic); the intrinsic and substantive merits of the case or cases which are being
(4) one (1) ALINCO ELH 230D Base; prepared against respondent Tambasen, and insofar as Tambasen is
(5) one (1) DC Regulator Supply 150 V. 13.8 V 12 AMP — 128 VAC; concerned involve matters of defense which should be properly raised at the
(6) one (1) brown Academy Notebook & Assorted papers; and criminal action or actions that may be filed against respondent Leon
(7) Four (4) handsets battery pack (Rollo, p. 16). Tambasen (see DOH v. Sy Chi Siong Co., Inc. et. al., G.R. No. 85289, Feb. 20,
On September 19, 1988, the MTCC, acting on petitioner's urgent motion for the return of the 1989). They cannot be addressed to the respondent Judge because the
seized articles, issued an order directing Sgt. Natuel to make a return of the search warrant. The respondent Judge has no jurisdiction over the said issue. It is clear therefore
following day, Sgt. Natuel submitted a report to the court. Not considering the report as a that respondent Judge has transcended the boundaries of his limited
"return in contemplation of law," petitioner filed another motion praying that Sgt. Natuel be jurisdiction and had in effect encroached upon the jurisdiction of the
required to submit a complete and verified inventory of the seized articles. Thereafter, Sgt. appropriate trial court or courts that will try the criminal case or cases
Natuel manifested that although he was the applicant for the issuance of the search warrant, he against respondent Leon Tambasen, in issuing the assailed order dated
was not present when it was served. December 23, 1988. Ostensibly, the assailed order, if not corrected, will
On October 7, 1988, petitioner filed before the MTCC a motion praying that the search and unduly deprive the prosecution of its right to present the evidence in
seizure be declared illegal and that the seized articles be returned to him. In his answer to the question and, consequently, will improperly oust the trial court, which will
motion, Lt. Col. Nicolas Torres, the station commander of the Bacolod City Police, said that the try the criminal case or cases against private respondent Leon Tambasen of
amount of P14,000.00 had been earmarked for the payment of the allowance of the Armed City its original and exclusive jurisdiction to rule on the admissibility and legality
Partisan (ACP) and other "known NPA personalities" operating in the City of Bacolod. of the said evidence. This order of respondent court is tantamount to a
On December 23, 1988, the MTCC issued an order directing Lt. Col. Torres to return the money denial of due process. It may be considered as a grave abuse of discretion
seized to petitioner. The court opined that in the implementation of the search warrant, any reviewable by certiorari (Esparagoza v. Tan, 94 Phil. 749) (Rollo, pp.
seizure should be limited to the specific items covered thereby. It said that the money could not 47-48).
be considered as "subversive documents"; it was neither stolen nor the effects of gambling. Consequently, petitioner filed the instant petition for certiorari and prohibition praying for the
Three months later, the Solicitor General filed before the RTC, Branch 44, Bacolod City a petition issuance of a temporary restraining order commanding the city prosecutor to cease and desist
for certiorariseeking the annulment of the order of the MTCC (Civil Case No. 5331). The petition from continuing with the preliminary investigation in BC I.S. No. 88-1239 and the RTC from taking
alleged that assuming that the seizure of the money had been invalid, petitioner was not entitled any step with respect to Civil Case No. 5331. He also prayed that Search Warrant No. 365 and the
to its return citing the rulings in Alih v. Castro, 151 SCRA 279 (1987) and Roan v. Gonzales, 145 seizure of his personal effects be declared illegal and that the Order of July 20, 1989 be reversed
SCRA 687 (1986). In those cases, the Court held that pending the determination of the legality of and annulled.
the seizure of the articles, they should remain in custodia legis. The petition also averred that a Petitioner contended that the search warrant covered three offenses: "(1) illegal possession of
criminal complaint for "any of the crimes against public order as provided under Chapter I, Title armalite rifle and .45 cal. pistol; (2) illegal possession of hand grenade and dynamite sticks; and
III of the Revised Penal Code" had been filed with the City Fiscal (BC I.S. No. 88-1239) and (3) illegal possession of subversive documents" (Rollo, pp. 3-4) in violation of Section 3 of Rule
33

126 of the Revised Rules of Court. He assailed the legality of the seizure of the articles which [1937]). As the Court aptly puts it inBagalihog v. Fernandez, 198 SCRA 614 (1991), "[z]eal in the
were not mentioned in the search warrant. Moreover, since a complaint against him was filed pursuit of criminals cannot ennoble the use of arbitrary methods that the Constitution itself
only after his house had been searched, petitioner claimed that the police were "on a fishing abhors."
expedition." For the retention of the money seized by the police officers, approval of the court which issued
During the pendency of the instant petition, a series of events related to the questioned search the search warrant is necessary (People v. Gesmundo, 219 SCRA 743 [1993]). In like manner, only
and seizure transpired. At around 10:30 P.M. of March 1, 1990, petitioner, who was then on the court which issued the search warrant may order their release (Temple v. Dela Cruz, 60 SCRA
board a passenger vehicle, was arrested by intelligence operatives in Barangay Mandalagan, 295 [1974]; Pagkalinawan v. Gomez, 21 SCRA 1275 [1967]).
Bacolod City and forthwith detained. On the strength of sworn statements of two rebel Section 3(2) of Article III of the 1987 Constitution provides that evidence obtained in violation of
returnees, the police filed a complaint for subversion against petitioner with the Office of the the right against unreasonable searches and seizures shall be inadmissible for any purpose in any
City Prosecutor. The following day, the City Prosecutor filed an information for violation of the proceeding.
Anti-Subversion Law against petitioner with RTC, Branch 42, Bacolod City (Criminal Case No. The information in Criminal Case No. 8517, with petitioner as the sole accused, was ordered
8517). An order for the arrest of petitioner was issued on March 2, 1990. quashed by the trial court and the prosecution's motion for the reconsideration of the quashal
On March 6, 1990, petitioner filed a motion to quash the information in Criminal Case No. 8517. order had been denied. Even in BC I.S. Case No. 88-1239, which was being investigated by
On March 15, 1990, RTC, Branch 42 granted petitioner's motion to quash and recalled the Assistant Provincial Prosecutor Marcos, petitioner was dropped as a respondent. Hence, there
warrant of arrest. The court also directed the City Prosecutor to resolve BC-I.S. Case No. 88-1239. appears to be no criminal prosecution which can justify the retention of the seized articles
On March 20, 1990, Assistant Provincial Prosecutor Gloria Lastimosa Marcos manifested before in custodia legis.
RTC, Branch 42 that petitioner had been "dropped" from BC-I.S. No. 88-1239. However, the City A subsequent legal development added another reason for the return to him of all the seized
Prosecutor had, by then, filed a motion for the reconsideration of said Resolution of March 15, articles: R.A. No. 1700, the Anti-Subversion Law, was repealed by R.A. No. 7636 and, therefore,
1990. The motion was denied. the crimes defined in the repealed law no longer exist.
Under this factual matrix, this Court is confronted with the question of whether RTC, Branch 44 WHEREFORE, the petition is GRANTED and the People of the Philippines is ORDERED to RETURN
gravely abused its discretion in directing that the money seized from petitioner's house, the money seized to petitioner.
specifically the amount of P14,000.00, be retained and kept in custodia legis. SO ORDERED.
On its face, the search warrant violates Section 3, Rule 126 of the Revised Rules of Court, which
prohibits the issuance of a search warrant for more than one specific offense. The caption of
Search Warrant No. 365 reflects the violation of two special laws: P.D. No. 1866 for illegal
possession of firearms, ammunition and explosives; and R.A. No. 1700, the Anti-Subversion Law.
Search Warrant No. 365 was therefore a "scatter-shot warrant" and totally null and void (People
v. Court of Appeals, 216 SCRA 101 [1992]).
Moreover, by their seizure of articles not described in the search warrant, the police acted
beyond the parameters of their authority under the search warrant. Section 2, Article III of the
1987 Constitution requires that a search warrant should particularly describe the things to be
seized. "The evident purpose and intent of the requirement is to limit the things to be seized to
those, and only those, particularly described in the search warrant — to leave the officers of the
law with no discretion regarding what articles they should seize, to the end that unreasonable
searches and seizures may not be made and that abuses may not be committed" (Corro v. Lising,
137 SCRA 541, 547 [1985]); Bache & Co. [Phil.] Inc. v. Ruiz, 37 SCRA 823 [1971]; Uy Kheytin v.
Villareal, 42 Phil. 886 [1920]). The same constitutional provision is also aimed at preventing
violations of security in person and property and unlawful invasions of the sanctity of the home,
and giving remedy against such usurpations when attempted (People v. Damaso, 212 SCRA 547
[1992] citing Alvero v. Dizon, 76 Phil. 637, 646 [1946]).
Clearly then, the money which was not indicated in the search warrant, had been illegally seized
from petitioner. The fact that the members of the police team were doing their task of pursuing
subversives is not a valid excuse for the illegal seizure. The presumption juris tantum of
regularity in the performance of official duty cannot by itself prevail against the constitutionally
protected rights of an individual (People v. Cruz, 231 SCRA 759 [1994]; People v. Veloso, 48 Phil.
169, 176 [1925]). Although public welfare is the foundation of the power to search and seize,
such power must be exercised and the law enforced without transgressing the constitutional
rights of the citizens (People v. Damaso, supra, citing Rodriguez v. Evangelista, 65 Phil. 230, 235
34

PEOPLE VS VELOSO advisable to set forth further facts, relating particularly to the search warrant, before passing to
This is an appeal from a judgment of the Court of First Instance of Manila finding the accused, the law.
Jose Ma. Veloso, guilty of the crime of resistance of the agents of the authority, in violation of There are found in the record the application for search warrant, the affidavit for search warrant,
article 252 of the Penal Code, and sentencing him to four months and one day and the search warrant. The application reads:
imprisonment, arresto mayor, with the accessory penalties, to pay a fine of P200, with the UNITED STATES OF AMERICA
corresponding subsidiary imprisonment in case of insolvency, and to pay the costs. The errors PHILIPPINE ISLANDS
assigned by counsel for the accused as appellant, go to the proposition that the resistance of the IN THE MUNICIPAL COURT OF THE CITY OF MANILA
THE PEOPLE OF THE PHILIPPINE ISLANDS, plaintiff, vs. JOHN DOE, Defendant.
police was justifiable on account of the illegality of the John Doe search warrant.
APPLICATION FOR (G)
In May, 1923, the building located at No. 124 Calle Arzobispo, City of Manila, was used by an SEARCH WARRANT
organization known as the Parliamentary Club. Jose Ma. Veloso was at that time a member of Testimony taken before Hon. L. Garduño, Judge, Municipal Court, Manila.
the House of Representative of the Philippine Legislature. He was also the manager of the club. Andres Geronimo, being duly sworn, testifies as follows:
The police of Manila had reliable information that the so-called Parliamentary Club was nothing Q. What is your name, residence and occupation? — A. Andres Geronimo, No. 47
more than a gambling house. Indeed, on May 19, 1923, J. F. Townsend, the chief of the gambling Revellin, detective.
squad, had been to the club and verified this fact. As a result, on May 25, 1923, Detective Andres Q. Are you the applicant of this search warrant? — A. Yes, sir.
Geronimo of the secret service of the City of Manila, applied for, and obtained a search warrant Q. Do you know the premises situated at No. 124 Calle Arzobispo, District of W. C., City
of Manila? — A. Yes. sir.
from Judge Garduño of the municipal court. Thus provided, the police attempted to raid the
Q. Do you know who occupies said premises? — A. I do not know. According to the
Parliamentary Club a little after three in the afternoon of the date above- mentioned. They best of my information the house is occupied by John Doe.
found the doors to the premises closed and barred. Accordingly, one band of police including Q . What are your reasons for applying for this search warrant? — A. It has been
policeman Rosacker, ascended a telephone pole, so as to enter a window of the house. Other reported to me by a person whom I consider to be reliable that in said premises there
policemen, headed by Townsend, broke in the outer door. are instruments and devices used in gambling games, such as cards, dice, chips, lottery
Once inside the Parliamentary Club, nearly fifty persons were apprehended by the police. One of tickets, lists of drawing and lists used in prohibited games kept. It has been reported to
them was the defendant Veloso. Veloso asked Townsend what he wanted, and the latter showed me by a person whom I consider to be reliable that there are or there will be gambling
him the search warrant. Veloso read it and told Townsend that he was Representative Veloso conducted in said premises. The aforesaid premises are known as gambling house. I
have watched the foregoing premises and believed it to be a gambling house and a
and not John Doe, and that the police had no right to search the house. Townsend answered that
place where instruments and devices used in gambling games, such as cards, dice,
Veloso was considered as John Doe. As Veloso's pocket was bulging, as if it contained gambling chips, lottery tickets, lists of drawing and lists used in prohibited games are kept.
utensils, Townsend required Veloso to show him the evidence of the game. About five minutes I, Andres Geronimo, being duly sworn, depose and say that I have read the foregoing questions
was consumed in conversation between the policemen and the accused the policemen insisting and answers and that I find the same to correct and true to the best of my knowledge and belief.
on searching Veloso, and Veloso insisting in his refusal to submit to the search. (Sgd.) ANDRES GERONIMO
At last the patience of the officers was exhausted. So policeman Rosacker took hold of Veloso Subscribed and sworn to before me this 25th day of May, 1923.
only to meet with his resistance. Veloso bit Rosacker in the right forearm, and gave him a blow in (Sgd.) L. GARDUÑO Judge, Municipal Court
another part of the body, which injured the policeman quite severely. Through the combined The affidavit and the search warrant are so nearly alike that it will suffice to copy the search
efforts of Townsend and Rosacker, Veloso was finally laid down on the floor, and long sheets of warrant alone. This document reads:
paper, of reglas de monte, cards, cardboards, and chips were taken from his pockets. UNITED STATES OF AMERICA
All of the persons arrested were searched and then conducted to the patrol wagons. Veloso PHILIPPINE ISLANDS
IN THE MUNICIPAL COURT OF THE CITY OF MANILA
again refused to obey and shouted offensive epithets against the police department. It was
THE PEOPLE OF THE PHILIPPINE ISLANDS, Plaintiff,
necessary for the policemen to conduct him downstairs. At the door, Veloso resisted so vs.
tenaciously that three policemen were needed to place him in the patrol wagon. 1awph!l.net JOHN DOE, Defendant.
In the municipal court of the City of Manila, the persons arrest in the raid were accused of SEARCH WARRANT (G)
gambling. All of them were eventually acquitted in the Court of First Instance for lack of proof, The People of the Philippine Islands, to any member of the
with the sole exception of Veloso, who was found guilty of maintaining a gambling house. This Police Force of the City of Manila.
case reached the appellate court where the accused was finally sentenced to pay a fine of P500. GREETING:
(No. 22163. 1 ) Proof by affidavit having this day been made before me by Andres Geronimo that he has good
reason to believe and does believe that John Doe has illegally in his possession in the building
The foregoing are the principal facts taken mainly from the findings of the trial judge, the
occupied by him and which is under his control, namely in the building numbered 124 Calle
Honorable Vicente Nepomuceno. Counsel for the appellant makes no effort to impugn these Arzobispo, City of Manila, Philippines Islands, certain devices and effects used in violation of the
findings, except that he stresses certain points as more favorable to the case of his client. The Gambling Law, to wit: money, cards, chips, reglas, pintas, tables and chairs and other utensils
defense, as previously indicated, is planted squarely on the contention that since the name of used in connection with the game commonly known as monte and that the said John Doe keeps
Veloso did not appear in the search warrant, but instead the pseudonym John Doe was used, and conceals said devices and effects with the illegal and criminal intention of using them in
Veloso had a legal right to resist the police by force. The nature of this defense makes it violation of the Gambling Law.
35

Now therefore, you are hereby commanded that at any time in the day or night within ten (10) Name and description of the accused should be inserted in the body of the warrant and
days on or after this date to make a search on the person of said John Doe and in the house where the name is unknown there must be such a description of the person accused as
situated at No. 124 Calle Arzobispo, City of Manila, Philippine Islands, in quest of the above will enable the officer to identify him when found.
described devices and effects and if you find the same or any part thereof, you are commanded to
xxx xxx xxx
bring it forthwith before me as provided for by law.
Given under my hand, this 25th day of May, 1923. Warrant for apprehension of unnamed party, or containing a wrong name for the party
(Sgd.) L. GARDUÑO to be apprehended is void, except in those cases where it contains a descriptio
Judge, Municipal Court personae such as will enable the officer to identify the accused.
Coming now to the legal aspects of the case it is first worthy of mention that by reason of the xxx xxx xxx
Fourth Amendment to the United States Constitution and the eleventh and eighteenth John Doe' Warrants. It follows, on principle, from what has already been said regarding
paragraphs of the Philippine Bill of Rights, as found in the present Organic Act, the security of the the essential requirements of warrants for the apprehension of persons accused, and
dwelling and the person is guaranteed. The organic act provides "that the right to be secured about blank warrants, that a warrant for the apprehension of a person whose true
against unreasonable searches and seizures shall not be violated." It further provides "that no name is unknown, by the name of "John Doe" or "Richard Roe," "whose other or true
warrant shall issue but upon probable cause, supported by oath or affirmation and particularly name in unknown," is void, without other and further descriptions of the person to be
describing the place to be searched and the person or things to be seized." apprehended, and such warrant will not justify the officer in acting under it. Such a
In the Philippine Code of Criminal Procedure are found provisions of the same import although warrant must, in addition, contain the best descriptio personae possible to be obtained
naturally entering more into detail. It is therein provided, among other things, that "a search of the person or persons to be apprehended, and this description must be sufficient to
warrant shall not issue except for probable cause and upon application supported by oath indicate clearly the proper person or persons upon whom the warrant is to be served;
particularly describing the place to be searched and the person of thing to be seized." (Section and should state his personal appearance and peculiarities, give his occupation and
97.) After the judge or justice shall have examined on oath the complainant and any witnesses place of residence, and any other circumstances by means of which he can be
he may produce, and shall have taken their depositions in writing (section 98), and after the identified.
judge or justice is satisfied of the existence of facts upon which the application is based, or that Person apprehended in act of committing a crime, under a "John Doe" warrant, on the
there is probable cause to believe that they exist, he must issue the warrant which must be other hand, the apprehension will not be illegal, or the officer liable, because under
substantially in the following form: such circumstances it is not necessary that a warrant should have been issued.
. . . You are, therefore, commanded, . . . to make immediate search on the person of The authority most often cited to sustain the text, and quoted with approval by the United
............................, or in the house situated ...................................... (describing it or any States Supreme Court, is the case of Commonwealth vs. Crotty ([1865], 10 Allen [Mass.], 403). It
other place to be searched with reasonable particularity, as the case may be) for the there appeared that one Peaslee had made a complaint to the police court Lee, charging that
following property: . . . ." (Section 99.) It is finally provided that "a person charged with "John Doe or Richard Roe, whose other or true name is to your complainant unknown," had
a crime may be searched for dangerous weapons or anything which may be used as committed an assault and battery upon him; upon which complaint a warrant was issued against
proof of the commission of the crime. (Section 105). "John Doe or Richard Roe, whose other or true name is to your complainant unknown, named in
A search warrant must conform strictly to the requirements of the constitutional and statutory the foregoing complaint." Neither the complaint nor the warrant contained any further
provisions under which it is issued. Otherwise it has rightly been held, must be absolutely legal, description or means of identification of the person to be arrested. Crotty resisted the arrest
"for there is not a description of process known to the law, the execution of which is more upon the ground that the warrant was invalid. Mr. Chief Justice Bigelow, as the organ of the
distressing to the citizen. Perhaps there is none which excites such intense feeling in Supreme Court of Massachusetts, said:
consequence of its humiliating and degrading effect." The warrant will always be construed We cannot entertain a doubt that the warrant on which the officer attempted to arrest one of the
defendant at the time of the alleged riot was insufficient, illegal and void. It did not contain the
strictly without, however, going the full length of requiring technical accuracy. No presumptions
name of the defendant, nor any description or designation by which he could be known and
of regularity are to be invoked in aid of the process when an officer undertakes to justify under identified as the person against whom it was issued. It was in effect a general warrant, upon
it. (24 R. C. L., pp. 711, et seq.; Reed vs. Rice [1829], 2 J. J. Marshall [Ky.] 44; 19 Am. Dec., 122; which any other individual might as well have been arrested, as being included in the description,
Smith vs. McDuffee [1914], 72 Ore., 276; Ann. Cas. 1916 D, 947.) as the defendant himself. Such a warrant was contrary to elementary principles, and in direct
The search warrant has been likened to a warrant of arrest. Although apprehending that there violation of the constitutional right of the citizen, as set forth in the Declaration of Rights, article
are material differences between the two, in view of the paucity of authority pertaining to John 14, which declares that every subject has a right to be secure from all unreasonable searches and
Doe search warrants we propose to take into consideration the authorities relied upon by the seizures of his person, and that all warrants, therefore, are contrary to this right, if the order in
appellant, thus following the precedent of Uy Kheytin vs. Villareal ([1920], 42 Phil., 886), where the warrant to a civil officer to arrest one or more suspected persons or to seize their property be
not accompanied with a special designation of the persons or objects of search, arrest or seizure.
the regularity of the issuance of the search warrant was also questioned.
This is in fact only a declaration of an ancient common law right. It was always necessary to
In the lower court, and again in this court, the attorneys for the defense quoted from Wharton's express the name or give some description of a party to be arrested on a warrant; and if one was
Criminal Procedure. In that text at pages 51, 52, 54, 55, and 56 of volume 1 of the Tenth Edition, granted with the name in blank, and without other designation of the person to be arrested, it
is found the following: was void. (1 Hale P. C. 577. 2 Ib. 119. Foster, 312. 7 Dane Ab. 248. 1 Chit. Crim. Law, 39. Mead vs.
Form and Sufficiency of Warrant. Technical accuracy is not required. . . . Haws, 7 Cow., 332, and cases cited.)
xxx xxx xxx
36

This rule or principle does not prevent the issue and service of a warrant against a party whose occupied by him at No. 124 Calle Arzobispo, City of Manila, and as this John Doe was Jose Ma.
name is unknown. In such case the best description possible of the person to be arrested is to be Veloso, the manager of the club, the police could identify John Doe as Jose Ma. Veloso without
given in the warrant; but it must be sufficient to indicate clearly on whom it is to be served, by difficulty.
stating his occupation, his personal appearance and peculiarities, the place of his residence, or
Again, it must be remembered that No. 124 Calle Arzobispo was supposed to be used for club
other circumstances by which he can be identified. (1 Chit. Crim. Law, 39, 40.)
The warrant being defective and void on its face, the officer had no right to arrest the person on purposes. It was not the home of Veloso; not the place of abode of the family, which the law
whom he attempted to serve it. He acted without warrant and was a trespasser. The defendant carefully protects in all of its sanctity. It was a club partially public in nature. It was, moreover, a
whom he sought to arrest had a right to resist by force, using no more than was necessary to camouflaged club with a high sounding name calculated to mislead the police, but intended for
resist the unlawful acts of the officer . . . nefarious practices. In a club of such a character, unlike in the home, there would commonly be
The defendants, therefore, in resisting the officer in making an arrest under the warrant in varying occupancy, a number of John Does and Richard Roes whose names would be unknown to
question, if they were guilty of no improper or excessive force or violence, did not do an unlawful the police.
act by lawful means, or a lawful act by unlawful means, and so could not be convicted of the It is also borne out by the authorities that, in defense of himself, any member of his family or his
misdemeanor of a riot, with which they are charged in the indictment.
dwelling, a man has a right to employ all necessary violence. But even in the home, and much
Appellant's argument, as based on these authorities, runs something like this. The law,
less so in a club or public place, the person sought to be arrested or to be searched should use
constitutional and statutory, requires that the search warrant shall not issue unless the
no more force than is necessary to repel the unlawful act of the officers. To authorize resistance
application "particularly" describe the person to be seized. A failure thus to name the person is
to the agents of the authority, the illegality of the invasion must be clearly manifest. Here, there
fatal to the validity of the search warrant. To justify search and arrest, the process must be legal.
was possibly a proper case for protest. There was no case for excessive violence to enforce the
Illegal official action may be forcibly resisted.
defendant's idea of a debatable legal question. (Commonwealth vs. Crotty, supra; People vs.
For the prosecution, however, as the arguments are advanced by the Attorney-General, and as
Chan Fook [1921], 42 Phil., 230; 3 Groizard, Codigo Penal, pp. 456, 457.)
the law was summarized by the trial judge, there is much to be said. Careful and logical reflection
The trial judge deduced from the searched warrant that the accused Veloso was sufficiently
brings forth certain points of paramount force and exercising a decisive influence. We will now
identified therein. Mention was made by his Honor of the code provision relating to a complaint
make mention of them by correlating the facts and the law.
or information, permitting a fictitious name to be inserted in the complaint or information, in
In the first place, the affidavit for the search warrant and the search warrant itself described the
lieu of the true name. The Attorney-General adds to this the argument that the police were
building to be searched as "the building No. 124 Calle Arzobispo, City of Manila, Philippine
authorized to arrest without a warrant since a crime was being committed. We find it
Islands." This, without doubt, was a sufficient designation of the premises to be searched. It is
unnecessary to comment on this contention.
the prevailing rule that a description of a place to be searched is sufficient if the officer with the
John Doe search warrants should be the exception and not the rule. The police should
warrant can, with reasonable effort, ascertain and identify the place intended. (Steele vs. U. S.
particularly describe the place to be searched and the person or things to be seized, wherever
[1925], U. S. Supreme Court Advance Opinions 1924-1925; 69 Law. ed., 757). The police officers
and whenever it is feasible. The police should not be hindered in the performance of their duties,
were accordingly authorized to break down the door and enter the premises of the building
which are difficult enough of performance under the best of conditions, by superficial adherence
occupied by the so-called Parliamentary Club. When inside, they then had the right to arrest the
to technicality or far fetched judicial interference.
persons presumably engaged in a prohibited game, and to confiscate the evidence of the
We agree with the trial judge and with the Attorney-General in their conclusions to the effect
commission of the crime. It has been held that an officer making an arrest may take from the
that the search warrant was valid, and that the defendant has been proved guilty beyond a
person arrested any money or property found upon his person, which was used in the
reasonable doubt, of the crime of resistance of the agents of the authority.
commission of the crime or was the fruit of the crime, or which may furnish the person arrested
The information alleges that at the time of the commission of the crime, the accused was a
with the means of committing violence or of escaping, or which may be used as evidence on the
member of the House of Representatives. The trial court was led to consider this allegation in
trial of the cause, but not otherwise. (Moreno vs. Ago Chi [1909], 12 Phil., 439.)
relation with the facts as an aggravating circumstance, and to sentence the accused accordingly.
Proceeding along a different line of approach, it is undeniable that the application for the search
We doubt, however, that advantage was taken by the offender of his public position when he
warrant, the affidavit, and the search warrant failed to name Jose Ma. Veloso as the person to be
resisted the officers of the law. The offender did not necessarily make use of the prestige of his
seized. But the affidavit and the search warrant did state that "John Doe has illegally in his
office as a means to commit a crime. Undoubtedly, Jose Ma. Veloso, as Juan de la Cruz, would
possession in the building occupied by him, and which is under his control, namely, in the
have resisted the police just as stoutly, as the Honorable Jose Ma. Veloso did. The penalty,
building numbered 124 Calle Arzobispo, City of Manila, Philippine Islands, certain devices and
accordingly, falls within the medium of that provided by the Penal Code.
effects used in violation of the Gambling Law." Now, in this connection, it must not be forgotten
Finding present no reversible error, agreeing in all respects with the findings of facts as made by
that the Organic Act requires a particular description of the place to be searched, and the person
the trial judge, and concurring with the trial judge in his legal conclusion, with one exception, it
or things to be seized, and that the warrant in this case sufficiently described the place and the
results that the judgment appealed from must be, as it is hereby, affirmed, with the sole
gambling apparatus, and, in addition, contained a description of the person to be seized. Under
modification that the defendant and appellant shall be sentenced to two months and one day
the authorities cited by the appellant, it is invariably recognized that the warrant for the
imprisonment, arresto mayor, with the costs of this instance against him. Let the corresponding
apprehension of an unnamed party is void, "except in those cases where it contains a description
order to carry this judgment into effect issue.
personae such as will enable the officer to identify the accused." The description must be
sufficient to indicate clearly the proper person upon whom the warrant is to be served. As the
search warrant stated that John Doe had gambling apparatus in his possession in the building
37

YOUSEF AL GHOUL VS CA imprisonment, when evidence of guilt is strong shall be admitted to bail regardless of the stage
Petitioners assail the decision[1] dated September 30, 1996, of the Court of Appeals, which of the criminal prosecution.[8]
affirmed the orders of the Regional Trial Court of Kalookan City, Branch 123, thereby dismissing As petitioners action before respondent appellate court also proved futile, petitioners filed
petitioners special civil action for certiorari.[2] the instant petition on the ground that it had acted with grave abuse of discretion tantamount to
The facts leading to the present petition under Rule 65 are as follows: lack or in excess of jurisdiction. They present for our consideration the following issues:
On March 31, 1995, Judge Geronimo S. Mangay, presiding judge of the Regional Trial I. WHETHER OR NOT THE EVIDENCE OFFERED BY THE PROSECUTION ARE
Court, National Capital Judicial Region, Branch 125, Kalookan City, issued search warrants 54- ADMISSIBLE;
95[3] and 55-95[4]for the search and seizure of certain items in Apartment No. 2 at 154 Obiniana II. WHETHER OR NOT ACCUSED HAVE THE RIGHT TO BAIL.[9]
Compound, Deparo Road, Kalookan City. The issue on bail has been resolved in our resolution dated November 24, 1998, where this
On April 1, 1995, the police searched Apartment No. 8, in the same compound and found Court ruled:
one (1) .45 caliber pistol. Found in Apartment No. 2 were: Consequent to the enactment of RA 8294, the penalty prescribed in Section 1 and 3 of P.D. 1866
2 M-16 rifles with 2 magazines and 20 live M-16 ammunitions for illegal possession of firearms, ammunitions and explosives under which petitioners were
1 Bar of demolition charge charged, has now been reduced to prision mayor in its minimum period and prision mayor in its
1 Caliber Pistol with no. 634 and other nos. were placed with magazine of Caliber .45 and 3 live maximum period to reclusion temporal, respectively. Evidently, petitioners are now entitled to
45 ammunitions bail as a matter of right prior to their conviction by the trial court pursuant to Section 4 of SC
1 22 Caliber handgun with 5 live ammunitions in its cylinder Administrative Circular No. 12-94 [10]
1 Box containing 40 pieces of .25 caliber ammunitions xxx
2 pieces of fragmentation grenade WHEREFORE, the petitioners motion is hereby GRANTED. The Temporary Restraining Order
1 roll of detonating cord color yellow issued by this Court in the Resolution of November 20, 1996 is hereby PARTIALLY LIFTED in so far
2 big bags of ammonium nitrate suspected to be explosives substance as petitioners pending motion for bail before the RTC of Kalookan City, Branch 123 is
22 detonating cords with blasting caps concerned. The trial court is hereby ordered to proceed with the hearing of the motion for bail
and pound of high explosives TNT and resolve the same with dispatch.[11]
1 timer alarm clock The issue that remains is whether the respondent court erred and gravely abused its
2 bags of suspected gun powder discretion when it ruled that the search and seizure orders in question are valid and the objects
2 small plastic bag of suspected explosive substance seized admissible in evidence.
1 small box of plastic bag of suspected dynamites Petitioners contend that the search and seizure orders violated Sections 2 and 3 of the Bill
One weighing scale of Rights[12] as well as Section 3 of Rule 126 of the Rules of Court on Criminal
Two (2) batteries 9 volts with blasting caps and detonating cord.[5] Procedure[13] because the place searched and articles seized were not described with
The firearms, ammunitions, explosives and other incendiary devices seized at the particularity. They argue that the two-witness requirement under Section 10 of Rule 126[14] was
apartments were acknowledged in the receipt signed by SPO2 Melanio de la Cruz. ignored when only one witness signed the receipt for the properties seized during the search,
Petitioners were charged before the Regional Trial Court of Kalookan City, Branch 123, in and said witness was not presented at the trial. Petitioners also aver that the presumption of
informations docketed as Criminal Cases Nos. C-48666-67, accusing them with illegal possession regularity of the implementation of the search warrant was rebutted by the defense during
of firearms, ammunitions and explosives, pursuant to Presidential Decree No. cross-examination of prosecution witnesses. According to petitioners, respondent court failed to
1866.[6] Thereafter, petitioners were arrested and detained. appreciate the fact that the items seized were not turned over to the police evidence custodian
Petitioners filed a motion for bail on May 24, 1995, the resolution of which was held in as required under Section 18 of the Department of Justice Circular No. 61 dated September 21,
abeyance by the RTC pending the presentation of evidence from the prosecution to determine 1993. Finally, they fault the lower courts finding that petitioners were in possession of the items
whether or not the evidence presented is strong.[7] allegedly confiscated from them.[15]
On February 7, 1996, at the hearing for bail, the RTC admitted all exhibits being offered for For the State, the Office of the Solicitor General avers that the search of Apartment 2 was
whatever purpose that they maybe worth after the prosecution had finished adducing its legal, and the items seized therein are admissible in evidence. However, the OSG agrees with
evidence despite the objection by the petitioners on the admissibility of said evidence. petitioners that the search warrants issued by the RTC, Branch 125, Kalookan City on March 31,
On February 19, 1996, the RTC denied petitioners motion for bail earlier filed, giving as 1995, namely search warrant 54-95[16] and search warrant 55-95,[17] specified the place to be
reasons the following: searched, namely Apartment No. 2, 154 Obiniana Compound, Deparo Road, Kalookan City. There
To begin with, the accused are being charged of two criminal offenses and both offenses under was no mention of Apartment No. 8. Thus, we find that the search conducted at Apartment No.
Presidential Decree 1866, Sections 1 and 3 thereof prescribe the penalty of Reclusion Temporal 8 clearly violated Sections 2 and 3 (2) of the Bill of Rights, in relation to Section 3 of Rule 126 of
in its maximum period to Reclusion Perpetua. Under Rule 114 of the Rules on Criminal Procedure the Rules of Court.
as amended by Supreme Court Administrative Circular No. 12-94, particularly Section 7 thereof, As held in PICOP v. Asuncion,[18] the place to be searched cannot be changed, enlarged nor
no person charged with a capital offense or an offense punishable by reclusion perpetua or life amplified by the police. Policemen may not be restrained from pursuing their task with vigor, but
in doing so, care must be taken that constitutional and legal safeguards are not
38

disregarded. Exclusion of unlawfully seized evidence is the only practical means of enforcing the 1. One (1) 5.56 M16 Rifle with corresponding ammunitions
constitutional injunction against unreasonable searches and seizures. Hence, we are constrained 2. One (1) 9MM Pistol with corresponding ammunitions
to declare that the search made at Apartment No. 8 is illegal and the .45 caliber pistol taken 3. Three (3) boxes of explosives
thereat is inadmissible in evidence against petitioners. 4. More or less ten (10) sticks of dymanites (sic)
Now, in contrast, the search conducted at Apartment No. 2 could not be similarly 5. More or less thirty (30) pieces of blasting caps pieces of detonating cords
faulted. The search warrants in question specifically mentioned Apartment No. 2. The search was and bring to this Court to be dealt with as the law may direct.[23]
done in the presence of its occupants, herein petitioners,[19] in accordance with Section 7 of Rule That the articles seized during the search of Apartment No. 2 are of the same kind and
126, Revised Rules of Court.[20] nature as those items enumerated in the search warrant above-quoted appears to us beyond
Petitioners allege lack of particularity in the description of objects to be seized pursuant to cavil. The items seized from Apartment No. 2 were described with specificity in the warrants in
the warrants. Hence, they also question the seizure of the following articles from Apartment No. question. The nature of the items ordered to be seized did not require, in our view, a technical
2, namely: description. Moreover, the law does not require that the things to be seized must be described
One M16 rifles, Colt AR-15 with 2 magazines and 20 rds ammo live in precise and minute details as to leave no room for doubt on the part of the searching
One (1) bar demolition charge authorities, otherwise, it would be virtually impossible for the applicants to obtain a search
One (1) .45 caliber pistol numbers were defaced with magazine and with three (3) live .45 cal warrant as they would not know exactly what kind of things they are looking for. [24] Once
ammos described, however, the articles subject of the search and seizure need not be so invariant as to
One (1) .22 caliber handgun with live ammos in its cylinder require absolute concordance, in our view, between those seized and those described in the
One (1) box containing (40) forty pieces of .22 cal. live ammos (magnum) warrant. Substantial similarity of those articles described as a class or species would suffice.
Two (2) pieces fragmentation grenade In People v. Rubio, 57 Phil. 384, 389 (1932), this Court said, While it is true that the
Two (2) magazines of M16 rifles with live ammos.[21] property to be seized under a warrant must be particularly described therein and no other
To appreciate them fully, we quote the search warrants in question: property can be taken thereunder, yet the description is required to be specific only in so far as
Search Warrant 54-95 the circumstances will ordinarily allow. Where by the nature of the goods to be seized, their
It appearing to the satisfaction of the undersigned, after examining under oath P/Sr Insp description must be rather general, it is not required that a technical description be given, as this
Joel D. Pagdilao, Chief, DPIU, OADDI NPDC, Applicant and his witness SPO1 Cesar R. Rivera would mean that no warrant could issue. As a corollary, however, we could not logically
of District Police Intelligence Unit, Northern Police District Command with postal address conclude that where the description of those goods to be seized have been expressed
c/o NPIU, NPDC, PNP, Langaray St., Kaunlaran Village, Kalookan City that a.k.a. AL, a.k.a. technically, all others of a similar nature but not bearing the exact technical descriptions could
GHUL a.k.a. NADI, a.k.a. NABIL and several John Does of Apartment Nr. 2, Nr. 154 Obiniana not be lawfully subject to seizure. Otherwise, the reasonable purpose of the warrant issued
Compound, Deparo Road, Kalookan City have in their possession and control the following: would be defeated by mere technicalities.
1. One (1) 45 Caliber Pistol The case of Bache and Co. (Phil.), Inc. v. Ruiz, 37 SCRA 823, 835 (1971), pointed out that
You are hereby commanded to make an immediate search anytime of the DAY and NIGHT of the one of the tests to determine the particularity in the description of objects to be seized under a
premises above-mentioned and forthwith, seize and take possession of the foregoing property, search warrant is when the things described are limited to those which bear direct relation to
to wit: the offense for which the warrant is being issued. A careful examination of Search Warrant Nos.
1. One (1) .45 Caliber Pistol 54-95[25] and 55-95[26] shows that they were worded in such a manner that the enumerated
and bring to this Court to be dealt with as the law may direct.[22] items to be seized could bear a direct relation to the offense of violation of Section 1 [27] and
Search Warrant 55-95 3[28] of Presidential Decree No. 1866, as amended, penalizing illegal possession of firearms,
It appearing to the satisfaction of the undersigned after examining under oath P/Sr. Insp. ammunitions and explosives. What the warrants authorized was the seizure of articles
Joel D. Pagdilao, Chief, DPIU, OADDI NPDC, Applicant and his witness SPO1 Cesar R. Rivera proscribed by that decree, and no other.
of District Police Intelligence Unit, Northern Police District Command with postal address Lastly, on this score, we note that the Court of Appeals ruled that petitioners waived their
c/o NPIU, NPDC, PNP, Langaray St., Kaunlaran Village, Kalookan City that a.k.a. AL, a.k.a. right to raise any attack on the validity of the search warrants at issue by their failure to file a
GHUL a.k.a. NADI, a.k.a. NABIL and several John Does of Apartment Nr. 2, Nr. 154 Obiniana motion to quash.[29]But, in conducting the search at Apartment No. 8, not just Apartment No. 2
Compound, Deparo Road, Kalookan City have in their possession and control the following: as ordered specifically in the search warrants, the police committed a gross violation we cannot
1. One (1) 5.56 M16 Rifle with corresponding ammunitions condone. Thus, we conclude that the gun seized in Apartment No. 8 cannot be used in evidence,
2. One (1) 9MM Pistol with corresponding ammunitions but those articles including guns, ammunitions, and explosives seized in Apartment No. 2 are
3. Three (3) boxes of explosives admissible in evidence.
4. More or less ten (10) sticks of dymanites (sic) Coming now to the two-witness requirement under Section 10, Rule 126 of the Revised
5. More or less thirty (30) pieces of blasting caps pieces of detonating cords Rules of Court, petitioners claim the rule was violated because only one witness signed the
You are hereby commanded to make an immediate search anytime of the DAY or NIGHT of receipt for the properties seized. For clarity, let us reproduce the pertinent section:
the premises above-mentioned and forthwith seize and take possession of the foregoing SEC. 10. Receipt for the property seized.The officer seizing property under the warrant must give
properties, to wit: a detailed receipt for the same to the lawful occupant of the premises in whose presence the
39

search and seizure were made, or in the absence of such occupant, must, in the presence of at
least two witnesses of sufficient age and discretion residing in the same locality, leave a receipt
in the place in which he found the seized property.
Clearly, the two-witness rule applies only in the absence of the lawful occupants of the
premises searched. In the case at bar, petitioners were present when the search and seizure
operation was conducted by the police at Apartment No. 2. More importantly, petitioner Nabeel
Al-Riyami y Nasser admitted being an actual occupant/resident of Apartment No. 2.[30] Hence, we
find here no violation of Section 10, Rule 126 of the Revised Rules of Court.
Petitioners contend that they could not be charged with violation of P.D. 1866 because the
seized items were not taken actually from their possession. This contention, however, cannot
prosper in the light of the settled rule that actual possession of firearms and ammunitions is not
an indispensable element for prosecution under P.D. No. 1866. In People v. Dela Rosa, 284 SCRA
158, 168-169 (1998), we clarified that the kind of possession punishable under P.D. 1866 is one
where the accused possessed a firearm either physically or constructively with animus
possidendi or intent to possess said firearm.Whether or not the evidence would show all the
elements of P.D. 1866 in this case is a different matter altogether. We shall not preempt issues
properly still within the cognizance of courts below.
Likewise, whether or not the articles seized were planted by the police, as claimed by the
petitioners, is a matter that must be brought before the trial court. In the same vein, petitioners
claim that the properties seized were not turned over to the proper police custodian is a
question of fact best ventilated during trial.
WHEREFORE, the petition is PARTIALLY GRANTED. The search conducted at Apartment No.
8 is hereby declared illegal and the item (.45 caliber pistol) seized therein inadmissible in
evidence.However, the search at Apartment No. 2 pursuant to Search Warrant 55-95 is hereby
declared valid and legal, and the articles seized from Apartment No. 2 are found admissible in
evidence. Let this case be remanded to the Regional Trial Court of Kalookan City, Branch 123, for
trial on the merits of Criminal Cases Nos. C-48666-67 with dispatch.
No pronouncement as to costs.
SO ORDERED.
40

UY VS BIR 7. He is executing this affidavit to attest under oath the veracity of the foregoing allegations and
Petitioners assail the validity of the warrants issued for the search of the premises of the he is reserving his right to claim for reward under the provisions of Republic Act No. 2338.
Unifish Packing Corporation, and pray for the return of the items seized by virtue thereof. On 1 October 1993, Nestor N. Labaria, Assistant Chief of the Special Investigation Branch of
On 30 September 1993, a certain Rodrigo Abos reported to the Bureau of Internal Revenue the BIR, applied for search warrants from Branch 28 of the Regional Trial Court of Cebu.The
(BIR) that petitioners Unifish Packing Corporation and Uy Chin Ho alias Frank Uy were engaged in application sought permission to search the premises of Unifish.
activities constituting violations of the National Internal Revenue Code. Abos, who claimed to be After hearing the depositions of Labaria and Abos, Judge Mercedes Gozo-Dadole issued the
a former employee of Unifish, executed an Affidavit[1] stating: disputed search warrants. The first[2] is docketed as SEARCH WARRANT NO. 93-10-79
1. He has personal knowledge that UNIFISH PACKING CORPORATION (hereinafter referred to as FOR: VIOLATION OF SECTION 253 ("Search Warrant A-1"), and consists of two pages. A verbatim
UNIFISH), a canning factory located at Hernan Cortes Street, under the active management of UY reproduction of Search Warrant A-1 appears below:
CHIN HO alias Frank Uy [,] is selling by the thousands of [sic] cartons of canned sardines without REPUBLIC OF THE PHILIPPINES
issuing receipt. This is in violation of Sections 253 and 263 of the Internal Revenue Code. REGIONAL TRIAL COURT OF CEBU
2. This grand scale tax fraud is perpetrated through the following scheme: 7th Judicial Region
(1) Uy Chin Ho a director of UNIFISH buys in bulk from the company; Branch 28
(2) Being a director, Uy Chin Ho has a lot of clout in the distribution of Mandaue City
the canned sardines processed by UNIFISH; THE PEOPLE OF THE PHILIPPINES,
(3) Uy Chin Ho dictates the value of canned sardines that he orders and Plaintiff,
buys from UNIFISH without any receipt of his purchases; - versus - SEARCH WARRANT NO. 93-10-79
(4) The moment he has the quantity he wants, UNIFISH through Uy FOR: VIOLATION OF SEC. 253
Chin Ho delivers to the different supermarkets such as White Gold, UY CHIN HO alias FRANK UY,
Gaisano, etc.; Unifish Packing Corporation
(5) Payments made by these tax evading establishments are made by Hernan Cortes St., Cebu City
checks drawn payable to cash and delivered to Uy Chin Ho; These x-------------------------/
payments are also not receipted (sic); (with sketch)
(6) Uy Chin Ho will then pay UNIFISH for the quantity of sardines he had SEARCH WARRANT
withdrawn from the corporation; TO ANY PEACE OFFICER:
3. Another fraudulent practice perpetrated by UNIFISH through Uy Chin Hos direction is the sale G R E E T I N G S:
of imported oil locally to different customers. This is a case of smuggling in the sense that It appearing to the satisfaction of the undersigned, after examination underoath (sic), Nestor N.
UNIFISH, being an export company registered with the Board of Investments, is enjoying certain Labaria, Asst. Chief, Special Investigation Branch, BIR and witness Rodrigo Abos that there is a
exemptions in their importation of oil as one of the raw materials in its processing of canned (sic) probable cause to believe that the crime of violation of Section 253 - attempt to evade or
tuna for export. These tax exemptions are granted by the government on the condition that the defeat the tax has been committed and there is good and sufficient reason to believe that Uy
oil is to be used only in the processing of tuna for export and that it is not to be sold unprocessed Chin Ho c/o Unifish Packing Corporation, Hernan Cortes St., Mandaue City has in his possession,
as is to local customers. care and control, the following:
4. Another fraudulent practice involves the sales of unused cans; UNIFISH also enjoys tax 1. Multiple sets of Books of Accounts; Ledgers, Journals, Columnar Books, Cash
exemptions in its purchases of tin cans subject to the condition that these are to be used as Register Books, Sales Books or Records; Provisional & Official Receipts;
containers for its processed tuna for export. These cans are never intended to be sold locally to 2. Production Record Books/Inventory Lists [,] Stock Cards;
other food processing companies. 3. Unregistered Delivery Receipts;
5. Prior to 1990, that is from 1980 to 1990, the factory of the UNIFISH PACKING CORPORATION 4. Unregistered Purchase & Sales Invoices;
was then run by the PREMIER INDUSTRIAL & DEVELOPMENT CORPORATION (hereinafter 5. Sales Records, Job Order;
referred to as PREMIER) [,] which corporation was being controlled by the same majority 6. Corporate Financial Records; and
stockholders as those now running and controlling UNIFISH; [a]t that time, PREMIER was also 7. Bank Statements/Cancelled Checks
committing the same fraudulent acts as what is being perpetrated by UNIFISH at present. You are hereby commanded to make an immediate search at any time of day or night of said
6. The records containing entries of actual volume of production and sales, of both UNIFISH AND premises and its immediate vicinity and to forthwith seize and take possession of the articles
PREMIER, are found in the office of the corporation at its factory site at H. Cortes Street, above-mentioned and other properties relative to such violation and bring said properties to the
Mandaue City.The particular place or spot where these records [official receipts, sales invoices, undersigned to be dealt with as the law directs.
delivery receipts, sales records or sales books, stock cards, accounting records (such as ledgers, WITNESS MY HAND this 1st day of October, 1993.
journals, cash receipts books, and check disbursements books)] are kept and may be found is (sgd.)
best described in the herein attached sketch of the arrangement of the offices furniture and MERCEDES GOZO-DADOLE
fixture of the corporation which is made an integral part hereof and marked as Annex A, Judge
41

The second warrant[3]is similarly docketed as SEARCH WARRANT 93-10-79 FOR: VIOLATION unregistered delivery receipts and/or sales invoices), Search Warrant B is a verbatim
OF SEC. 253 ("Search Warrant A-2"). Search Warrant A-2, reproduced below, is almost identical reproduction of Search Warrant A-2.
in content to Search Warrant A-1, save for the portions indicated in bold print. It consisted of On the strength of these warrants, agents of the BIR, accompanied by members of the
only one page. Philippine National Police, on 2 October 1993, searched the premises of the Unifish Packing
REPUBLIC OF THE PHILIPPINES Corporation. They seized, among other things, the records and documents of petitioner
REGIONAL TRIAL COURT OF CEBU corporation. A return of said search was duly made by Nestor Labaria with the RTC of Cebu ,
7th Judicial Region Branch 28.
Branch 28 On 8 February 1995, the BIR filed against petitioners a case before the Department of
Mandaue City Justice. The records, however, do not reveal the nature of this case.
THE PEOPLE OF THE PHILIPPINES, On 31 March 1995, petitioners filed motions to quash the subject search warrants with
Plaintiff, Branch 28 of the Cebu RTC.
- versus - SEARCH WARRANT NO. 93-10-79 The RTC, however, denied petitioners' motions to quash as well as their subsequent
FOR: VIOLATION OF SEC. 253 motion for reconsideration, prompting petitioners to file a petition for certiorari with the Court
UY CHIN HO alias FRANK UY, and of Appeals (CA). The CA dismissed their petition, holding that petitioners failed to comply with
Unifish Packing Corporation Section 2(a), Rule 6 of the Revised Internal Rules of the Court of Appeals (RIRCA), which states:
Hernan Cortes St., Mandaue City a. What Should be Filed. - The petition shall be filed in seven (7) legible copies and a copy thereof
x-------------------------/ shall be served on each of the respondents, and must be accompanied by a certified true copy of
(with sketch) the decision or order complained of and true copies of the pleadings and other pertinent
SEARCH WARRANT documents and papers. (As amended by S.Ct. Res., dated November 24, 1992).
TO ANY PEACE OFFICER: The CA found that petitioners did not submit certified true copies of (1) the Motions to Quash,
G R E E T I N G S: (2) the Motion for Reconsideration, and (3) the Affidavit of Rodrigo Abos.
It appearing to the satisfaction of the undersigned, after examination underoath [sic], Nestor N. The CA also held that certiorari was not the proper remedy to question the resolution
Labaria, Asst. Chief, Special Investigation Branch, BIR and witness Rodrigo Abos that there is a denying the motion to quash.
[sic] probable cause to believe that the crime of violation of Section 253 - attempt to evade or In this case now before us, the available remedies to the petitioners, assuming that the
defeat the tax has been committed and there is good and sufficient reason to believe that Uy Department of Justice will eventually file the case, are: a petition for reinvestigation; the right to
Chin Ho alias Frank Uy andUnifish Packing Corporation, Hernan Cortes St., Mandaue City has in post bail; a Motion to Quash the Information; and in case of denial, an appeal, after judgment on
his possession, care and control, the following: the merits, or after the case shall have been tried. This brings us to the case of Lai vs.
1. Multiple sets of Books of Accounts; Ledgers, Journals, Columnar Books, Cash Intermediate 220 SCRA 149 and the pronouncement, thus:
Register Books, Sales Books or Records; Provisional & Official Receipts; Criminal Procedure: Certiorari: Certiorari should not be allowed where petitioner has other
2. Production Record Books/Inventory Lists [,] Stock Cards; remedies available. -- Anent the remedy resorted to by petitioners (referring to the petition for
3. Unregistered Delivery Receipts; certiorari) from the Regional Trial Court of Negros Oriental presided by Judge Diez, the same
4. Unregistered Purchase & Sales Invoices; should not have been granted. Petitioners were not without plain, speedy and adequate
5. Sales Records, Job Order; remedies in the ordinary course of law against Judge Lomeda's order for their arrest. These
6. Corporate Financial Records; and remedies are as enumerated by respondent appellate court in its decision: "1. they can post bail
7. Bank Statements/Cancelled Checks for their provisional release; 2. They can ask the Provincial Fiscal for a reinvestigation of the
You are hereby commanded to make an immediate search at any time of day or night of said charge against them. If unsatisfied with the fiscal's resolution they can ask for a review by the
premises and its immediate vicinity and to forthwith seize and take possession of the articles Minister of Justice; (Sec. 1(), RA 5180 as amended by P.D.
above-mentioned and other properties relative to such violation and bring said properties to the 911); 3. if their petitionfor review does not prosper, they can file a motion to quash the informati
undersigned to be dealt with as the law directs. on in the trial court. (Rule 117, Rules of
WITNESS MY HAND this 1st day of October, 1993. Court). 4. If the motion is denied, they can appeal the judgment of the court after the caseshall h
(sgd.) ave been tried on the merits.
MERCEDES GOZO-DADOLE x x x Where motion to quash is denied, remedy is not certiorari, but to go to trial.-- Moreover, in
Judge the case of Acharon vs. Purisima, this Court held
Judge Gozo-Dadole issued a third warrant,[4] which was docketed as SEARCH WARRANT 93- that when a motion to quash a criminal case is denied, theremedy is not certiorari but to go to tri
10-80 FOR: VIOLATION OF SEC. 238 in relation to SEC. 263 (hereinafter, "Search Warrant al without prejudice to reiterating the special defenses involved in said Motion. In the event that
B"). Except for the docket number and the designation of the crime in the body of the warrant an adverse decision is rendered after trial on the merits, an appealtherefrom should be the next
(Section 238 in relation to Sec. 263 - non-issuance of sales invoice and use and possession of legal step.
xxx
42

In this case now before Us, there is no pretention [sic] that the Court issued the Search Warrants describing the place to be searched and the persons or things to be seized; and that no search
without jurisdiction. On the contrary, it had jurisdiction. The argument therefore that the Court warrant shall issue for more than one specific offense.
committed an error in not describing the persons or things to be searched; that the Search The utter disregard by Judge de Guzman of the requirements laid down by the said rule renders
Warrants did not describe with particularity the things to be seized/taken; the absence of the warrant in question absolutely null and void. It has been held that where the order
probable cause; and for having allegedly condoned the discriminating manner in which the complained of is a patent nullity, a petition for certiorari and mandamus may properly be
properties were taken, to us, are merely errors in the Court's finding, certainly not correctible by entertained despite the existence of the remedy of appeal.
certiorari, but instead thru an appeal.[5] Moreover, an appeal from the order of Judge de Guzman would neither be an adequate nor
In any event, the CA ruled, no grave abuse of discretion amounting to lack of jurisdiction speedy remedy to relieve appellee of the injurious effects of the warrant. The seizure of her
was committed by the RTC in the issuance of the warrants. personal property had resulted in the total paralization of the articles and documents which had
As petitioners' motion for reconsideration proved futile, petitioners filed the instant been improperly seized. Where the remedy of appeal cannot afford an adequate and expeditious
petition for review. relief, certiorari can be allowed as a mode of redress to prevent irreparable damage and injury to
Petitioners claim that they did submit to the CA certified true copies of the pleadings and a party.
documents listed above along with their Petition, as well as in their Motion for This Court had occasion to reiterate the above pronouncement in Silva vs.
Reconsideration.An examination of the CA Rollo, however, reveals that petitioners first Presiding Judge, RTC of Negros Oriental, Br. XXXIII,[10] which also involved a special civil action
submitted the same in their Reply, after respondents, in their Comment, pointed out petitioners forcertiorari:[11]
failure to attach them to the Petition. Thus, in issuing a search warrant, the judge must strictly comply with the constitutional
Nevertheless, the CA should not have dismissed the petition on this ground although, to its requirement that he must determine the existence of probable cause by examining the applicant
credit, it did touch upon the merits of the case. First, it appears that the case could have been and his witnesses in the form of searching questions and answers. His failure to comply with this
decided without these pleadings and documents. Second, even if the CA deemed them essential requirement constitutes grave abuse of discretion. As declared in Marcelo vs. De Guzman, G.R.
to the resolution of the case, it could have asked for the records from the RTC. Third, in a similar No. L-29077, June 29, 1982, 114 SCRA 657, the capricious disregard by the judge in not
case,[6] we held that the submission of a document together with the motion for reconsideration complying with the requirements before issuance of search warrants constitutes grave abuse of
constitutes substantial compliance with Section 3, Rule 46 of the Rules of Court, requiring the discretion.
submission of a certified true copy of material portions of the record as are referred to [in the In this case, petitioners alleged in their petition before the CA that the issuing judge
petition], and other documents relevant or pertinent thereto along with the petition.So should it violated the pertinent provisions of the Constitution and the Rules of Court in issuing the
be in this case, especially considering that it involves an alleged violation of a constitutionally disputed search warrants, which, if true, would have constituted grave abuse of
guaranteed right. The rules of procedure are not to be applied in a very rigid, technical sense; discretion. Petitioners also alleged that the enforcers of the warrants seized almost all the
rules of procedure are used only to help secure substantial justice. If a technical and rigid records and documents of the corporation thus resulting in the paralysis of its business. Appeal,
enforcement of the rules is made, their aim could be defeated.[7] therefore, would not be an adequate remedy that would afford petitioners expeditious relief.
The CA likewise erred in holding that petitioners cannot avail of certiorari to question the We now proceed to the merits of the case.
resolution denying their motions to quash the subject search warrants. We note that the case of Section 2, Article III of the Constitution guarantees the right of the people against
Lai vs. Intermediate, cited by the appellate court as authority for its ruling does not appear in 220 unreasonable searches and seizures:
SCRA 149. The excerpt of the syllabus quoted by the court, as observed by petitioners, [8]appears The right of the people to be secure in their persons, houses, papers, and effects against
to have been taken from the case of Yap vs. Intermediate Appellate Court, 220 unreasonable searches and seizures of whatever nature and for any purpose shall be inviolable,
SCRA 245 (1993). Yap, however, is inapplicable since that case involved a motion to quash and no search warrant or warrant of arrest shall issue except upon probable cause to be
acomplaint for qualified theft, not a motion to quash a search warrant. determined personally by the judge after examination under oath or affirmation of the
The applicable case is Marcelo vs. De Guzman,[9] where we held that the issuing judges complainant and the witnesses he may produce, and particularly describing the place to be
disregard of the requirements for the issuance of a search warrant constitutes grave abuse of searched and the persons or things to be seized.
discretion, which may be remedied by certiorari: In relation to the above provision, Rule 126 of the Rules of Court provides:
Expressly announced in Section 1, Rule 65 of the Rules of Court is the general rule that certiorari SEC. 3. Requisite for issuing search warrant. - A search warrant shall not issue but upon probable
is available where a tribunal or officer exercising judicial functions has acted without or in excess cause in connection with one specific offense to be determined personally by the judge after
of its or his jurisdiction, or with grave abuse of discretion and there is no appeal, nor any plain, examination under oath or affirmation of the complainant and the witnesses he may produce,
speedy, and adequate remedy in the ordinary course of law. and particularly describing the place to be searched and the things to be seized.
In the light of the findings of the lower court, herein above quoted, it is indisputable that Judge SEC. 4. Examination of complainant; record. - The judge must, before issuing the warrant,
de Guzman gravely abused his discretion in issuing the said search warrant. Indeed, he acted personally examine in the form of searching questions and answers, in writing and under oath
whimsically and capriciously when he ignored the explicit mandate of Section 3, Rule 126 of the the complainant and any witnesses he may produce on facts personally known to them and
Rules of Court that a search warrant shall not issue but upon probable cause in connection with attach to the record their sworn statements together with any affidavits submitted.
one specific offense to be determined by the municipal or city judge after examination under A search warrant must conform strictly to the requirements of the foregoing constitutional
oath or affirmation of the complainant and the witnesses he may produce, and particularly and statutory provisions. These requirements, in outline form, are:
43

(1) the warrant must be issued upon probable cause; Inconsistencie


(2) the probable cause must be determined by the judge himself and not by the s in the
applicant or any other person; description of
(3) in the determination of probable cause, the judge must examine, under oath or the persons
affirmation, the complainant and such witnesses as the latter may produce; and named in the
(4) the warrant issued must particularly describe the place to be searched and two warrants
persons or things to be seized.[12] Petitioners also find fault in the description of the names of the persons in Search
The absence of any of these requisites will cause the downright nullification of the search Warrants A-1 and A-2. Search Warrant A-1 was issued solely against Uy Chin Ho alias Frank Uy.
warrants.[13] The proceedings upon search warrants must be absolutely legal, for there is not a Search Warrant A-2, on the other hand, was directed against UY CHIN HO alias FRANK
description of process known to the law, the execution of which is more distressing to the UY, and Unifish Packing Corporation.
citizen. Perhaps there is none which excites such intense feeling in consequence of its These discrepancies are hardly relevant.
humiliating and degrading effect. The warrants will always be construed strictly without, In Miller v. Sigler,[21] it was held that the Fourth Amendment of the United States
however, going the full length of requiring technical accuracy. No presumptions of regularity are Constitution, from which Section 2, Article III of our own Constitution is historically derived, does
to be invoked in aid of the process when an officer undertakes to justify under it.[14] not require the warrant to name the person who occupies the described premises. Where the
Petitioners contend that there are several defects in the subject warrants that command search warrant is issued for the search of specifically described premises only and not for the
their nullification. They point out inconsistencies in the description of the place to be searched in search of a person, the failure to name the owner or occupant of such property in the affidavit
Search Warrant A-1, as well as inconsistencies in the names of the persons against whom Search and search warrant does not invalidate the warrant; and where the name of the owner of the
Warrants A-1 and A-2 were issued. That two search warrants (Search Warrants A-1 and A-2) premises sought to be searched is incorrectly inserted in the search warrant, it is not a fatal
were issued for the same crime, for the same place, at a single occasion is cited as another defect if the legal description of the premises to be searched is otherwise correct so that no
irregularity. Petitioners also dispute the existence of probable cause that would justify the discretion is left to the officer making the search as to the place to be searched. [22]
issuance of the warrants. Finally, they claim that the things to be seized were not described with Since, in the case at bar, the warrant was issued not for search of the persons owning or
particularity. These defects, according to petitioners, render the objects seized inadmissible in occupying the premises, but only a search of the premises occupied by them, the search could
evidence.[15] not be declared unlawful or in violation of the constitutional rights of the owner or occupants of
Inconsistencie the premises, because of inconsistencies in stating their names. [23]
s in the Two
description of warrants
the place to be issued at
searched one time for
Petitioners observe that the caption of Search Warrant A-1 indicates the address of Uy one crime
Chin Ho alias Frank Uy as Hernan Cortes St., Cebu City while the body of the same warrant states and one
the address as Hernan Cortes St., Mandaue City. Parenthetically, Search Warrants A-2 and B place
consistently state the address of petitioner as Hernan Cortes St., Mandaue City. In any event, Search Warrant A-1 should be deemed superseded by Search Warrant A-2.
The Constitution requires, for the validity of a search warrant, that there be a particular Two warrants, Search Warrants A-1 and A-2, were actually issued by the trial court for the
description of the place to be searched and the persons of things to be seized.[16] The rule is that same crime (violation of SEC. 253 of the National Internal Revenue Code). It appears, however,
a description of a place to be searched is sufficient if the officer with the warrant can, with that Search Warrant A-2 was issued merely to correct the inconsistencies in the address in
reasonable effort, ascertain and identify the place intended[17]and distinguish it from other Search Warrant A-1, as well as to include Unifish Packing Corporation as a party against whom
places in the community.[18] Any designation or description known to the locality that points out the warrant was issued. Search Warrant A-2 was evidently an attempt by the issuing judge to be
the place to the exclusion of all others, and on inquiry leads the officers unerringly to it, satisfies more precise in the names of the persons against whom the warrant was issued and in the
the constitutional requirement.[19] Thus, in Castro vs. Pabalan,[20] where the search warrant description of the place to be searched. Indeed, it would be absurd for the judge to issue on a
mistakenly identified the residence of the petitioners therein as Barrio Padasil instead of the single occasion two warrants authorizing the search of a single place for a single
adjoining Barrio Maria Cristina, this Court "admitted that the deficiency in the writ is not of offense. Inasmuch as the apparent intent in issuing Search Warrant A-2 was to supersede Search
sufficient gravity to call for its invalidation." Warrant A-1, the latter should be deemed revoked by the former.
In this case, it was not shown that a street similarly named Hernan Cortes could be found The alleged
in Cebu City. Nor was it established that the enforcing officers had any difficulty in locating the absence of
premises of petitioner corporation. That Search Warrant A-1, therefore, inconsistently identified probable cause
the city where the premises to be searched is not a defect that would spell the warrants Petitioners claim there was no probable cause for Judge Gozo-Dadole to issue the subject
invalidation in this case. search warrants.
44

Probable cause is defined as such facts and circumstances which would lead a reasonably A Yes.
discreet and prudent man to believe that an offense has been committed and that the objects Q How is it done?
sought in connection with the offense are in the place sought to be searched.[24] A As an officer, he is an active member of the corporation who is at the same time making
In the determination of probable cause, the Constitution and the Rules of Court require an his authority as appointing himself as the distributor of the company's products. He
examination of the witnesses under oath. The examination must be probing and exhaustive, not sells these products thru supermarkets in Visayas and Mindanao, in fact, the whole
merely routine or pro forma. The examining magistrate must not simply rehash the contents of Philippines. He makes it appear that it is the company which is selling when actually it
the affidavit but must make his own inquiry on the intent and justification of the is him selling the goods and he does not issue any invoices.
application.[25] Asking of leading questions to the deponent in an application for search warrant, Q Since he does not issue any invoices, how is it done?
and conducting of examination in a general manner, would not satisfy the requirements for A Thru delivery receipts.
issuance of a valid search warrant.[26] Q Is the delivery receipt official?
The witnesses, in turn, must testify under oath to facts of their own personal A No. It is unregistered.
knowledge. The oath required must refer to the truth of the facts within the personal knowledge Q For how long has this been going on?
of the petitioner or his witnesses, because the purpose thereof is to convince the committing A As far as I know, it is still in 1986 since we started producing the sardines.
magistrate, not the individual making the affidavit and seeking the issuance of the warrant, of Q When was the last time that you observed that that is what he is doing?
the existence of probable cause.[27] Search warrants are not issued on loose, vague or doubtful A August, 1993, last month.
basis of fact, nor on mere suspicion or belief.[28] Q How did you happen to know about this last month?
It may be recalled that before issuing the warrants, the judge deposed two witnesses, A Because he delivered to certain supermarkets and the payments of that supermarket did
namely, Nestor Labaria of the BIR, and Rodrigo Abos, who claimed to be an old employee of not go directly to the company. It went to him and he is the one who paid the company
Unifish. Petitioners claim that the testimonies of Labaria and Abos are hearsay. We agree with for the goods that he sold.
this contention, but only as to the testimony of Labaria, who stated during the examination: Q Can you tell this Court the name of that certain supermarkets?
Q. Do you know of a certain Uy Chin Ho alias Frank Uy? A White Gold and Gaisano.
A. No. Q How did you know this fact?
Q. Do you know his establishment known as Unifish Packing Corporation? A As a manager of the company I have access to all the records of that company for the last
A. I have only heard of that thru the affidavit of our informer, Mr. Abos. three years. I was the Operating Chief.
Q. Why are you applying for search warrant in the premises of Unifish Packing Corporation? Q Until now?
A. Because of that information we received that they are using only delivery receipts instead A No. I was separated already.
of the legal sales invoices. It is highly indicative of fraud. Q When?
Q. From where did you get that information? A August, 1993.
A. From our informer, the former employee of that establishment.[29] Q How does he do this manipulation?
The above portion of the transcript shows that Labarias knowledge of the alleged illegal A He sells the goods to the supermarkets afterwhich the company, Unifish will deliver to his
activities of petitioners was acquired not through his own perception but was merely supplied by customers, then his customers will pay directly to him and in turn, he pays to the
Abos. Therefore, the deposition of Labaria, which is based on hearsay, standing alone, cannot company.
justify the issuance of the search warrants.[30] Q And these transactions, were they reflected in their books of account or ledger or
The application for the warrants, however, is not based solely on Labarias deposition but is whatever?
supported by that of Abos, whose knowledge of petitioners alleged illegal practices was A It is written but it is supposed to be a secret transaction. It is not for the public, not for the
apparently obtained during his employment with Unifish. In his deposition, Abos detailed the BIR but it is only for the purpose of keeping the transactions between the company
schemes employed by Frank Uy and Unifish to evade the payment of taxes, and described the and him. It is not made to be shown to the BIR.
place where the documents supposedly evidencing these schemes were located: Q In that books of account, is it reflected that they have made some deliveries to certain
Q Do you know Frank Uy? supermarkets?
A Yes. A Yes.
Q Why do you know him? Q For the consumption of the BIR what are the papers that they show?
A Because I were (sic) an employee of his from 1980 until August of 1993. A It is the private accounting firm that prepares everything.
Q Where is this Unifish Packing Corporation located? Q Based on what?
A Hernan Cortes St. A Based on some fictitious records just as they wish to declare.
Q What is it being engaged of? Q In your affidavit you stated that there are sales invoices, official receipts, delivery receipts,
A It is engaged in canning of fish. sales records, etc. These documents are records that you have stated, in your affidavit,
Q You have executed an affidavit here to the effect that it seems that in his business dealings which are only for the consumption of the company?
that he is actually doing something that perpetrated tax evasion. Is that correct? A Yes, not for the BIR.
45

Q Where are they kept now? Q In other words, the company imports soya oil supposedly to be used as a raw material but
A They are kept on the table which I have drawn in the sketch. This is the bird's eyeview (sic) instead they are selling it locally?
of the whole office. When you enter thru the door this Gina Tan is the one recording all A Yes. ([W]itness showing DR No. 3053 dated November 13, 1991.) This delivery receipt was
the confidential transactions of the company. In this table you can find all the ledgers the delivery receipt to Celebes Canning Corp. of the 90 grams soya oil.
and notebooks. Q In other words, this soya oil should have to be used by Unifish but instead they are seeling
Q This sketch is a blow-up of this portion, Exh. "A"? (sic) it?
A Yes. Exh. "B" is the blow-up of Exh. "A" inside the office. A Yes, at a profit.
In this blow-up there are four personnel plus one new personnel. Gina Tan collects all the Q You also said that there is tax evasion in the selling of cans. What do you mean by this?
records from this girl and this girl makes the statements. This first girl delivers the A There is another privileged [sic] by the BOI for a special price given to packaging
receipts. The second girl prepares the bill of lading. The third girl keeps the inventory materials. When you export the product there is a 50% price difference. Now, taking
of all the stocks. that advantage of that exemption, they sold it to certain company here, again to
This sketch here is the bodega where the records are kept. The records from these people Virginia Farms.
are stored in this place which is marked as "C". Q Do you have proof to that effect?
Q So what you want to impress on that now is that only current records are kept by Gina A No, but we can get it there.
because according to you the whole records are already placed in the bodega? Q Will that fact be shown in any listed articles in the application for search warrant since
A Yes. according to you, you have seen this manipulation reflected on the books of account
Q But how can you enter the bodega? kept by Gina? Are you sure that these documents are still there?
A Here, from the main entrance there is a door which will lead to this part here. If you go A Yes. I have received information.
straight there is a bodega there and there is also a guard from this exit right after COURT: Alright.[31]
opening the door. Abos stated that, as former Operating Chief of Unifish, he had access to the company
Q The problem is that, when actually in August have you seen the current records kept by records, and even showed the issuing judge photocopies thereof. Thus, we reject the contention
Gina? that this witness did not have personal knowledge of the facts to which he testified. The
A I cannot exactly recall but I have the xerox copies of the records. contents of the deposition clearly demonstrate otherwise.
Q Where are they now? The deposition also shows that, contrary to petitioners submission, the inquiries made by
A They are in my possession (witness handling [sic] to the Court a bunch of records). the judge were far from leading or being a rehash of the witness affidavit. We find such inquiries
Q The transactions that are reflected in these xerox copies that you have given me, especially to be sufficiently probing.
this one which seems to be pages of a ledger, they show that these are for the months Alleged lack
of January, February, March, April and May. Are these transactions reflected in these of
xerox copies which appear in the ledger being shown to the BIR? particularity
A As far as I know, it did not appear. in the
Q What about this one which says Columnar Book Cash Receipt for the month of January, description
what does it show? of the
A It shows that Frank Uy is the one purchasing from the company and these are his things
customers. seized
Q Do these entries appear in the columnar books which are the basis for the report to the Petitioners note the similarities in the description of the things to be seized in the subject
BIR? warrants and those in Stonehill vs. Diokno,[32] Bache & Co. (Phil.), Inc. vs. Ruiz,[33] and Asian Surety
A As far as I know, it does not reflect. & Insurance Co., Inc. vs. Herrera.[34]
Q What are these xerox copies of checks? In Stonehill, the effects to be searched and seized were described as:
A I think we cannot trace it up. These ones are the memos received by Unifish for payment Books of accounts, financial records, vouchers, journals correspondence, receipts, ledgers,
of sardines. This is the statement of the company given to Uy Chin Ho for collection. portfolios, credit journals, typewriters, and other documents and/or papers showing all business
Q It is also stated in your affidavit that the company imported soya oil. How is it done? transactions including disbursement receipts, balance sheets and related profit and loss
A The company imports soya oil to be used as a component in the processing of canned tuna statements.
for export. The company enjoys certain BOI privilege and so it is tax free. As far as I This Court found that the foregoing description failed to conform to the requirements set
know, they profit more to dispose the product locally. Whatever excess of this soya oil forth by the Constitution since:
are sold to another company. x x x the warrants authorized the search for and seizure of records pertaining to all business
Q Is that fact reflected in the xerox copies? transactions of petitioners herein, regardless of whether the transactions
A No. I have the actual delivery receipt. were legal or illegal. The warrants sanctioned the seizure of all records of the petitioners and the
aforementioned corporations, whatever their nature, thus openly contravening the explicit
46

command of our Bill of Rights - that the things to be seized be particularly described - as well as x x x Because of this all embracing description which includes all conceivable records of
tending to defeat its major object: the elimination of general warrants. petitioner corporation, which if seized x x x, could paralyze its business, petitioner in several
In Bache & Co., this Court struck down a warrant containing a similar description as those motions filed for early resolution of this case, manifested that the seizure of TWO carloads of
in Stonehill: their papers has paralyzed their business to the grave prejudice of not only the company, its
The documents, papers, and effects sought to be seized are described in Search Warrant No. 2- workers, agents, employees but also of its numerous insured and beneficiaries of bonds issued
M-70 in this manner: by it, including the government itself, and of the general public. And correlating the same to the
Unregistered and private books of accounts (ledgers, journals, columnars, receipts and charges for which the warrant was issued, We have before Us the infamous general warrants of
disbursements books, customers' ledgers); receipts for payments received; certificates of stocks old.
and securities; contracts, promissory notes and deeds of sale; telex and coded messages; In the case at bar, the things to be seized were described in the following manner:
business communications; accounting and business records; checks and check stubs; records of 1. Multiple sets of Books of Accounts; Ledgers, Journals, Columnar Books, Cash
bank deposits and withdrawals; and records of foreign remittances, covering the years 1966 to Register Books, Sales Books or Records; Provisional & Official Receipts;
1970. 2. Production Record Books/Inventory Lists [,] Stock Cards;
The description does not meet the requirement in Art. III, Sec. 1, of the Constitution, and of Sec. 3. Unregistered Delivery Receipts;
3, Rule 126 of the Revised Rules of Court, that the warrant should particularly describe the things 4. Unregistered Purchase & Sales Invoices;
to be seized. 5. Sales Records, Job Order;
xxx 6. Corporate Financial Records; and
In Uy Kheytin, et al. vs. Villareal, etc., et al., 42 Phil. 886, 896, this Court had occasion to explain 7. Bank Statements/Cancelled Checks
the purpose of the requirement that the warrant should particularly describe the place to be We agree that most of the items listed in the warrants fail to meet the test of particularity,
searched and the things to be seized, to wit: especially since witness Abos had furnished the judge photocopies of the documents sought to
x x x Both the Jones Law (sec. 3) and General Orders No. 68 (sec. 97) specifically require that a be seized. The issuing judge could have formed a more specific description of these documents
search warrant should particularly describe the place to be searched and the things to be from said photocopies instead of merely employing a generic description thereof. The use of a
seized. The evident purpose and intent of this requirement is to limit the things to be seized to generic term or a general description in a warrant is acceptable only when a more specific
those, and only those, particularly described in the search warrant - to leave the officers of the description of the things to be seized is unavailable. The failure to employ the specificity
law with no discretion regarding what articles they shall seize, to the end that unreasonable available will invalidate a general description in a warrant.[35] The use by the issuing judge of the
searches and seizures may not be made, - that abuses may not be committed. That is the correct terms multiple sets of books of accounts, ledgers, journals, columnar books, cash register books,
interpretation of this constitutional provision borne out by the American authorities. sales books or records, provisional & official receipts, production record books/inventory lists,
The purpose as thus explained could, surely and effectively, be defeated under the search stock cards, sales records, job order, corporate financial records, and bank statements/cancelled
warrant issued in this case. checks is therefore unacceptable considering the circumstances of this case.
A search warrant may be said to particularly describe the things to be seized when the As regards the terms unregistered delivery receipts and unregistered purchase & sales
description therein is as specific as the circumstances will ordinarily allow (People vs. Rubio, 57 invoices, however, we hold otherwise. The Solicitor General correctly argues that the serial
Phil, 384); or when the description expresses a conclusion of fact - not of law - by which the markings of these documents need not be specified as it is not possible to do so precisely
warrant officer may be guided in making the search and seizure (idem., dissent of Abad because they are unregistered.[36] Where, by the nature of the goods to be seized, their
Santos, J.,); or when the things described are limited to those which bear direct relation to the description must be rather general, it is not required that a technical description be given, as this
offense for which the warrant is being issued (Sec. 2, Rule 126, Revised Rules of Court). The would mean that no warrant could issue. Taking into consideration the nature of the articles so
herein search warrant does not conform to any of the foregoing tests. If the articles desired to described, it is clear that no other more adequate and detailed description could have been
be seized have any direct relation to an offense committed, the applicant must necessarily have given, particularly because it is difficult to give a particular description of the contents
some evidence, other than those articles, to prove the said offense; and the articles subject of thereof.[37] Although it appears that photocopies of these unregistered documents were among
search and seizure should come in handy merely to strengthen such evidence. In this event, the those handed by Abos to the issuing judge, it would be impractical to require the latter to specify
description contained in the herein disputed warrant should have mentioned, at least, the dates, each and every receipt and invoice, and the contents thereof, to the minutest detail.
amounts, persons, and other pertinent data regarding the receipts of payments, certificates of The general description of most of the documents listed in the warrants does not render
stocks and securities, contracts, promissory notes, deeds of sale, messages and communications, the entire warrant void. Insofar as the warrants authorize the search and seizure of unregistered
checks, bank deposits and withdrawals, records of foreign remittances, among others, delivery receipts and unregistered purchase and sales invoices, the warrants remain valid. The
enumerated in the warrant. search warrant is severable, and those items not particularly described may be cut off without
In Asian Surety & Insurance Co., Inc. vs. Herrera, the description of the things to be seized, destroying the whole warrant. In United States v. Cook,[38] the United States Court of Appeals
i.e., Fire Registers, Loss, Bordereau, Adjusters' Report, including subrogation receipts and proof (Fifth Circuit) made the following pronouncement:
of loss, Loss Registers, Book of Accounts including cash receipts and disbursements and general x x x. The leading decision is Aday v. Superior Court, 53 Cal.2d 789, 362 P.2d 47, 13 Cal.Rptr. 415
ledger, etc. was held to be an omnibus description and, therefore, invalid: (1961). In Aday, a warrant was issued authorizing the seizure of two particularly described books
and myriad other generally described items. On appeal, the California Supreme Court held that
47

only the books were particularly described in the warrant and lawfully seized. The court law with no discretion regarding what articles they should seize, to the end that unreasonable
acknowledged that the warrant was flawed, but rather than suppress everything seized, the searches and seizures may not be made and that abuses may not be committed (Corro v. Lising,
court chose to sever the defective portions of the warrant and suppress only those items that 137 SCRA 541, 547 [1985]); Bache & Co. [Phil.], Inc. v. Ruiz, 37 SCRA 823 [1971]; Uy Kheytin v.
were not particularly described. Villareal, 42 Phil. 886 [1920]). The same constitutional provision is also aimed at preventing
Although the warrant was defective x x x it does not follow that it was invalid as a whole. Such a violations of security in person and property and unlawful invasions of the sanctity of the home,
conclusion would mean that the seizure of certain articles, even though proper if viewed and giving remedy against such usurpations when attempted (People v. Damaso, 212 SCRA 547
separately, must be condemned merely because the warrant was defective with respect to other [1992] citing Alvero v. Dizon, 76 Phil. 637, 646 [1946]).
articles. The invalid portions of the warrant are severable from the authorization relating to the Clearly then, the money which was not indicated in the search warrant, had been illegally seized
named books x x x. The search for and seizure of these books, if otherwise valid, were not from petitioner. The fact that the members of the police team were doing their task of pursuing
rendered illegal by the defects concerning other articles. subversives is not a valid excuse for the illegal seizure. The presumption juris tantum of
xxx regularity in the performance of official duty cannot by itself prevail against the constitutionally
x x x We agree with the reasoning of the Supreme Court of California and the majority of state protected right of an individual (People v. Cruz, 231 SCRA 759 [1994]; People v. Veloso, 48 Phil.
courts that have considered this question and hold that in the usual case the district judge 169, 176 [1925]). Although public welfare is the foundation of the power to search and seize,
should sever the infirm portion of the search warrant as passes constitutional muster. See United such power must be exercised and the law enforced without transgressing the constitutional
States v. Giresi, 488 F.Supp. 445, 459-60 (D.N.J.1980). Items that were not described with the rights of the citizens (People v. Damaso, supra, citing Rodriguez v. Evangelista, 65 Phil. 230, 235
requisite particularity in the warrant should be suppressed, but suppression of all of the fruits of [1937]). As the Court aptly puts it in Bagahilog v. Fernandez, 198 SCRA 614 (1991), [z]eal in the
the search is hardly consistent with the purposes underlying exclusion. Suppression of only the pursuit of criminals cannot ennoble the use of arbitrary methods that the Constitution itself
items improperly described prohibits the Government from profiting from its own wrong and abhors.
removes the court from considering illegally obtained evidence. Moreover, suppression of only The seizure of the items not specified in the warrants cannot be justified by the directive in
those items that were not particularly described serves as an effective deterrent to those in the the penultimate paragraph thereof to "seize and take possession of other properties relative to
Government who would be tempted to secure a warrant without the necessary description. As such violation," which in no way can be characterized as a particular description of the things to
the leading commentator has observed, it would be harsh medicine indeed if a warrant which be seized.
was issued on probable cause and which did particularly describe certain items were to be As regards the articles supposedly belonging to PIDC, we cannot order their return in the
invalidated in toto merely because the affiant and the magistrate erred in seeking and permitting present proceedings. The legality of a seizure can be contested only by the party whose rights
a search for other items as well. 2 W. LaFave, Search and Seizure: A Treatise on the Fourth have been impaired thereby, and the objection to an unlawful search and seizure is purely
Amendment 4.6(f) (1978). personal and cannot be availed of by third parties.[42]
Accordingly, the items not particularly described in the warrants ought to be returned to WHEREFORE, the Resolutions of respondent Court of Appeals dated 27 June 1996 and 14
petitioners. May 1987, affirming the Order of the Regional Trial Court dated 17 July 1995, are hereby
Petitioners allege that the following articles, though not listed in the warrants, were also AFFIRMED insofar as said Resolutions upheld the validity of the subject Search Warrants
taken by the enforcing officers: authorizing the seizure of the unregistered delivery receipts and unregistered purchase and sales
1. One (1) composition notebook containing Chinese characters, invoices, but REVERSED with respect to the rest of the articles subject of said warrants. The
2. Two (2) pages writing with Chinese characters, respondent Bureau of Internal Revenue is hereby ordered to return to petitioners all items
3. Two (2) pages Chinese character writing, seized from the subject premises and belonging to petitioners, except the unregistered delivery
4. Two (2) packs of chemicals, receipts and unregistered purchase and sales invoices.
5. One (1) bound gate pass, SO ORDERED.
6. Surety Agreement.[39]
In addition, the searching party also seized items belonging to the Premier Industrial and
Development Corporation (PIDC), which shares an office with petitioner Unifish.
The things belonging to petitioner not specifically mentioned in the warrants, like those
not particularly described, must be ordered returned to petitioners. In order to comply with the
constitutional provisions regulating the issuance of search warrants, the property to be seized
under a warrant must be particularly described therein and no other property can be taken
thereunder.[40] In Tambasen vs. People,[41] it was held:
Moreover, by their seizure of articles not described in the search warrant, the police acted
beyond the parameters of their authority under the search warrant. Section 2, Article III of the
1987 Constitution requires that a search warrant should particularly describe the things to be
seized. The evident purpose and intent of the requirement is to limit the things to be seized to
those, and only those, particularly described in the search warrant, to leave the officers of the
48

GO VS CA Supreme Court challenging an order of the trial judge denying a motion for inhibition will not
1. REMEDIAL LAW; CRIMINAL PROCEDURE; BAIL; NATURE OF BAIL PROCEEDINGS. — Respondent deprive the judge of authority to proceed with the case. Otherwise, by the expedient of filing
judge is correct in appreciating the nature of the bail proceedings. "[T]he hearing of an such motion or petition, although the same be lacking in merit, a party can unduly delay the trial.
application for bail should be summary or otherwise in the discretion of the court. By 'summary 6. ID.; ID.; EVIDENCE REQUIRED IN DISQUALIFICATION OF JUDGE; BIAS AND PREJUDICE NOT
hearing' [is] meant such brief and speedy method of receiving and considering the evidence of PRESUMED. — While bias and prejudice, which are relied upon by petitioner, have been
guilt as is practicable and consistent with the purpose of the hearing which is merely to recognized as valid reasons for the voluntary inhibition of the judge under Rule 137, Sec. 1, par.
determine the weight of the evidence for the purpose of bail. In such a hearing, the court 'does 2, the established rule is that mere suspicion that a judge is partial is not enough. There should
not sit to try the merits or to enter into any nice inquiry as to the weight that ought to be be clear and convincing evidence to prove the charge. Bare allegations of partiality and
allowed to the evidence for or against accused, nor will it speculate on the outcome of the trial prejudgment will not suffice. Bias and prejudice cannot be presumed especially if weighed
or on what further evidence may be therein offered is admitted.' . . . The course of the inquiry against a judge's sacred obligation under his oath of office to administer justice without respect
may be left to the discretion of the court which may confine itself to receiving such evidence as to person and do equal right to the poor and the rich.
has reference to substantial matters avoiding unnecessary thoroughness in the examination and 7. ID.; ID.; IN CASE AT BAR, THE ACT OF RESPONDENT JUDGE OF NOT SUSPENDING HEARING OF
cross-examination of witnesses and reducing to a reasonable minimum the amount of CASE AFTER DENIAL OF PETITIONER'S MOTION FOR RECUSATION AND DURING PENDENCY OF
corroboration particularly on details that are not essential to the purpose of the hearing." PETITION CHALLENGING HIS ORDERS DENYING THE MOTION FOR RECUSATION AND THE
2. ID.; ID.; ID.; PROCEDURE IN GRANT OF BAIL IS SIMILAR TO PROCEDURE IN CANCELLATION OF MOTION TO SUSPEND PROCEEDINGS AND TRANSFER VENUE OUTSIDE METRO MANILA NOT
BAIL. — Although the proceedings conducted by respondent judge were not for an application PROOF OF PARTIALITY. — In the case at hand, respondent judge acted in accordance with the
for bail but to cancel that which was issued to petitioner, the principles and procedure governing Rules and prevailing jurisprudence when he proceeded with the trial after denying petitioner's
hearings on an application for bail were correctly applied by respondent judge in the cancellation Motion for Recusation. Petitioner cannot, therefore, cite the fact that respondent judge did not
of bail proceedings since the bail was issued by this Court in G.R. No. 101837 without prejudice suspend hearing the case during the pendency of this petition as proof of his claim that the judge
to any lawful order which the trial court may issue in case the Provincial Prosecutor moves for is partial.
the cancellation of the bail. The grant of bail was made without prejudice because where bail is 8. ID.; ID.; IN CASE AT BAR, THE ACT OF RESPONDENT JUDGE OF OVERRULING PETITIONER'S
not a matter of right, as in this case, the prosecution must be given the opportunity to prove that OBJECTION TO THE ADMISSIBILITY OF THE EXTRAJUDICIAL STATEMENT OF A PROSECUTION
there is a strong evidence of guilt. In the cancellation of bail proceedings before him, the judge WITNESS WHO DID NOT TESTIFY THEREON, NOT PROOF OF BIAS; REASON. — The first of these
was confronted with the same issue as in an application for bail, i.e., whether the evidence of allegedly "palpably biased and hostile orders" was that issued by respondent judge on August 14,
guilt is so strong as to convince the court that the accused is not entitled to bail. Hence, the 1992 overruling petitioner's objection to the admissibility of an affidavit of Geronimo Gonzaga
similarity of the nature and procedure of the hearings for an application for bail and the offered by the prosecution. Petitioner contends that respondent judge should not have admitted
cancellation of the same. the extrajudicial statements of Gonzaga because the latter did not take the witness stand to
3. LEGAL AND JUDICIAL ETHICS; DISQUALIFICATION OF JUDGES; ESSENCE OF RULE ON affirm the statements contained in the document presented by the prosecution. Petitioner
DISQUALIFICATION OF JUDGES. — The Constitution commands that in all criminal prosecutions, suspected that respondent judge was trying to bolster the evidence for the prosecution. This
the accused shall enjoy the right to have "a speedy, impartial, and public trial." This right is a contention is without merit. The mere fact that the trial judge overruled petitioner's objection to
derivation and elaboration of the more fundamental right to due process of law. The rule on the the admissibility of a particular piece of evidence is not proof of bias. In Jandionco v. Peñaranda,
disqualification of judges is a mechanism for enforcing the requirements of due process. it was held that "[d]ivergence of opinions between a judge hearing a case and a party's counsel,
4. ID.; ID.; REASON FOR REQUIREMENT OF IMPARTIALITY ON PART OF JUDGE. — "It is now as to applicable laws and jurisprudence, is not a sufficient ground to disqualify the judge from
beyond dispute that due process cannot be satisfied in the absence of that degree of objectivity hearing the case on the ground of bias and manifest partiality." If petitioner disagrees with the
on the part of a judge sufficient to reassure litigants of his being fair and being just. Thereby judge's ruling, he may still question the admissibility of the evidence when he files an appeal, in
there is the legitimate expectation that the decision arrived at would be the application of the case a judgment of conviction is rendered. To conclude, however, that respondent judge, by
law to the facts as found by a judge who does not play favorites." The "cold neutrality of an overruling the objection raised by petitioner's counsel, was trying to strengthen the
impartial judge," although required primarily for the benefit of the litigants, is also designed to prosecution's evidence is not only baseless because there was no evidence given to support this
preserve the integrity of the judiciary and more fundamentally, to gain and maintain the people's conclusion, but also premature because at that stage, the judge was not yet appreciating the
faith in the institutions they have erected when they adopted our Constitution. The notion that merits and weight of the particular piece of evidence in question but was merely ruling on its
"justice must satisfy the appearance of justice" is an imposition by the citizenry, as the final admissibility. Petitioner's conclusion that "the offer and admission of Gonzaga's hearsay
judge of the conduct of public business, including trials, upon the courts of a high and 'eyewitness' statement suggest a sinister concert to simulate evidential strength" is, if not
uncompromising standard in the proper dispensation of justice. suggestive of paranoia, at the very least, an overreaction.
5. ID.; ID.; EFFECT OF MOTION FOR INHIBITION, DENIAL OF MOTION FOR INHIBITION, AND 9. ID.; ID.; IN CASE AT BAR, THE ACT OF RESPONDENT JUDGE OF CONSIDERING THE
PETITION CHALLENGING DENIAL OF MOTION FOR INHIBITION. — Hence, if the trial judge decides CANCELLATION OF BAIL PROCEEDINGS RIPE FOR RESOLUTION AND REFUSING TO ALLOW
to deny a motion for inhibition based on Rule 137, Sec. 1, par. 2, he shall proceed with the trial, PETITIONER'S COUNSEL TO PRESENT ANYMORE WITNESSES, NOT MOTIVATED BY BIAS; REASON.
unless of course restrained by either the Court of Appeals or by this Court. The mere filing of a — The other supervening event allegedly demonstrating the judge's partiality occurred during
motion for inhibition before the trial court or a petition before either the Court of Appeals or the one of the hearings concerning the prosecution's motion for the cancellation of petitioner's bail.
49

On September 28, 1992, after eleven (11) witnesses had been presented for the prosecution and 1991 and the end of last year or December 31, 1992, the records of the case now consist of four
two (2) for the defense, respondent judge considered the cancellation of bail proceedings ripe (4) volumes and the transcript of stenographic notes have reached a total of one thousand five
for resolution and refused to allow petitioner's counsel to present anymore witnesses. The hundred and twenty three (1523) pages. Hearings are still being conducted. When taken in the
reasons given by respondent judge for his ruling were: (1) the proceeding in the cancellation of light of petitioner's repeated attempts to have the proceedings in the murder case suspended
bail is summary and different from the hearing on the merits; (2) the court need not receive and his lawyers' transparent maneuvers for the needless protraction of the case, the Motion for
exactly the same number of witnesses from both the prosecution and the defense; and (3) the Recusation can only be viewed as another dilatory move and the present Motion for
counsel for petitioner previously limited himself to two (2) witnesses as borne out by the record Reconsideration a further ploy to stall hearings.
of the case . . . Having determined that respondent judge made a proper appreciation of the 12. ID.; CONDUCT REQUIRED OF A LAWYER BEFORE COURTS AND JUDICIAL OFFICERS. — The
nature of the bail proceedings before him, we likewise hold that it was within his discretion to Rules of Court commands members of the bar "[t]o observe and maintain the respect due to the
limit the number of witnesses for petitioner. The power of the court in the bail proceedings to courts of justice and judicial officers." Reinforcing this rule of conduct is the Code of Professional
make a determination as to whether or not the evidence of guilt is strong "implies a full exercise Responsibility which states in Canon 11 the following: "A lawyer shall observe and maintain the
of judicial discretion." If the trial judge believes that the evidence before him is sufficient for him respect due to the courts and to judicial officers and should insist on similar conduct by others."
to rule on the bail issue, after giving both parties their opportunity to present evidence, it is Rule 11.03 of the Code further states: "A lawyer shall abstain from scandalous, offensive or
within his authority to consider the bail proceedings ripe for resolution. In any case, respondent menacing language or behaviour before the courts." The next succeeding rule, Rule 11.03 adds:
judge acceded to petitioner's request and allowed him to present more witnesses in the bail "A lawyer shall not attribute to a judge motives not supported by the record or having
proceedings. In fine, the Court holds that the respondent judge's ruling on September 28, 1992 materiality to the case."
considering the prosecution's motion for cancellation of bail ripe for resolution on the basis of 13. ID.; ID.; REASON FOR THE REQUIREMENT. — To be sure, the adversarial nature of our legal
the evidence already presented was not motivated by bias or prejudice. system has tempted members of the bar, in pursuing their duty to advance the interests of their
10. ID.; ID.; IN CASE AT BAR, RESPONDENT JUDGE'S ORDER ALLOWING PETITIONER'S clients, to use strong language. But this privilege is not a license to malign our courts of justice.
ARRAIGNMENT AND TRIAL WITHOUT BENEFIT OF PRELIMINARY INVESTIGATION, NOT Irreverent behavior towards the courts by members of the bar is proscribed, not so much for the
NECESSARILY PROOF OF PARTIALITY. — Petitioner, in this Motion for Reconsideration, restates sake of the temporary incumbent of the judicial office, but more importantly, for the
his argument in the Petition that the respondent judge is biased, as evidenced by his Order dated maintenance of respect for our judicial system, so necessary for the country's stability. "Time
July 17, 1991 which in effect allowed petitioner's arraignment and trial without the benefit of a and again, this Court has admonished and punished, in varying degrees, members of the bar for
preliminary investigation. It is true that in Go v. Court of Appeals, et al., G.R. No. 101837, statements, disrespectful or irreverent, acrimonious or defamatory, of this Court or the lower
February 11, 1992, a divided Court nullified respondent judge's July 17, 1991 Order and ordered courts . . . To be sure, lawyers may come up with various methods, perhaps more effective, in
that a preliminary investigation be conducted. But the erroneous Order of respondent judge is calling the Court's attention to the issues involved. The language vehicle does not run short of
not necessarily proof of partiality. In People v. Lacson, we held that erroneous rulings do not expressions, emphatic but respectful, convincing but not derogatory, illuminating but not
always constitute evidence of bias. In Luciano v. Mariano, we made the pronouncement that offensive."
"[t]he mere fact that the judge has erroneously ruled against the same litigant on two or more 14. ID.; ID.; AN INSTANCE OF HIGHLY DEROGATORY, OFFENSIVE AND CONTEMPTUOUS
occasions does not create in our minds a decisive pattern of malice on the part of the judge LANGUAGE TOWARD A JUDGE. — Indeed, in the Motion for Reconsideration, counsels for
against that particular litigant. This is not an unusual occurrence on our courts . . ." Moreover, petitioner describe as "unparalleled for sheer malevolence" respondent judge's allegedly
the fact that the erroneous order issued by a judge can be remedied and was actually corrected, erroneous assumptions. Petitioner's lawyers further stated: "Petitioner's counsel, citing the
as in this case, militates against the disqualification of the judge on the ground of bias or above proceedings, contested the trial judge's baseless, nay despotic attempt to muzzle his right
partiality. to be heard in his defense. . ." The trial judge's actions were also branded as an "obviously
11. ID.; ID.; IN CASE AT BAR, MOTION FOR RECUSATION IS ANOTHER DILATORY MOVE. — In the unholy rush to do petitioner in . . ." In the Urgent Motion filed by petitioner on December 16,
case at hand, the Motion for Recusation filed by petitioner must be viewed in the light of his 1992, respondent judge is alleged to have: (a) "generated belief of his being under contract to do
lawyers' many attempts to suspend the proceedings before the respondent judge. Before the the prosecution's bidding;" (2) "evinced contempt for Supreme Court case law;" and (3)
trial court, petitioner tried at least eight (8) times, not merely to reset the scheduled hearings, "dishonored his judicial oath and duty to hear before he condemns, proceed upon inquiry, and
but to suspend the trial of the case itself. The following pleadings filed by petitioner before render judgment on a man's liberty only after a full trial of the facts." . . . In light of the above
respondent judge all prayed either to suspend the proceedings entirely or for the respondent doctrines and jurisprudence, as well as the inherent power and authority of this Court to cite
judge to delay the disposition of a particular issue . . . Before this Court, petitioner has already members of the Bar in contempt and to discipline them, we are of the opinion that the language
filed three (3) petitions assailing various orders of respondent judge in connection with the single used by petitioner's lawyers is highly derogatory, offensive and contemptuous.
murder case pending against him. Apart from the present petition which is docketed as G.R. No. RESOLUTION
106087, petitioner has previously filed two (2) other petitions docketed as G.R. Nos. 101837 and ROMERO, J p:
105424. In all three (3) petitions, petitioner applied for a temporary restraining order to have the This is a Motion for Reconsideration of this Court's Resolution dated September 23, 1992
proceedings before the trial court held in abeyance. The murder case involving only one accused, denying petitioner's Petition and affirming the Decision and Resolution promulgated on March 9,
the petitioner, has become unnecessarily complicated and the proceedings before the trial court 1992 and June 26, 1992, respectively, of the Court of Appeals in CA-G.R. SP No. 26305. 1 The CA
protracted, as can be gleaned from the fact that between the filing of the information on July 11, Decision and Resolution upheld the following: (1) respondent Judge Benjamin V. Pelayo's Order
50

dated September 4, 1991 which denied petitioner's Motion for Recusation; and (2) respondent An Order 11 was issued by the trial Court dated August 1, 1991 setting for hearing the issue
judge's Order dated September 17, 1991 denying petitioner's Motion to Suspend Proceedings concerning the proper venue of petitioner's detention.
and to Transfer Venue Outside Metro Manila. After the hearing on petitioner's custody, the trial court issued an Order 12 dated August 2, 1991
A review of the antecedent facts of this case, particularly those wherein respondent Judge ordering the CAPCOM to bring the person of petitioner to the court not later than August 5,
participated, is in order to arrive at a just and correct assessment of his acts vis-a-vis the 1991 so that a commitment order for his detention at the Rizal Provincial Jail could be issued.
petitioner. The Commitment Order 13 ordering the Provincial Warden of the Provincial Jail of Pasig to take
On July 2, 1991, Eldon Maguan was shot inside his car along Wilson Street, San Juan, Metro custody of petitioner was issued on August 5, 1991.
Manila. After conducting an investigation of the shooting incident, the police identified On August 8, 1991, petitioner filed a Motion for Recusation 14 praying that respondent judge
petitioner Rolito Go as the prime suspect in the commission of the crime. On July 8, 1991, inhibit himself from hearing the case. The motion was denied by respondent judge in his Order
petitioner, accompanied by two lawyers, presented himself before the San Juan Police Station. dated September 4, 1991. 15
He was arrested and booked for the shooting of Maguan. The police filed a complaint for On August 22, 1991, petitioner filed a Motion to Suspend Proceedings and Transfer Venue
frustrated homicide with the Office of the Provincial Prosecutor of Rizal. Outside Metro Manila which was denied by respondent judge on September 17, 1991. 16
On July 11, 1991, an information for murder was filed against petitioner before the Regional Trial Petitioner was arraigned on August 23, 1991. In view of his refusal to enter a plea, a plea of "Not
Court, Pasig, Metro Manila, the victim Eldon Maguan having died on July 9, 1991. Guilty" was entered for him by the trial court. 17
On the same day, July 11, 1991, counsel for petitioner filed with the Prosecutor an omnibus In the meantime, this Court, by Resolution dated July 24, 1991, remanded to the Court of
motion praying for petitioner's immediate release and for a preliminary investigation. Provincial Appeals the petition filed by petitioner assailing the July 17, 1991 Order of the trial court.
Prosecutor Mauro Castro interposed no objection to petitioner's being granted provisional On August 27, 1991, petitioner filed a petition for habeas corpus before the Court of Appeals.
liberty on a cash bond of P100,000.00. On September 23, 1991, the Court of Appeals rendered a consolidated decision dismissing the
The case was raffled to the sala of respondent judge, the Hon. Benjamin V. Pelayo, who, on July two petitions. However, upon petition by petitioner, this Court by an 8-6 vote in G.R. No. 101837
12, 1991, approved the cash bond posted by petitioner and ordered his release. issued a decision reversing the, CA decision and ordering (a) the Provincial Prosecutor to conduct
On July 16, 1991, respondent judge issued an Order granting leave for the Provincial Prosecutor a preliminary investigation; and (b) the release of petitioner without prejudice to any order that
of Rizal to conduct a preliminary investigation. the trial court may issue, should the Provincial Prosecutor move for cancellation of bail at the
However, on July 17, 1991, respondent judge motu proprio issued an Order 2 which: (a) recalled conclusion of the preliminary investigation.
the July 12, 1991 Order granting bail; (b) directed petitioner to surrender within 48 hours from After conducting a preliminary investigation pursuant to this Court's decision in G.R. No. 101837,
notice; (c) cancelled the July 16, 1991 Order granting leave for the Provincial Prosecutor to the Assistant Provincial Prosecutor issued a Resolution dated February 27, 1992 finding probable
conduct a preliminary investigation; (d) treated petitioner's omnibus motion for immediate cause to charge petitioner with the crime of murder. The Resolution was approved by the
release and preliminary investigation dated July 11, 1991 as a petition for bail. Provincial Prosecutor who filed with the trial court a motion to cancel the bail of petitioner and a
On July 19, 1991, petitioner filed a petition for certiorari, prohibition and mandamus questioning motion to set the criminal case for resumption of the trial on the merits.
the July 17, 1991 Order of respondent judge. On the same day, petitioner filed before the trial Petitioner attempted to have the Resolution of the Prosecutor reversed by appealing to the
court a motion to suspend all the proceedings pending the resolution of the petition filed before Department of Justice, and filing petitions with the Court of Appeals (CA, G.R. SP No. 27738) and
the Supreme Court. 3 This motion was denied by respondent judge. 4 finally to this Court (G.R. No. 105424), but his efforts did not meet with success.
On July 23, 1991, petitioner voluntarily surrendered to the CAPCOM. Upon motion 5 by On October 1, 1991, petitioner filed another petition for certiorari, prohibition and mandamus
petitioner, the respondent judge issued an Order 6 directing "the accused's continued detention before this Court seeking to annul: (1) the Order of the trial court dated September 4, 1991
at the CAPCOM until such time as the Court shall have properly determined the place where denying petitioner's Motion for Recusation; and (2) the Order dated September 17, 1991
accused should be detained." denying petitioner's Motion to Suspend Proceedings and Transfer Venue Outside Metro Manila.
On July 29, 1991, the National Bureau of Investigation wrote a letter 7 to the trial court The petition, docketed as G.R. No. 101772, was remanded to the Court of Appeals.
requesting that custody of petitioner be transferred to the Bureau in view of an investigation for On March 9, 1992, the Court of Appeals (13th Division) rendered a decision dismissing the
illegal possession of firearms involving petitioner. petition. As to the denial of petitioner's Motion for Recusation, the Court of Appeals held in part:
On the same date, July 29, 1991, the trial court issued an Order 8 granting the NBI temporary "On the basis of the allegation of the petition, the Court is not inclined to strike down the denial
custody of petitioner subject to the following conditions: (a) the petitioner is to be accorded his of petitioner's motion for recusation as a grave abuse of discretion on the part of the respondent
constitutional rights during the investigation; (b) the NBI investigation is to be conducted only judge absent any clear showing of such grave abuse of his discretion. The allegation of petitioner
during office hours and petitioner is to be returned to the custody of the CAPCOM at the end of in support of his motion for recusation are conclusions based on his own fears and are therefore
each day; and (c) the NBI should report to the trial court the status of the investigation. speculations than anything else.
On July 30, 1991, petitioner filed a motion 9 before the trial court praying that the Order dated In order to warrant a finding of 'prejudicial' publicity as urged by the petitioner, there must be
July 29, 1991 be nullified and recalled. allegation and proof that the judge has been unduly influenced, not simply that he might be, by
The following day, July 31, 1991, the NBI filed a motion 10 praying that it be granted full custody the "barrage" of publicity (Martelino vs. Alejandro, 32 SCRA 106; emphasis supplied). While
of petitioner pending the investigation of the case involving illegal possession of firearms. there is such allegation in the petition, the Court has however found no proof so far adduced
51

sufficient to accept the petitioner's claim that the respondent judge has been unduly influenced parte of a writ of preliminary mandatory injunction directing respondent judge to allow
by the alleged publicity. petitioner to complete his defense evidence by presenting his last witness on the bail issue . . ."
Additionally, We quote hereunder the pronouncement of the Supreme Court in the case of 24
Aparicio vs. Andal, 175 SCRA 569 where, citing the case of Pimentel vs. Salanga, 21 SCRA 160, it On December 29, 1992, the Court passed a Resolution 25 issuing a temporary restraining order
said: (TRO) restraining respondent judge from resolving the bail issue and directing him to allow
Efforts to attain fair, just and impartial trial and decision, have a natural and alluring appeal. But, petitioner to present his last witness. This Resolution was clarified and the TRO confirmed in
we are not licensed to indulge in unjustified assumptions, or make a speculative approval to this another Resolution issued by the Court on January 11, 1993. 26
ideal. It ill behooves this Court to tar and feather a judge as biased or prejudiced, simply because On January 8, 1993, the OSG filed a Comment on petitioner's Motion for Reconsideration.
counsel for a party litigant happens to complain against him. As applied here, respondent judge At the outset, it is noteworthy to observe that petitioner in this Motion for Reconsideration no
has not as yet crossed the line that divides partiality and impartiality. He has not thus far stepped longer raises the question of change of venue. Moreover, the Motion for Reconsideration is
to one side of the fulcrum. No act or conduct of his would show arbitrariness or prejudice. predicated on what petitioner alleges are "the supervening events demonstrating partiality to
Therefore, we are not to assume what respondent judge, not otherwise legally disqualified, will the prosecution, on one hand, and hostility against petitioner, on the other." 27 Perforce, this
do in a case before him. We have had occasion to rule in a criminal case that a charge made Resolution shall only consider the allegations and issues raised in this Motion for
before trial that a party 'will not be given a fair, impartial and just hearing' is 'premature.' Reconsideration and in the Comment thereon filed by the OSG.
Prejudice is not to be presumed. Especially if weighed against a judge's legal obligation under his Petitioner's Motion for Recusation filed before the trial court is based on Rule 137, sec. 1, par. 2
oath to administer justice without respect to person and to equal right to the poor and the rich.' of the Rules of Court on disqualification of judges.
To disqualify or not to disqualify himself then, as far as respondent judge is concerned, is a The Constitution commands that in all criminal prosecutions, the accused shall enjoy the right to
matter of conscience." 18 have "a speedy, impartial, and public trial." 28 (Emphasis supplied) This right is a derivation and
The Court of Appeals also sustained the trial court's denial of petitioner's Motion to Suspend elaboration of the more fundamental right to due process of law. 29 The rule on the
Proceedings and Transfer Venue Outside Metro Manila with the following pronouncement: disqualification of judges is a mechanism for enforcing the requirements of due process. "It is
"On the question of the denial by the respondent court of petitioner's motion to suspend now beyond dispute that due process cannot be satisfied in the absence of that degree of
proceedings and transfer venue outside of Metro Manila, suffice it to say that the respondent objectivity on the part of a judge sufficient to reassure litigants of his being fair and being just.
court was correct in denying petitioner's motion. For indeed, the authority to order a change of Thereby there is the legitimate expectation that the decision arrived at would be the application
venue or place of trial to avoid a miscarriage of justice is vested in the Supreme Court by Article of the law to the facts as found by a judge who does not play favorites." 30
VIII, Section 5, paragraph 4 of the Constitution. Neither the respondent court nor this Court has The "cold neutrality of an impartial judge," 31 although required primarily for the benefit of the
the authority to grant petitioner's motion for transfer of venue. The cases cited by petitioner in litigants, is also designed to preserve the integrity of the judiciary and more fundamentally, to
support of this issue were all decided by the Supreme Court before the advent of the 1973 gain and maintain the people's faith in the institutions they have erected when they adopted our
Constitution where the provision on transfer of venue was first adopted, hence not applicable to Constitution. The notion that "justice must satisfy the appearance of justice" 32 is an imposition
the instant case." 19 by the citizenry, as the final judge of the conduct of public business, including trials, upon the
Petitioner's Motion for Reconsideration of the CA decision having been denied, 20 a petition courts of a high and uncompromising standard in the proper dispensation of justice.
under Rule 45 was filed before this Court on July 29, 1992 assailing the decision of the Court of While bias and prejudice, which are relied upon by petitioner, have been recognized as valid
Appeals. On September 9, 1992, the Office of the Solicitor General (OSG), representing reasons for the voluntary inhibition 33 of the judge under Rule 137, sec. 1, par. 2, 34 the
respondent People of the Philippines, filed a Comment on the Petition. established rule is that mere suspicion that a judge is partial is not enough. There should be clear
On September 23, 1992, the Court, after considering the allegations contained, issues raised and and convincing evidence to prove the charge. 35 Bare allegations of partiality and prejudgment
the arguments adduced in the Petition, as well as the Comment filed by the OSG, issued a will not suffice. 36 Bias and prejudice cannot be presumed especially if weighed against a judge's
Resolution denying the Petition on the ground that the respondent Court of Appeals committed sacred obligation under his oath of office to administer justice without respect to person and do
no reversible error in its assailed decision. equal right to the poor and the rich. 37
On October 12, 1992, the present Motion for Reconsideration 21 was filed in which petitioner In the Motion for Reconsideration now before the Court, petitioner, to prove his allegation of
reiterates his position that respondent judge should inhibit himself from the case. bias on the part of respondent judge, takes the latter to task for continuing with the trial during
On December 16, 1992, petitioner filed a pleading captioned "Urgent Motion (for preliminary the pendency of this petition stating that:
mandatory injunction)." In said Motion, petitioner questioned the Order of the trial court dated "Even as the instant petition for the trial judge's recusation pends, the latter did not see fit to
December 9, 1992 denying petitioner's Motion to Reopen Hearing (of the cancellation of bail suspend the hearings. Indeed the trial judge has been conducting marathon hearings which, in
proceedings) and to Present Last Witness. It appears that after the presentation of eleven (11) the context of his questioned fairness and impartiality, roars out as a railroad rush to make
witnesses by the prosecution and six (6) by the defense, the trial court considered the question official a pre-determined verdict of guilt." 38
concerning the cancellation of petitioner's bail ripe for resolution. Thereafter, petitioner filed a The Court draws the attention of petitioner and his counsels 39 to the procedure to be followed
Motion to Reopen and Present Last Witness. 22 But the trial court issued an Order 23 dated by the judge before whom a motion for disqualification has been filed. Rule 137, sec. 2 provides:
December 9, 1992 which, among other things, denied the Motion. In the Urgent Motion filed "If it be claimed that an official is disqualified from sitting as above provided, the party objecting
with this Court on December 16, 1992, petitioner prayed "for the issuance forthwith and ex to his competency may, in writing, file with the official his objection, stating the grounds
52

therefor, and the official shall thereupon proceed with the trial, or withdraw therefrom, in The first of these allegedly "palpably biased and hostile orders" was that issued by respondent
accordance with his determination of the question of his disqualification . . ." (Emphasis judge on August 14, 1992 overruling petitioner's objection to the admissibility of an affidavit of
supplied) Geronimo Gonzaga offered by the prosecution. Petitioner contends that respondent judge
In People v. Moreno, 40 we stated that if a judge denies the motion for disqualification and rules should not have admitted the extrajudicial statements of Gonzaga because the latter did not
favorably on his competency to try the case, it becomes a matter of official duty for him to take the witness stand to affirm the statements contained in the document presented by the
proceed with the trial and decision of the case. He cannot shirk the responsibility without the prosecution. Petitioner suspected that respondent judge was trying to bolster the evidence for
risk of being called upon to account for his dereliction. Although this case was decided prior to the prosecution.
the introduction of par. 2 of Rule 137, sec. 1, there is no reason why the procedure laid down in This contention is without merit. The mere fact that the trial judge overruled petitioner's
Rule 137, sec. 2 and applied in People v. Moreno should not likewise apply to a motion for objection to the admissibility of a particular piece of evidence is not proof of bias. In Jandionco v.
inhibition filed pursuant to Rule 137, sec. 1, par. 2. 41 In fact, in Genoblazo v. Court of Appeals, Peñaranda, 47 it was held that "[d]ivergence of opinions between a judge hearing a case and a
42 the Court applied the procedure prescribed in Rule 137, sec. 2 when the trial judge denied a party's counsel, as to applicable laws and jurisprudence, is not a sufficient ground to disqualify
party's motion for inhibition under Rule 137, sec. 1, par. 2, thus: the judge from hearing the case on the ground of bias and manifest partiality." 48 If petitioner
"Moreover, the trial judge acted correctly in proceeding with the case by setting it for pre-trial disagrees with the judge's ruling, he may still question the admissibility of the evidence when he
since it is within her sound discretion, after her decision in favor of her own competency, to files an appeal, in case a judgment of conviction is rendered. To conclude, however, that
either proceed with the trial or refrain from acting on the case until determination of the issue of respondent judge, by overruling the objection raised by petitioner's counsel, was trying to
her disqualification by the appellate court [Section 2 of Rule 137 of the Revised Rules of Court; strengthen the prosecution's evidence is not only baseless because there was no evidence given
De la Paz v. Intermediate Appellate Court, supra, at 76]. 43 to support this conclusion, but also premature because at that stage, the judge was not yet
Hence, if the trial judge decides to deny a motion for inhibition based on Rule 137, sec. 1, par. 2, appreciating the merits and weight of the particular piece of evidence in question but was
he shall proceed with the trial, unless of course restrained by either the Court of Appeals or by merely ruling on its admissibility. Petitioner's conclusion that "the offer and admission of
this Court. The mere filing of a motion for inhibition before the trial court or a petition before Gonzaga's hearsay 'eyewitness' statement suggest a sinister concert to simulate evidential
either the Court of Appeals or the Supreme Court challenging an order of the trial judge denying strength" 49 is, if not suggestive of paranoia, at the very least, an overreaction.
a motion for inhibition will not deprive the judge of authority to proceed with the case. The other supervening event allegedly demonstrating the judge's partiality occurred during one
Otherwise, by the expedient of filing such motion or petition, although the same be lacking in of the hearings concerning the prosecution's motion for the cancellation of petitioner's bail. On
merit, a party can unduly delay the trial. September 28, 1992, after eleven (11) witnesses had been presented for the prosecution and
In the case at hand, respondent judge acted in accordance with the Rules and prevailing two (2) for the defense, respondent judge considered the cancellation of bail proceedings ripe
jurisprudence when he proceeded with the trial after denying petitioner's Motion for for resolution and refused to allow petitioner's counsel to present anymore witnesses. The
Recusation. Petitioner cannot, therefore, cite the fact that respondent judge did not suspend reasons given by respondent judge for his ruling were: (1) the proceeding in the cancellation of
hearing the case during the pendency of this petition as proof of his claim that the judge is bail is summary and different from the hearing on the merits; (2) the court need not receive
partial. This Court has not, in connection with the petition, issued a temporary restraining order exactly the same number of witnesses from both the prosecution and the defense; and (3) the
(TRO) enjoining respondent judge from further hearing the case. The TRO which this Court issued counsel for petitioner previously limited himself to two (2) witnesses as borne out by the record
on December 29, 1992 after the petition was denied and pending this Motion for of the case. 50
Reconsideration ordered the judge to desist from resolving the question on the cancellation of Respondent judge is correct in appreciating the nature of the bail proceedings. "[T]he hearing of
bail until the last witness of petitioner was heard. The TRO did not restrain the judge from an application for bail should be summary or otherwise in the discretion of the court. By
hearing the case. On the contrary, the judge was ordered to hear petitioner's last witness in the 'summary hearing' [is] meant such brief and speedy method of receiving and considering the
cancellation of bail proceedings. 44 Because it was his duty to continue trying the case and there evidence of guilt as is practicable and consistent with the purpose of the hearing which is merely
was no order from this Court not to do so, respondent judge committed no impropriety evincing to determine the weight of the evidence for the purpose of bail. In such a hearing, the court
partiality when he continued hearing the case during the pendency of the petition before this 'does not sit to try the merits or to enter into any nice inquiry as to the weight that ought to be
Court. allowed to the evidence for or against accused, nor will it speculate on the outcome of the trial
Petitioner, in this Motion for Reconsideration, claims that since the issuance of the Court or on what further evidence may be therein offered is admitted.' . . . The course of the inquiry
Resolution dated September 23, 1992 denying his Petition, there have been "supervening events may be left to the discretion of the court which may confine itself to receiving such evidence as
demonstrating partiality to the prosecution on one hand, and hostility against petitioner, on the has reference to substantial matters avoiding unnecessary thoroughness in the examination and
other hand." 45 Petitioner alleges: cross-examination of witnesses and reducing to a reasonable minimum the amount of
"Pursuing his unconstitutional bent first evinced when, without benefit of preliminary corroboration particularly on details that are not essential to the purpose of the hearing." 51
investigation, petitioner's arraignment and trial, then arrest and detention for almost a year was Although the proceedings conducted by respondent judge were not for an application for bail
peremptorily ordered — which this Court reversed and rebuked (G.R. no. 101837, promulgated but to cancel that which was issued to petitioner, the principles and procedure governing
11 February 1992) — the unchastened trial judge let out yet with two palpably biased and hostile hearings on an application for bail were correctly applied by respondent judge in the cancellation
orders, infra, clearly and unmistakably demonstrating an unconstitutional prejudgment of of bail proceedings since the bail was issued by this Court in G.R. No. 101837 without prejudice
petitioner's culpability." 46 to any lawful order which the trial court may issue in case the Provincial Prosecutor moves for
53

the cancellation of the bail. 52 The grant of bail was made without prejudice because where bail CAPTION OF PLEADING DATE OF FILING
is not a matter of right, as in this case, the prosecution must be given the opportunity to prove 1. Urgent Ex-Parte Motion July 19, 1991
that there is a strong evidence of guilt. 53 In the cancellation of bail proceedings before him, the 2. Motion to Hold in Abeyance August 2, 1991
judge was confronted with the same issue as in an application for bail, i.e., whether the evidence 3. Motion for Recusation August 8, 1991
of guilt is so strong as to convince the court that the accused is not entitled to bail. Hence, the 4. Motion to Suspend Proceedings
similarity of the nature and procedure of the hearings for an application for bail and the and Transfer Venue Outside
cancellation of the same. Metro Manila August 22, 1991
Having determined that respondent judge made a proper appreciation of the nature of the bail 5. Motion to Suspend Proceedings March 4, 1991
proceedings before him, we likewise hold that it was within his discretion to limit the number of 6. Second Motion to Inhibit March 2, 1992
witnesses for petitioner. The power of the court in the bail proceedings to make a determination 7. Motion to Suspend Action on
as to whether or not the evidence of guilt is strong "implies a full exercise of judicial discretion." Formal Offer of Evidence and on
54 If the trial judge believes that the evidence before him is sufficient for him to rule on the bail Submission of Memorandum Dec. 21, 1992
issue, after giving both parties their opportunity to present evidence, it is within his authority to 8. Motion to Reopen Hearing and
consider the bail proceedings ripe for resolution. In any case, respondent judge acceded to Present Last Witness Dec. 1, 1992
petitioner's request and allowed him to present more witnesses in the bail proceedings. Before this Court, petitioner has already filed three (3) petitions assailing various orders of
In fine, the Court holds that the respondent judge's ruling on September 28, 1992 considering respondent judge in connection with the single murder case pending against him. Apart from the
the prosecution's motion for cancellation of bail ripe for resolution on the basis of the evidence present petition which is docketed as G.R. No. 106087, petitioner has previously filed two (2)
already presented was not motivated by bias or prejudice. other petitions docketed as G.R. Nos. 101837 and 105424. In all three (3) petitions, petitioner
Finally, petitioner, in this Motion for Reconsideration, restates his argument in the Petition that applied for a temporary restraining order to have the proceedings before the trial court held in
the respondent judge is biased, as evidenced by his Order dated July 17, 1991 55 which in effect abeyance.
allowed petitioner's arraignment and trial without the benefit of a preliminary investigation. The murder case involving only one accused, the petitioner, has become unnecessarily
It is true that in Go v. Court of Appeals, et al., G.R. No. 101837, February 11, 1992, a divided complicated and the proceedings before the trial court protracted, as can be gleaned from the
Court nullified respondent judge's July 17, 1991 Order and ordered that a preliminary fact that between the filing of the information on July 11, 1991 and the end of last year or
investigation be conducted. But the erroneous Order of respondent judge is not necessarily December 31, 1992, the records of the case now consist of four (4) volumes and the transcript of
proof of partiality. In People v. Lacson, 56 we held that erroneous rulings do not always stenographic notes have reached a total of one thousand five hundred and twenty three (1523)
constitute evidence of bias. 57 In Luciano v. Mariano, 58 we made the pronouncement that pages. Hearings are still being conducted.
"[t]he mere fact that the judge has erroneously ruled against the same litigant on two or more When taken in the light of petitioner's repeated attempts to have the proceedings in the murder
occasions does not create in our minds a decisive pattern of malice on the part of the judge case suspended and his lawyers' transparent maneuvers for the needless protraction of the case,
against that particular litigant. This is not an unusual occurrence in our courts . . ." Moreover, the the Motion for Recusation can only be viewed as another dilatory move and the present Motion
fact that the erroneous order issued by a judge can be remedied and was actually corrected, as for Reconsideration a further ploy to stall hearings.
in this case, militates against the disqualification of the judge on the ground of bias or partiality. In sum, after a careful examination of the records of the case, including the transcript of
59 stenographic notes, and considering the applicable law, the pertinent rules and prevailing
We have earlier underscored the importance of the rule of disqualification of judges, not only in jurisprudence, we reiterate our holding in the Court Resolution dated September 23, 1992 that
safeguarding the rights of litigants to due process of law but also in earning for the judiciary the the Court of Appeals committed no reversible error in affirming the respondent judge's Order
people's confidence, an element so essential in the effective administration of justice. The rule which denied petitioner's Motion for Recusation. This extended Resolution should put an end to
should, therefore, not be used cavalierly to suit a litigant's personal designs or to defeat the ends petitioner's obvious attempts at deferring the trial of his principal case by dwelling on incidental
of justice. "While We are exacting on the conduct of judges confronted with motions for matters. The motion for reconsideration must, perforce, be denied with finality.
disqualification's, We cannot, however, tolerate acts of litigants who, for any conceivable reason, In the Comment on the petitioner's Motion for Recusation, the Solicitor General prays that Attys.
seek to disqualify a judge for their own purpose, under a plea of bias, hostility, prejudice or Raymundo A. Armovit, Miguel R. Armovit and Rafael R. Armovit, be disciplinarily dealt with by
prejudgment . . . [T]his Court does not approve the tactic of some litigants of filing of baseless this Court for allegedly using abusive and intemperate language against respondent judge which
motion for disqualification of the judge as a means of delaying the case and/or of forum- betrays disrespect to the trial court.
shopping for a more friendly judge." 60 Indeed, in the Motion for Reconsideration, counsels for petitioner describe as "unparalleled for
In the case at hand, the Motion for Recusation filed by petitioner must be viewed in the light of sheer malevolence" 62 respondent judge's allegedly erroneous assumptions. Petitioner's lawyers
his lawyers' many attempts to suspend the proceedings before the respondent judge. Before the further stated: "Petitioner's counsel, citing the above proceedings, contested the trial judge's
trial court, petitioner tried at least eight (8) times, not merely to reset the scheduled hearings, 61 baseless, nay despotic attempt to muzzle his right to be heard in his defense . . ." 63 The trial
but to suspend the trial of the case itself. The following pleadings filed by petitioner before judge's actions were also branded as an "obviously unholy rush to do petitioner in . . ." 64
respondent judge all prayed either to suspend the proceedings entirely or for the respondent In the Urgent Motion filed by petitioner on December 16, 1992, respondent judge is alleged to
judge to delay the disposition of a particular issue: have: (1) "generated belief of his being under contract to do the prosecution's bidding;" (2)
54

"evinced contempt for Supreme Court case law;" and (3) "dishonored his judicial oath and duty
to hear before he condemns, proceed upon inquiry, and render judgment on a man's liberty only
after a full trial of the facts." 65
The Rules of Court commands members of the bar "[t]o observe and maintain the respect due to
the courts of justice and judicial officers." 66 Reinforcing this rule of conduct is the Code of
Professional Responsibility which states in Canon 11 the following: "A lawyer shall observe and
maintain the respect due to the courts and to judicial officers and should insist on similar
conduct by others." Rule 11.03 of the Code further states: "A lawyer shall abstain from
scandalous, offensive or menacing language or behaviour before the courts." The next
succeeding rule, Rule 11.03 adds: "A lawyer shall not attribute to a judge motives not supported
by the record or having materiality to the case."
To be sure, the adversarial nature of our legal system has tempted members of the bar, in
pursuing their duty to advance the interests of their clients, to use strong language. But this
privilege is not a license to malign our courts of justice. Irreverent behavior towards the courts
by members of the bar is proscribed, not so much for the sake of the temporary incumbent of
the judicial office, but more importantly, for the maintenance of respect for our judicial system,
so necessary for the country's stability. "Time and again, this Court has admonished and
punished, in varying degrees, members of the bar for statements, disrespectful or irreverent,
acrimonious or defamatory, of this Court or the lower courts . . . To be sure, lawyers may come
up with various methods, perhaps more effective, in calling the Court's attention to the issues
involved. The language vehicle does not run short of expressions, emphatic but respectful,
convincing but not derogatory, illuminating but not offensive." 67
In light of the above doctrines and jurisprudence, as well as the inherent power and authority of
this Court to cite members of the Bar in contempt and to discipline them, we are of the opinion
that the language used by petitioner's lawyers is highly derogatory, offensive and contemptuous.
WHEREFORE, in view of the foregoing, the motion for reconsideration is DENIED with FINALITY.
Attys. Raymundo A. Armovit, Miguel R. Armovit and Rafael R. Armovit are hereby ordered to pay
a FINE of P500.00 each with a stern WARNING that a repetition of this or similar act and
language will be dealt with more severely. Let a copy of this Resolution be attached to their
records.
SO ORDERED.
55

PADILLA VS CA petitioner on September 27, 1995. The next day, September 28, petitioner filed the instant
On October 26, 1992, high-powered firearms with live ammunitions were found in the petition for review on certiorari with application for bail[19] followed by two "supplemental
possession of petitioner Robin Padilla @ Robinhood Padilla, i.e.: petitions" filed by different counsels,[20] a "second supplemental petition"[21] and an urgent
"(1) One .357 Caliber revolver, Smith and Wesson, SN-32919 with six (6) live motion for the separate resolution of his application for bail. Again, the Solicitor-
ammunitions; General[22] sought the denial of the application for bail, to which the Court agreed in a Resolution
"(2) One M-16 Baby Armalite rifle, SN-RP 131120 with four (4) long and one (1) promulgated on July 31, 1996.[23] The Court also granted the Solicitor-General's motion to file a
short magazine with ammunitions; consolidated comment on the petitions and thereafter required the petitioner to file his
"(3) One .380 Pietro Beretta, SN-A 35723 Y with clip and eight (8) ammunitions; and reply.[24] However, after his vigorous resistance and success on the intramural of bail (both in the
"(4) Six additional live double action ammunitions of .38 caliber revolver."[1] respondent court and this Court) and thorough exposition of petitioner's guilt in his 55-page
Petitioner was correspondingly charged on December 3, 1992, before the Regional Trial Brief in the respondent court, the Solicitor-General now makes a complete turnabout by filing a
Court (RTC) of Angeles City with illegal possession of firearms and ammunitions under P.D. "Manifestation In Lieu Of Comment" praying for petitioner's acquittal.[25]
1866[2] thru the following Information:[3] The People's detailed narration of facts, well-supported by evidence on record and given
"That on or about the 26th day of October, 1992, in the City of Angeles, Philippines, credence by respondent court, is as follows:[26]
and within the jurisdiction of this Honorable Court, the above-named accused, did "At about 8:00 o'clock in the evening of October 26, 1992, Enrique Manarang and his
then and there willfully, unlawfully and feloniously have in his possession and under compadre Danny Perez were inside the Manukan sa Highway Restaurant in Sto.
his custody and control one (1) M-16 Baby Armalite rifle, SN-RP 131120 with four (4) Kristo, Angeles City where they took shelter from the heavy downpour (pp. 5-6, TSN,
long and one (1) short magazines with ammunitions, one (1) .357 caliber revolver February 15, 1993) that had interrupted their ride on motorcycles (pp. 5-6, ibid.)
Smith and Wesson, SN-32919 with six (6) live ammunitions and one (1) .380 Pietro along McArthur Highway (ibid). While inside the restaurant, Manarang noticed a
Beretta, SN-A35723Y with clip and eight (8) ammunitions, without having the vehicle, a Mitsubishi Pajero, running fast down the highway prompting him to remark
necessary authority and permit to carry and possess the same. that the vehicle might get into an accident considering the inclement weather. (p.
ALL CONTRARY TO LAW."[4] 7, Ibid) In the local vernacular, he said thus: 'Ka bilis na, mumuran pa naman pota
The lower court then ordered the arrest of petitioner, [5] but granted his application for makaaksidente ya.' (p. 7, ibid). True enough, immediately after the vehicle had
bail.[6] During the arraignment on January 20, 1993, a plea of not guilty was entered for passed the restaurant, Manarang and Perez heard a screeching sound produced by
petitioner after he refused,[7] upon advice of counsel,[8] to make any plea.[9] Petitioner waived in the sudden and hard braking of a vehicle running very fast (pp. 7-8, ibid) followed by
writing his right to be present in any and all stages of the case.[10] a sickening sound of the vehicle hitting something (p. 8, ibid). Danny Cruz, quite sure
After trial, Angeles City RTC Judge David Rosete rendered judgment dated April 25, 1994 of what had happened, remarked 'oy ta na' signifying that Manarang had been right
convicting petitioner of the crime charged and sentenced him to an "indeterminate penalty from in his observation (pp. 8-9, ibid).
17 years, 4 months and 1 day of reclusion temporal as minimum, to 21 years of reclusion "Manarang and Cruz went out to investigate and immediately saw the vehicle
perpetua, as maximum".[11] Petitioner filed his notice of appeal on April 28, 1994.[12] Pending the occupying the edge or shoulder of the highway giving it a slight tilt to its side (pp. 9-
appeal in the respondent Court of Appeals,[13] the Solicitor-General, convinced that the 10, ibid). Manarang, being a member of both the Spectrum, a civic group and the
conviction shows strong evidence of guilt, filed on December 2, 1994 a motion to cancel Barangay Disaster Coordinating Council, decided to report the incident to the
petitioner's bail bond. The resolution of this motion was incorporated in the now assailed Philippine National Police of Angeles City (p. 10, ibid). He took out his radio and
respondent court's decision sustaining petitioner's conviction, [14] the dispositive portion of which called the Viper, the radio controller of the Philippine National Police of Angeles City
reads: (p. 10, ibid). By the time Manarang completed the call, the vehicle had started to
"WHEREFORE, the foregoing circumstances considered, the appealed decision is leave the place of the accident taking the general direction to the north (p. 11, ibid).
hereby AFFIRMED, and furthermore, the P200,000.00 bailbond posted by accused- "Manarang went to the location of the accident and found out that the vehicle had
appellant for his provisional liberty, FGU Insurance Corporation Bond No. JCR (2) hit somebody (p. 11, ibid).
6523, is hereby cancelled. The Regional Trial Court, Branch 61, Angeles City, is "He asked Cruz to look after the victim while he went back to the restaurant, rode on
directed to issue the Order of Arrest of accused-appellant and thereafter his his motorcycle and chased the vehicle (p. 11 ibid). During the chase he was able to
transmittal to the National Bureau of Prisons thru the Philippine National Police make out the plate number of the vehicle as PMA 777 (p. 33, TSN, February 15,
where the said accused-appellant shall remain under confinement pending 1993). He called the Viper through the radio once again (p. 34, ibid) reporting that a
resolution of his appeal, should he appeal to the Supreme Court. This shall be vehicle heading north with plate number PMA 777 was involved in a hit and run
immediately executory. The Regional Trial Court is further directed to submit a accident (p. 20, TSN, June 8, 1993). The Viper, in the person of SPO2 Ruby Buan,
report of compliance herewith. upon receipt of the second radio call flashed the message to all units of PNP Angeles
SO ORDERED."[15] City with the order to apprehend the vehicle (p. 20, ibid). One of the units of the PNP
Petitioner received a copy of this decision on July 26, 1995.[16] On August 9, 1995 he filed a Angeles City reached by the alarm was its Patrol Division at Jake Gonzales Street near
"motion for reconsideration (and to recall the warrant of arrest)" [17] but the same was denied by the Traffic Division (pp. 5-7, TSN, February 23, 1993). SPO2 Juan C. Borja III and SPO2
respondent court in its September 20, 1995 Resolution,[18] copy of which was received by Emerlito Miranda immediately borded a mobile patrol vehicle (Mobile No. 3) and
56

positioned themselves near the south approach of Abacan bridge since it was the "While SPO2 Borja and appellant were arguing, Mobile No. 7 with SPO Ruben
only passable way going to the north (pp. 8-9, ibid). It took them about ten (10) Mercado, SPO3 Tan and SPO2 Odejar on board arrived (pp. 11-12, TSN, March 8,
seconds to cover the distance between their office and the Abacan bridge (p. 9, ibid). 1993). As the most senior police officer in the group, SPO Mercado took over the
"Another PNP mobile patrol vehicle that responded to the flash message from SPO2 matter and informed appellant that he was being arrested for the hit and run
Buan was Mobile No. 7 of the Pulongmaragal Detachment which was then incident (p. 13, ibid). He pointed out to appellant the fact that the plate number of
conducting patrol along Don Juico Avenue (pp. 8-9, TSN, March 8, 1993). On board his vehicle was dangling and the railing and the hood were dented (p.
were SPO Ruben Mercado and SPO3 Tan and SPO2 Odejar (p. 8, ibid). SPO Ruben 12, ibid). Appellant, however, arrogantly denied his misdeed and, instead, played
Mercado immediately told SPO3 Tan to proceed to the MacArthur Highway to with the crowd by holding their hands with one hand and pointing to SPO3 Borja with
intercept the vehicle with plate number PMA 777 (p. 10, ibid). his right hand saying 'iyan, kinuha ang baril ko' (pp. 13-15, ibid). Because appellant's
"In the meantime, Manarang continued to chase the vehicle which figured in the hit jacket was short, his gesture exposed a long magazine of an armalite rifle tucked in
and run incident, even passing through a flooded portion of the MacArthur Highway appellant's back right pocket (p. 16, ibid). SPO Mercado saw this and so when
two (2) feet deep in front of the Iglesia ni Kristo church but he could not catch up appellant turned around as he was talking and proceeding to his vehicle, Mercado
with the same vehicle (pp. 11-12, February 15, 1993). When he saw that the car he confiscated the magazine from appellant (pp. 16-17, ibid). Suspecting that appellant
was chasing went towards Magalang, he proceeded to Abacan bridge because he could also be carrying a rifle inside the vehicle since he had a magazine, SPO2
knew Pulongmaragal was not passable (pp. 12-14, ibid). When he reached the Mercado prevented appellant from going back to his vehicle by opening himself the
Abacan bridge, he found Mobile No. 3 and SPO2 Borja and SPO2 Miranda watching door of appellant's vehicle (16-17, ibid). He saw a baby armalite rifle (Exhibit D) lying
all vehicles coming their way (p. 10, TSN, February 23, 1993). He approached them horizontally at the front by the driver's seat. It had a long magazine filled with live
and informed them that there was a hit and run incident (p. 10, ibid). Upon learning bullets in a semi-automatic mode (pp. 17-21, ibid). He asked appellant for the papers
that the two police officers already knew about the incident, Manarang went back to covering the rifle and appellant answered angrily that they were at his home (pp. 26-
where he came from (pp. 10-11; ibid). When Manarang was in front of Tina's 27, ibid). SPO Mercado modified the arrest of appellant by including as its ground
Restaurant, he saw the vehicle that had figured in the hit and run incident emerging illegal possession of firearms (p. 28, ibid). SPO Mercado then read to appellant his
from the corner adjoining Tina's Restaurant (p. 15, TSN, February 15, 1993). He saw constitutional rights (pp. 28-29,ibid).
that the license plate hanging in front of the vehicle bore the identifying number "The police officers brought appellant to the Traffic Division at Jake Gonzales
PMA 777 and he followed it (p. 15, ibid) towards the Abacan bridge. Boulevard (pp. 31-32, ibid) where appellant voluntarily surrendered a third firearm, a
"Soon the vehicle was within sight of SPO2 Borja and SPO2 Miranda of Mobile No. 3 pietro berreta pistol (Exhibit 'L') with a single round in its chamber and a magazine
(p. 10, TSN, February 23, 1993). When the vehicle was about twelve (12) meters (pp. 33-35, ibid) loaded with seven (7) other live bullets. Appellant also voluntarily
away from their position, the two police officers boarded their Mobile car, switched surrendered a black bag containing two additional long magazines and one short
on the engine, operated the siren and strobe light and drove out to intercept the magazine (Exhibits M, N, and O, pp. 36-37, ibid). After appellant had been
vehicle (p. 11, ibid). They cut into the path of the vehicle forcing it to stop (p. interrogated by the Chief of the Traffic Division, he was transferred to the Police
11, ibid). Investigation Division at Sto. Rosario Street beside the City Hall Building where he
"SPO2 Borja and SPO2 Miranda alighted from Mobile No. 3 (P. 12, TSN, February 23, and the firearms and ammunitions were turned over to SPO2 Rene Jesus Gregorio
1993). SPO2 Miranda went to the vehicle with plate number PMA 777 and instructed (pp. 5-10, TSN, July 13, 1993). During the investigation, appellant admitted
its driver to alight (p. 12,ibid). The driver rolled down the window and put his head possession of the firearms stating that he used them for shooting (p. 14, ibid). He was
out while raising both his hands. They recognized the driver as Robin C. Padilla, not able to produce any permit to carry or memorandum receipt to cover the three
appellant in this case (p. 13, ibid). There was no one else with him inside the vehicle firearms (pp. 16-18, TSN, January 25, 1994).
(p. 24). At that moment, Borja noticed that Manarang arrived and stopped his "On November 28, 1992, a certification (Exhibit 'F') was issued by Captain, Senior
motorcycle behind the vehicle of appellant (p. 14, ibid). SPO2 Miranda told appellant Inspector Mario Espino, PNP, Chief, Record Branch of the Firearms and Explosives
to alight to which appellant complied. Appellant was wearing a short leather jacket Office (pp. 7-8, TSN, March 4, 1993). The Certification stated that the three firearms
(p. 16, TSN, March 8, 1993) such that when he alighted with both his hands raised, a confiscated from appellant, an M-16 Baby armalite rifle SN-RP 131280, a .357 caliber
gun (Exhibit 'C') tucked on the left side of his waist was revealed (p. 15, TSN, February revolver Smith and Wesson SN 32919 and a .380 Pietro Beretta SN-A35720, were not
23, 1993), its butt protruding (p. 15, ibid). SPO2 Borja made the move to confiscate registered in the name of Robin C. Padilla (p. 6, ibid). A second Certification dated
the gun but appellant held the former's hand alleging that the gun was covered by December 11, 1992 issued by Captain Espino stated that the three firearms were not
legal papers (p. 16, ibid). SPO2 Borja, however, insisted that if the gun really was also registered in the name of Robinhood C. Padilla (p. 10, ibid)."
covered by legal papers, it would have to be shown in the office (p. 16, ibid). After Petitioner's defenses are as follows: (1) that his arrest was illegal and consequently, the
disarming appellant, SPO2 Borja told him about the hit and run incident which was firearms and ammunitions taken in the course thereof are inadmissible in evidence under the
angrily denied by appellant (p. 17, ibid). By that time, a crowd had formed at the exclusionary rule; (2) that he is a confidential agent authorized, under a Mission Order and
place (p. 19, ibid). SPO2 Borja checked the cylinder of the gun and find six (6) live Memorandum Receipt, to carry the subject firearms; and (3) that the penalty for simple illegal
bullets inside (p. 20, ibid). possession constitutes excessive and cruel punishment proscribed by the 1987 Constitution.
57

After a careful review of the records[27]of this case, the Court is convinced that petitioner's raining nighttime - all created a situation in which speed is essential and delay
guilt of the crime charged stands on terra firma, notwithstanding the Solicitor-General's change improvident.[35] The Court acknowledges police authority to make the forcible stop since
of heart. they had more than mere "reasonable and articulable" suspicion that the occupant of the vehicle
Anent the first defense, petitioner questions the legality of his arrest. There is no dispute has been engaged in criminal activity.[36] Moreover, when caught in flagrante delicto with
that no warrant was issued for the arrest of petitioner, but that per se did not make his possession of an unlicensed firearm (Smith & Wesson) and ammunition (M-16 magazine),
apprehension at the Abacan bridge illegal. petitioner's warrantless arrest was proper as he was again actually committing another offense
Warrantless arrests are sanctioned in the following instances:[28] (illegal possession of firearm and ammunitions) and this time in the presence of a peace
"Sec. 5. Arrest without warrant; when lawful. - A peace officer or a private officer.[37]
person may, without a warrant, arrest a person: Besides, the policemen's warrantless arrest of petitioner could likewise be justified under
(a) When, in his presence, the person to be arrested has committed, is paragraph (b) as he had in fact just committed an offense. There was no supervening event or a
actually committing, or is attempting to commit an offense; considerable lapse of time between the hit and run and the actual apprehension. Moreover,
(b) When an offense has in fact just been committed, and he has personal after having stationed themselves at the Abacan bridge in response to Manarang's report, the
knowledge of facts indicating that the person to be arrested has committed policemen saw for themselves the fast approaching Pajero of petitioner,[38] its dangling plate
it. number (PMA 777 as reported by Manarang), and the dented hood and railings thereof. [39]These
(c) When the person to be arrested is a prisoner who has escaped from a formed part of the arresting police officer's personal knowledge of the facts indicating that
penal establishment or place where he is serving final judgment or petitioner's Pajero was indeed the vehicle involved in the hit and run incident. Verily then, the
temporarily confined while his case is pending, or has escaped while being arresting police officers acted upon verified personal knowledge and not on unreliable hearsay
transferred from one confinement to another. information.[40]
Paragraph (a) requires that the person be arrested (i) after he has committed or while he is Furthermore, in accordance with settled jurisprudence, any objection, defect or irregularity
actually committing or is at least attempting to commit an offense, (ii) in the presence of the attending an arrest must be made before the accused enters his plea. [41] Petitioner's belated
arresting officer or private person.[29] Both elements concurred here, as it has been established challenge thereto aside from his failure to quash the information, his participation in the trial
that petitioner's vehicle figured in a hit and run - an offense committed in the "presence" of and by presenting his evidence, placed him in estoppel to assail the legality of his
Manarang, a private person, who then sought to arrest petitioner. It must be stressed at this arrest.[42]Likewise, by applying for bail, petitioner patently waived such irregularities and
point that "presence" does not only require that the arresting person sees the offense, but also defects.[43]
when he "hears the disturbance created thereby AND proceeds at once to the scene." [30] As We now go to the firearms and ammunitions seized from petitioner without a search
testified to by Manarang, he heard the screeching of tires followed by a thud, saw the warrant, the admissibility in evidence of which, we uphold.
sideswiped victim (balut vendor), reported the incident to the police and thereafter gave chase The five (5) well-settled instances when a warrantless search and seizure of property is
to the erring Pajero vehicle using his motorcycle in order to apprehend its driver. After having valid,[44] are as follows:
sent a radio report to the PNP for assistance, Manarang proceeded to the Abacan bridge where 1. warrantless search incidental to a lawful arrest recognized under Section 12,
he found responding policemen SPO2 Borja and SPO2 Miranda already positioned near the Rule 126 of the Rules of Court[45] and by prevailing jurisprudence[46],
bridge who effected the actual arrest of petitioner.[31] 2. Seizure of evidence in "plain view", the elements of which are:[47]
Petitioner would nonetheless insist on the illegality of his arrest by arguing that the (a). a prior valid intrusion based on the valid warrantless arrest in
policemen who actually arrested him were not at the scene of the hit and run. [32] We beg to which the police are legally present in the pursuit of their official
disagree.That Manarang decided to seek the aid of the policemen (who admittedly were duties;
nowhere in the vicinity of the hit and run) in effecting petitioner's arrest, did not in any way (b). the evidence was inadvertently discovered by the police who
affect the propriety of the apprehension. It was in fact the most prudent action Manarang could had the right to be where they are;
have taken rather than collaring petitioner by himself, inasmuch as policemen are (c). the evidence must be immediately apparent, and
unquestionably better trained and well-equipped in effecting an arrest of a suspect (like herein (d). "plain view" justified mere seizure of evidence without further
petitioner) who , in all probability, could have put up a degree of resistance which an untrained search.[48]
civilian may not be able to contain without endangering his own life. Moreover, it is a reality that 3. search of a moving vehicle.[49] Highly regulated by the government, the
curbing lawlessness gains more success when law enforcers function in collaboration with vehicle's inherent mobility reduces expectation of privacy especially when its
private citizens. It is precisely through this cooperation, that the offense herein involved transit in public thoroughfares furnishes a highly reasonable suspicion
fortunately did not become an additional entry to the long list of unreported and unsolved amounting to probable cause that the occupant committed a criminal
crimes. activity.[50]
It is appropriate to state at this juncture that a suspect, like petitioner herein, cannot 4. consented warrantless search, and
defeat the arrest which has been set in motion in a public place for want of a warrant as the 5. customs search.
police was confronted by an urgent need to render aid or take action.[33] The exigent In conformity with respondent court's observation, it indeed appears that the authorities
circumstances of - hot pursuit,[34] a fleeing suspect, a moving vehicle, the public place and the stumbled upon petitioner's firearms and ammunitions without even undertaking any active
58

search which, as it is commonly understood, is a prying into hidden places for that which is suspicious circumstances. On this score, we lift from respondent court's incisive
concealed.[51] The seizure of the Smith & Wesson revolver and an M-16 rifle magazine was observation. Thus:
justified for they came within "plain view" of the policemen who inadvertently discovered the "Appellant's contention is predicated on the assumption that the Memorandum
revolver and magazine tucked in petitioner's waist and back pocket respectively, when he raised Receipts and Mission Order were issued before the subject firearms were seized and
his hands after alighting from his Pajero. The same justification applies to the confiscation of the confiscated from him by the police officers in Angeles City. That is not so. The
M-16 armalite rifle which was immediately apparent to the policemen as they took a casual evidence adduced indicate that the Memorandum Receipts and Mission Order were
glance at the Pajero and saw said rifle lying horizontally near the driver's seat. [52] Thus it has prepared and executed long after appellant had been apprehended on October 26,
been held that: 1992.
"(W)hen in pursuing an illegal action or in the commission of a criminal offense, the . "Appellant, when apprehended, could not show any document as proof of his
. . police officers should happen to discover a criminal offense being committed by authority to possess and carry the subject firearms. During the preliminary
any person, they are not precluded from performing their duties as police officers for investigation of the charge against him for illegal possession of firearms and
the apprehension of the guilty person and the taking of the corpus delicti."[53] ammunitions he could not, despite the ample time given him, present any proper
"Objects whose possession are prohibited by law inadvertently found in plain view document showing his authority. If he had, in actuality, the Memorandum Receipts
are subject to seizure even without a warrant."[54] and Missions Order, he could have produced those documents easily, if not at the
With respect to the Berreta pistol and a black bag containing assorted magazines, petitioner time of apprehension, at least during the preliminary investigation. But neither
voluntarily surrendered them to the police.[55] This latter gesture of petitioner indicated a waiver appellant nor his counsel inform the prosecutor that appellant is authorized to
of his right against the alleged search and seizure[56], and that his failure to quash the possess and carry the subject firearms under Memorandum Receipt and Mission
information estopped him from assailing any purported defect.[57] Order. At the initial presentation of his evidence in court, appellant could have
Even assuming that the firearms and ammunitions were products of an active search done produced these documents to belie the charged against him. Appellant did not. He
by the authorities on the person and vehicle of petitioner, their seizure without a search warrant did not even take the witness stand to explain his possession of the subject firearms.
nonetheless can still be justified under a search incidental to a lawful arrest (first instance). Once "Even in appellant's Demurrer to Evidence filed after the prosecution rested contain
the lawful arrest was effected, the police may undertake a protective search[58] of the passenger no allegation of a Memorandum Receipts and Mission Order authorizing appellant to
compartment and containers in the vehicle[59] which are within petitioner's grabbing distance possess and carry the subject firearms.
regardless of the nature of the offense.[60] This satisfied the two-tiered test of an incidental "At the initial presentation of appellant's evidence, the witness cited was one James
search: (i) the item to be searched (vehicle) was within the arrestee's custody or area of Neneng to whom a subpoena was issued. Superintendent Gumtang was not even
immediate control[61] and (ii) the search was contemporaneous with the arrest. [62] The products mentioned. James Neneng appeared in court but was not presented by the
of that search are admissible evidence not excluded by the exclusionary rule. Another defense. Subsequent hearings were reset until the defense found Superintendent
justification is a search of a moving vehicle (third instance). In connection therewith, a Gumtang who appeared in court without subpoena on January 13, 1994."[67]
warrantless search is constitutionally permissible when, as in this case, the officers conducting The Court is baffled why petitioner failed to produce and present the Mission Order and
the search have reasonable or probable cause to believe, before the search, that either the Memorandum Receipt if they were really issued and existing before his
motorist is a law-offender (like herein petitioner with respect to the hit and run) or the contents apprehension.Petitioner's alternative excuses that the subject firearms were intended for
or cargo of the vehicle are or have been instruments or the subject matter or the proceeds of theatrical purposes, or that they were owned by the Presidential Security Group, or that his
some criminal offense.[63] Mission Order and Memorandum Receipt were left at home, further compound their
Anent his second defense, petitioner contends that he could not be convicted of violating irregularity. As to be reasonably expected, an accused claiming innocence, like herein petitioner,
P.D. 1866 because he is an appointed civilian agent authorized to possess and carry the subject would grab the earliest opportunity to present the Mission Order and Memorandum Receipt in
firearms and ammunition as evidenced by a Mission Order [64] and Memorandum Receipt duly question and save himself from the long and agonizing public trial and spare him from proffering
issued by PNP Supt. Rodialo Gumtang, the deputy commander of Task Force Aguila, Lianga, inconsistent excuses. In fact, the Mission Order itself, as well as the Letter-Directive of the AFP
Surigao del Sur. The contention lacks merit. Chief of Staff, is explicit in providing that:
In crimes involving illegal possession of firearm, two requisites must be established, viz.: (1) "VIII. c. When a Mission Order is requested for verification by enforcement
the existence of the subject firearm and, (2) the fact that the accused who owned or possessed units/personnels such as PNP, Military Brigade and other Military Police Units of
the firearm does not have the corresponding license or permit to possess. [65] The first element is AFP, the Mission Order should be shown without resentment to avoid
beyond dispute as the subject firearms and ammunitions[66] were seized from petitioner's embarrassment and/or misunderstanding.
possession via a valid warrantless search, identified and offered in evidence during trial. As to "IX. d. Implicit to this Mission Order is the injunction that the confidential
the second element, the same was convincingly proven by the prosecution.Indeed, petitioner's instruction will be carried out through all legal means and do not cover an
purported Mission Order and Memorandum Receipt are inferior in the face of the more actuation in violation of laws. In the latter event, this Mission Order is rendered
formidable evidence for the prosecution as our meticulous review of the records reveals that the inoperative in respect to such violation."[68]
Mission Order and Memorandum Receipt were mere afterthoughts contrived and issued under which directive petitioner failed to heed without cogent explanation.
59

The authenticity and validity of the Mission Order and Memorandum Receipt, moreover, being asked whether it is registered or not, I did not find any records, the M-
were ably controverted. Witness for the prosecution Police Supt. Durendes denied under oath 16 and the caliber .357 and the caliber .380 but there is a firearm with the
his signature on the dorsal side of the Mission Order and declared further that he did not same serial number which is the same as that licensed and/or registered in
authorize anyone to sign in his behalf.[69] His surname thereon, we note, was glaringly misspelled the name of one Albert Villanueva Fallorina.
as "Durembes."[70] In addition, only Unit Commanders and Chief of Offices have the authority to "Q. So in short, the only licensed firearms in the name of accused Robin C. Padilla is
issue Mission Orders and Memorandum Receipts under the Guidelines on the Issuance of MOs, a pistol, Smith and Wesson, caliber 9 mm with Serial No. TCT 8214?
MRs, & PCFORs.[71] PNP Supt. Rodialo Gumtang who issued petitioner's Mission Order and "A. Yes, sir.
Memorandum Receipt is neither a Unit Commander nor the Chief of Office, but a mere deputy "Q. And the firearms that were the subject of this case are not listed in the names
commander. Having emanated from an unauthorized source, petitioner's Mission Order and of the accused in this case?
Memorandum Receipt are infirm and lacking in force and effect. Besides, the Mission Order "A. Yes, sir.[77]
covers "Recom 1-12-Baguio City,"[72] areas outside Supt. Gumtang's area of responsibility thereby xxx xxx xxx
needing prior approval "by next higher Headquarters"[73] which is absent in this case. The And the certification which provides as follows:
Memorandum Receipt is also unsupported by a certification as required by the March 5, 1988 Republic of the Philippines
Memorandum of the Secretary of Defense which pertinently provides that: Department of the Interior and Local Government
"No memorandum receipt shall be issued for a CCS firearms without corresponding GENERAL HEADQUARTERS PHILIPPINE NATIONAL POLICE
certification from the corresponding Responsible Supply Officer of the appropriate FIREARMS AND EXPLOSIVES OFFICE
AFP unit that such firearm has been officially taken up in that units property book, Camp Crame, Quezon City
and that report of such action has been reported to higher AFP authority." "PNPFEO5 28 November 1992
Had petitioner's Memorandum Receipt been authentic, we see no reason why he cannot present "C E R T I F I C A T I O N
the corresponding certification as well. "TO WHOM IT MAY CONCERN:
What is even more peculiar is that petitioner's name, as certified to by the Director for "THIS IS TO CERTIFY that Robin C. Padilla of 59 Labo St., Quezon City is a licensed/registered
Personnel of the PNP, does not even appear in the Plantilla of Non-Uniform Personnel or in the holder of Pistol Smith and Wesson Cal 9mm with serial number TCT8214 covered by License No.
list of Civilian Agents or Employees of the PNP which could justify the issuance of a Mission RL M76C4476687.
Order, a fact admitted by petitioner's counsel.[74] The implementing rules of P.D. 1866 issued by "Further certify that the following firearms are not registered with this Office per verification
the then PC-INP Chief and Director-General Lt. Gen. Fidel V. Ramos are clear and unambiguous, from available records on file this Office as of this date:
thus: M16 Baby Armalite SN-RP131120
"No Mission Order shall be issued to any civilian agent authorizing the same to Revolver Cal 357 SN-3219
carry firearms outside residence unless he/she is included in the regular plantilla of Pistol Cal 380 Pietro Beretta SN-35723
the government agencyinvolved in law enforcement and is receiving regular "However, we have on file one Pistol Cal 380, Beretta with serial number 35723Y,
compensation for the services he/she is rendering in the agency. Further, the licensed/registered to one Albert Villanueva Fallorina of 29 San Juan St., Capitol Pasig, MM under
civilian agent must be included in a specific law enforcement/police/intelligence Re-Registered License.
project proposal or special project which specifically required the use of firearms(s) "This certification is issued pursuant to Subpoena from City of Angeles.
to insure its accomplishment and that the project is duly approved at the PC Regional "FOR THE CHIEF, FEO:
Command level or its equivalent level in other major services of the AFP, INP and (Sgd.)
NBI, or at higher levels of command."[75] JOSE
Circular No. 1, dated January 6, 1986, of the then Ministry of Justice likewise provides as follows: MARIO M. ESPINO
"If mission orders are issued to civilians (not members of the uniformed service), Sr.
they must be civilian agents included in the regular plantilla of the government Inspector, PNP
agency involved in law enforcement and are receiving regular compensation for the Chief, Records
service they are rendering." Branch" [78]
That petitioner's Mission Order and Memorandum Receipt were fabricated pieces of evidence is In several occasions, the Court has ruled that either the testimony of a representative of,
accentuated all the more by the testimony and certification of the Chief of the Records Branch of or a certification from, the PNP Firearms and Explosives Office (FEO) attesting that a person is
the firearms and Explosives Office of the PNP declaring that petitioner's confiscated firearms are not a licensee of any firearm would suffice to prove beyond reasonable doubt the second
not licensed or registered in the name of the petitioner.[76] Thus: element of illegal possession of firearm.[79] In People vs. Tobias,[80] we reiterated that such
"Q. In all these files that you have just mentioned Mr. Witness, what did you find, if certification is sufficient to show that a person has in fact no license. From the foregoing
any? discussion, the fact that petitioner does not have the license or permit to possess was
"A. I found that a certain Robin C. Padilla is a licensed registered owner of one 9 mm overwhelmingly proven by the prosecution. The certification may even be dispensed with in the
pistol, Smith and Wesson with Serial No. TCT 8214 and the following firearms light of the evidence[81] that an M-16 rifle and any short firearm higher than a .38 caliber pistol,
60

akin to the confiscated firearms, cannot be licensed to a civilian,[82] as in the case of range of ten (10) years and one (1) day to twelve years (12) of prision mayor, as minimum, to
petitioner. The Court, therefore, entertains no doubt in affirming petitioner's conviction eighteen (18) years, eight (8) months and one (1) day to twenty (20) of reclusion temporal, as
especially as we find no plausible reason, and none was presented, to depart from the factual maximum. This is discernible from the following explanation by the Court:
findings of both the trial court and respondent court which, as a rule, are accorded by the Court "In the case at bar, no mitigating or aggravating circumstances have been alleged or proved, In
with respect and finality.[83] accordance with the doctrine regarding special laws explained in People v. Simon,[94] although
Anent his third defense, petitioner faults respondent court "in applying P.D. 1866 in a Presidential Decree No. 1866 is a special law, the penalties therein were taken from the Revised
democratic ambience (sic) and a non-subversive context" and adds that respondent court should Penal Code, hence the rules in said Code for graduating by degrees or determining the proper
have applied instead the previous laws on illegal possession of firearms since the reason for the period should be applied. Consequently, the penalty for the offense of simple illegal possession
penalty imposed under P.D. 1866 no longer exists.[84] He stresses that the penalty of 17 years and of firearm is the medium period of the complex penalty in said Section 1, that is, 18 years, 8
4 months to 21 years for simple illegal possession of firearm is cruel and excessive in months and 1 day to 20 years.
contravention of the Constitution.[85] "This penalty, being that which is to be actually imposed in accordance with the rules therefor
The contentions do not merit serious consideration. The trial court and the respondent and not merely imposable as a general prescription under the law, shall be the maximum of the
court are bound to apply the governing law at the time of appellant's commission of the offense range of the indeterminate sentence. The minimum thereof shall be taken, as aforesaid, from
for it is a rule that laws are repealed only by subsequent ones.[86] Indeed, it is the duty of judicial any period of the penalty next lower in degree, which is, prision mayor in its maximum period
officers to respect and apply the law as it stands.[87] And until its repeal, respondent court can to reclusion temporal in its medium period.[95]
not be faulted for applying P.D. 1866 which abrogated the previous statutes adverted to by WHEREFORE, premises considered, the decision of the Court of Appeals sustaining
petitioner. petitioner's conviction by the lower court of the crime of simple illegal possession of firearms
Equally lacking in merit is appellant's allegation that the penalty for simple illegal and ammunitions is AFFIRMED EXCEPT that petitioner's indeterminate penalty is MODIFIED to
possession is unconstitutional. The penalty for simple possession of firearm, it should be ten (10) years and one (1) day, as minimum, to eighteen (18) years, eight (8) months and one (1)
stressed, ranges from reclusion temporal maximum to reclusion perpetua contrary to appellant's day, as maximum.
erroneous averment. The severity of a penalty does not ipso facto make the same cruel and SO ORDERED
excessive.
"It takes more than merely being harsh, excessive, out of proportion, or severe for a penalty to
be obnoxious to the Constitution. 'The fact that the punishment authorized by the statute is
severe does not make it cruel and unusual.' (24 C.J.S., 1187-1188). Expressed in other terms, it
has been held that to come under the ban, the punishment must be 'flagrantly and plainly
oppressive', 'wholly disproportionate to the nature of the offense as to shock the moral sense of
the community' "[88]
It is well-settled that as far as the constitutional prohibition goes, it is not so much the extent as
the nature of the punishment that determines whether it is, or is not, cruel and unusual and that
sentences of imprisonment, though perceived to be harsh, are not cruel or unusual if within
statutory limits.[89]
Moreover, every law has in its favor the presumption of constitutionality. The burden of
proving the invalidity of the statute in question lies with the appellant which burden, we note,
was not convincingly discharged. To justify nullification of the law, there must be a clear and
unequivocal breach of the Constitution, not a doubtful and argumentative implication, [90] as in
this case. In fact, the constitutionality of P.D. 1866 has been upheld twice by this Court. [91] Just
recently, the Court declared that "the pertinent laws on illegal possession of firearms [are not]
contrary to any provision of the Constitution. . ."[92] Appellant's grievance on the wisdom of the
prescribed penalty should not be addressed to us. Courts are not concerned with the wisdom,
efficacy or morality of laws. That question falls exclusively within the province of Congress which
enacts them and the Chief Executive who approves or vetoes them. The only function of the
courts, we reiterate, is to interpret and apply the laws.
With respect to the penalty imposed by the trial court as affirmed by respondent court (17
years 4 months and 1 day of reclusion temporal, as minimum, to 21 years of reclusion perpetua,
as maximum), we reduce the same in line with the fairly recent case of People v. Lian[93] where
the Court en banc provided that the indeterminate penalty imposable for simple illegal
possession of firearm, without any mitigating or aggravating circumstance, should be within the
61

ESPANO VS CA Petitioner contends that the trial and appellate courts erred in convicting him on the basis of the
This is a petition for review of the decision of the Court of Appeals in CA-G.R. CR No. 13976 dated following: (a) the pieces of evidence seized were inadmissible; (b) the superiority of his
January 16, 1995, 1 which affirmed in toto the judgment of the Regional Trial Court of Manila, constitutional right to be presumed innocent over the doctrine of presumption of regularity, (c)
Branch 1, convincing petitioner Rodolfo Espano for violation of Article II, Section 8 of Republic he was denied the constitutional right of confrontation and to compulsory process; and (d) his
Act No. 6425, as amended, otherwise known as the Dangerous Drugs Act. conviction was based on evidence which was irrelevant and not properly identified.
Petitioner was charged under the following information: After a careful examination of the records of the case, this Court finds no compelling reason
That on or about July 14, 1991, in the City of Manila, Philippines, the said accused not sufficient to reverse the decisions of the trial and appellate courts.
being authorized by law to possess or use any prohibited drug, did then and there First, it is a well settled doctrine that findings of trial courts on the credibility of witnesses
willfully, unlawfully and knowingly have in his possession and under his custody and deserve a high degree of respect. Having observed the deportment of witnesses during the trial,
control twelve (12) plastic cellophane (bags) containing crushed flowering tops, the trial judge is in a better position to determine the issue of credibility and, thus, his findings
marijuana weighing 5.5 grams which is a prohibited drug. will not be disturbed during appeal in the absence of any clear showing that he had overlooked,
Contrary to law. 2 misunderstood or misapplied some facts or circumstances of weight and substance which could
The evidence for the prosecution, based on the testimony of Pat. Romeo Pagilagan, shows that have altered the conviction of the appellants. 6
on July 14, 1991, at about 12:30 a.m., he and other police officers, namely, Pat. Wilfredo Aquino, In this case, the findings of the trial court that the prosecution witnesses were more credible
Simplicio Rivera, and Erlindo Lumboy of the Western Police District (WPD), Narcotics Division than those of the defense must stand. Petitioner failed to show that Pat. Pagilagan, in testifying
went to Zamora and Pandacan Streets, Manila to confirm reports of drug pushing in the area. against him, was motivated by reasons other than his duty to curb drug abuse and had any intent
They saw petitioner selling "something" to another person. After the alleged buyer left, they to falsely impute to him such a serious crime as possession of prohibited drugs. In the absence of
approached petitioner, identified themselves as policemen, and frisked him. The search yielded such ill motive, the presumption of regularity in the performance of his official duty must prevail.
two plastic cellophane tea bags of marijuana. When asked if he had more marijuana, he replied In People v. Velasco, 7 this Court reiterated the doctrine of presumption of regularity in the
that there was more in his house. The policemen went to his residence where they found ten performance of official duty which provides:
more cellophane tea bags of marijuana. Petitioner was brought to the police headquarters . . . Appellant failed to establish that Pat. Godoy and the other members of the buy-
where he was charged with possession of prohibited drugs. On July 24, 1991, petitioner posted bust team are policemen engaged in mulcting or other unscrupulous activities who
bail 3 and the trial court issued his order of release on July 29, 1991. 4 were motivated either by the desire to extort money or exact personal vengeance, or
Annabelle Alip, forensic chemist of the WPD Criminal Investigation Laboratory Section, testified by sheer whim and caprice, when they entrapped her. And in the absence of proof of
that the articles sent to her by Pat. Wilfredo Aquino regarding the apprehension of a certain any intent on the part of the police authorities to falsely impute such a serious crime
Rodolfo Espano for examination tested positive for marijuana, with a total weight of 5.5 grams. against appellant, as in this case, the presumption of regularity in the performance of
By way of defense, petitioner testified that on said evening, he was sleeping in his house and was official duty, . . . , must prevail over the self-serving and uncorroborated claim of
awakened only when the policemen handcuffed him. He alleged that the policemen were appellant that she had been framed. 8
looking for his brother-in-law Lauro, and when they could not find the latter, he was instead Furthermore, the defense set up by petitioner does not deserve any consideration. He simply
brought to the police station for investigation and later indicted for possession of prohibited contended that he was in his house sleeping at the time of the incident. This Court has
drugs. His wife Myrna corroborated his story. consistently held that alibi is the weakest of all defenses; and for it to prosper, the accused has
The trial court rejected petitioner's, defense as a "mere afterthought" and found the version of the burden of proving that he was not at the scene of the crime at the time of its commission
the prosecution "more credible and trustworthy." and that it was physically impossible for him to be there. Moreover, the "claim of a 'frame-up',
Thus, on August 14, 1992, the trial court rendered a decision, convicting petitioner of the crime like alibi, is a defense that has been invariably viewed by the Court with disfavor for it can just as
charged, the dispositive portion of which reads: easily be concocted but difficult to prove, and is a common and standard line of defense in most
WHEREFORE there being proof beyond reasonable doubt, the court finds the accused prosecutions arising from violations of the Dangerous Drugs Act." 9 No clear and convincing
Rodolfo Espano y Valeria guilty of the crime of violation of Section 8, Article II, in evidence was presented by petitioner to prove his defense of alibi.
relation to Section 2 (e-L) (I) of Republic Act No. 6425 as amended by Batas Pambansa Second, petitioner contends that the prosecution's failure to present the alleged informant in
Blg. 179, and pursuant to law hereby sentences him to suffer imprisonment of six (6) court cast a reasonable doubt which warrants his acquittal. This is again without merit, since
years and one (1) day to twelve (12) years and to pay a fine of P6,000.00 with failure of the prosecution to produce the informant in court is of no moment especially when he
subsidiary imprisonment in case of default plus costs. is not even the best witness to establish the fact that a buy-bust operation had indeed been
The marijuana is declared forfeited in favor of government and shall be turned over to conducted. In this case, Pat. Pagilagan, one of the policemen who apprehended petitioner,
the Dangerous Drugs Board without delay. testified on the actual incident of July 14, 1991, and identified him as the one they caught in
SO ORDERED. 5 possession of prohibited drugs. Thus,
Petitioner appealed the decision to the Court of Appeals. The appellate court, however, affirmed We find that the prosecution had satisfactorily proved its case against appellants.
the decision of the trial court in toto. There is no compelling reason for us to overturn the finding of the trial court that the
Hence, this petition. testimony of Sgt. Gamboa, the lone witness for the prosecution, was straightforward
62

spontaneous and convincing. The testimony of a sole witness, if credible and positive In sum, this Court finds petitioner Rodolfo Espano guilty beyond reasonable doubt of violating
and satisfies the court beyond reasonable doubt, is sufficient to convict. 10 Article II, Section 8, in relation to Section 2 (e-L) (I) of Republic Act No. 6425, as amended. Under
Thus on the basis of Pat. Pagilagan's testimony, the prosecution was able to prove that petitioner the said provision, the penalty imposed is six years and one day to twelve years and a fine
indeed committed the crime charged; consequently, the finding of conviction was proper. ranging from six thousand to twelve thousand pesos. With the passage of Republic Act No. 7659,
Lastly, the issue on the admissibility of the marijuana seized should likewise be ruled upon. Rule which took effect on December 31, 1993, the imposable penalty shall now depend on the
113 Section 5(a) of the Rules of Court provides: quantity of drugs recovered. Under the provisions of Republic Act No. 7629, Section 20, and as
A peace officer or a private person may, without a warrant, arrest a person: interpreted in People v. Simon 13 and People v. Lara, 14 if the quantity of marijuana involved is less
a. when, in his presence, the person to be arrested has committed, is actually than 750 grams, the imposable penalty ranges from prision correccional to reclusion temporal.
committing, or is attempting to commit an offense; Taking into consideration that petitioner is not a habitual delinquent, the amendatory provision
xxx xxx xxx is favorable to him and the quantity of marijuana involved is less than 750 grams, the penalty
Petitioner's arrest falls squarely under the aforecited rule. He was caught in flagranti as a result imposed under Republic Act No. 7659 should be applied. There being no mitigating nor
of a buy-bust operation conducted by police officers on the basis of information received aggravating circumstances, the imposable penalty shall be prision correccional in its medium
regarding the illegal trade of drugs within the area of Zamora and Pandacan Streets, Manila. The period. Applying the Indeterminate Sentence Law, the maximum penalty shall be taken from the
police officer saw petitioner handing over something to an alleged buyer. After the buyer left, medium period of prision correccional, which is two (2) years, four (4) months and one (1) day to
they searched him and discovered two cellophanes of marijuana. His arrest was, therefore, four (4) years and two (2) months, while the minimum shall be taken from the penalty next
lawful and the two cellophane bags of marijuana seized were admissible in evidence, being the lower in degree, which is one (1) month and one (1) day to six (6) months of arresto mayor.
fruits of the crime. WHEREFORE, the instant petition is hereby DENIED. The decision of the Court of Appeals in C.A.-
As for the ten cellophane bags of marijuana found at petitioner's residence, however, the same G.R. CR No. 13976 dated January 16, 1995 is AFFIRMED with the MODIFICATION that petitioner
are inadmissible in evidence. Rodolfo Espano is sentenced to suffer an indeterminate penalty of TWO (2) months and ONE (1)
The 1987 Constitution guarantees freedom against unreasonable searches and seizures under day of arresto mayor, as minimum to TWO (2) years, FOUR (4) months and ONE (1) day of prision
Article III, Section 2 which provides: correccional, as maximum.
The right of the people to be secure in their persons, houses, papers and effects SO ORDERED.
against unreasonable searches and seizures of whatever nature and for any purpose
shall be inviolable, and no search warrant or warrant of arrest shall issue except upon
probable cause to be determined personally by the judge after examination under oath
or affirmation of the complainant and the witnesses he may produce, and particularly
describing the place to be searched and the persons or things to be seized.
An exception to the said rule is a warrantless search incidental to a lawful arrest for dangerous
weapons or anything which may be used as proof of the commission of an offense. 11 It may
extend beyond the person of the one arrested to include the premises or surroundings under his
immediate control. In this case, the ten cellophane bags of marijuana seized at petitioner's house
after his arrest at Pandacan and Zamora Streets do not fall under the said exceptions.
In the case of People v. Lua, 12 this Court held:
As regards the brick of marijuana found inside the appellant's house, the trial court
correctly ignored it apparently in view of its inadmissibility. While initially the arrest as
well as the body search was lawful, the warrantless search made inside the appellant's
house became unlawful since the police operatives were not armed with a search
warrant. Such search cannot fall under "search made incidental to a lawful arrest," the
same being limited to body search and to that point within reach or control of the
person arrested, or that which may furnish him with the means of committing violence
or of escaping. In the case at bar, appellant was admittedly outside his house when he
was arrested. Hence, it can hardly be said that the inner portion of his house was
within his reach or control.
The articles seized from petitioner during his arrest were valid under the doctrine of search
made incidental to a lawful arrest. The warrantless search made in his house, however, which
yielded ten cellophane bags of marijuana became unlawful since the police officers were not
armed with a search warrant at the time. Moreover, it was beyond the reach and control of
petitioner.
63

PEOPLE VS MENGOTE the testimony regarding the alleged robbery in Danganan's house was irrelevant and should also
Accused-appellant Rogelio Mengote was convicted of illegal possession of firearms on the have been disregarded by the trial court.
strength mainly of the stolen pistol found on his person at the moment of his warrantless arrest. The following are the pertinent provision of the Bill of Rights:
In this appeal, he pleads that the weapon was not admissible as evidence against him because it Sec. 2. The right of the people to be secure in their persons, houses, papers, and
had been illegally seized and was therefore the fruit of the poisonous tree. The Government effects against unreasonable searches and seizures of whatever nature and for any
disagrees. It insists that the revolver was validly received in evidence by the trial judge because purpose shall be inviolable, and no search warrant or warrant of arrest shall issue
except upon probable cause to be determined personally by the judge after
its seizure was incidental to an arrest that was doubtless lawful even if admittedly without
examination under oath or affirmation of the complainant and the witnesses he may
warrant. produce, and particularly describing the place to be searched and the persons or things
The incident occurred shortly before noon of August 8, 1987, after the Western Police District to be seized.
received a telephone call from an informer that there were three suspicious-looking persons at Sec. 3 (1). The privacy of communication and correspondence shall be inviolable
the corner of Juan Luna and North Bay Boulevard in Tondo, Manila. A surveillance team of except upon lawful order of the court, or when public safety or order requires
plainclothesmen was forthwith dispatched to the place. As later narrated at the trial by otherwise as prescribed by law.
Patrolmen Rolando Mercado and Alberto Juan, 1 they there saw two men "looking from side to (2) Any evidence obtained in violation of this or the preceding section shall be
side," one of whom was holding his abdomen. They approached these persons and identified inadmissible for any purpose in any proceeding.
There is no question that evidence obtained as a result of an illegal search or seizure is inadmissible in any
themselves as policemen, whereupon the two tried to run away but were unable to escape
proceeding for any purpose. That is the absolute prohibition of Article III, Section 3(2), of the Constitution.
because the other lawmen had surrounded them. The suspects were then searched. One of This is the celebrated exclusionary rule based on the justification given by Judge Learned Hand that "only in
them, who turned out to be the accused-appellant, was found with a .38 caliber Smith and case the prosecution, which itself controls the seizing officials, knows that it cannot profit by their wrong will
Wesson revolver with six live bullets in the chamber. His companion, later identified as Nicanor the wrong be repressed." The Solicitor General, while conceding the rule, maintains that it is not applicable
Morellos, had a fan knife secreted in his front right pants pocket. The weapons were taken from in the case at bar. His reason is that the arrest and search of Mengote and the seizure of the revolver from
them. Mengote and Morellos were then turned over to police headquarters for investigation by him were lawful under Rule 113, Section 5, of the Rules of Court reading as follows:
the Intelligence Division. Sec. 5. Arrest without warrant when lawful. — A peace officer or private person may,
On August 11, 1987, the following information was filed against the accused-appellant before without a warrant, arrest a person;
(a) When, in his presence, the person to be arrested has committed, is actually
the Regional Trial Court of Manila:
committing, or is attempting to commit an offense;
The undersigned accuses ROGELIO MENGOTE y TEJAS of a violation of Presidential
(b) When an offense has in fact just been committed, and he has personal knowledge
Decree No. 1866, committed as follows:
of facts indicating that the person to be arrested has committed it; and
That on or about August 8, 1987, in the City of Manila, Philippines, the said accused did
(c) When the person to be arrested is a prisoner who has escaped from a penal
then and there wilfully, unlawfully and knowingly have in his possession and under his
establishment or place where he is serving final judgment or temporarily confined
custody and control a firearm, to wit:
while his case is pending, or has escaped while being transferred from one
one (1) cal. 38 "S & W" bearing
confinement to another.
Serial No. 8720-T
In cases failing under paragraphs (a) and (b) hereof, the person arrested without a
without first having secured the necessary license or permit therefor from the proper
warrant shall be forthwith delivered to the nearest police station or jail, and he shall be
authorities.
proceeded against in accordance with Rule 112, Section 7.
Besides the police officers, one other witness presented by the prosecution was Rigoberto
We have carefully examined the wording of this Rule and cannot see how we can agree with the
Danganan, who identified the subject weapon as among the articles stolen from him during the
prosecution.
robbery in his house in Malabon on June 13, 1987. He pointed to Mengote as one of the robbers.
Par. (c) of Section 5 is obviously inapplicable as Mengote was not an escapee from a penal
He had duly reported the robbery to the police, indicating the articles stolen from him, including
institution when he was arrested. We therefore confine ourselves to determining the lawfulness
the revolver. 2 For his part, Mengote made no effort to prove that he owned the firearm or that
of his arrest under either Par. (a) or Par. (b) of this section.
he was licensed to possess it and claimed instead that the weapon had been "Planted" on him at
Par. (a) requires that the person be arrested (1) after he has committed or while he is actually
the time of his arrest. 3
committing or is at least attempting to commit an offense, (2) in the presence of the arresting
The gun, together with the live bullets and its holster, were offered as Exhibits A, B, and C and
officer.
admitted over the objection of the defense. As previously stated, the weapon was the principal
These requirements have not been established in the case at bar. At the time of the arrest in
evidence that led to Mengote's conviction for violation of P.D. 1866. He was sentenced
question, the accused-appellant was merely "looking from side to side" and "holding his
to reclusion
abdomen," according to the arresting officers themselves. There was apparently no offense that
perpetua. 4
had just been committed or was being actually committed or at least being attempted by
It is submitted in the Appellant's Brief that the revolver should not have been admitted in
Mengote in their presence.
evidence because of its illegal seizure. no warrant therefor having been previously obtained.
The Solicitor General submits that the actual existence of an offense was not necessary as long
Neither could it have been seized as an incident of a lawful arrest because the arrest of Mengote
as Mengote's acts "created a reasonable suspicion on the part of the arresting officers and
was itself unlawful, having been also effected without a warrant. The defense also contends that
induced in them the belief that an offense had been committed and that the accused-appellant
had committed it." The question is, What offense? What offense could possibly have been
64

suggested by a person "looking from side to side" and "holding his abdomen" and in a place not Before these events, the Peace officers had no knowledge even of Mengote' identity, let alone
exactly forsaken? the fact (or suspicion) that he was unlawfully carrying a firearm or that he was involved in the
These are certainly not sinister acts. And the setting of the arrest made them less so, if at all. It robbery of Danganan's house.
might have been different if Mengote bad been apprehended at an ungodly hour and in a place In the landmark case of People v. Burgos, 9 this Court declared:
where he had no reason to be, like a darkened alley at 3 o'clock in the morning. But he was Under Section 6(a) of Rule 113, the officer arresting a person who has just committed,
arrested at 11:30 in the morning and in a crowded street shortly after alighting from a passenger is committing, or is about to commit an offense must have personal knowledge of the
jeep with I his companion. He was not skulking in the shadows but walking in the clear light of fact. The offense must also be committed in his presence or within his view. (Sayo v.
Chief of Police, 80 Phil. 859). (Emphasis supplied)
day. There was nothing clandestine about his being on that street at that busy hour in the blaze
xxx xxx xxx
of the noonday sun. In arrests without a warrant under Section 6(b), however, it is not enough that there is
On the other hand, there could have been a number of reasons, all of them innocent, why his reasonable ground to believe that the person to be arrested has committed a crime. A
eyes were darting from side to side and be was holding his abdomen. If they excited suspicion in crime must in fact or actually have been committed first. That a crime has actually
the minds of the arresting officers, as the prosecution suggests, it has nevertheless not been been committed is an essential precondition. It is not enough to suspect that a crime
shown what their suspicion was all about. In fact, the policemen themselves testified that they may have been committed. The fact of the commission of the offense must be
were dispatched to that place only because of the telephone call from the informer that there undisputed. The test of reasonable ground applies only to the identity of the
were "suspicious-looking" persons in that vicinity who were about to commit a robbery at North perpetrator. (Emphasis supplied)
This doctrine was affirmed in Alih v. Castro, 10 thus:
Bay Boulevard. The caller did not explain why he thought the men looked suspicious nor did he
If the arrest was made under Rule 113, Section 5, of the Rules of Court in connection
elaborate on the impending crime. with a crime about to be committed, being committed, or just committed, what was
In the recent case of People v. Malmstedt, 5 the Court sustained the warrantless arrest of the that crime? There is no allegation in the record of such a falsification. Parenthetically,
accused because there was a bulge in his waist that excited the suspicion of the arresting officer it may be observed that under the Revised Rule 113, Section 5(b), the officer making
and, upon inspection, turned out to be a pouch containing hashish. In People v. Claudio, 6 the the arrest must have personal knowledge of the ground therefor as stressed in the
accused boarded a bus and placed the buri bag she was carrying behind the seat of the arresting recent case of People v. Burgos. (Emphasis supplied)
officer while she herself sat in the seat before him. His suspicion aroused, be surreptitiously It would be a sad day, indeed, if any person could be summarily arrested and searched just
examined the bag, which he found to contain marijuana. He then and there made the because he is holding his abdomen, even if it be possibly because of a stomach-ache, or if a
warrantless arrest and seizure that we subsequently upheld on the ground that probable cause peace officer could clamp handcuffs on any person with a shifty look on suspicion that he may
had been sufficiently established. have committed a criminal act or is actually committing or attempting it. This simply cannot be
The case before us is different because there was nothing to support the arresting officers' done in a free society. This is not a police state where order is exalted over liberty or, worse,
suspicion other than Mengote's darting eyes and his hand on his abdomen. By no stretch of the personal malice on the part of the arresting officer may be justified in the name of security.
imagination could it have been inferred from these acts that an offense had just been There is no need to discuss the other issues raised by the accused-appellant as the ruling we
committed, or was actually being committed, or was at least being attempted in their presence. here make is sufficient to sustain his exoneration. Without the evidence of the firearm taken
This case is similar to People v. Aminnudin, 7 where the Court held that the warrantless arrest of from him at the time of his illegal arrest, the prosecution has lost its most important exhibit and
the accused was unconstitutional. This was effected while be was coming down a vessel, to all must therefore fail. The testimonial evidence against Mengote (which is based on the said
appearances no less innocent than the other disembarking passengers. He had not committed firearm) is not sufficient to prove his guilt beyond reasonable doubt of the crime imputed to him.
nor was be actually committing or attempting to commit an offense in the presence of the We commend Atty. Violeta Calvo-Drilon for her able and spirited defense of the accused-
arresting officers. He was not even acting suspiciously. In short, there was no probable cause appellant not only in the brief but also in the reply brief, which she did not have to file but did so
that, as the prosecution incorrectly suggested, dispensed with the constitutional requirement of just the same to stress the constitutional rights of her client. The fact that she was acting only as
a warrant. a counsel de oficio with no expectation of material reward makes her representation even more
Par. (b) is no less applicable because its no less stringent requirements have also not been commendable.
satisfied. The prosecution has not shown that at the time of Mengote's arrest an offense had in The Court feels that if the peace officers had been more mindful of the provisions of the Bill of
fact just been committed and that the arresting officers had personal knowledge of facts Rights, the prosecution of the accused-appellant might have succeeded. As it happened, they
indicating that Mengote had committed it. All they had was hearsay information from the allowed their over-zealousness to get the better of them, resulting in their disregard of the
telephone caller, and about a crime that had yet to be committed. requirements of a valid search and seizure that rendered inadmissible the vital evidence they
The truth is that they did not know then what offense, if at all, had been committed and neither had invalidly seized.
were they aware of the participation therein of the accused-appellant. It was only later, after This should be a lesson to other peace officers. Their impulsiveness may be the very cause of the
Danganan had appeared at the Police headquarters, that they learned of the robbery in his acquittal of persons who deserve to be convicted, escaping the clutches of the law because,
house and of Mengote's supposed involvement therein. 8 As for the illegal possession of the ironically enough, it has not been observed by those who are supposed to enforce it.
firearm found on Mengote's person, the policemen discovered this only after he had been WHEREFORE, the appealed decision is REVERSED and SET ASIDE. The accused-appellant is
searched and the investigation conducted later revealed that he was not its owners nor was he ACQUITTED and ordered released immediately unless he is validly detained for other offenses.
licensed to possess it. No costs.
SO ORDERED.
65

CABALLES VS CA was promised P1,000.00 for the job. Upon crossing a bridge, the two vehicles
This is an appeal by certiorari from the decision1 of respondent Court of Appeals dated separated but in his case, he was intercepted by Sgt. Noceja and Pat. De Castro. When
September 15, 1998 which affirmed the judgment rendered by the Regional Trial Court of Santa they discovered the cables, he told the police officers that the cables were loaded in
Cruz, Laguna, finding herein petitioner, Rudy Caballes y Taiño, guilty beyond reasonable doubt of his jeep by the owner, Resty Fernandez. But despite his explanation, he was ordered to
the crime of theft, and the resolution2 dated November 9, 1998 which denied petitioner's proceed to police headquarters where he was interrogated. The police officers did not
motion for reconsideration. believe him and instead locked him up in jail for a week."4
In an Information3 dated October 16, 1989, petitioner was charged with the crime of theft On April 27, 1993, the court a quo rendered judgment5 the dispositive portion of which reads:
committed as follows: "WHEREFORE, finding the accused guilty beyond reasonable doubt of the crime of
"That on or about the 28th day of June, 1989, in the Municipality of Pagsanjan, and/or Theft of property worthP55,244.45, the Court hereby sentences him to suffer
elsewhere in the Province of Laguna, and within the jurisdiction of this Honorable imprisonment from TWO (2) [YEARS], FOUR (4) MONTHS, and ONE (1) DAY of Prision
Court, the above-named accused, with intent of gain, and without the knowledge and Correccional, as minimum, to TEN (10) YEARS of Prision Mayor, as maximum, to
consent of the owner thereof, the NATIONAL POWER CORPORATION, did then and indemnify the complainant National Power Corporation in the amount of P55, 244.45,
there wilfully, unlawfully and feloniously take, steal and carry away about 630-kg of and to pay the costs."
Aluminum Cable Conductors, valued at P27, 450.00, belonging to and to the damage On appeal, the Court of Appeals affirmed the judgment of conviction but deleted the award for
and prejudice of said owner National Power Corp., in the aforesaid amount. damages on the ground that the stolen materials were recovered and modified the penalty
CONTRARY TO LAW." imposed, to wit:
During the arraignment, petitioner pleaded not guilty and hence, trial on the merits ensued. "WHEREFORE, the appealed decision is hereby AFFIRMED with the modification that
The facts are summarized by the appellate court as follows: appellant RUDY CABALLES is found guilty beyond reasonable doubt as principal in
"[At] about 9:15 p.m. of June 28, 1989, Sgt. Victorino Noceja and Pat. Alex de Castro, theft, defined and penalized under Articles 308 and 309, par. 1, Revised Penal Code,
while on a routine patrol in Barangay Sampalucan, Pagsanjan, Laguna, spotted a and there being no modifying circumstances, he is hereby meted an indeterminate
passenger jeep unusually covered with "kakawati" leaves. penalty of Four (4) years, Nine (9) months and Eleven (11) days of prision correccional,
Suspecting that the jeep was loaded with smuggled goods, the two police officers as minimum term, to Eight (8) years, Eight (8) months and one (1) day of prision mayor,
flagged down the vehicle. The jeep was driven by appellant. When asked what was as maximum term. No civil indemnity and no costs."6
loaded on the jeep, he did not answer; he appeared pale and nervous. Petitioner comes before us and raises the following issues:
With appellant's consent, the police officers checked the cargo and they discovered "(a) Whether or not the constitutional right of petitioner was violated when the police
bundles of 3.08 mm aluminum/galvanized conductor wires exclusively owned by officers searched his vehicle and seized the wires found therein without a search
National Power Corporation (NPC). The conductor wires weighed 700 kilos and valued warrant and when samples of the wires and references to them were admitted in
at P55, 244.45. Noceja asked appellant where the wires came from and appellant evidence as basis for his conviction;
answered that they came from Cavinti, a town approximately 8 kilometers away from (b) Whether or not respondent Court erred in rejecting petitioner's defense that he
Sampalucan. Thereafter, appellant and the vehicle with the high-voltage wires were was engaged in an entrapment operation and in indulging in speculation and
brought to the Pagsanjan Police Station. Danilo Cabale took pictures of the appellant conjecture in rejecting said defense; and
and the jeep loaded with the wires which were turned over to the Police Station (c) Whether or not the evidence of the prosecution failed to establish the guilt of
Commander of Pagsanjan, Laguna. Appellant was incarcerated for 7 days in the petitioner beyond reasonable doubt and thus failed to overcome the constitutional
Municipal jail. right of petitioner to presumption of innocence."
In defense, appellant interposed denial and alibi. He testified that he is a driver and The conviction or acquittal of petitioner hinges primarily on the validity of the warrantless search
resident of Pagsanjan, Laguna; a NARCOM civilian agent since January, 1988 although and seizure made by the police officers, and the admissibility of the evidence obtained by virtue
his identification card (ID) has already expired. In the afternoon of June 28, 1989, while thereof.
he was driving a passenger jeepney, he was stopped by one Resty Fernandez who In holding that the warrantless search and seizure is valid, the trial court ruled that:
requested him to transport in his jeepney conductor wires which were in Cavinti, "As his last straw of argument, the accused questions the constitutionality of the
Laguna. He told Resty to wait until he had finished his last trip for the day from Santa search and validity of his arrest on the ground that no warrant was issued to that
Cruz, Laguna. On his way to Santa Cruz, Laguna, he dropped by the NARCOM effect. The Court cannot again sustain such view. In the case of People v. Lo Ho [Wing],
headquarters and informed his superior, Sgt. Callos, that something unlawful was G.R. No. 88017, January 21, 1991, it has been held that 'considering that before a
going to happen. Sgt. Callos advised him to proceed with the loading of the wires and warrant can be obtained, the place, things and persons to be searched must be
that the former would act as back-up and intercept the vehicle at the Sambat Patrol described to the satisfaction of the issuing judge - a requirement which borders on the
Base in Pagsanjan. impossible in the case of smuggling effected by the use of a moving vehicle that can
After receiving those instructions, he went back to see Resty. Although Resty had his transport contraband from one place to another with impunity, a warrantless search of
own vehicle, its tires were old so the cable wires were loaded in appellant's jeep and a moving vehicle is justified on grounds of practicability.' The doctrine is not of recent
covered with kakawati leaves. The loading was done by about five (5) masked men. He vintage. In the case of Valmonte vs. de Villa, G.R. No. 83988, May 24, 1990 (Resolution
66

on Motion for Reconsideration, September 29, 1989), it was ruled that 'automobiles Highly regulated by the government, the vehicle's inherent mobility reduces expectation of
because of their mobility may be searched without a warrant upon facts not justifying privacy especially when its transit in public thoroughfares furnishes a highly reasonable suspicion
warrantless search of a resident or office. x x x To hold that no criminal can, in any amounting to probable cause that the occupant committed a criminal activity.15 Thus, the rules
case, be arrested and searched for the evidence and tokens of his crime without a governing search and seizure have over the years been steadily liberalized whenever a moving
warrant, would be to leave society, to a large extent, at the mercy of the shrewdest, vehicle is the object of the search on the basis of practicality. This is so considering that before a
the most expert, and the most depraved of criminals, facilitating their escape in many warrant could be obtained, the place, things and persons to be searched must be described to
instances' (Ibid.). In Umil v. Ramos, 187 SCRA 311, and People vs. Ortiz, 191 SCRA 836, the satisfaction of the issuing judge — a requirement which borders on the impossible in the
the Supreme Court held that a search may be made even without a warrant where the case of smuggling effected by the use of a moving vehicle that can transport contraband from
accused is caught in flagrante. Under the circumstances, the police officers are not only one place to another with impunity. We might add that a warrantless search of a moving vehicle
authorized but are also under obligation to arrest the accused even without a is justified on the ground that it is not practicable to secure a warrant because the vehicle can be
warrant."7 quickly moved out of the locality or jurisdiction in which the warrant must be sought.16 Searches
Petitioner contends that the flagging down of his vehicle by police officers who were on routine without warrant of automobiles is also allowed for the purpose of preventing violations of
patrol, merely on "suspicion" that "it might contain smuggled goods," does not constitute smuggling or immigration laws, provided such searches are made at borders or 'constructive
probable cause that will justify a warrantless search and seizure. He insists that, contrary to the borders' like checkpoints near the boundary lines of the State.17
findings of the trial court as adopted by the appellate court, he did not give any consent, express The mere mobility of these vehicles, however, does not give the police officers unlimited
or implied, to the search of the vehicle. Perforce, any evidence obtained in violation of his right discretion to conduct indiscriminate searches without warrants if made within the interior of the
against unreasonable search and seizure shall be deemed inadmissible. territory and in the absence of probable cause.18 Still and all, the important thing is that there
Enshrined in our Constitution is the inviolable right of the people to be secure in their persons was probable cause to conduct the warrantless search, which must still be present in such a
and properties against unreasonable searches and seizures, as defined under Section 2, Article III case.
thereof, which reads: Although the term eludes exact definition, probable cause signifies a reasonable ground of
"Sec. 2. The right of the people to be secure in their persons, houses, papers, and suspicion supported by circumstances sufficiently strong in themselves to warrant a cautious
effects against unreasonable searches and seizures of whatever nature and for any man's belief that the person accused is guilty of the offense with which he is charged; or the
purpose shall be inviolable, and no search warrant or warrant of arrest shall issue existence of such facts and circumstances which could lead a reasonably discreet and prudent
except upon probable cause to be determined personally by the judge after man to believe that an offense has been committed and that the items, articles or objects sought
examination under oath or affirmation of the complainant and the witnesses he may in connection with said offense or subject to seizure and destruction by law is in the place to be
produce, and particularly describing the place to be searched and the persons or things searched.19 The required probable cause that will justify a warrantless search and seizure is not
to be seized." determined by a fixed formula but is resolved according to the facts of each case.20
The exclusionary rule under Section 3(2), Article III of the Constitution bars the admission of One such form of search of moving vehicles is the "stop-and-search" without warrant at military
evidence obtained in violation of such right. or police checkpoints which has been declared to be not illegal per se,21 for as long as it is
The constitutional proscription against warrantless searches and seizures is not absolute but warranted by the exigencies of public order22 and conducted in a way least intrusive to
admits of certain exceptions, namely: (1) warrantless search incidental to a lawful arrest motorists.23 A checkpoint may either be a mere routine inspection or it may involve an extensive
recognized under Section 12, Rule 126 of the Rules of Court and by prevailing jurisprudence;8 (2) search.
seizure of evidence in plain view;9 (3) search of moving vehicles;10 (4) consented warrantless Routine inspections are not regarded as violative of an individual's right against unreasonable
search;11 (5) customs search; (6) stop and frisk situations (Terry search);12 and (7) exigent and search. The search which is normally permissible in this instance is limited to the following
emergency circumstances.13 instances: (1) where the officer merely draws aside the curtain of a vacant vehicle which is
In cases where warrant is necessary, the steps prescribed by the Constitution and reiterated in parked on the public fair grounds;24 (2) simply looks into a vehicle;25 (3) flashes a light therein
the Rules of Court must be complied with. In the exceptional events where warrant is not without opening the car's doors;26 (4) where the occupants are not subjected to a physical or
necessary to effect a valid search or seizure, or when the latter cannot be performed except body search;27 (5) where the inspection of the vehicles is limited to a visual search or visual
without a warrant, what constitutes a reasonable or unreasonable search or seizure is purely a inspection;28 and (6) where the routine check is conducted in a fixed area.29
judicial question, determinable from the uniqueness of the circumstances involved, including the None of the foregoing circumstances is obtaining in the case at bar. The police officers did not
purpose of the search or seizure, the presence or absence of probable cause, the manner in merely conduct a visual search or visual inspection of herein petitioner's vehicle. They had to
which the search and seizure was made, the place or thing searched and the character of the reach inside the vehicle, lift the kakawati leaves and look inside the sacks before they were able
articles procured.14 to see the cable wires. It cannot be considered a simple routine check.
It is not controverted that the search and seizure conducted by the police officers in the case at In the case of United States vs. Pierre,30 the Court held that the physical intrusion of a part of
bar was not authorized by a search warrant. The main issue is whether the evidence taken from the body of an agent into the vehicle goes beyond the area protected by the Fourth Amendment,
the warrantless search is admissible against the appellant. Without said evidence, the to wit:
prosecution cannot prove the guilt of the appellant beyond reasonable doubt.1âwphi1.nêt "The Agent . . . stuck his head through the driver's side window. The agent thus
I. Search of moving vehicle effected a physical intrusion into the vehicle. . . [W]e are aware of no case holding that
67

an officer did not conduct a search when he physically intruded part of his body into a We hold that the fact that the vehicle looked suspicious simply because it is not common for
space in which the suspect had a reasonable expectation of privacy. [The] Agent['s] . . . such to be covered with kakawati leaves does not constitute "probable cause" as would justify
physical intrusion allowed him to see and to smell things he could not see or smell the conduct of a search without a warrant.
from outside the vehicle. . . In doing so, his inspection went beyond that portion of the In People vs. Chua Ho San,36 we held that the fact that the watercraft used by the accused was
vehicle which may be viewed from outside the vehicle by either inquisitive passersby different in appearance from the usual fishing boats that commonly cruise over the Bacnotan
or diligent police officers, and into the area protected by the Fourth amendment, just seas coupled with the suspicious behavior of the accused when he attempted to flee from the
as much as if he had stuck his head inside the open window of a home." police authorities do not sufficiently establish probable cause. Thus:
On the other hand, when a vehicle is stopped and subjected to an extensive search, such a "In the case at bar, the Solicitor General proposes that the following details are
warrantless search would be constitutionally permissible only if the officers conducting the suggestive of probable cause - persistent reports of rampant smuggling of firearm and
search have reasonable or probable cause to believe, before the search, that either the motorist other contraband articles, CHUA's watercraft differing in appearance from the usual
is a law-offender or they will find the instrumentality or evidence pertaining to a crime in the fishing boats that commonly cruise over the Bacnotan seas, CHUA's illegal entry into
vehicle to be searched.31 the Philippines x x x, CHUA's suspicious behavior, i.e., he attempted to flee when he
This Court has in the past found probable cause to conduct without a judicial warrant an saw the police authorities, and the apparent ease by which CHUA can return to and
extensive search of moving vehicles in situations where (1) there had emanated from a package navigate his speedboat with immediate dispatch towards the high seas, beyond the
the distinctive smell of marijuana; (2) agents of the Narcotics Command ("Narcom") of the reach of Philippine laws.
Philippine National Police ("PNP") had received a confidential report from informers that a This Court, however, finds that these do not constitute "probable cause." None of the
sizeable volume of marijuana would be transported along the route where the search was telltale clues, e.g., bag or package emanating the pungent odor of marijuana or other
conducted; (3) Narcom agents had received information that a Caucasian coming from Sagada, prohibited drug, confidential report and/or positive identification by informers of
Mountain Province, had in his possession prohibited drugs and when the Narcom agents courier of prohibited drug and/or the time and place where they will transport/deliver
confronted the accused Caucasian, because of a conspicuous bulge in his waistline, he failed to the same, suspicious demeanor or behavior, and suspicious bulge in the waist -
present his passport and other identification papers when requested to do so; (4) Narcom agents accepted by this Court as sufficient to justify a warrantless arrest exists in this case.
had received confidential information that a woman having the same physical appearance as There was no classified information that a foreigner would disembark at Tammocalao
that of the accused would be transporting marijuana;32 (5) the accused who were riding a beach bearing prohibited drug on the date in question. CHUA was not identified as a
jeepney were stopped and searched by policemen who had earlier received confidential reports drug courier by a police informer or agent. The fact that the vessel that ferried him to
that said accused would transport a large quantity of marijuana; and (6) where the moving shore bore no resemblance to the fishing boats of the area did not automatically
vehicle was stopped and searched on the basis of intelligence information and clandestine mark him as in the process of perpetrating an offense. x x x." (emphasis supplied)
reports by a deep penetration agent or spy - one who participated in the drug smuggling In addition, the police authorities do not claim to have received any confidential report or tipped
activities of the syndicate to which the accused belonged - that said accused were bringing information that petitioner was carrying stolen cable wires in his vehicle which could otherwise
prohibited drugs into the country.33 have sustained their suspicion. Our jurisprudence is replete with cases where tipped information
In the case at bar, the vehicle of the petitioner was flagged down because the police officers who has become a sufficient probable cause to effect a warrantless search and
were on routine patrol became suspicious when they saw that the back of the vehicle was seizure.37 Unfortunately, none exists in this case.
covered with kakawati leaves which, according to them, was unusual and uncommon. II. Plain view doctrine
Pat. Alex de Castro recounted the incident as follows: It cannot likewise be said that the cable wires found in petitioner's vehicle were in plain view,
"ATTY. SANTOS making its warrantless seizure valid.
Q Now on said date and time do you remember of any unusual incident while you were Jurisprudence is to the effect that an object is in plain view if the object itself is plainly exposed
performing your duty? to sight. Where the object seized was inside a closed package, the object itself is not in plain
A Yes, sir, at that time and date myself and Police Sgt. Noceja were conducting patrol in the
view and therefore cannot be seized without a warrant. However, if the package proclaims its
said place when we spotted a suspicious jeepney so we stopped the jeepney and searched the
load of the jeepney and we found out (sic) these conductor wires. contents, whether by its distinctive configuration, its transparency, or if its contents are obvious
Q You mentioned about the fact that when you saw the jeepney you became suspicious, why to an observer, then the contents are in plain view and may be seized. In other words, if the
did you become suspicious? package is such that an experienced observer could infer from its appearance that it contains the
A Because the cargo was covered with leaves and branches, sir. prohibited article, then the article is deemed in plain view. It must be immediately apparent to
Q When you became suspicious upon seeing those leaves on top of the load what did you do the police that the items that they observe may be evidence of a crime, contraband or otherwise
next, if any? subject to seizure.38
A We stopped the jeepney and searched the contents thereof, sir."34 It is clear from the records of this case that the cable wires were not exposed to sight because
The testimony of Victorino Noceja did not fare any better:
they were placed in sacks39 and covered with leaves. The articles were neither transparent nor
"ATTY SANTOS
Q When you saw the accused driving the said vehicle, what did you do? immediately apparent to the police authorities. They had no clue as to what was hidden
A Because I saw that the vehicle being drawn by Caballes was covered by kakawati leaves, I underneath the leaves and branches. As a matter of fact, they had to ask petitioner what was
became suspicious since such vehicle should not be covered by those and I flagged him, sir."35
68

loaded in his vehicle. In such a case, it has been held that the object is not in plain view which A He did not answer and I observed him to be pale, "nagpapamutla" (sic), so I told
could have justified mere seizure of the articles without further search.40 him I will look at the contents of his vehicle and he answered in the positive.
III. Consented search Q And after you saw for yourself the aluminum wires loaded on the jeep, what did
Petitioner contends that the statement of Sgt. Victorino Noceja that he checked the vehicle you do?
"with the consent of the accused" is too vague to prove that petitioner consented to the search. A I asked him where those wires came from and he answered those came from the
He claims that there is no specific statement as to how the consent was asked and how it was Cavinti area, sir."48
given, nor the specific words spoken by petitioner indicating his alleged "consent." At most, This Court is not unmindful of cases upholding the validity of consented warrantless searches
there was only an implied acquiescence, a mere passive conformity, which is no "consent" at all and seizure. But in these cases, the police officers' request to search personnel effects was orally
within the purview of the constitutional guarantee. articulated to the accused and in such language that left no room for doubt that the latter fully
Doubtless, the constitutional immunity against unreasonable searches and seizures is a personal understood what was requested. In some instance, the accused even verbally replied to the
right which may be waived. The consent must be voluntary in order to validate an otherwise request demonstrating that he also understood the nature and consequences of such request. 49
illegal detention and search, i.e., the consent is unequivocal, specific, and intelligently given, In Asuncion vs. Court of Appeals,50 the apprehending officers sought the permission of
uncontaminated by any duress or coercion.41 Hence, consent to a search is not to be lightly petitioner to search the car, to which the latter agreed. Petitioner therein himself freely gave his
inferred, but must be shown by clear and convincing evidence.42 The question whether a consent consent to said search. In People vs. Lacerna,51 the appellants who were riding in a taxi were
to a search was in fact voluntary is a question of fact to be determined from the totality of all the stopped by two policemen who asked permission to search the vehicle and the appellants readily
circumstances.43 Relevant to this determination are the following characteristics of the person agreed. In upholding the validity of the consented search, the Court held that appellant himself
giving consent and the environment in which consent is given: (1) the age of the defendant; (2) who was "urbanized in mannerism and speech" expressly said that he was consenting to the
whether he was in a public or secluded location; (3) whether he objected to the search or search as he allegedly had nothing to hide and had done nothing wrong. In People vs.
passively looked on;44 (4) the education and intelligence of the defendant; (5) the presence of Cuizon,52 the accused admitted that they signed a written permission stating that they freely
coercive police procedures; (6) the defendant's belief that no incriminating evidence will be consented to the search of their luggage by the NBI agents to determine if they were carrying
found;45 (7) the nature of the police questioning; (8) the environment in which the questioning shabu. In People vs. Montilla,53 it was held that the accused spontaneously performed
took place; and (9) the possibly vulnerable subjective state of the person consenting.46 It is the affirmative acts of volition by himself opening the bag without being forced or intimidated to do
State which has the burden of proving, by clear and positive testimony, that the necessary so, which acts should properly be construed as a clear waiver of his right. In People vs.
consent was obtained and that it was freely and voluntarily given. 47 Omaweng,54 the police officers asked the accused if they could see the contents of his bag to
In the case at bar, Sgt. Victorino Noceja testified on the manner in which the search was which the accused said "you can see the contents but those are only clothings." Then the
conducted in this wise: policemen asked if they could open and see it, and accused answered "you can see it." The Court
"WITNESS said there was a valid consented search.1âwphi1.nêt
Q On June 28, 1989, where were you? In case of consented searches or waiver of the constitutional guarantee against obtrusive
A We were conducting patrol at the poblacion and some barangays, sir. searches, it is fundamental that to constitute a waiver, it must first appear that (1) the right
xxx xxx xxx exists; (2) that the person involved had knowledge, either actual or constructive, of the existence
Q After conducting the patrol operation, do you remember of any unusual incident of such right; and (3) the said person had an actual intention to relinquish the right.55
on said date and time? In the case at bar, the evidence is lacking that the petitioner intentionally surrendered his right
A Yes, sir. against unreasonable searches. The manner by which the two police officers allegedly obtained
Q What is that incident? the consent of petitioner for them to conduct the search leaves much to be desired. When
A While I was conducting my patrol at barangay Sampalucan, I saw Rudy Caballes petitioner's vehicle was flagged down, Sgt. Noceja approached petitioner and "told him I will
driving a vehicle and the vehicle contained aluminum wires, sir. look at the contents of his vehicle and he answered in the positive." We are hard put to believe
xxx xxx xxx that by uttering those words, the police officers were asking or requesting for permission that
Q When you saw the accused driving the said vehicle, what did you do? they be allowed to search the vehicle of petitioner. For all intents and purposes, they
A Because I saw that the vehicle being driven by Caballes was covered by kakawati wereinforming, nay, imposing upon herein petitioner that they will search his vehicle. The
leaves, I became suspicious since such vehicle should not be covered by those and I "consent" given under intimidating or coercive circumstances is no consent within the purview of
flagged him, sir. the constitutional guaranty. In addition, in cases where this Court upheld the validity of
Q Did the vehicle stop? consented search, it will be noted that the police authorities expressly asked, in no uncertain
A Yes, sir, and after said vehicle stop[ped], I removed the cover of said vehicle and terms, for the consent of the accused to be searched. And the consent of the accused was
by so doing, I saw the aluminum wires. established by clear and positive proof. In the case of herein petitioner, the statements of the
Q Before you saw the aluminum wires, did you talk to the accused? police officers were not asking for his consent; they were declaring to him that they will look
A Yes, sir, I asked him what his load was. inside his vehicle. Besides, it is doubtful whether permission was actually requested and granted
Q What was the answer of Caballes? because when Sgt. Noceja was asked during his direct examination what he did when the vehicle
of petitioner stopped, he answered that he removed the cover of the vehicle and saw the
69

aluminum wires. It was only after he was asked a clarificatory question that he added that he
told petitioner he will inspect the vehicle. To our mind, this was more of an afterthought.
Likewise, when Pat. de Castro was asked twice in his direct examination what they did when they
stopped the jeepney, his consistent answer was that they searched the vehicle. He never
testified that he asked petitioner for permission to conduct the search.56
Neither can petitioner's passive submission be construed as an implied acquiescence to the
warrantless search. In People vs. Barros,57 appellant Barros, who was carrying a carton box,
boarded a bus where two policemen were riding. The policemen inspected the carton and found
marijuana inside. When asked who owned the box, appellant denied ownership of the box and
failed to object to the search. The Court there struck down the warrantless search as illegal and
held that the accused is not to be presumed to have waived the unlawful search conducted
simply because he failed to object, citing the ruling in the case of People vs. Burgos,58 to wit:
"As the constitutional guaranty is not dependent upon any affirmative act of the
citizen, the courts do not place the citizens in the position of either contesting an
officer's authority by force, or waiving his constitutional rights; but instead they hold
that a peaceful submission to a search or seizure is not a consent or an invitation
thereto, but is merely a demonstration of regard for the supremacy of the law."
Casting aside the cable wires as evidence, the remaining evidence on record are insufficient to
sustain petitioner's conviction. His guilt can only be established without violating the
constitutional right of the accused against unreasonable search and seizure.
WHEREFORE, the impugned decision is REVERSED and SET ASIDE, and accused Rudy Caballes is
herebyACQUITTED of the crime charged. Cost de oficio.
SO ORDERED.
70

MANALILI VS CA because of information that drug addicts were roaming the area in front of the Kalookan City
When dealing with a rapidly unfolding and potentially criminal situation in the city streets Cemetery.
where unarguably there is no time to secure an arrest or a search warrant, policemen should Upon reaching the Kalookan City Cemetery, the policemen alighted from their vehicle. They then
employ limited, flexible responses -- like stop-and-frisk -- which are graduated in relation to the chanced upon a male person in front of the cemetery who appeared high on drugs. The male
amount of information they possess, the lawmen being ever vigilant to respect and not to violate person was observed to have reddish eyes and to be walking in a swaying manner. When this
or to treat cavalierly the citizens constitutional rights against unreasonable arrest, search and male person tried to avoid the policemen, the latter approached him and introduced themselves
seizure. as police officers. The policemen then asked the male person what he was holding in his
The Case hands. The male person tried to resist. Pat. Romeo Espiritu asked the male person if he could see
This rule is reiterated as we resolve this petition for review on certiorari under Rule 45 of what said male person had in his hands. The latter showed the wallet and allowed Pat. Romeo
the Rules of Court, seeking the reversal of the Decision of the Court of Appeals dated April 19, Espiritu to examine the same. Pat. Espiritu took the wallet and examined it. He found suspected
1993 and its Resolution dated January 20, 1994 in CA G.R. CR No. 07266, entitled People of the crushed marijuana residue inside. He kept the wallet and its marijuana contents.
Philippines vs. Alain Manalili y Dizon. The male person was then brought to the Anti-Narcotics Unit of the Kalookan City Police
In an Information dated April 11, 1988,[1] Petitioner Alain Manalili y Dizon was charged by Headquarters and was turned over to Cpl. Wilfredo Tamondong for investigation. Pat. Espiritu
Assistant Caloocan City Fiscal E. Juan R. Bautista with violation of Section 8, Article II of Republic also turned over to Cpl. Tamondong the confiscated wallet and its suspected marijuana
Act No. 6425, allegedly committed as follows:[2] contents. The man turned out to be the accused ALAIN MANALILI y DIZON.
That on or about the 11th day of April 1988 in Caloocan City, MM, Philippines and within the Upon receipt of the confiscated suspected marijuana residue from Pat. Espiritu, Cpl. Tamondong
jurisdiction of this Honorable Court, the above-named accused without any authority of law, did wrapped the same with a white sheet of paper on which he wrote Evidence A 4/11/88 Alain
then and there wilfully, unlawfully and feloniously have in his custody, possession and control Manalili. The white sheet of paper was marked as Exhibit E-3. The residue was originally wrapped
crushed marijuana residue, which is a prohibited drug and knowing the same to be such. in a smaller sheet of folded paper. (Exhibit E-4).
Contrary to Law. Cpl. Tamondong next prepared a referral slip addressed to the NBI Forensic Chemistry Section
Upon his arraignment on April 21, 1988, appellant pleaded not guilty to the charge. [3] With requesting a chemical analysis of the subject marijuana residue (Exhibit D). Cpl. Tamondong
the agreement of the public prosecutor, appellant was released after filing a P10,000.00 bail thereafter prepared a Joint Affidavit of the apprehending policemen (Exhibit A). Pat. Angel
bond.[4] After trial in due course, the Regional Trial Court of Caloocan City, Branch 124, acting as Lumabas handcarried the referral slip (Exhibit D) to the National Bureau of Investigation (NBI),
a Special Criminal Court, rendered on May 19, 1989 a decision[5] convicting appellant of illegal including the subject marijuana residue for chemical analysis. The signature of Pat. Lumabas
possession of marijuana residue. The dispositive portion of the decision reads:[6] appears on the left bottom corner of Exhibit D.
WHEREFORE, in view of all the foregoing, this Court finds the accused ALAIN MANALILI Y DIZON The Forensic Chemistry Section of the NBI received the aforesaid referral slip and the subject
guilty beyond reasonable doubt of violation of Section 8, Article II, of Republic Act No. 6425, as marijuana residue at 7:40 oclock in the evening of April 11, 1988 as shown on the stamped
amended (Illegal Possession of Marijuana residue), and hereby snetences (sic) said accused to portion of Exhibit D.
suffer imprisonment of SIX (6) YEARS and ONE (1) DAY; and to pay a fine of P6,000.00; and to pay It was NBI Aida Pascual who conducted the microscopic and chemical examinations of the
the costs. specimen which she identified. (Exhibit E)[13] Mrs. Pascual referred to the subject specimen as
xxx xxx xxx. crushed marijuana leaves in her Certification dated April 11, 1988 (Exhibit F).[14] These crushed
Appellant remained on provisional liberty.[7] Atty. Benjamin Razon, counsel for the marijuana leaves gave positive results for marijuana, according to the Certificate.
defense, filed a Notice of Appeal[8] dated May 31, 1989. On April 19, 1993, Respondent Mrs. Pascual also conducted a chromatographic examination of the specimen. In this
Court[9]promulgated its assailed Decision, denying the appeal and affirming the trial court:[10] examination, she also found that the crushed marijuana leaves gave positive results for
ACCORDINGLY, the decision appealed from dated May 19, 1989 is hereby AFFIRMED in all marijuana. She then prepared a Final Report of her examinations (Exhibit G).
respects. Costs against appellant. After conducting the examinations, Ms. Pascual placed the specimen in a white letter-envelope
Respondent Court[11] denied reconsideration via its assailed Resolution dated January 20, and sealed it. (Exhibit E). She then wrote identification notes on this letter-envelope. (Exhibit E-
1994, disposing: 1).
ACCORDINGLY, accused-appellants motion for reconsideration is, as is hereby DENIED. Pat. Lumabas carried the Certification marked as Exhibit F from the NBI Forensic Chemistry
The Facts Section to Cpl. Tamondong. Upon receipt thereof, Cpl. Tamondong prepared a referral slip
Version of the Prosecution addressed to the City Fiscal of Kalookan City. (Exhibit C)
The facts, as found by the trial court, are as follows:[12] On rebuttal, Pat. Espiritu testified that appellant was not riding a tricycle but was walking
At about 2:10 oclock in the afternoon of April 11, 1988, policemen from the Anti-Narcotics Unit in front of the cemetery when he was apprehended.[15]
of the Kalookan City Police Station were conducting a surveillance along A. Mabini street, Version of the Defense
Kalookan City, in front of the Kalookan City Cemetery. The policemen were Pat. Romeo Espiritu The trial court summarized the testimonies of the defense witnesses as follows:[16]
and Pat. Anger Lumabas and a driver named Arnold Enriquez was driving a Tamaraw vehicle At about 2:00 oclock in the afternoon of April 11, 1988, the accused ALAIN MANALILI was aboard
which was the official car of the Police Station of Kalookan City. The surveillance was being made a tricycle at A. Mabini street near the Kalookan City Cemetery on the way to his boarding
house. Three policemen ordered the driver of the tricycle to stop because the tricycle driver and
71

his lone passenger were under the influence of marijuana. The policemen brought the accused contention -- that he could not be convicted of illegal possession of marijuana residue -- to be
and the tricycle driver inside the Ford Fiera which the policemen were riding in. The policemen without merit, because the forensic chemist reported that what she examined were marijuana
then bodily searched the accused and the tricycle driver. At this point, the accused asked the leaves.
policemen why he was being searched and the policemen replied that he (accused) was carrying Issues
marijuana. However, nothing was found on the persons of the accused and the driver. The Petitioner assigns the following errors on the part of Respondent Court:
policemen allowed the tricycle driver to go while they brought the accused to the police I
headquarters at Kalookan City where they said they would again search the accused. The Court of Appeals erred in upholding the findings of fact of the trial court.
On the way to the police headquarters, the accused saw a neighbor and signaled the latter to II
follow him. The neighbor thus followed the accused to the Kalookan City Police The Court of Appeals erred in upholding the conviction of (the) accused (and) in ruling that
Headquarters. Upon arrival thereat, the accused was asked to remove his pants in the presence the guilt of the accused had been proved (beyond) reasonable doubt.
of said neighbor and another companion. The policemen turned over the pants of the accused III
over a piece of bond paper trying to look for marijuana. However, nothing was found, except for The Court of Appeals erred in not ruling that the inconsistencies in the testimonies of the
some dirt and dust. This prompted the companion of the neighbor of the accused to tell the prosecution witnesses were material and substantial and not minor.
policemen to release the accused. The accused was led to a cell.The policemen later told the IV
accused that they found marijuana inside the pockets of his pants. The Court of Appeals erred in not appreciating the evidence that the accused was framed
At about 5:00 oclock in the afternoon on the same day, the accused was brought outside the cell for the purpose of extorting money.
and was led to the Ford Fiera. The accused was told by the policemen to call his parents in order V
to settle the case. The policemen who led the accused to the Ford Fiera were Pat. Lumabas, Pat. The Court of Appeals erred in not acquitting the accused when the evidence presented is
Espiritu and Cpl. Tamondong. Pat. Lumabas was the policeman who told the accused to call his consistent with both innocence and guilt.
parents. The accused did not call his parents and he told the policemen that his parents did not VI
have any telephone. The Court of Appeals erred in admitting the evidence of the prosecution which are
At about 5:30 oclock in the afternoon of the same day, the accused was brought in the office of inadmissible in evidence.
an inquest Fiscal. There, the accused told the Fiscal that no marijuana was found on his person Restated more concisely, petitioner questions (1) the admissibility of the evidence against
but the Fiscal told the accused not to say anything. The accused was then brought back to the him, (2) the credibility of prosecution witnesses and the rejection by the trial and the appellate
Kalookan City Jail. courts of the defense of extortion, and (3) the sufficiency of the prosecution evidence to sustain
Loreto Medenilla, the tricycle driver who was allegedly with the accused when he and the his conviction.
accused were stopped by policemen and then bodily searched on April 11, 1988, testified. He The Courts Ruling
said that the policemen found nothing either on his person or on the person of the accused The petition has no merit.
when both were searched on April 11, 1988. First Issue: Admissibility of the Evidence Seized During a Stop-and-Frisk
Roberto Abes, a neighbor of the accused, testified that he followed the accused at the Kalookan Petitioner protests the admission of the marijuana leaves found in his possession,
City Police Headquarters on April 11, 1988. He said that the police searched the accused who contending that they were products of an illegal search. The Solicitor General, in his Comment,
was made to take off his pants at the police headquarters but no marijuana was found on the dated July 5, 1994, which was adopted as memorandum for respondent, counters that the
body of the accused. inadmissibility of the marijuana leaves was waived because petitioner never raised this issue in
Appellant, who was recalled to the stand as sur-rebuttal witness, presented several the proceedings below nor did he object to their admissibility in evidence. He adds that, even
pictures showing that tricycles were allowed to ply in front of the Caloocan Cemetery.[17] assuming arguendo that there was no waiver, the search was legal because it was incidental to a
The Rulings of the Trial and the Appellate Courts warrantless arrest under Section 5 (a), Rule 113 of the Rules of Court.
The trial court convicted petitioner of illegal possession of marijuana residue largely on the We disagree with petitioner and hold that the search was valid, being akin to a stop-and-
strength of the arresting officers testimony. Patrolmen Espiritu and Lumabas were neutral and frisk. In the landmark case of Terry vs. Ohio,[18] a stop-and-frisk was defined as the vernacular
disinterested witnesses, testifying only on what transpired during the performance of their designation of the right of a police officer to stop a citizen on the street, interrogate him, and pat
duties. Substantially, they asserted that the appellant was found to be in possession of a him for weapon(s):
substance which was later identified as crushed marijuana residue. x x x (W)here a police officer observes an unusual conduct which leads him reasonably to
The trial court disbelieved appellants defense that this charge was merely trumped up, conclude in light of his experience that criminal activity may be afoot and that the persons with
because the appellant neither took any legal action against the allegedly erring policemen nor whom he is dealing may be armed and presently dangerous, where in the course of investigating
moved for a reinvestigation before the city fiscal of Kalookan City. this behavior he identified himself as a policeman and makes reasonable inquiries, and where
On appeal, Respondent Court found no proof that the decision of the trial court was based nothing in the initial stages of the encounter serves to dispel his reasonable fear for his own or
on speculations, surmises or conjectures. On the alleged serious discrepancies in the testimonies others safety, he is entitled for the protection of himself and others in the area to conduct a
of the arresting officers, the appellate court ruled that the said inconsistencies were carefully limited search of the outer clothing of such persons in an attempt to discover weapons
insubstantial to impair the essential veracity of the narration. It further found petitioners which might be used to assault him. Such a search is a reasonable search under the Fourth
72

Amendment, and any weapon seized may properly be introduced in evidence against the person instances where a search and seizure can be effected without necessarily being preceded by an
from whom they were taken.[19] arrest, one of which is stop-and-frisk. In said case, members of the Integrated National Police of
In allowing such a search, the United States Supreme Court held that the interest of Davao stopped petitioner, who was carrying a buri bag and acting suspiciously. They found inside
effective crime prevention and detection allows a police officer to approach a person, in petitioners bag one .38-cal. revolver with two rounds of live ammunition, two live ammunitions
appropriate circumstances and manner, for purposes of investigating possible criminal behavior for a .22-cal. gun and a tear gas grenade. In upholding the legality of the search, the Court said
even though there is insufficient probable cause to make an actual arrest. This was the legitimate that to require the police officers to search the bag only after they had obtained a search
investigative function which Officer McFadden discharged in that case, when he approached warrant might prove to be useless, futile and much too late under the circumstances. In such a
petitioner and his companion whom he observed to have hovered alternately about a street situation, it was reasonable for a police officer to stop a suspicious individual briefly in order to
corner for an extended period of time, while not waiting for anyone; paused to stare in the same determine his identity or to maintain the status quo while obtaining more information, rather
store window roughly 24 times; and conferred with a third person. It would have been sloppy than to simply shrug his shoulders and allow a crime to occur.
police work for an officer of 30 years experience to have failed to investigate this behavior In the case at hand, Patrolman Espiritu and his companions observed during their
further. surveillance that appellant had red eyes and was wobbling like a drunk along the Caloocan City
In admitting in evidence two guns seized during the stop-and-frisk, the US Supreme Court Cemetery, which according to police information was a popular hangout of drug addicts. From
held that what justified the limited search was the more immediate interest of the police officer his experience as a member of the Anti-Narcotics Unit of the Caloocan City Police, such
in taking steps to assure himself that the person with whom he was dealing was not armed with suspicious behavior was characteristic of drug addicts who were high. The policemen therefore
a weapon that could unexpectedly and fatally be used against him. had sufficient reason to stop petitioner to investigate if he was actually high on drugs. During
It did not, however, abandon the rule that the police must, whenever practicable, obtain such investigation, they found marijuana in petitioners possession:[25]
advance judicial approval of searches and seizures through the warrant procedure, excused only FISCAL RALAR:
by exigent circumstances. Q And why were you conducting surveillance in front of the Caloocan Cemetery, Sangandaan,
In Philippine jurisprudence, the general rule is that a search and seizure must be validated Caloocan City?
by a previously secured judicial warrant; otherwise, such search and seizure is unconstitutional A Because there were some informations that some drug dependents were roaming around at
A. Mabini Street in front of the Caloocan Cemetery, Caloocan City.
and subject to challenge.[20] Section 2, Article III of the 1987 Constitution, gives this guarantee:
xxx xxx xxx
SEC. 2. The right of the people to be secure in their persons, houses, papers, and effects against Q While you were conducting your surveillance, together with Pat. Angel Lumabas and one
unreasonable searches and seizures of whatever nature and for any purpose shall be inviolable, Arnold Enriquez, what happened, if any?
and no search warrant or warrant of arrest shall issue except upon probable cause to be A We chanced upon one male person there in front of the Caloocan Cemetery then when we
determined personally by the judge after examination under oath or affirmation of the called his attention, he tried to avoid us, then prompting us to approach him and
complainant and the witnesses he may produce, and particularly describing the place to be introduce ourselves as police officers in a polite manner.
searched and the persons or things to be seized. xxx xxx xxx
Any evidence obtained in violation of the mentioned provision is legally inadmissible in Q Could you describe to us the appearance of that person when you chanced upon him?
A That person seems like he is high on drug.
evidence as a fruit of the poisonous tree, falling under the exclusionary rule:
Q How were you able to say Mr. Witness that that person that you chanced upon was high on
SEC. 3. x x x drug?
(2) Any evidence obtained in violation of x x x the preceding section shall be inadmissible for any A Because his eyes were red and he was walking on a swaying manner.
purpose in any proceeding. Q What was he doing in particular when you chanced upon him?
This right, however, is not absolute.[21] The recent case of People vs. Lacerna enumerated A He was roaming around, sir.
five recognized exceptions to the rule against warrantless search and seizure, viz.: (1) search Q You said that he avoided you, what did you do when he avoided you?
incidental to a lawful arrest, (2) search of moving vehicles, (3) seizure in plain view, (4) customs A We approached him and introduced ourselves as police officers in a polite manner, sir.
search, and (5) waiver by the accused themselves of their right against unreasonable search and Q How did you introduce yourselves?
A In a polite manner, sir.
seizure.[22] In People vs. Encinada,[23] the Court further explained that [i]n these cases, the search
Q What did you say when you introduced yourselves?
and seizure may be made only with probable cause as the essential requirement.Although the A We asked him what he was holding in his hands, sir.
term eludes exact definition, probable cause for a search is, at best, defined as a reasonable Q And what was the reaction of the person when you asked him what he was holding in his
ground of suspicion, supported by circumstances sufficiently strong in themselves to warrant a hands?
cautious man in the belief that the person accused is guilty of the offense with which he is A He tried to resist, sir.
charged; or the existence of such facts and circumstances which could lead a reasonably discreet Q When he tried to resist, what did you do?
and prudent man to believe that an offense has been committed and that the item(s), article(s) A I requested him if I can see what was he was(sic) holding in his hands.
or object(s) sought in connection with said offense or subject to seizure and destruction by law is Q What was the answer of the person upon your request?
A He allowed me to examine that something in his hands, sir.
in the place to be searched.
xxx xxx xxx
Stop-and-frisk has already been adopted as another exception to the general rule against a Q What was he holding?
search without a warrant. In Posadas vs. Court of Appeals ,[24] the Court held that there are many
73

A He was holding his wallet and when we opened it, there was a marijuana (sic) crushed Furthermore, like the trial and the appellate courts, we have not been given sufficient
residue. grounds to believe the extortion angle in this case. Petitioner did not file any administrative or
Furthermore, we concur with the Solicitor Generals contention that petitioner effectively criminal case against the arresting officers or present any evidence, other than his bare claim. His
waived the inadmissibility of any evidence illegally obtained when he failed to raise this issue or argument that he feared for his life was lame and unbelievable, considering that he was released
to object thereto during the trial. A valid waiver of a right, more particularly of the constitutional on bail and continued to be on bail as early as April 26, 1988. [32] Since then, he could have made
right against unreasonable search, requires the concurrence of the following requirements: (1) the charge in relative safety, as he was no longer in the custody of the police. His defense of
the right to be waived existed; (2) the person waiving it had knowledge, actual or constructive, frame-up, like alibi, is viewed by this Court with disfavor, because it is easy to concoct and
thereof; and (3) he or she had an actual intention to relinquish the right.[26] Otherwise, the fabricate.[33]
Courts will indulge every reasonable presumption against waiver of fundamental safeguards and The Proper Penalty
will not deduce acquiescence from the failure to exercise this elementary right. In the present The trial and the appellate courts overlooked the Indeterminate Sentence Law (Act No.
case, however, petitioner is deemed to have waived such right for his failure to raise its violation 4103, as amended) by sentencing petitioner to a straight penalty of six years and one day of
before the trial court. In petitions under Rule 45, as distinguished from an ordinary appeal of imprisonment, aside from the imposed fine of six thousand pesos. This Act requires the
criminal cases where the whole case is opened for review, the appeal is generally limited to the imposition of an indeterminate penalty:
errors assigned by petitioner. Issues not raised below cannot be pleaded for the first time on SECTION 1. Hereafter, in imposing a prison sentence for an offense punished by the Revised
appeal.[27] Penal Code, or its amendments, the court shall sentence the accused to an indeterminate
Second Issue: Assessment of Evidence sentence the maximum term of which shall be that which, in view of the attending
Petitioner also contends that the two arresting officers testimony contained polluted, circumstances, could be properly imposed under the rules of the said Code, and the minimum
irreconcilable and unexplained contradictions which did not support petitioners conviction. which shall be within the range of the penalty next lower to that prescribed by the Code for the
We disagree. Time and again, this Court has ruled that the trial courts assessment of the offense; and if the offense is punished by any other law, the court shall sentence the accused to
credibility of witnesses, particularly when affirmed by the Court of Appeals as in this case, is an indeterminate sentence, the maximum term of which shall not exceed the maximum fixed by
accorded great weight and respect, since it had the opportunity to observe their demeanor and said law and the minimum shall not be less than the minimum term prescribed by the same. (As
deportment as they testified before it. Unless substantial facts and circumstances have been amended by Act No. 4225.)
overlooked or misappreciated by the trial court which, if considered, would materially affect the SEC. 2. This Act shall not apply to persons convicted of offenses punished with death penalty or
result of the case, we will not countenance a departure from this rule.[28] life-imprisonment; to those convicted of treason; to those convicted of misprision of treason,
We concur with Respondent Courts ruling: rebellion, sedition or espionage; to those convicted of piracy; to those who are habitual
(e)ven assuming as contended by appellant that there had been some inconsistencies in the delinquents; to those who shall have escaped from confinement or evaded sentence; to those
prosecution witnesses testimonies, We do not find them substantial enough to impair the who having been granted conditional pardon by the Chief Executive shall have violated the terms
essential veracity of their narration. In People vs. Avila, it was held that As long as the witnesses thereof; to those whose maximum term of imprisonment does not exceed one year, not to those
concur on the material points, slight differences in their remembrance of the details, do not already sentenced by final judgment at the time of approval of this Act, except as provided in
reflect on the essential veracity of their statements. Section 5 hereof. (Underscoring supplied)
However, we find that, aside from the presumption of regularity in the performance of The Dangerous Drugs Law, R.A. 6425, as amended by B.P. 179, imposes the following
duty, the bestowal of full credence on Pat. Espiritus testimony is justified by tangible evidence on penalty for illegal possession of marijuana:
record. Despite Pat. Lumabas contradictory testimony, that of Espiritu is supported by the Joint Sec. 8. x x x x
Affidavit[29] signed by both arresting policemen. The question of whether the marijuana was The penalty of imprisonment ranging from six years and one day to twelve years and a fine
found inside petitioners wallet or inside a plastic bag is immaterial, considering that petitioner ranging from six thousand to twelve thousand pesos shall be imposed upon any person who,
did not deny possession of said substance. Failure to present the wallet in evidence did not unless authorized by law, shall possess or use Indian hemp.
negate that marijuana was found in petitioners possession. This shows that such contradiction is Prescinding from the foregoing, the Court holds that the proper penalty is an
minor, and does not destroy Espiritus credibility.[30] indeterminate sentence of imprisonment ranging from six years and one day to twelve years.[34]
Third Issue: Sufficiency of Evidence WHEREFORE, the assailed Decision and Resolution are
The elements of illegal possession of marijuana are: (a) the accused is in possession of an hereby AFFIRMED with MODIFICATION. Petitioner is sentenced to suffer IMPRISONMENT of SIX
item or object which is identified to be a prohibited drug; (b) such possession is not authorized (6) YEARS, as minimum, to TWELVE (12) YEARS, as maximum, and to PAY a FINE of SIX
by law; and (c) the accused freely and consciously possessed the said drug.[31] THOUSAND PESOS. Costs against petitioner.
The substance found in petitioners possession was identified by NBI Forensic Chemist Aida SO ORDERED.
Pascual to be crushed marijuana leaves. Petitioners lack of authority to possess these leaves was
established. His awareness thereof was undeniable, considering that petitioner was high on
drugs when stopped by the policemen and that he resisted when asked to show and identify the
thing he was holding. Such behavior clearly shows that petitioner knew that he was holding
marijuana and that it was prohibited by law.
74

PEOPLE VS COGAED obtained were indeed marijuana.23 The marijuana collected from Cogaed’s blue bag had a total
The mantle of protection upon one's person and one's effects through Article III, Section 2 of the weight of 8,091.5 grams.24 The marijuana from Cogaed’s sack weighed 4,246.1 grams. 25 The
Constitution is essential to allow citizens to evolve their autonomy and, hence, to avail marijuana collected from Dayao’s bag weighed 5,092 grams.26 A total of 17,429.6 grams
themselves of their right to privacy. The alleged compromise with the battle against dangerous werecollected from Cogaed’s and Dayao’s bags.27
drugs is more apparent than real. Often, the compromise is there because law enforcers neglect According to Cogaed’s testimony during trial, he was at Balbalayan, La Union, "waiting for a
to perform what could have been done to uphold the Constitution as they pursue those who jeepney to take him"28to the Poblacion of San Gabriel so he could buy pesticide.29 He boarded a
traffic this scourge of society. jeepney and recognized Dayao, his younger brother’s friend.30 Upon arrival at the Poblacion of
Squarely raised in· this appeal1 is the admissibility of the evidence seized as a result of a San Gabriel, Dayao and Cogaed alighted from the jeepney.31 Dayao allegedly "asked for
warrantless arrest. The police officers identified the alleged perpetrator through facts that were [Cogaed’s] help in carrying his things, which included a travelling bag and a sack."32 Cogaed
not based on their personal knowledge. The information as to the accused’s whereabouts was agreed because they were both going to the market. 33 This was when SPO1 Taracatac
sent through a text message. The accusedwho never acted suspicious was identified by a driver. approached them, and when SPO1 Taracatac asked Cogaed what was inside the bags, Cogaed
The bag that allegedly contained the contraband was required to be opened under intimidating replied that he did not know.34 SPO1 Taracatac then talked to Dayao, however, Cogaed was not
circumstances and without the accused having been fully apprised of his rights. This was not a privy to their conversation.35Thereafter, SPO1 Taracatac arrested Dayao and Cogaed and brought
reasonable search within the meaning of the Constitution. There was no reasonable suspicion them to the police station.36 These facts were corroborated by an eyewitness,Teodoro Nalpu-ot,
that would allow a legitimate "stop and frisk" action. The alleged waiver of rights by the accused who was standing across the parking lot where Cogaed was apprehended. 37
was not done intelligently, knowingly, and without improper pressure or coercion. At the police station, Cogaed said that "SPO1 Taracatac hit [him] on the head." 38 The bags were
The evidence, therefore, used against the accused should be excluded consistent with Article III, also opened, but Cogaed never knew what was inside.39
Section 3 (2) of the Constitution. There being no possible admissible evidence, the accused It was only later when Cogaed learned that it was marijuana when he and Dayao were charged
should be acquitted. with illegal possession of dangerous drugs under Republic Act No. 9165.40 The information
I against them states:
According to the prosecution, at about 6:00 a.m. of November 25, 2005, Police Senior Inspector That on or about the 25th day of November, 2005, in the Municipality of San Gabriel, Province of
Sofronio Bayan (PSI Bayan) of the San Gabriel Police Station in San Gabriel,La Union, "received a La Union, and within the jurisdiction of this Honorable Court, the above-named accused VICTOR
text message from an unidentified civilian informer"2 that one Marvin Buya (also known as COGAED Y ROMANA and SANTIAGO DAYAO Y SACPA (who acted with discernment) and JOHN
Marvin Bugat) "[would]be transporting marijuana"3 from Barangay LunOy, San Gabriel, La Union DOE,conspiring, confederating and mutually helping one another, did then there wilfully,
to the Poblacion of San Gabriel, La Union.4 unlawfully, feloniously and knowingly, without being authorized by law, have in their control,
PSI Bayan organized checkpoints in order "to intercept the suspect."5 PSI Bayan ordered SPO1 custody and possession dried marijuana, a dangerous drug, with a total weight of seventeen
Jaime Taracatac, Jr. (SPO1 Taracatac), a member of the San Gabriel Police, to set up a checkpoint thousand,four hundred twenty-nine and sixtenths (17, 429.6) grams.
in the waiting area of passengers from San Gabriel bound for San Fernando City.6 A passenger CONTRARY TO Section 11 (Possession of Dangerous Drugs), Article II, of Republic Act No. 9165
jeepney from Barangay Lun-Oy arrived at SPO1 Taracatac’s checkpoint.7 The jeepney driver (otherwise known as the "Comprehensive Dangerous Drugs Act of 2002").41
disembarked and signalled to SPO1 Taracatac indicating the two male passengers who were The case was raffled to Regional Trial Court, Branch 28 of San Fernando City, La Union.42 Cogaed
carrying marijuana.8 SPO1 Taracatac approached the two male passengers who were later and Dayao pleaded not guilty.43 The case was dismissed against Dayao because he was only 14
identified as Victor RomanaCogaed and Santiago Sacpa Dayao.9 Cogaed was carrying a blue bag years old at that time and was exempt from criminal liability under the Juvenile Justice and
and a sack while Dayao was holding a yellow bag.10 Welfare Act of 2006 or Republic Act No. 9344.44Trial against Cogaed ensued. In a
SPO1 Taracatac asked Cogaed and Dayao about the contents of their bags.11 Cogaed and Dayao decision45 dated May 21, 2008, the Regional Trial Court found Cogaed guilty. The dispositive
told SPO1 Taracatac that they did not know since they were transporting the bags as a favor for portion of the decision states:
their barriomatenamed Marvin.12 After this exchange, Cogaed opened the blue bag, revealing WHEREFORE, the Court finds accused Victor Cogaed y Romana GUILTY beyond reasonable doubt
three bricks of what looked like marijuana.13 Cogaed then muttered, "nagloko daytoy nga for Violation of Section 11, Article II of Republic Act No. 9165 (otherwise known as the
Marvinen, kastoymet gayam ti nagyanna,"which translates to "Marvin is a fool, this is what [is] "Comprehensive Dangerous Drugs Act of 2002") and sentences him to suffer life imprisonment,
contained in the bag."14 "SPO1 Taracatac arrested [Cogaed] and . . . Dayao and brought them to and to pay a fine of one million pesos (Php 1,000,000.00).46
the police station."15 Cogaed and Dayao "were still carrying their respective bags"16inside the The trial court judge initiallyfound Cogaed’s arrest illegal considering that "Cogaed at that time
station.17 was not, at the moment of his arrest, committing a crime nor was shown that hewas about to do
While at the police station, the Chief of Police and Investigator PO3 Stanley Campit (PO3 Campit) so or that had just done so. He just alighted from the passenger jeepney and there was no
requested Cogaed and Dayao to empty their bags.18 Inside Cogaed’s sack was "four (4) rolled outward indication that called for his arrest."47 Since the arrest was illegal, the warrantless
pieces of suspected marijuana fruiting tops,"19 and inside Dayao’s yellow bag was a brick of search should also be considered illegal.48 However, the trial court stated that notwithstanding
suspected marijuana.20 the illegality of the arrest, Cogaed "waived his right to object to such irregularity"49 when "he did
PO3 Campit prepared the suspected marijuana for laboratory testing.21 PSI Bayan personally not protest when SPO1 Taracatac, after identifying himself, asked him to open his bag." 50
delivered the suspected marijuana to the PNP Crime Laboratory.22 Forensic Chemical Officer Cogaed appealed51 the trial court’s decision.However, the Court of Appeals denied his appeal
Police Inspector Valeriano Panem Laya II performed the tests and found that the objects and affirmed the trial court’s decision.52 The Court of Appeals found that Cogaed waived his right
75

against warrantless searches when "[w]ithout any prompting from SPO1 Taracatac, [he] 2. Seizure of evidence in "plain view," . . . ;
voluntarily opened his bag."53 Hence, this appeal was filed. 3. Search of a moving vehicle. Highly regulated by the government, the vehicle’s
The following errors were assigned by Cogaed in his appellant’s brief: inherent mobility reduces expectation of privacy especially when its transit in public
I thoroughfares furnishes a highly reasonable suspicion amounting to probable cause
THE TRIAL COURT GRAVELY ERRED IN ADMITTING THE SEIZED DANGEROUS DRUGS AS EVIDENCE that the occupant committed a criminal activity;
AGAINST THE ACCUSED-APPELLANT DESPITE BEING THE RESULT OF AN UNLAWFUL 4. Consentedwarrantless search;
WARRANTLESS SEARCH AND SEIZURE. 5. Customs search;
II 6. Stop and frisk; and
THE TRIAL COURT GRAVELY ERRED IN CONVICTING THE ACCUSED-APPELLANT DESPITE THE 7. Exigent and emergency circumstances.62 (Citations omitted)
ARRESTING OFFICER’S NON-COMPLIANCE WITH THE REQUIREMENTS FOR THE PROPER CUSTODY III
OF SEIZED DANGEROUS DRUGS UNDER REPUBLIC ACT NO. 9165. One of these jurisprudential exceptionsto search warrants is "stop and frisk". "Stop and frisk"
III searches are often confused with searches incidental to lawful arrests under the Rules of
THE TRIAL COURT GRAVELY ERRED IN CONVICTING THE ACCUSED-APPELLANT DESPITE THE Court.63 Searches incidental to a lawful arrest require that a crime be committed in flagrante
ARRESTING OFFICER’S FAILURE TO PRESERVE THE INTEGRITY AND EVIDENTIARY VALUE OF THE delicto, and the search conducted within the vicinity and withinreach by the person arrested is
SEIZED DANGEROUS DRUGS.54 done to ensure that there are no weapons, as well as to preserve the evidence. 64
For our consideration are the following issues: (1) whether there was a valid search and seizure On the other hand, "stop and frisk"searches are conducted to prevent the occurrence of a crime.
of marijuana as against the appellant; (2) whether the evidence obtained through the search For instance, the search in Posadas v. Court of Appeals65 was similar "to a ‘stop and frisk’
should be admitted; and (3) whether there was enough evidence to sustain the conviction of the situation whose object is either to determine the identity of a suspicious individual or to
accused. maintain the status quomomentarily while the police officer seeks to obtain more
In view of the disposition of this case, we deem that a discussion with respect to the information."66 This court stated that the "stop and frisk" search should be used "[w]hen dealing
requirements on the chain of custody of dangerous drugs unnecessary.55 with a rapidly unfolding and potentially criminal situation in the city streets where unarguably
We find for the accused. there is no time to secure . . . a search warrant."67
II The search involved in this case was initially a "stop and frisk" search, but it did not comply with
The right to privacy is a fundamental right enshrined by implication in our Constitution. It has all the requirements of reasonability required by the Constitution.
many dimensions. One of its dimensions is its protection through the prohibition of "Stop and frisk" searches (sometimes referred to as Terrysearches68) are necessary for law
unreasonable searches and seizures in Article III, Section 2 of the Constitution: enforcement. That is, law enforcers should be given the legal arsenal to prevent the commission
The right of the people to be secure in their persons, houses, papers, and effects against of offenses. However, this should be balanced with the need to protect the privacy of citizens in
unreasonable searches and seizures of whatever nature and for any purpose shall be inviolable, accordance with Article III, Section 2 of the Constitution.
and no search warrant or warrant of arrest shall issue except upon probable cause to be The balance lies in the concept of"suspiciousness" present in the situation where the police
determinedpersonally by the judge after examination under oath or affirmation of the officer finds himself or herself in. This may be undoubtedly based on the experience ofthe police
complainant and the witnesses he may produce, and particularly describing the place to be officer. Experienced police officers have personal experience dealing with criminals and criminal
searched and the persons or things to be seized. behavior. Hence, they should have the ability to discern — based on facts that they themselves
This provision requires that the court examine with care and diligence whether searches and observe — whether an individual is acting in a suspicious manner. Clearly, a basic criterion would
seizures are "reasonable." As a general rule, searches conducted with a warrant that meets all be that the police officer, with his or her personal knowledge, must observe the facts leading to
the requirements of this provision are reasonable. This warrant requires the existence of the suspicion of an illicit act.
probable cause that can only be determined by a judge.56 The existence of probable cause must In Manalili v. Court of Appeals,69 the police officers were initially informed about a place
be established by the judge after asking searching questions and answers.57 Probable cause at frequented by people abusing drugs.70 When they arrived, one of the police officers saw a man
this stage can only exist if there is an offense alleged to be committed. Also, the warrant frames with "reddish eyes and [who was] walking in a swaying manner."71 The suspicion increased when
the searches done by the law enforcers. There must be a particular description of the place and the man avoided the police officers.72 These observations led the police officers to conclude that
the things to be searched.58 the man was high on drugs.73 These were sufficient facts observed by the police officers "to
However, there are instances when searches are reasonable even when warrantless.59 In the stop[the] petitioner [and] investigate."74
Rules of Court, searchesincidental to lawful arrests are allowed even without a separate In People v. Solayao,75 police officers noticed a man who appeared drunk.76 This man was also
warrant.60 This court has taken into account the "uniqueness of circumstances involved including "wearing a camouflage uniform or a jungle suit."77 Upon seeing the police, the man fled.78 His
the purpose of the search or seizure, the presence or absence of probable cause, the manner in flight added to the suspicion.79After stopping him, the police officers found an unlicensed
which the search and seizure was made, the place or thing searched, and the character of the "homemade firearm"80 in his possession.81 This court ruled that "[u]nder the circumstances, the
articles procured."61 The known jurisprudential instances of reasonable warrantless searches and government agents could not possibly have procured a search warrant first."82 This was also a
seizures are: valid search.
1. Warrantless search incidental to a lawful arrest. . . ;
76

In these cases, the police officers using their senses observed facts that led to the suspicion. There was not a single suspicious circumstance in this case, and there was no approximation for
Seeing a man with reddish eyes and walking in a swaying manner, based on their experience, is the probable cause requirement for warrantless arrest. The person searched was noteven the
indicative of a person who uses dangerous and illicit drugs. A drunk civilian in guerrilla wear is person mentioned by the informant. The informant gave the name of Marvin Buya, and the
probably hiding something as well. person searched was Victor Cogaed. Even if it was true that Cogaed responded by saying that he
The case of Cogaed was different. He was simply a passenger carrying a bag and traveling was transporting the bag to Marvin Buya, this still remained only as one circumstance. This
aboarda jeepney. There was nothing suspicious, moreover, criminal, about riding a jeepney or should not have been enough reason to search Cogaed and his belongings without a valid search
carrying a bag. The assessment of suspicion was not made by the police officer but by the warrant.
jeepney driver. It was the driver who signalled to the police that Cogaed was "suspicious." V
This is supported by the testimony of SPO1 Taracatac himself: Police officers cannot justify unbridled searches and be shielded by this exception, unless there is
COURT: compliance with the "genuine reason" requirement and that the search serves the purpose of
Q So you don’t know what was the content while it was still being carried by him in the passenger jeep? protecting the public. As stated in Malacat:
WITNESS: [A] "stop-and-frisk" serves a two-fold interest: (1) the general interest of effective crime
A Not yet, Your Honor.83 prevention and detection, which underlies the recognition that a police officer may, under
SPO1 Taracatac likewise stated:
appropriate circumstances and in an appropriate manner, approach a person for purposes of
COURT:
Q If the driver did not make a gesture pointing to the accused, did you have reason to believe that the investigating possible criminal behavior even without probable cause; and (2) the more pressing
accused were carrying marijuana? interest of safety and self-preservationwhich permit the police officer to take steps to assure
WITNESS: himself that the person with whom he deals is not armed with a deadly weapon that could
A No, Your Honor.84 unexpectedly and fatally be used against the police officer.99 (Emphasis supplied)
The jeepney driver had to point toCogaed. He would not have been identified by the police officers The "stop and frisk" searchwas originally limited to outer clothing and for the purpose of
otherwise. detecting dangerous weapons.100 As in Manalili,101 jurisprudence also allows "stop and frisk" for
It is the police officer who should observe facts that would lead to a reasonable degree of suspicion of a cases involving dangerous drugs.
person. The police officer should not adopt the suspicion initiated by another person. This is necessary to
The circumstances of thiscase are analogous to People v. Aruta.102 In that case, an informant told
justify that the person suspected be stopped and reasonably searched.85 Anything less than this would be an
infringementupon one’s basic right to security of one’s person and effects. the police that a certain "Aling Rosa" would be bringing in drugs from Baguio City by bus. 103 At
IV the bus terminal, the police officers prepared themselves.104 The informant pointed at a woman
Normally, "stop and frisk" searches do not give the law enforcer an opportunity to confer with a crossing the street105 and identified her as "Aling Rosa."106 The police apprehended "Aling Rosa,"
judge to determine probable cause. In Posadas v. Court of Appeals,86 one of the earliest cases and they alleged that she allowed them to look inside her bag. 107 The bag contained marijuana
adopting the "stop and frisk" doctrine in Philippine jurisprudence, this court approximatedthe leaves.108
suspicious circumstances as probable cause: In Aruta, this court found that the search and seizure conducted was illegal.109 There were no
The probable causeis that when the petitioner acted suspiciously and attempted to flee with the suspicious circumstances that preceded Aruta’s arrest and the subsequent search and
buri bag there was a probable cause that he was concealing something illegal in the bag and it seizure.110 It was only the informant that prompted the police to apprehend her.111 The evidence
was the right and duty of the police officers to inspect the same. 87 (Emphasis supplied) obtained was not admissible because of the illegal search.112 Consequently, Aruta was
For warrantless searches, probable cause was defined as "a reasonable ground of acquitted.113
suspicionsupported by circumstances sufficiently strong in themselves to warrant a cautious man Arutais almost identical to this case, except that it was the jeepney driver, not the police’s
to believe that the person accused is guilty of the offense with which he is charged."88 informant, who informed the police that Cogaed was "suspicious."
Malacat v. Court of Appeals89 clarifies the requirement further. It does not have to be probable The facts in Arutaare also similar to the facts in People v. Aminnudin.114 Here, the National
cause,but it cannot be mere suspicion.90 It has to be a "genuine reason"91 to serve the purposes Bureau ofInvestigation (NBI) acted upon a tip, naming Aminnudin as somebody possessing
of the "stop and frisk" exception:92 drugs.115 The NBI waited for the vessel to arrive and accosted Aminnudin while he was
Other notable points of Terryare that while probable cause is not required to conduct a "stop disembarking from a boat.116 Like in the case at bar, the NBI inspected Aminnudin’s bag and
and frisk," it nevertheless holds that mere suspicion or a hunch will not validate a "stop and found bundles of what turnedout to be marijuana leaves.117 The court declared that the
frisk." A genuine reason must exist, in light of the police officer’s experience and surrounding searchand seizure was illegal.118 Aminnudin was acquitted.119
conditions, to warrant the belief that the person detained has weapons concealed about People v. Chua120 also presents almost the same circumstances. In this case, the police had been
him.93 (Emphasis supplied, footnotes omitted) receiving information that the accused was distributing drugs in "different karaoke bars in
In his dissent for Esquillo v. People,94 Justice Bersamin reminds us that police officers must not Angeles City."121 One night, the police received information that thisdrug dealer would be
rely on a single suspicious circumstance.95 There should be "presence of more than dealing drugs at the Thunder Inn Hotel so they conducted a stakeout.122 A car "arrived and
oneseemingly innocent activity, which, taken together, warranted a reasonable inference of parked"123 at the hotel.124The informant told the police that the man parked at the hotel was
criminal activity."96 The Constitution prohibits "unreasonable searches and seizures."97 Certainly, dealing drugs.125 The man alighted from his car.126 He was carrying a juice box.127 The police
reliance on only one suspicious circumstance or none at all will not result in a reasonable immediately apprehended him and discovered live ammunition and drugs in his person and in
search.98 the juice box he was holding.128
77

Like in Aruta, this court did not find anything unusual or suspicious about Chua’s situation when Q So that there was not any order from you for them to open the bags?
the police apprehended him and ruled that "[t]here was no valid‘stop-and-frisk’."129 A None, ma’am.
VI Q Now, Mr. witness when you went near them and asked them what were the contents ofthe bag, you have
not seen any signs of hesitation or fright from them, is it not?
None of the other exceptions to warrantless searches exist to allow the evidence to be
A It seems they were frightened, ma’am.
admissible.The facts of this case do not qualify as a search incidental to a lawful arrest. Q But you actually [claimed] that there was not any hesitation from them in opening the bags, is it not?
Rule 126, Section 13 of the Rules of Court allows for searches incidental to a lawful arrest. For A Yes, ma’am but when I went near them it seems that they were surprised.133 (Emphasis supplied)
there to be a lawful arrest, there should be either a warrant of arrest or a lawful warrantless The state of mind of Cogaed was further clarified with SPO1 Taracatac’s responses to Judge Florendo’s
arrest as enumerated in Rule 113, Section 5 of the Rules of Court: questions:
Section 5. Arrest without warrant; when lawful. – A peace officer or a private person may, COURT:
withouta warrant, arrest a person: ....
(a) When, in his presence, the person to be arrested has committed, is actually Q Did you have eye contact with Cogaed?
A When I [sic] was alighting from the jeepney, Your Honor I observed that he was somewhat
committing, or is attempting to commit an offense;
frightened.1âwphi1 He was a little apprehensive and when he was already stepping down and he put down
(b) When an offense has just been committed and he has probable cause to believe the bag I asked him, "what’s that," and he answered, "I don’t know because Marvin only asked me to
based on personal knowledge of facts or circumstances that the person to be arrested carry."134
has committed it; and For a valid waiver by the accused of his or her constitutional right, it is not sufficient that the
(c) When the person to be arrested is a prisoner who has escaped from a penal police officerintroduce himself or herself, or be known as a police officer.1âwphi1 The police
establishment or place where he is serving final judgment or temporarily confined officer must also inform the person to be searched that any inaction on his orher part will
while his case is pending, or has escaped while being transferred from one amount to a waiver of any of his or her objections that the circumstances do not amount to a
confinement to another. reasonable search. The police officer must communicate this clearly and in a language known to
The apprehension of Cogaed was not effected with a warrant of arrest. None of the instances the person who is about to waive his or her constitutional rights. There must be anassurance
enumerated in Rule 113, Section 5 of the Rules of Court were present whenthe arrest was made. given to the police officer that the accused fully understands his or her rights. The fundamental
At the time of his apprehension, Cogaed has not committed, was not committing, or was about nature of a person’s constitutional right to privacy requires no less.
to commit a crime. As in People v. Chua, for a warrantless arrest of in flagrante delictoto be VIII
affected, "two elements must concur: (1) the person to bearrested must execute anovert act The Constitution provides:
indicating that he has just committed, is actually committing, or is attempting to commit a crime; Any evidence obtained in violation of [the right against unreasonable searches and seizures] shall
and (2) such overt act is done inthe presence or within the view of the arresting officer."130 Both be inadmissible for any purpose in any proceeding. 135
elements were missing when Cogaed was arrested.131 There were no overt acts within plain view Otherwise known as the exclusionary rule or the fruit of the poisonous tree doctrine, this
of the police officers that suggested that Cogaed was in possession of drugs at that time. constitutional provision originated from Stonehill v. Diokno.136 This rule prohibits the issuance of
Also, Cogaed was not an escapee prisoner that time; hence, he could not have qualified for the general warrants that encourage law enforcers to go on fishing expeditions. Evidence obtained
last allowable warrantless arrest. through unlawful seizures should be excluded as evidence because it is "the only practical means
VII of enforcing the constitutional injunction against unreasonable searches and seizures."137 It
There can be no valid waiver of Cogaed’s constitutional rights even if we assume that he did not ensures that the fundamental rights to one’s person, houses, papers, and effects are not lightly
object when the police asked him to open his bags. As this court previously stated: infringed upon and are upheld.
Appellant’s silence should not be lightly taken as consent to such search. The implied Considering that the prosecution and conviction of Cogaed were founded on the search of his
acquiescence to the search, if there was any, could not have been more than mere passive bags, a pronouncement of the illegality of that search means that there is no evidence left to
conformity given under intimidating or coercive circumstances and is thus considered no consent convict Cogaed.
at all within the purview of the constitutional guarantee.132(Citations omitted) Cogaed’s silence Drugs and its illegal traffic are a scourgeto our society. In the fight to eradicate this menace, law
or lack of aggressive objection was a natural reaction to a coercive environment brought about enforcers should be equipped with the resources to be able to perform their duties better.
by the police officer’s excessive intrusion into his private space. The prosecution and the police However, we cannot, in any way, compromise our society’s fundamental values enshrined in our
carry the burden of showing that the waiver of a constitutional right is one which is knowing, Constitution. Otherwise, we will be seen as slowlydismantling the very foundations of the society
intelligent, and free from any coercion. In all cases, such waivers are not to be presumed. that we seek to protect.
The coercive atmosphere created by the presence of the police officer can be discerned again WHEREFORE, the decisions of the Regional Trial Court, Branch 28, San Fernando City, La Union
from the testimony of SPO1 Taracatac during cross-examination: and of the Court of Appeals in CA-G.R. CR-HC No. 03394 are hereby REVERSEDand SET ASIDE. For
ATTY. BINWAG: lack of evidence to establish his guilt beyond reasonable doubt, accused-appellant VICTOR
Q Now, Mr. witness, you claimed that you only asked them what are the contents of their bags, is it not? COGAED Y ROMANA is hereby ACQUITTED and ordered RELEASED from confinement unless he is
WITNESS:
being heldfor some other legal grounds. No costs.
A Yes, ma’am.
Q And then without hesitation and voluntarily they just opened their bags, is it not? SO ORDERED.
A Yes, ma’am.
78

FAJARDO VS PEOPLE In the evening of August 27, 2002, members of the Provincial Intelligence Special
At bar is a Petition for Review on Certiorari under Rule 45 of the Rules of Court, seeking Operations Group (PISOG) were instructed by Provincial Director Police Superintendent Edgardo
the reversal of the February 10, 2009 Decision[1] of the Court of Appeals (CA), which affirmed Mendoza (P/Supt. Mendoza) to respond to the complaint of concerned citizens residing on Ilang-
with modification the August 29, 2006 decision[2] of the Regional Trial Court (RTC), Branch 5, Ilang and Sampaguita Roads, Park Homes III Subdivision, Barangay Andagao, Kalibo, Aklan, that
Kalibo, Aklan, finding petitioner guilty of violating Presidential Decree (P.D.) No. 1866, as armed men drinking liquor at the residence of petitioner were indiscriminately firing guns.
amended.
The facts: Along with the members of the Aklan Police Provincial Office, the elements of the
Petitioner, Elenita Fajardo, and one Zaldy Valerio (Valerio) were charged with violation of P.D. PISOG proceeded to the area. Upon arrival thereat, they noticed that several persons scampered
No. 1866, as amended, before the RTC, Branch 5, Kalibo, Aklan, committed as follows: and ran in different directions. The responding team saw Valerio holding two .45 caliber pistols.
He fired shots at the policemen before entering the house of petitioner.
That on or about the 28th day of August, 2002, in the morning, in Barangay Andagao,
Municipality of Kalibo, Province of Aklan, Republic of the Petitioner was seen tucking a .45 caliber handgun between her waist and the
Philippines, and within the jurisdiction of this Honorable Court, the waistband of her shorts, after which, she entered the house and locked the main door.
above-named accused, conspiring, confederating and mutually To prevent any violent commotion, the policemen desisted from entering petitioners house but,
helping one another, without authority of law, permit or license, in order to deter Valerio from evading apprehension, they cordoned the perimeter of the house
did then and there, knowingly, willfully, unlawfully and feloniously as they waited for further instructions from P/Supt. Mendoza. A few minutes later, petitioner
have in their possession, custody and control two (2) receivers of went out of the house and negotiated for the pull-out of the police troops. No agreement
caliber .45 pistol, [M]odel [No.] M1911A1 US with SN 763025 and materialized.
Model [No.] M1911A1 US with defaced serial number, two (2)
pieces short magazine of M16 Armalite rifle, thirty-five (35) pieces At around 2:00 a.m. and 4:00 a.m. of August 28, 2002, Senior Police Officer 2
live M16 ammunition 5.56 caliber and fourteen (14) pieces live Clemencio Nava (SPO2 Nava), who was posted at the back portion of the house, saw Valerio
caliber .45 ammunition, which items were confiscated and emerge twice on top of the house and throw something. The discarded objects landed near the
recovered from their possession during a search conducted by wall of petitioners house and inside the compound of a neighboring residence. SPO2 Nava,
members of the Provincial Intelligence Special Operation Group, together with SPO1 Teodoro Neron and Jerome T. Vega (Vega), radio announcer/reporter of
Aklan Police Provincial Office, Kalibo, Aklan, by virtue of Search RMN DYKR, as witness, recovered the discarded objects, which turned out to be two (2) receivers
Warrant No. 01 (9) 03 issued by OIC Executive Judge Dean Telan of of .45 caliber pistol, model no. M1911A1 US, with serial number (SN) 763025, and model no.
the Regional Trial Court of Aklan.[3] M1911A1 US, with a defaced serial number. The recovered items were then surrendered to
SPO1 Nathaniel A. Tan (SPO1 Tan), Group Investigator, who utilized them in applying for and
When arraigned on March 25, 2004, both pleaded not guilty to the offense charged.[4] During obtaining a search warrant.
pre-trial, they agreed to the following stipulation of facts:
The warrant was served on petitioner at 9:30 a.m. Together with
1. The search warrant subject of this case exists; a barangay captain, barangay kagawad, and members of the media, as witnesses, the police
2. Accused Elenita Fajardo is the same person subject of the search team proceeded to search petitioners house. The team found and was able to confiscate the
warrant in this case who is a resident of Sampaguita Road, Park Homes, following:
Andagao, Kalibo, Aklan;
3. Accused Zaldy Valerio was in the house of Elenita Fajardo in the evening 1. Two (2) pieces of Short Magazine of M16 Armalite Rifle;
of August 27, 2002 but does not live therein; 2. Thirty five (35) pieces of live M16 ammos 5.56 Caliber; and
4. Both accused were not duly licensed firearm holders; 3. Fourteen (14) pieces of live ammos of Caliber 45 pistol.
5. The search warrant was served in the house of accused Elenita Fajardo
in the morning of August 28, 2002; and Since petitioner and Valerio failed to present any documents showing their authority
6. The accused Elenita Fajardo and Valerio were not arrested immediately to possess the confiscated firearms and the two recovered receivers, a criminal information for
upon the arrival of the military personnel despite the fact that the latter violation of P.D. No. 1866, as amended by Republic Act (R.A.) No. 8294, was filed against them.
allegedly saw them in possession of a firearm in the evening of August
27, 2002.[5] For their exoneration, petitioner and Valerio argued that the issuance of the search
warrant was defective because the allegation contained in the application filed and signed by
As culled from the similar factual findings of the RTC and the CA,[6] these are the chain SPO1 Tan was not based on his personal knowledge. They quoted this pertinent portion of the
of events that led to the filing of the information: application:
79

That this application was founded on confidential information effective. The same evils, the same perils to public security, which the law
received by the Provincial Director, Police Supt. Edgardo Mendoza.[7] penalizes exist whether the unlicensed holder of a prohibited weapon be its
owner or a borrower. To accomplish the object of this law[,] the proprietary
They further asserted that the execution of the search warrant was infirm since concept of the possession can have no bearing whatsoever.
petitioner, who was inside the house at the time of the search, was not asked to accompany the xxxx
policemen as they explored the place, but was instead ordered to remain in the living room x x x. [I]n order that one may be found guilty of a violation of the
(sala). decree, it is sufficient that the accused had no authority or license to possess
a firearm, and that he intended to possess the same, even if such possession
Petitioner disowned the confiscated items. She refused to sign the inventory/receipt was made in good faith and without criminal intent.
prepared by the raiding team, because the items allegedly belonged to her brother, Benito
Fajardo, a staff sergeant of the Philippine Army. xxxx
Petitioner denied that she had a .45 caliber pistol tucked in her waistband when the
raiding team arrived. She averred that such situation was implausible because she was wearing To convict an accused for illegal possession of firearms and
garterized shorts and a spaghetti-strapped hanging blouse.[8] explosive under P.D. 1866, as amended, two (2) essential elements must be
indubitably established, viz.: (a) the existence of the subject firearm
Ruling of the RTC ammunition or explosive which may be proved by the presentation of the
The RTC rejected the defenses advanced by accused, holding that the same were subject firearm or explosive or by the testimony of witnesses who saw
already denied in the Orders dated December 31, 2002 and April 20, 2005, respectively denying accused in possession of the same, and (b) the negative fact that the accused
the Motion to Quash Search Warrant and Demurrer to Evidence. The said Orders were not has no license or permit to own or possess the firearm, ammunition or
appealed and have thus attained finality. The RTC also ruled that petitioner and Valerio were explosive which fact may be established by the testimony or certification of a
estopped from assailing the legality of their arrest since they participated in the trial by representative of the PNP Firearms and Explosives Unit that the accused has
presenting evidence for their defense. Likewise, by applying for bail, they have effectively waived no license or permit to possess the subject firearm or explosive (Exhibit G).
such irregularities and defects.
The judicial admission of the accused that they do not have permit
In finding the accused liable for illegal possession of firearms, the RTC explained: or license on the two (2) receivers of caliber .45 pistol, model M1911A1 US
with SN 763025 and model M1911A1 of M16 Armalite rifle, thirty-five (35)
Zaldy Valerio, the bodyguard of Elenita Fajardo, is a former soldier, pieces live M16 ammunition, 5.56 caliber and fourteen (14) pieces live caliber
having served with the Philippine Army prior to his separation from his .45 ammunition confiscated and recovered from their possession during the
service for going on absence without leave (AWOL). With his military search conducted by members of the PISOG, Aklan Police Provincial Office by
background, it is safe to conclude that Zaldy Valerio is familiar with and virtue of Search Warrant No. 01 (9) 03 fall under Section 4 of Rule 129 of the
knowledgeable about different types of firearms and ammunitions. As a Revised Rules of Court.[9]
former soldier, undoubtedly, he can assemble and disassemble firearms.
It must not be de-emphasize[d] that the residence of Elenita Consequently, petitioner and Valerio were convicted of illegal possession of firearms
Fajardo is definitely not an armory or arsenal which are the usual and explosives, punishable under paragraph 2, Section 1 of P.D. No. 1866, as amended by R.A.
depositories for firearms, explosives and ammunition. Granting arguendo No. 8294, which provides:
that those firearms and ammunition were left behind by Benito Fajardo, a
member of the Philippine army, the fact remains that it is a government The penalty of prision mayor in its minimum period and a fine of
property. If it is so, the residence of Elenita Fajardo is not the proper place to Thirty thousand pesos (P30,000.00) shall be imposed if the firearm is
store those items. The logical explanation is that those items are stolen classified as high powered firearm which includes those with bores bigger in
property. diameter than .38 caliber and 9 millimeter such as caliber .40, .41, .44, .45
and also lesser calibered firearms but considered powerful such as caliber
xxxx .357 and caliber .22 center-fire magnum and other firearms with firing
capability of full automatic and by burst of two or three: Provided, however,
The rule is that ownership is not an essential element of illegal That no other crime was committed by the person arrested.
possession of firearms and ammunition. What the law requires is merely
possession which includes not only actual physical possession but also Both were sentenced to suffer the penalty of imprisonment of six (6) years and one (1)
constructive possession or the subjection of the thing to ones control and day to twelve (12) years of prision mayor, and to pay a fine of P30,000.00.
management. This has to be so if the manifest intent of the law is to be
80

On September 1, 2006, only petitioner filed a Motion for Reconsideration, which was The penalty of prision mayor in its minimum period and a fine of
denied in an Order dated October 25, 2006. Petitioner then filed a Notice of Appeal with the CA. Thirty thousand pesos (P30,000.00) shall be imposed if the firearm is
classified as high powered firearm which includes those with bores bigger in
Ruling of the CA diameter than .38 caliber and 9 millimeter such as caliber .40, 41, .44,
The CA concurred with the factual findings of the RTC, but disagreed with its .45 and also lesser calibered firearms but considered powerful such as caliber
conclusions of law, and held that the search warrant was void based on the following .357 and caliber .22 center-fire magnum and other firearms with firing
observations: capability of full automatic and by burst of two or three: Provided, however,
That no other crime was committed by the person arrested.[14]
[A]t the time of applying for a search warrant, SPO1 Nathaniel A. Tan did not
have personal knowledge of the fact that appellants had no license to On the other hand, illegal possession of the two (2) receivers of a .45 caliber pistol,
possess firearms as required by law. For one, he failed to make a categorical model no. M1911A1 US, with SN 763025, and Model M1911A1 US, with a defaced serial
statement on that point during the application. Also, he failed to attach to number, is penalized under paragraph 1, which states:
the application a certification to that effect from the Firearms and Explosives
Office of the Philippine National Police. x x x, this certification is the best Sec. 1. Unlawful manufacture, sale, acquisition, disposition or
evidence obtainable to prove that appellant indeed has no license or permit possession of firearms or ammunition or instruments used or intended to be
to possess a firearm. There was also no explanation given why said used in the manufacture of firearms or ammunition. The penalty of prision
certification was not presented, or even deemed no longer necessary, during correccional in its maximum period and a fine of not less than Fifteen
the application for the warrant. Such vital evidence was simply ignored.[10] thousand pesos (P15,000.00) shall be imposed upon any person who shall
unlawfully manufacture, deal in, acquire, dispose, or possess any low
Resultantly, all firearms and explosives seized inside petitioners residence were powered firearm, such as rimfire handgun, .380 or .32 and other firearm of
declared inadmissible in evidence. However, the 2 receivers recovered by the policemen outside similar firepower, part of firearm, ammunition, or machinery, tool or
the house of petitioner before the warrant was served were admitted as evidence, pursuant to instrument used or intended to be used in the manufacture of any firearm or
the plain view doctrine. ammunition: Provided, That no other crime was committed.[15]

Accordingly, petitioner and Valerio were convicted of illegal possession of a part of a This is the necessary consequence of the amendment introduced by R.A. No. 8294,
firearm, punishable under paragraph 1, Section 1 of P.D. No. 1866, as amended. They were which categorized the kinds of firearms proscribed from being possessed without a license,
sentenced to an indeterminate penalty of three (3) years, six (6) months, and twenty-one (21) according to their firing power and caliber. R.A. No. 8294 likewise mandated different penalties
days to five (5) years, four (4) months, and twenty (20) days of prision correccional, and ordered for illegal possession of firearm according to the above classification, unlike in the old P.D. No.
to pay a P20,000.00 fine. 1866 which set a standard penalty for the illegal possession of any kind of firearm. Section 1 of
the old law reads:
Petitioner moved for reconsideration,[11] but the motion was denied in the CA
Resolution dated December 3, 2009.[12] Hence, the present recourse. Section 1. Unlawful Manufacture, Sale, Acquisition, Disposition or
Possession of Firearms or Ammunition or Instruments Used or Intended to be Used in
At the onset, it must be emphasized that the information filed against petitioner and the Manufacture of Firearms of Ammunition. The penalty of reclusion temporal in its
maximum period to reclusion perpetua shall be imposed upon any person who shall
Valerio charged duplicitous offenses contrary to Section 13 of Rule 110 of the Rules of Criminal
unlawfully manufacture, deal in, acquire dispose, or possess any firearms, part of
Procedure, viz.: firearm, ammunition, or machinery, tool or instrument used or intended to be used in
the manufacture of any firearm or ammunition. (Emphasis ours.)
Sec. 13. Duplicity of offense. A complaint or information must
charge but one offense, except only in those cases in which existing laws By virtue of such changes, an information for illegal possession of firearm should now
prescribe a single punishment for various offenses. particularly refer to the paragraph of Section 1 under which the seized firearm is classified, and
should there be numerous guns confiscated, each must be sorted and then grouped according to
A reading of the information clearly shows that possession of the enumerated articles the categories stated in Section 1 of R.A. No. 8294, amending P.D. No. 1866. It will no longer
confiscated from Valerio and petitioner are punishable under separate provisions of Section 1, suffice to lump all of the seized firearms in one information, and state Section 1, P.D. No. 1866 as
P.D. No. 1866, as amended by R.A. No. 8294.[13] Illegal possession of two (2) pieces of short the violated provision, as in the instant case,[16]because different penalties are imposed by the
magazine of M16 Armalite rifle, thirty-five (35) pieces of live M16 ammunition 5.56 caliber, and law, depending on the caliber of the weapon. To do so would result in duplicitous charges.
fourteen (14) pieces of live caliber .45 ammunition is punishable under paragraph 2 of the said
section, viz.: Ordinarily, an information that charges multiple offenses merits a quashal, but
petitioner and Valerio failed to raise this issue during arraignment. Their failure constitutes a
81

waiver, and they could be convicted of as many offenses as there were charged in the search in violation of custom laws; (4) seizure of evidence in plain view; and (5) when the
information.[17] This accords propriety to the diverse convictions handed down by the courts a accused himself waives his right against unreasonable searches and seizures.[18]
quo. Under the plain view doctrine, objects falling in the plain view of an officer, who has a
Further, the charge of illegal possession of firearms and ammunition under paragraph 2, Section right to be in the position to have that view, are subject to seizure and may be presented as
1 of P.D. No. 1866, as amended by R.A. No. 8294, including the validity of the search warrant evidence.[19] It applies when the following requisites concur: (a) the law enforcement officer in
that led to their confiscation, is now beyond the province of our review since, by virtue of the search of the evidence has a prior justification for an intrusion or is in a position from which he
CAs Decision, petitioner and Valerio have been effectively acquitted from the said charges. The can view a particular area; (b) the discovery of the evidence in plain view is inadvertent; and (c) it
present review is consequently only with regard to the conviction for illegal possession of a part is immediately apparent to the officer that the item he observes may be evidence of a crime,
of a firearm. contraband, or otherwise subject to seizure. The law enforcement officer must lawfully make an
initial intrusion or properly be in a position from which he can particularly view the area. In the
The Issues course of such lawful intrusion, he came inadvertently across a piece of evidence incriminating
the accused. The object must be open to eye and hand, and its discovery inadvertent.[20]
Petitioner insists on an acquittal and avers that the discovery of the two (2) receivers
does not come within the purview of the plain view doctrine. She argues that no valid intrusion Tested against these standards, we find that the seizure of the two receivers of the .45 caliber
was attendant and that no evidence was adduced to prove that she was with Valerio when he pistol outside petitioners house falls within the purview of the plain view doctrine.
threw the receivers. Likewise absent is a positive showing that any of the two receivers
recovered by the policemen matched the .45 caliber pistol allegedly seen tucked in the First, the presence of SPO2 Nava at the back of the house and of the other law
waistband of her shorts when the police elements arrived. Neither is there any proof that enforcers around the premises was justified by the fact that petitioner and Valerio were earlier
petitioner had knowledge of or consented to the alleged throwing of the receivers. seen respectively holding .45 caliber pistols before they ran inside the structure and sought
refuge. The attendant circumstances and the evasive actions of petitioner and Valerio when the
Our Ruling law enforcers arrived engendered a reasonable ground for the latter to believe that a crime was
being committed. There was thus sufficient probable cause for the policemen to cordon off the
We find merit in the petition. house as they waited for daybreak to apply for a search warrant.

First, we rule on the admissibility of the receivers. We hold that the receivers were seized in plain Secondly, from where he was situated, SPO2 Nava clearly saw, on two different instances,
view, hence, admissible. Valerio emerge on top of the subject dwelling and throw suspicious objects. Lastly, considering
the earlier sighting of Valerio holding a pistol, SPO2 Nava had reasonable ground to believe that
No less than our Constitution recognizes the right of the people to be secure in their persons, the things thrown might be contraband items, or evidence of the offense they were then
houses, papers, and effects against unreasonable searches and seizures. This right is suspected of committing. Indeed, when subsequently recovered, they turned out to be two (2)
encapsulated in Article III, Section 2, of the Constitution, which states: receivers of .45 caliber pistol.

Sec. 2. The right of the people to be secure in their persons, houses, papers, The pertinent portions of SPO2 Navas testimony are elucidating:
and effects against unreasonable searches and seizures of whatever nature and for any
purpose shall be inviolable, and no search warrant or warrant of arrest shall issue Q When you arrived in that place, you saw policemen?
except upon probable cause to be determined personally by the judge after A Yes, sir.
examination under oath or affirmation of the complainant and the witnesses he may Q What were they doing?
produce, and particularly describing the place to be searched and the persons or things A They were cordoning the house.
to be seized. Q You said that you asked your assistant team leader Deluso about that incident. What did he tell you?
A Deluso told me that a person ran inside the house carrying with him a gun.
Complementing this provision is the exclusionary rule embodied in Section 3(2) of the Q And this house you are referring to is the house which you mentioned is the police officers were
same article surrounding?
A Yes, sir.
(2) Any evidence obtained in violation of this or the preceding section shall Q Now, how long did you stay in that place, Mr. Witness?
A I stayed there when I arrived at past 10:00 oclock up to 12:00 oclock the following day.
be inadmissible for any purpose in any proceeding.
Q At about 2:00 oclock in the early morning of August 28, 2002, can you recall where were you?
A Yes, sir.
Q Where were you?
There are, however, several well-recognized exceptions to the foregoing rule. Thus, evidence A I was at the back of the house that is being cordoned by the police.
obtained through a warrantless search and seizure may be admissible under any of the following Q While you were at the back of this house, do you recall any unusual incident?
circumstances: (1) search incident to a lawful arrest; (2) search of a moving motor vehicle; (3) A Yes, sir.
82

Q Can you tell the Honorable Court what was that incident? Q When you entered the premises of the house of the lady, what did you find?
A Yes, sir. A person went out at the top of the house and threw something. A We saw the lower receiver of this .45 cal. (sic)[21]
Q And did you see the person who threw something out of this house?
A Yes, sir. The ensuing recovery of the receivers may have been deliberate; nonetheless, their initial
xxxx discovery was indubitably inadvertent. It is not crucial that at initial sighting the seized contraband be
Q Can you tell the Honorable Court who was that person who threw that something outside the house? identified and known to be so. The law merely requires that the law enforcer observes that the seized
A It was Zaldy Valerio. item may be evidence of a crime, contraband, or otherwise subject to seizure.
COURT: (to witness)
Q Before the incident, you know this person Zaldy Valerio? Hence, as correctly declared by the CA, the two receivers were admissible as evidence. The
A Yes, sir. liability for their possession, however, should fall only on Valerio and not on petitioner.
Q Why do you know him?
A Because we were formerly members of the Armed Forces of the Philippines.
xxxx The foregoing disquisition notwithstanding, we find that petitioner is not liable for illegal
PROS. PERALTA: possession of part of a firearm.
Q When you saw something thrown out at the top of the house, did you do something if any? In dissecting how and when liability for illegal possession of firearms attaches, the
A I shouted to seek cover. following disquisitions in People v. De Gracia[22] are instructive:
xxxx
Q So, what else did you do if any after you shouted, take cover? The rule is that ownership is not an essential element of illegal possession of firearms
A I took hold of a flashlight after five minutes and focused the beam of the flashlight on the place where and ammunition. What the law requires is merely possession which includes
something was thrown.
not only actual physical possession but also constructive possession or the
Q What did you see if any?
A I saw there the lower [part] of the receiver of cal. 45. subjection of the thing to one's control and management. This has to be so if
xxxx the manifest intent of the law is to be effective. The same evils, the same
Q Mr. Witness, at around 4:00 oclock that early morning of August 28, 2002, do you recall another unusual perils to public security, which the law penalizes exist whether the
incident? unlicensed holder of a prohibited weapon be its owner or a borrower. To
A Yes, sir. accomplish the object of this law the proprietary concept of the possession
Q And can you tell us what was that incident? can have no bearing whatsoever.
A I saw a person throwing something there and the one that was thrown fell on top of the roof of another But is the mere fact of physical or constructive possession sufficient to
house.
convict a person for unlawful possession of firearms or must there be an
Q And you saw that person who again threw something from the rooftop of the house?
A Yes, sir. intent to possess to constitute a violation of the law? This query assumes
Q Did you recognize him? significance since the offense of illegal possession of firearms is a malum
A Yes, sir. prohibitum punished by a special law, in which case good faith and absence
Q Who was that person? of criminal intent are not valid defenses.
A Zaldy Valerio again. When the crime is punished by a special law, as a rule, intent to
xxxx commit the crime is not necessary. It is sufficient that the offender has the
Q Where were you when you saw this Zaldy Valerio thr[o]w something out of the house? intent to perpetrate the act prohibited by the special law. Intent to commit
A I was on the road in front of the house.
the crime and intent to perpetrate the act must be distinguished. A person
Q Where was Zaldy Valerio when you saw him thr[o]w something out of the house?
A He was on top of the house. may not have consciously intended to commit a crime; but he did intend to
xxxx commit an act, and that act is, by the very nature of things, the crime itself.
Q Later on, were you able to know what was that something thrown out? In the first (intent to commit the crime), there must be criminal intent; in the
A Yes, sir. second (intent to perpetrate the act) it is enough that the prohibited act is
Q What was that? done freely and consciously.
A Another lower receiver of a cal. 45. In the present case, a distinction should be made between criminal intent
xxxx and intent to possess. While mere possession, without criminal intent, is
Q And what did he tell you?
sufficient to convict a person for illegal possession of a firearm, it must still
A It [was] on the wall of another house and it [could] be seen right away.
xxxx be shown that there was animus possidendi or an intent to possess on the
Q What did you do if any? part of the accused. Such intent to possess is, however, without regard to
A We waited for the owner of the house to wake up. any other criminal or felonious intent which the accused may have harbored
xxxx in possessing the firearm. Criminal intent here refers to the intention of the
Q Who opened the fence for you? accused to commit an offense with the use of an unlicensed firearm. This is
A It was a lady who is the owner of the house.
83

not important in convicting a person under Presidential Decree No. 1866. indubitably proving that petitioner participated in the decision to commit the criminal act
Hence, in order that one may be found guilty of a violation of the decree, it is committed by Valerio.
sufficient that the accused had no authority or license to possess a firearm,
and that he intended to possess the same, even if such possession was made Hence, this Court is constrained to acquit petitioner on the ground of reasonable
in good faith and without criminal intent. doubt. The constitutional presumption of innocence in her favor was not adequately overcome
Concomitantly, a temporary, incidental, casual, or harmless by the evidence adduced by the prosecution.
possession or control of a firearm cannot be considered a violation of a
statute prohibiting the possession of this kind of weapon, such as The CA correctly convicted Valerio with illegal possession of part of a firearm.
Presidential Decree No. 1866. Thus, although there is physical or constructive
possession, for as long as the animus possidendi is absent, there is no offense In illegal possession of a firearm, two (2) things must be shown to exist: (a) the existence of
committed.[23] the subject firearm; and (b) the fact that the accused who possessed the same does not have the
corresponding license for it.[26]
Certainly, illegal possession of firearms, or, in this case, part of a firearm, is committed when the
holder thereof: By analogy then, a successful conviction for illegal possession of part of a firearm must
yield these requisites:
(1) possesses a firearm or a part thereof
(2) lacks the authority or license to possess the firearm.[24] (a) the existence of the part of the firearm; and
(b) the accused who possessed the same does not have the license for the firearm
We find that petitioner was neither in physical nor constructive possession of the to which the seized part/component corresponds.
subject receivers. The testimony of SPO2 Nava clearly bared that he only saw Valerio on top of
the house when the receivers were thrown. None of the witnesses saw petitioner holding the In the instant case, the prosecution proved beyond reasonable doubt the
receivers, before or during their disposal. elements of the crime. The subject receivers - one with the markings United States Property and
the other bearing Serial No. 763025 - were duly presented to the court as Exhibits E and E-1,
At the very least, petitioners possession of the receivers was merely incidental because Valerio, respectively. They were also identified by SPO2 Nava as the firearm parts he retrieved af ter
the one in actual physical possession, was seen at the rooftop of petitioners house. Absent any Valerio discarded them.[27] His testimony was corroborated by DYKR radio announcer Vega, who
evidence pointing to petitioners participation, knowledge or consent in Valerios actions, she witnessed the recovery of the receivers.[28]
cannot be held liable for illegal possession of the receivers.
Anent the lack of authority, SPO1 Tan testified that, upon verification, it was ascertained
Petitioners apparent liability for illegal possession of part of a firearm can only proceed from the that Valerio is not a duly licensed/registered firearm holder of any type, kind, or caliber of
assumption that one of the thrown receivers matches the gun seen tucked in the waistband of firearms.[29] To substantiate his statement, he submitted a certification[30] to that effect and
her shorts earlier that night. Unfortunately, the prosecution failed to convert such assumption identified the same in court.[31] The testimony of SPO1 Tan, or the certification, would suffice to
into concrete evidence. prove beyond reasonable doubt the second element.[32]

Mere speculations and probabilities cannot substitute for proof required to establish WHEREFORE, premises considered, the February 10, 2009 Decision of the Court of Appeals is
the guilt of an accused beyond reasonable doubt. The rule is the same whether the offenses are hereby REVERSED with respect to petitioner Elenita Fajardo y Castro, who is
punishable under the Revised Penal Code, which are mala in se, or in crimes, which are malum hereby ACQUITTED on the ground that her guilt was not proved beyond reasonable doubt.
prohibitum by virtue of special law.[25] The quantum of proof required by law was not adequately SO ORDERED.
met in this case in so far as petitioner is concerned.

The gun allegedly seen tucked in petitioners waistband was not identified with
sufficient particularity; as such, it is impossible to match the same with any of the seized
receivers. Moreover, SPO1 Tan categorically stated that he saw Valerio holding two guns when
he and the rest of the PISOG arrived in petitioners house. It is not unlikely then that the receivers
later on discarded were components of the two (2) pistols seen with Valerio.

These findings also debunk the allegation in the information that petitioner conspired
with Valerio in committing illegal possession of part of a firearm. There is no evidence
84

MALACAT VS CA the affidavit of arrest and booking sheet of petitioner and Casan. Later, Serapio turned over the
In an Information[1] filed on 30 August 1990, in Criminal Case No. 90-86748 before the grenade to the Intelligence and Special Action Division (ISAD) of the Explosive Ordnance Disposal
Regional Trial Court (RTC) of Manila, Branch 5, petitioner Sammy Malacat y Mandar was charged Unit for examination.[11]
with violating Section 3 of Presidential Decree No. 1866,[2] as follows: On cross-examination, Serapio admitted that he took petitioners confession knowing it was
That on or about August 27, 1990, in the City of Manila, Philippines, the said accused did then inadmissible in evidence.[12]
and there willfully, unlawfully and knowingly keep, possess and/or acquire a hand grenade, Orlando Ramilo, a member of the Bomb Disposal Unit, whose principal duties included,
without first securing the necessary license and/or permit therefor from the proper authorities. among other things, the examination of explosive devices, testified that on 22 March 1991, he
At arraignment[3] on 9 October 1990, petitioner, assisted by counsel de oficio, entered a received a request dated 19 March 1991 from Lt. Eduardo Cabrera and PO Diosdado Diotoy for
plea of not guilty. examination of a grenade. Ramilo then affixed an orange tag on the subject grenade detailing his
At pre-trial on 11 March 1991, petitioner admitted the existence of Exhibits A, A-1, and A- name, the date and time he received the specimen. During the preliminary examination of the
2,[4] while the prosecution admitted that the police authorities were not armed with a search grenade, he [f]ound that [the] major components consisting of [a] high filler and fuse assembly
warrant nor warrant of arrest at the time they arrested petitioner. [5] [were] all present, and concluded that the grenade was [l]ive and capable of exploding. On even
At trial on the merits, the prosecution presented the following police officers as its date, he issued a certification stating his findings, a copy of which he forwarded to Diotoy on 11
witnesses: Rodolfo Yu, the arresting officer; Josefino G. Serapio, the investigating officer; and August 1991.[13]
Orlando Ramilo, who examined the grenade. Petitioner was the lone defense witness. He declared that he arrived in Manila on 22 July
Rodolfo Yu of the Western Police District, Metropolitan Police Force of the Integrated 1990 and resided at the Muslim Center in Quiapo, Manila. At around 6:30 in the evening of 27
National Police, Police Station No. 3, Quiapo, Manila, testified that on 27 August 1990, at about August 1990, he went to Plaza Miranda to catch a breath of fresh air. Shortly after, several
6:30 p.m., in response to bomb threats reported seven days earlier, he was on foot patrol with policemen arrived and ordered all males to stand aside. The policemen searched petitioner and
three other police officers (all of them in uniform) along Quezon Boulevard, Quiapo, Manila, near two other men, but found nothing in their possession. However, he was arrested with two
the Mercury Drug store at Plaza Miranda. They chanced upon two groups of Muslim-looking others, brought to and detained at Precinct No. 3, where he was accused of having shot a police
men, with each group, comprised of three to four men, posted at opposite sides of the corner of officer. The officer showed the gunshot wounds he allegedly sustained and shouted at
Quezon Boulevard near the Mercury Drug Store. These men were acting suspiciously with [t]heir petitioner [i]to ang tama mo sa akin. This officer then inserted the muzzle of his gun into
eyes moving very fast.[6] petitioners mouth and said, [y]ou are the one who shot me. Petitioner denied the charges and
Yu and his companions positioned themselves at strategic points and observed both explained that he only recently arrived in Manila. However, several other police officers mauled
groups for about thirty minutes. The police officers then approached one group of men, who him, hitting him with benches and guns. Petitioner was once again searched, but nothing was
then fled in different directions. As the policemen gave chase, Yu caught up with and found on him. He saw the grenade only in court when it was presented. [14]
apprehended petitioner. Upon searching petitioner, Yu found a fragmentation grenade tucked The trial court ruled that the warrantless search and seizure of petitioner was akin to a
inside petitioners front waist line.[7] Yus companion, police officer Rogelio Malibiran, stop and frisk, where a warrant and seizure can be effected without necessarily being preceded
apprehended Abdul Casan from whom a .38 caliber revolver was recovered. Petitioner and by an arrest and whose object is either to maintain the status quo momentarily while the police
Casan were then brought to Police Station No. 3 where Yu placed an X mark at the bottom of the officer seeks to obtain more information.[15] Probable cause was not required as it was not
grenade and thereafter gave it to his commander.[8] certain that a crime had been committed, however, the situation called for an investigation,
On cross-examination, Yu declared that they conducted the foot patrol due to a report that hence to require probable cause would have been premature.[16] The RTC emphasized that Yu
a group of Muslims was going to explode a grenade somewhere in the vicinity of Plaza and his companions were [c]onfronted with an emergency, in which the delay necessary to
Miranda. Yu recognized petitioner as the previous Saturday, 25 August 1990, likewise at Plaza obtain a warrant, threatens the destruction of evidence [17] and the officers [h]ad to act in haste,
Miranda, Yu saw petitioner and 2 others attempt to detonate a grenade. The attempt was as petitioner and his companions were acting suspiciously, considering the time, place and
aborted when Yu and other policemen chased petitioner and his companions; however, the reported cases of bombing. Further, petitioners group suddenly ran away in different directions
former were unable to catch any of the latter. Yu further admitted that petitioner and Casan as they saw the arresting officers approach, thus [i]t is reasonable for an officer to conduct a
were merely standing on the corner of Quezon Boulevard when Yu saw them on 27 August limited search, the purpose of which is not necessarily to discover evidence of a crime, but to
1990. Although they were not creating a commotion, since they were supposedly acting allow the officer to pursue his investigation without fear of violence.[18]
suspiciously, Yu and his companions approached them. Yu did not issue any receipt for the The trial court then ruled that the seizure of the grenade from petitioner was incidental to
grenade he allegedly recovered from petitioner.[9] a lawful arrest, and since petitioner [l]ater voluntarily admitted such fact to the police
Josefino G. Serapio declared that at about 9:00 a.m. of 28 August 1990, petitioner and a investigator for the purpose of bombing the Mercury Drug Store, concluded that sufficient
certain Abdul Casan were brought in by Sgt. Saquilla [10] for investigation. Forthwith, Serapio evidence existed to establish petitioners guilt beyond reasonable doubt.
conducted the inquest of the two suspects, informing them of their rights to remain silent and to In its decision[19] dated 10 February 1994 but promulgated on 15 February 1994, the trial
be assisted by competent and independent counsel. Despite Serapios advice, petitioner and court thus found petitioner guilty of the crime of illegal possession of explosives under Section 3
Casan manifested their willingness to answer questions even without the assistance of a lawyer. of P.D. No. 1866, and sentenced him to suffer:
Serapio then took petitioners uncounselled confession (Exh. E), there being no PAO lawyer
available, wherein petitioner admitted possession of the grenade. Thereafter, Serapio prepared
85

[T]he penalty of not less than SEVENTEEN (17) YEARS, FOUR (4) MONTHS AND ONE (1) DAY Finally, the Court of Appeals held that the rule laid down in People v. Mengote, [26] which
OF RECLUSION TEMPORAL, as minimum, and not more than THIRTY (30) YEARS OF RECLUSION petitioner relied upon, was inapplicable in light of [c]rucial differences, to wit:
PERPETUA, as maximum. [In Mengote] the police officers never received any intelligence report that someone [at] the
On 18 February 1994, petitioner filed a notice of appeal[20] indicating that he was appealing corner of a busy street [would] be in possession of a prohibited article. Here the police officers
to this Court. However, the record of the case was forwarded to the Court of Appeals which were responding to a [sic] public clamor to put a check on the series of terroristic bombings in
docketed it as CA-G.R. CR No. 15988 and issued a notice to file briefs.[21] the Metropolis, and, after receiving intelligence reports about a bomb threat aimed at the
In his Appellants Brief [22] filed with the Court of Appeals, petitioner asserted that: vicinity of the historically notorious Plaza Miranda, they conducted foot patrols for about seven
1. THE LOWER COURT ERRED IN HOLDING THAT THE SEARCH UPON THE PERSON OF days to observe suspicious movements in the area. Furthermore, in Mengote, the police officers
ACCUSED-APPELLANT AND THE SEIZURE OF THE ALLEGED HANDGRENADE FROM [had] no personal knowledge that the person arrested has committed, is actually committing, or
HIM WAS AN APPROPRIATE INCIDENT TO HIS ARREST. is attempting to commit an offense. Here, PO3 Yu [had] personal knowledge of the fact that he
2. THE LOWER COURT ERRED IN ADMITTING AS EVIDENCE AGAINST ACCUSED- chased Malacat in Plaza Miranda two days before he finally succeeded in apprehending him.
APPELLANT THE HANDGRENADE ALLEGEDLY SEIZED FROM HIM AS IT WAS A Unable to accept his conviction, petitioner forthwith filed the instant petition and assigns
PRODUCT OF AN UNREASONABLE AND ILLEGAL SEARCH. the following errors:
In sum, petitioner argued that the warrantless arrest was invalid due to absence of any of 1. THE RESPONDENT COURT ERRED IN AFFIRMING THE FINDING OF THE TRIAL
the conditions provided for in Section 5 of Rule 113 of the Rules of Court, citing People vs. COURT THAT THE WARRANTLESS ARREST OF PETITIONER WAS VALID AND LEGAL.
Mengote.[23] As such, the search was illegal, and the hand grenade seized, inadmissible in 2. THE RESPONDENT COURT ERRED IN HOLDING THAT THE RULING IN PEOPLE VS.
evidence. MENGOTE DOES NOT FIND APPLICATION IN THE INSTANT CASE.
In its Brief for the Appellee, the Office of the Solicitor General agreed with the trial court In support thereof, petitioner merely restates his arguments below regarding the validity of the
and prayed that its decision be affirmed in toto.[24] warrantless arrest and search, then disagrees with the finding of the Court of Appeals that he
In its decision of 24 January 1996,[25] the Court of Appeals affirmed the trial court, noting, was attempting to commit a crime, as the evidence for the prosecution merely disclosed that he
first, that petitioner abandoned his original theory before the court a quo that the grenade was was standing at the corner of Plaza Miranda and Quezon Boulevard with his eyes moving very
planted by the police officers; and second, the factual finding of the trial court that the grenade fast and looking at every person that come (sic) nearer (sic) to them. Finally, petitioner points
was seized from petitioners possession was not raised as an issue. Further, respondent court out the factual similarities between his case and that of People v. Mengote to demonstrate that
focused on the admissibility in evidence of Exhibit D, the hand grenade seized from the Court of Appeals miscomprehended the latter.
petitioner. Meeting the issue squarely, the Court of Appeals ruled that the arrest was lawful on In its Comment, the Office of the Solicitor General prays that we affirm the challenged
the ground that there was probable cause for the arrest as petitioner was attempting to commit decision.
an offense, thus: For being impressed with merit, we resolved to give due course to the petition.
We are at a loss to understand how a man, who was in possession of a live grenade and in the The challenged decision must immediately fall on jurisdictional grounds. To repeat, the
company of other suspicious character[s] with unlicensed firearm[s] lurking in Plaza Miranda at a penalty imposed by the trial court was:
time when political tension ha[d] been enkindling a series of terroristic activities, [can] claim that [N]ot less than SEVENTEEN (17) YEARS, FOUR (4) MONTHS AND ONE (1) DAY
he was not attempting to commit an offense. We need not mention that Plaza Miranda is OF RECLUSION TEMPORAL, as minimum, and not more than THIRTY (30) YEARS
historically notorious for being a favorite bomb site especially during times of political OF RECLUSION PERPETUA, as maximum.
upheaval. As the mere possession of an unlicensed grenade is by itself an offense, Malacats The penalty provided by Section 3 of P.D. No. 1866 upon any person who shall unlawfully
posture is simply too preposterous to inspire belief. possess grenades is reclusion temporal in its maximum period to reclusion perpetua.
In so doing, the Court of Appeals took into account petitioners failure to rebut the For purposes of determining appellate jurisdiction in criminal cases, the maximum of the
testimony of the prosecution witnesses that they received intelligence reports of a bomb threat penalty, and not the minimum, is taken into account. Since the maximum of the penalty
at Plaza Miranda; the fact that PO Yu chased petitioner two days prior to the latters arrest, or on isreclusion perpetua, the appeal therefrom should have been to us, and not the Court of Appeals,
27 August 1990; and that petitioner and his companions acted suspiciously, the accumulation of pursuant to Section 9(3) of the Judiciary Reorganization Act of 1980 (B.P. Blg. 129), [27] in relation
which was more than sufficient to convince a reasonable man that an offense was about to be to Section 17 of the Judiciary Act of 1948,[28] Section 5(2) of Article VIII of the Constitution[29] and
committed. Moreover, the Court of Appeals observed: Section 3(c) of Rule 122 of the Rules of Court.[30] The term life imprisonment as used in Section 9
The police officers in such a volatile situation would be guilty of gross negligence and dereliction of B.P. Blg. 129, the Judiciary Act of 1948, and Section 3 of Rule 122 must be deemed to
of duty, not to mention of gross incompetence, if they [would] first wait for Malacat to hurl the include reclusion perpetua in view of Section 5(2) of Article VIII of the Constitution.
grenade, and kill several innocent persons while maiming numerous others, before arriving at Petitioners Notice of Appeal indicated that he was appealing from the trial courts decision
what would then be an assured but moot conclusion that there was indeed probable cause for to this Court, yet the trial court transmitted the record to the Court of Appeals and the latter
an arrest. We are in agreement with the lower court in saying that the probable cause in such a proceeded to resolve the appeal.
situation should not be the kind of proof necessary to convict, but rather the practical We then set aside the decision of the Court of Appeals for having been rendered without
considerations of everyday life on which a reasonable and prudent mind, and not legal jurisdiction, and consider the appeal as having been directly brought to us, with the petition for
technicians, will ordinarily act. review as petitioners Brief for the Appellant, the comment thereon by the Office of the Solicitor
86

General as the Brief for the Appellee and the memoranda of the parties as their Supplemental Sec. 5. -- Arrest, without warrant; when lawful -- A peace officer or a private person may, without
Briefs. a warrant, arrest a person:
Deliberating on the foregoing pleadings, we find ourselves convinced that the prosecution (a) When, in his presence, the person to be arrested has committed, is actually
failed to establish petitioners guilt with moral certainty. committing, or is attempting to commit an offense;
First, serious doubt surrounds the story of police officer Yu that a grenade was found in (b) When an offense has in fact just been committed, and he has personal knowledge
and seized from petitioners possession. Notably, Yu did not identify, in court, the grenade he of facts indicating that the person to be arrested has committed it; and
allegedly seized. According to him, he turned it over to his commander after putting an X mark at (c) When the person to be arrested is a prisoner who has escaped ***
its bottom; however, the commander was not presented to corroborate this claim. On the other A warrantless arrest under the circumstances contemplated under Section 5(a) has been
hand, the grenade presented in court and identified by police officer Ramilo referred to what the denominated as one "in flagrante delicto," while that under Section 5(b) has been described as a
latter received from Lt. Eduardo Cabrera and police officer Diotoy not immediately after "hot pursuit" arrest.
petitioners arrest, but nearly seven (7) months later, or on 19 March 1991; further, there was no Turning to valid warrantless searches, they are limited to the following: (1) customs
evidence whatsoever that what Ramilo received was the very same grenade seized from searches; (2) search of moving vehicles; (3) seizure of evidence in plain view; (4) consent
petitioner. In his testimony, Yu never declared that the grenade passed on to Ramilo was the searches;[33] (5) a search incidental to a lawful arrest;[34] and (6) a "stop and frisk."[35]
grenade the former confiscated from petitioner. Yu did not, and was not made to, identify the In the instant petition, the trial court validated the warrantless search as a stop and frisk
grenade examined by Ramilo, and the latter did not claim that the grenade he examined was with the seizure of the grenade from the accused [as] an appropriate incident to his arrest,
that seized from petitioner. Plainly, the law enforcement authorities failed to safeguard and hence necessitating a brief discussion on the nature of these exceptions to the warrant
preserve the chain of evidence so crucial in cases such as these. requirement.
Second, if indeed petitioner had a grenade with him, and that two days earlier he was with At the outset, we note that the trial court confused the concepts of a "stop-and-frisk" and
a group about to detonate an explosive at Plaza Miranda, and Yu and his fellow officers chased, of a search incidental to a lawful arrest. These two types of warrantless searches differ in terms
but failed to arrest them, then considering that Yu and his three fellow officers were in uniform of the requisite quantum of proof before they may be validly effected and in their allowable
and therefore easily cognizable as police officers, it was then unnatural and against common scope.
experience that petitioner simply stood there in proximity to the police officers. Note that Yu In a search incidental to a lawful arrest, as the precedent arrest determines the validity of
observed petitioner for thirty minutes and must have been close enough to petitioner in order to the incidental search, the legality of the arrest is questioned in a large majority of these cases,
discern petitioners eyes moving very fast. e.g., whether an arrest was merely used as a pretext for conducting a search.[36] In this instance,
Finally, even assuming that petitioner admitted possession of the grenade during his the law requires that there first be a lawful arrest before a search can be made -- the process
custodial investigation by police officer Serapio, such admission was inadmissible in evidence for cannot be reversed.[37] At bottom, assuming a valid arrest, the arresting officer may search the
it was taken in palpable violation of Section 12(1) and (3) of Article III of the Constitution, which person of the arrestee and the area within which the latter may reach for a weapon or for
provide as follows: evidence to destroy, and seize any money or property found which was used in the commission
SEC. 12 (1). Any person under investigation for the commission of an offense shall have the right of the crime, or the fruit of the crime, or that which may be used as evidence, or which might
to be informed of his right to remain silent and to have competent and independent counsel furnish the arrestee with the means of escaping or committing violence.[38]
preferably of his own choice. If the person cannot afford the services of counsel, he must be Here, there could have been no valid in flagrante delicto or hot pursuit arrest preceding
provided with one. These rights cannot be waived except in writing and in the presence of the search in light of the lack of personal knowledge on the part of Yu, the arresting officer, or an
counsel. overt physical act, on the part of petitioner, indicating that a crime had just been committed,
xxx was being committed or was going to be committed.
(3) Any confession or admission obtained in violation of this or Section 17 hereof Having thus shown the invalidity of the warrantless arrest in this case, plainly, the search
shall be inadmissible in evidence against him. conducted on petitioner could not have been one incidental to a lawful arrest.
Serapio conducted the custodial investigation on petitioner the day following his arrest. No We now proceed to the justification for and allowable scope of a "stop-and-frisk" as a
lawyer was present and Serapio could not have requested a lawyer to assist petitioner as no PAO "limited protective search of outer clothing for weapons," as laid down in Terry, thus:
lawyer was then available. Thus, even if petitioner consented to the investigation and waived his We merely hold today that where a police officer observes unusual conduct which leads
rights to remain silent and to counsel, the waiver was invalid as it was not in writing, neither was him reasonably to conclude in light of his experience that criminal activity may be afoot
it executed in the presence of counsel. and that the persons with whom he is dealing may be armed and presently dangerous,
Even granting ex gratia that petitioner was in possession of a grenade, the arrest and where in the course of investigating this behavior he identifies himself as a policeman and
search of petitioner were invalid, as will be discussed below. makes reasonable inquiries, and where nothing in the initial stages of the encounter serves
The general rule as regards arrests, searches and seizures is that a warrant is needed in to dispel his reasonable fear for his own or others' safety, he is entitled for the protection
order to validly effect the same.[31] The Constitutional prohibition against unreasonable arrests, of himself and others in the area to conduct a carefully limited search of the outer clothing
searches and seizures refers to those effected without a validly issued warrant, [32] subject to of such persons in an attempt to discover weapons which might be used to assault
certain exceptions. As regards valid warrantless arrests, these are found in Section 5, Rule 113 of him. Such a search is a reasonable search under the Fourth Amendment ***[39]
the Rules of Court, which reads, in part:
87

Other notable points of Terry are that while probable cause is not required to conduct a "stop ground of reasonable doubt, the decision of 10 February 1994 of Branch 5 of the Regional Trial
and frisk,"[40] it nevertheless holds that mere suspicion or a hunch will not validate a "stop and Court of Manila is REVERSED and petitioner SAMMY MALACAT y MANDAR is hereby ACQUITTED
frisk." A genuine reason must exist, in light of the police officer's experience and surrounding and ORDERED immediately released from detention, unless his further detention is justified for
conditions, to warrant the belief that the person detained has weapons concealed about any other lawful cause.
him.[41]Finally, a "stop-and-frisk" serves a two-fold interest: (1) the general interest of effective Costs de oficio.
crime prevention and detection, which underlies the recognition that a police officer may, under SO ORDERED.
appropriate circumstances and in an appropriate manner, approach a person for purposes of
investigating possible criminal behavior even without probable cause; and (2) the more pressing
interest of safety and self-preservation which permit the police officer to take steps to assure
himself that the person with whom he deals is not armed with a deadly weapon that could
unexpectedly and fatally be used against the police officer.
Here, here are at least three (3) reasons why the stop-and-frisk was invalid:
First, we harbor grave doubts as to Yus claim that petitioner was a member of the group
which attempted to bomb Plaza Miranda two days earlier. This claim is neither supported by any
police report or record nor corroborated by any other police officer who allegedly chased that
group. Aside from impairing Yu's credibility as a witness, this likewise diminishes the probability
that a genuine reason existed so as to arrest and search petitioner. If only to further tarnish the
credibility of Yu's testimony, contrary to his claim that petitioner and his companions had to be
chased before being apprehended, the affidavit of arrest (Exh. "A") expressly declares otherwise,
i.e., upon arrival of five (5) other police officers, petitioner and his companions were
"immediately collared."
Second, there was nothing in petitioners behavior or conduct which could have reasonably
elicited even mere suspicion other than that his eyes were moving very fast an observation
which leaves us incredulous since Yu and his teammates were nowhere near petitioner and it
was already 6:30 p.m., thus presumably dusk. Petitioner and his companions were merely
standing at the corner and were not creating any commotion or trouble, as Yu explicitly declared
on cross-examination:
Q And what were they doing?
A They were merely standing.
Q You are sure of that?
A Yes, sir.
Q And when you saw them standing, there were nothing or they did not create any
commotion?
A None, sir.
Q Neither did you see them create commotion?
A None, sir.[42]
Third, there was at all no ground, probable or otherwise, to believe that petitioner was
armed with a deadly weapon. None was visible to Yu, for as he admitted, the alleged grenade
was discovered inside the front waistline of petitioner, and from all indications as to the distance
between Yu and petitioner, any telltale bulge, assuming that petitioner was indeed hiding a
grenade, could not have been visible to Yu. In fact, as noted by the trial court:
When the policemen approached the accused and his companions, they were not yet aware that
a handgrenade was tucked inside his waistline. They did not see any bulging object in [sic] his
person.[43]
What is unequivocal then in this case are blatant violations of petitioners rights solemnly
guaranteed in Sections 2 and 12(1) of Article III of the Constitution.
WHEREFORE, the challenged decision of the Seventeenth Division of the Court of Appeals
in CA-G.R. CR No. 15988 is SET ASIDE for lack of jurisdiction on the part of said Court and, on
88

PEOPLE VS MUSA places about 90 to 100 meters from Mari Musa's house. T/Sgt. Belarga could
The appellant, Mari Musa, seeks, in this appeal, the reversal of the decision, dated August 31, see what went on between Ani and suspect Mari Musa from where he was.
1990, 1 of the Regional Trial Court (RTC) of Zamboanga City, Branch XII, finding him guilty of Ani approached Mari Musa, who came out of his house, and asked Ani what
selling marijuana in violation of Article II, Section 4 of Republic Act No. 6425, as amended, he wanted. Ani said he wanted some more stuff. Ani gave Mari Musa the
otherwise known as the Dangerous Drugs Act of 1972. P20.00 marked money. After receiving the money, Mari Musa went back to
The information filed on December 15, 1989 against the appellant reads: his house and came back and gave Amado Ani two newspaper wrappers
That on or about December 14, 1989, in the City of Zamboanga, Philippines, containing dried marijuana. Ani opened the two wrappers and inspected the
and within the jurisdiction of this Honorable Court, the contents. Convinced that the contents were marijuana, Ani walked back
above-named accused, not being authorized by law, did then and there, towards his companions and raised his right hand. The two NARCOM teams,
wilfully, unlawfully and feloniously sell to one SGT. AMADO ANI, two (2) riding the two civilian vehicles, sped towards Sgt. Ani. Ani joined Belarga's
wrappers containing dried marijuana leaves, knowing the same to be a team and returned to the house.
prohibited drug. At the time Sgt. Ani first approached Mari Musa, there were four persons
CONTRARY TO LAW. 2 inside his house: Mari Musa, another boy, and two women, one of whom Ani
Upon his arraignment on January 11, 1990, the appellant pleaded not guilty. 3 and Belarga later came to know to be Mari Musa's wife. The second time, Ani
At the trial, the prosecution presented three (3) witnesses, namely: (1) Sgt. Amado Ani, Jr. of the with the NARCOM team returned to Mari Musa's house, the woman, who
9th Narcotics Command (NARCOM) of Zamboanga City, who acted as poseur-buyer in the buy- was later known as Mari Musa's wife, slipped away from the house. Sgt.
bust operation made against the appellant; (2) T/Sgt. Jesus Belarga, also of the 9th Narcotics Belarga frisked Mari Musa but could not find the P20.00 marked money with
Command of Zamboanga City, who was the NARCOM team leader of the buy-bust operation; him. Mari Musa was then asked where the P20.00 was and he told the
and (3) Athena Elisa P. Anderson, the Document Examiner and Forensic Chemist of PC-INP Crime NARCOM team he has given the money to his wife (who had slipped away).
Laboratory of Regional Command (RECOM) 9. The evidence of the prosecution was summarized Sgt. Belarga also found a plastic bag containing dried marijuana inside it
by the trial court as follows: somewhere in the kitchen. Mari Musa was then placed under arrest and
Prosecution evidence shows that in the morning of December 13, 1989, brought to the NARCOM office. At Suterville, Sgt. Ani turned over to Sgt.
T/Sgt. Jesus Belarga, leader of a NARCOTICS COMMAND (NARCOM) team Belarga the two newspaper-wrapped marijuana he had earlier bought from
based at Calarian, Zamboanga City, instructed Sgt. Amado Ani to conduct Mari Musa (Exhs. "C" & "D").
surveillance and test buy on a certain Mari Musa of Suterville, Zamboanga In the NARCOM office, Mari Musa first gave his name as Hussin Musa. Later
City. Information received from civilian informer was that this Mari Musa was on, Mari Musa gave his true name — Mari Musa. T/Sgt. Jesus Belarga turned
engaged in selling marijuana in said place. So Sgt. Amado Ani, another over the two newspaper-wrapped marijuana (bought at the buy-bust), the
NARCOM agent, proceeded to Suterville, in company with a NARCOM civilian one newspaper-wrapped marijuana (bought at the test-buy) and the plastic
informer, to the house of Mari Musa to which house the civilian informer had bag containing more marijuana (which had been taken by Sgt. Lego inside the
guided him. The same civilian informer had also described to him the kitchen of Mari Musa) to the PC Crime Laboratory, Zamboanga City, for
appearance of Mari Musa. Amado Ani was able to buy one newspaper- laboratory examination. The turnover of the marijuana specimen to the PC
wrapped dried marijuana (Exh. "E") for P10.00. Sgt. Ani returned to the Crime Laboratory was by way of a letter-request, dated December 14, 1989
NARCOM office and turned over the newspaper-wrapped marijuana to T/Sgt. (Exh. "B"), which was stamped "RECEIVED" by the PC Crime Laboratory (Exh.
Jesus Belarga. Sgt. Belarga inspected the stuff turned over to him and found "B-1") on the same day.
it to be marijuana. Mrs. Athena Elisa P. Anderson, the Forensic Chemist of the PC Crime
The next day, December 14, 1989, about 1:30 P.M., a buy-bust was planned. Laboratory, examined the marijuana specimens subjecting the same to her
Sgt. Amado Ani was assigned as the poseur buyer for which purpose he was three tests. All submitted specimens she examined gave positive results for
given P20.00 (with SN GA955883) by Belarga. The the presence of marijuana. Mrs. Anderson reported the results of her
buy-bust money had been taken by T/Sgt. Jesus Belarga from M/Sgt. Noh Sali examination in her Chemistry Report D-100-89, dated December 14, 1989,
Mihasun, Chief of Investigation Section, and for which Belarga signed a (Exh. "J", "J-1", "J-2", "J-3", "J-4" and "J-5"). Mrs. Anderson identified in court
receipt (Exh. "L" & "L-l" ) The team under Sgt. Foncargas was assigned as the two newspaper wrapped marijuana bought at the
back-up security. A pre-arranged signal was arranged consisting of Sgt. Ani's buy-bust on December 14, 1989, through her initial and the weight of each
raising his right hand, after he had succeeded to buy the marijuana. The two specimen written with red ink on each wrapper (Exhs. "C-1" and "D-1"). She
NARCOM teams proceeded to the target site in two civilian vehicles. also identified the one newspaper-wrapped marijuana bought at the test-buy
Belarga's team was composed of Sgt. Belarga, team leader, Sgt. Amado Ani, on December 13, 1989, through her markings (Exh. "E-1"). Mrs. Anderson
poseur buyer, Sgt. Lego and Sgt. Biong. also identified her Chemistry Report (Exh. "J" & sub-markings.)
Arriving at the target site, Sgt. Ani proceeded to the house of Mari Musa, T. Sgt. Belarga identified the two buy-bust newspaper wrapped marijuana
while the rest of the NARCOM group positioned themselves at strategic through his initial, the words "buy-bust" and the words "December 14, 1989,
89

2:45 P.M." (written on Exhs. "C" and "D"). Belarga also identified the receipt afraid that was against the law and that the person selling marijuana was
of the P20 marked money (with SN GA955883) (Exh. "L"), dated December caught by the authorities; and he had a wife and a very small child to
14, 1989, and his signature thereon (Exh. support. Mari Musa said he had not been arrested for selling marijuana
"L-1"). He also identified the letter-request, dated December 14, 1989, before. 5
addressed to the PC Crime Laboratory (Exh. "B") and his signature thereon After trial, the trial court rendered the assailed decision with the following disposition:
(Exh. "B-2") and the stamp of the PC Crime Laboratory marked "RECEIVED" WHEREFORE, finding accused Mari Musa y Hantatalu guilty beyond
(Exh. "B-1"). 4 reasonable doubt of selling marijuana and pursuant to Sec. 4, Art II of Rep.
For the defense, the following testified as witnesses: (1) the accused-appellant Mari H. Musa; Act No. 6425, he is sentenced to life imprisonment and to pay the fine of
and (2) Ahara R. Musa, his wife. The trial court summarized the version of the defense, thus: P20,000.00, the latter imposed without subsidiary imprisonment. 6
[O]n December 14, 1989, at about 1:30 in the afternoon, Mari Musa was in In this appeal, the appellant contends that his guilt was not proved beyond reasonable doubt
his house at Suterville, Zamboanga City. With him were his wife, Ahara Musa, and impugns the credibility of the prosecution witnesses.
known as Ara, his one-year old child, a woman manicurist, and a male cousin The appellant claims that the testimony of Sgt. Ani, the poseur-buyer, is not credible because: (1)
named Abdul Musa. About 1:30 that afternoon, while he was being prior to the buy-bust operation, neither Sgt. Ani nor the other NARCOM agents were personally
manicured at one hand, his wife was inside the one room of their house, known by the appellant or vice-versa; and (2) there was no witness to the alleged giving of the
putting their child to sleep. Three NARCOM agents, who introduced two wrappers of marijuana by the appellant to Sgt. Ani.
themselves as NARCOM agents, dressed in civilian clothes, got inside Mari Sgt. Ani testified that on December 13, 1989, upon instruction by T/Sgt. Jesus Belarga, he
Musa's house whose door was open. The NARCOM agents did not ask conducted a test-buy operation on the appellant whereby he bought one wrapper of marijuana
permission to enter the house but simply announced that they were for P15.00 from the latter. 7 He reported the successful operation to T/Sgt. Belarga on the same
NARCOM agents. The NARCOM agents searched Mari Musa's house and Mari day. 8 Whereupon, T/Sgt. Belarga conducted a conference to organize a buy-bust operation for
Musa asked them if they had a search warrant. The NARCOM agents were the following day. 9
just silent. The NARCOM agents found a red plastic bag whose contents, Mari On December 14, 1989, at 1:30 p.m., two NARCOM teams in separate vehicles headed by T/Sgt.
Musa said, he did not know. He also did not know if the plastic bag belonged Belarga and a certain Sgt. Foncardas went to the place of operation, which was the appellant's
to his brother, Faisal, who was living with him, or his father, who was living in house located in Laquian Compound, Suterville, Zamboanga City. Sgt. Ani was with the team of
another house about ten arms-length away. Mari Musa, then, was T/Sgt. Belarga, whose other members were Sgts. Lego and Biong. 10 Sgt. Ani was given a marked
handcuffed and when Mari Musa asked why, the NARCOM agents told him P20.00 bill by T/Sgt. Belarga, which was to be used in the operation.
for clarification. Upon reaching the place, the NARCOM agents positioned themselves at strategic places. 11 Sgt.
Mari Musa was brought in a pick-up, his wife joining him to the NARCOM Ani approached the house. Outside the house, the appellant asked Sgt. Ani what he wanted. Sgt.
Office at Calarian, Zamboanga City. Inside the NARCOM Office, Mari Musa Ani asked him for some more marijuana.12 Sgt. Ani gave him the marked P20.00 bill and the
was investigated by one NARCOM agent which investigation was reduced appellant went inside the house and brought back two paper wrappers containing marijuana
into writing. The writing or document was interpreted to Mari Musa in which he handed to Sgt. Ani. 13 From his position, Sgt. Ani could see that there were other people
Tagalog. The document stated that the marijuana belonged to Mari Musa in the house. 14
and Mari Musa was asked to sign it. But Mari Musa refused to sign because After the exchange, Sgt. Ani approached the other NARCOM agents and made the pre-arranged
the marijuana did not belong to him. Mari Musa said he was not told that he signal of raising his right hand. 15 The NARCOM agents, accompanied by Sgt. Ani, went inside the
was entitled to the assistance of counsel, although he himself told the house and made the arrest. The agents searched the appellant and unable to find the marked
NARCOM agents he wanted to be assisted by counsel. money, they asked him where it was. The appellant said that he gave it to his wife. 16
Mari Musa said four bullets were then placed between the fingers of his right The Court, after a careful reading of the record, finds the testimony of Sgt. Ani regarding the buy-
hand and his fingers were pressed which felt very painful. The NARCOM bust operation, which resulted in the apprehension, prosecution and subsequent conviction of
agents boxed him and Mari Musa lost consciousness. While Mari Musa was the appellant, to be direct, lucid and forthright. Being totally untainted by contradictions in any
maltreated, he said his wife was outside the NARCOM building. The very day of the material points, it deserves credence.
he was arrested (on cross-examination Mari Musa said it was on the next The contention that the appellant could not have transacted with Sgt. Ani because they do not
day), Mari Musa was brought to the Fiscal's Office by three NARCOM agents. know each other is without merit. The day before the
The fiscal asked him if the marijuana was owned by him and he said "not." buy-bust operation, Sgt. Ani conducted a test-buy and he successfully bought a wrapper of
After that single question, Mari Musa was brought to the City Jail. Mari Musa marijuana from the appellant. Through this previous transaction, Sgt. Ani was able to gain the
said he did not tell the fiscal that he had been maltreated by the NARCOM appellant's confidence for the latter to sell more marijuana to Sgt. Ani the following day, during
agents because he was afraid he might be maltreated in the fiscal's office. the buy-bust operation. Moreover, the Court has held that what matters is not an existing
Mari Musa denied the NARCOM agents' charge that he had sold two familiarity between the buyer and the seller, for quite often, the parties to the transaction may
wrappers of marijuana to them; that he had received from them a P20.00 bill be strangers, but their agreement and the acts constituting the sale and delivery of the
which he had given to his wife. He did not sell marijuana because he was marijuana. 17
90

The appellant, again to cast doubt on the credibility of Sgt. Ani, argues that it was impossible for Contrary to the contention of the appellant, it was not impossible for T/Sgt. Belarga to have
the appellant to sell marijuana while his wife, cousin and manicurist were present. But the place seen, from a distance of 90-100 meters, Sgt. Ani hand to the appellant "something" and for the
of the commission of the crime of selling prohibited drugs has been held to be not crucial 18 and latter to give to the former "something."
the presence of other people apart from the buyer and seller will not necessarily prevent the Notwithstanding the fact that T/Sgt. Belarga could not have been certain that what Sgt. Ani
consummation of the illegal sale. As the Court observed in People v. Paco, 19 these factors may received from the appellant was marijuana because of the distance, his testimony, nevertheless,
sometimes camouflage the commission of the crime. In the instant case, the fact that the other corroborated the direct evidence, which the Court earlier ruled to be convincing, presented by
people inside the appellant's house are known to the appellant may have given him some Sgt. Ani on the following material points: (1) T/Sgt. Belarga instructed Sgt. Ani to conduct a
assurance that these people will not report him to the authorities. surveillance and test-buy operation on the appellant at Suterville, Zamboanga City on December
The appellant, besides assailing Sgt. Ani's credibility, also questions the credibility of T/Sgt. 13, 1989; 23 (2) later that same day, Sgt. Ani went back to their office and reported a successful
Belarga. The appellant submits that since T/Sgt. Belarga admitted that he was about 90 meters operation and turned over to T/Sgt. Belarga one wrapper of marijuana; 24 (3) T/Sgt. Belarga then
away from Sgt. Ani and the appellant, he could not have possibly witnessed the sale. The organized a team to conduct a buy-bust operation the following day; 25 (4) on December 14,
appellant invokes People v. 1989, T/Sgt. Belarga led a team of NARCOM agents who went to Suterville, Zamboanga
Ale 20 where the Court observed that from a distance of 10-15 meters, a policeman cannot City; 26 (5) T/Sgt. Belarga gave a P20.00 marked bill to Sgt. Ani which was to be used in the buy-
distinguish between marijuana cigarette from ordinary ones by the type of rolling done on the bust operation; 27 (6) upon the arrival of the NARCOM agents in Suterville, Zamboanga City, Sgt.
cigarette sticks. And since T/Sgt. Belarga allegedly did not see the sale, the appellant contends Ani proceeded to the house of the appellant while some agents stayed in the vehicles and others
that the uncorroborated testimony of Sgt. Ani can not stand as basis for his conviction. positioned themselves in strategic places; 28 the appellant met Sgt. Ani and an exchange of
People v. Ale does not apply here because the policeman in that case testified that he and his articles took place. 29
companion were certain that the appellant therein handed marijuana cigarettes to the poseur- The corroborative testimony of T/Sgt. Belarga strengthens the direct evidence given by Sgt. Ani.
buyer based on the appearance of the cigarette sticks. The Court rejected this claim, stating that: Additionally, the Court has ruled that the fact that the police officers who accompanied the
This Court cannot give full credit to the testimonies of the prosecution poseur-buyer were unable to see exactly what the appellant gave the poseur-buyer because of
witnesses marked as they are with contradictions and tainted with their distance or position will not be fatal to the prosecution's case 30 provided there exists other
inaccuracies. evidence, direct or circumstantial, e.g., the testimony of the poseur-buyer, which is sufficient to
Biñan testified that they were able to tell that the four cigarettes were prove the consummation of the sale of the prohibited drug
marijuana cigarettes because according to him, the rolling of ordinary The appellant next assails the seizure and admission as evidence of a plastic bag containing
cigarettes are different from those of marijuana cigarettes. (tsn, November marijuana which the NARCOM agents found in the appellant's kitchen. It appears that after Sgt.
13, 1984, p. 10). Ani gave the pre-arranged signal to the other NARCOM agents, the latter moved in and arrested
It is however, incredible to believe that they could discern the type of rolling the appellant inside the house. They searched him to retrieve the marked money but didn't find
done on those cigarettes from the distance where they were observing the it. Upon being questioned, the appellant said that he gave the marked money to his
alleged sale of more or less 10 to 15 meters. 21 wife. 31 Thereafter, T/Sgt. Belarga and Sgt. Lego went to the kitchen and noticed what T/Sgt.
In the case at bar, however, T/Sgt. Belarga did not positively claim that he saw the appellant Belarga described as a "cellophane colored white and stripe hanging at the corner of the
hand over marijuana to Sgt. Ani. What he said was that there was an exchange of certain articles kitchen." 32 They asked the appellant about its contents but failing to get a response, they
between the two. The relevant portion of T/Sgt. Belarga's testimony reads: 22 opened it and found dried marijuana leaves. At the trial, the appellant questioned the
Q Now, do you remember whether Sgt. Ani was able to admissibility of the plastic bag and the marijuana it contains but the trial court issued an Order
reach the house of Mari Musa? ruling that these are admissible in evidence. 33
A Yes, ma'am. Built into the Constitution are guarantees on the freedom of every individual against
Q After reaching Mari Musa, did you see what happened unreasonable searches and seizures by providing in Article III, Section 2, the following:
(sic)? The right of the people to be secure in their persons, houses, papers, and
A Yes, ma'am. effects against unreasonable searches and seizures of whatever nature and
Q Could you please tell us? for any purpose shall be inviolable, and no search warrant or warrant of
A From our vehicle the stainless owner type jeep where arrest shall issue except upon probable cause to be determined personally by
Sgt. Lego, Sgt. Biong were boarded, I saw that Sgt. Ani the judge after examination under oath or affirmation of the complainant
proceeded to the house near the road and he was met by and the witness he may produce, and particularly describing the place to be
one person and later known as Mari Musa who was at searched and the persons or things to be seized.
the time wearing short pants and later on I saw that Sgt. Furthermore, the Constitution, in conformity with the doctrine laid down in Stonehill v.
Ani handed something to him, thereafter received by Diokno, 34 declares inadmissible, any evidence obtained in violation of the freedom from
Mari Musa and went inside the house and came back unreasonable searches and seizures. 35
later and handed something to Sgt. Ani. While a valid search warrant is generally necessary before a search and seizure may be effected,
exceptions to this rule are recognized. Thus, in Alvero v. Dizon, 36 the Court stated that. "[t]he
91

most important exception to the necessity for a search warrant is the right of search and seizure other legitimate reason for being present unconnected with a search directed against the
as an incident to a lawful arrest." 37 accused — and permits the warrantless seizure. Of course, the extension of the original
Rule 126, Section 12 of the Rules of Court expressly authorizes a warrantless search and seizure justification is legitimate only where it is immediately apparent to the police that they have
incident to a lawful arrest, thus: evidence before them; the "plain view" doctrine may not be used to extend a general
Sec. 12. Search incident to lawful arrest. — A person lawfully arrested may be exploratory search from one object to another until something incriminating at last emerges. 46
searched for dangerous weapons or anything which may be used as proof of It has also been suggested that even if an object is observed in "plain view," the "plain view"
the commission of an offense, without a search warrant. doctrine will not justify the seizure of the object where the incriminating nature of the object is
There is no doubt that the warrantless search incidental to a lawful arrest authorizes the not apparent from the "plain view" of the object. 47 Stated differently, it must be immediately
arresting officer to make a search upon the person of the person arrested. As early as 1909, the apparent to the police that the items that they observe may be evidence of a crime, contraband,
Court has ruled that "[a]n officer making an arrest may take from the person arrested any money or otherwise subject to seizure.
or property found upon his person which was used in the commission of the crime or was the In the instant case, the appellant was arrested and his person searched in the living room. Failing
fruit of the crime or which might furnish the prisoner with the means of committing to retrieve the marked money which they hoped to find, the NARCOM agents searched the
violence or of escaping, or which may be used as evidence in the trial of the cause . . . " 38 Hence, whole house and found the plastic bag in the kitchen. The plastic bag was, therefore, not within
in a buy-bust operation conducted to entrap a drug-pusher, the law enforcement agents may their "plain view" when they arrested the appellant as to justify its seizure. The NARCOM agents
seize the marked money found on the person had to move from one portion of the house to another before they sighted the plastic bag.
of the pusher immediately after the arrest even without arrest and search warrants. 39 Unlike Ker vs. California, where the police officer had reason to walk to the doorway of the
In the case at bar, the NARCOM agents searched the person of the appellant after arresting him adjacent kitchen and from which position he saw the marijuana, the NARCOM agents in this case
in his house but found nothing. They then searched the entire house and, in the kitchen, found went from room to room with the obvious intention of fishing for more evidence.
and seized a plastic bag hanging in a corner. Moreover, when the NARCOM agents saw the plastic bag hanging in one corner of the kitchen,
The warrantless search and seizure, as an incident to a suspect's lawful arrest, may extend they had no clue as to its contents. They had to ask the appellant what the bag contained. When
beyond the person of the one arrested to include the premises or surroundings under his the appellant refused to respond, they opened it and found the marijuana. Unlike Ker v.
immediate control. 40 Objects in the "plain view" of an officer who has the right to be in the California, where the marijuana was visible to the police officer's eyes, the NARCOM agents in
position to have that view are subject to seizure and may be presented as evidence. 41 this case could not have discovered the inculpatory nature of the contents of the bag had they
In Ker v. California 42 police officers, without securing a search warrant but having information not forcibly opened it. Even assuming then, that the NARCOM agents inadvertently came across
that the defendant husband was selling marijuana from his apartment, obtained from the the plastic bag because it was within their "plain view," what may be said to be the object in
building manager a passkey to defendants' apartment, and entered it. There they found the their "plain view" was just the plastic bag and not the marijuana. The incriminating nature of the
defendant husband in the living room. The defendant wife emerged from the kitchen, and one of contents of the plastic bag was not immediately apparent from the "plain view" of said object. It
the officers, after identifying himself, observed through the open doorway of the kitchen, a small cannot be claimed that the plastic bag clearly betrayed its contents, whether by its distinctive
scale atop the kitchen sink, upon which lay a brick-shaped package containing green leafy configuration, its transprarency, or otherwise, that its contents are obvious to an observer. 48
substance which he recognized as marijuana. The package of marijuana was used as evidence in We, therefore, hold that under the circumstances of the case, the "plain view" doctrine does not
prosecuting defendants for violation of the Narcotic Law. The admissibility of the package was apply and the marijuana contained in the plastic bag was seized illegally and cannot be
challenged before the U.S. Supreme Court, which held, after observing that it was not presented in evidence pursuant to Article III, Section 3(2) of the Constitution.
unreasonable for the officer to walk to the doorway of the adjacent kitchen on seeing the The exclusion of this particular evidence does not, however, diminish, in any way, the damaging
defendant wife emerge therefrom, that "the discovery of the brick of marijuana did not effect of the other pieces of evidence presented by the prosecution to prove that the appellant
constitute a search, since the officer merely saw what was placed before him in full view. 43 The sold marijuana, in violation of Article II, Section 4 of the Dangerous Drugs Act of 1972. We hold
U.S. Supreme Court ruled that the warrantless seizure of the marijuana was legal on the basis of that by virtue of the testimonies of Sgt. Ani and T/Sgt. Belarga and the two wrappings of
the "plain view" doctrine and upheld the admissibility of the seized drugs as part of the marijuana sold by the appellant to Sgt. Ani, among other pieces of evidence, the guilt of the
prosecution's evidence. 44 appellant of the crime charged has been proved beyond reasonable doubt.
The "plain view" doctrine may not, however, be used to launch unbridled searches and WHEREFORE, the appeal is DISMISSED and the judgment of the Regional Trial Court AFFIRMED.
indiscriminate seizures nor to extend a general exploratory search made solely to find evidence SO ORDERED.
of defendant's guilt. The "plain view" doctrine is usually applied where a police officer is not
searching for evidence against the accused, but nonetheless inadvertently comes across an
incriminating object. 45 Furthermore, the U.S. Supreme Court stated the following limitations on
the application of the doctrine:
What the "plain view" cases have in common is that the police officer in each of them had a prior
justification for an intrusion in the course of which he came inadvertently across a piece of
evidence incriminating the accused. The doctrine serves to supplement the prior justification —
whether it be a warrant for another object, hot pursuit, search incident to lawful arrest, or some
92

PEOPLE VS SALANGUIT PO3 Duazo requested a laboratory examination of the confiscated evidence.[13] The white
This is an appeal from the decision,[1] dated January 27, 1998, of the Regional Trial Court, crystalline substance with a total weight of 2.77 grams and those contained in a small box with a
Branch 96, Quezon City, finding accused-appellant Roberto Salanguit y Ko guilty of violation of 16 total weight of 8.37 grams were found to be positive for methamphetamine hydrochloride. On
of Republic Act No. 6425, as amended, and sentencing him accordingly to suffer imprisonment the other hand, the two bricks of dried leaves, one weighing 425 grams and the other 850 grams,
ranging from six (6) months of arresto mayor, as minimum, to four (4) years and two (2) months were found to be marijuana.[14]
of prision correccional, as maximum, and of 8 of the same law and sentencing him for such For the defense, accused-appellant testified in his own behalf. His testimony was
violation to suffer the penalty of reclusion perpetua and to pay a fine of P700,000.00. corroborated by his mother-in-law, Soledad Arcano.
Charges against accused-appellant for violations of R.A. No. 6425 were filed on December Accused-appellant testified that on the night of December 26, 1995, as they were about to
28, 1995. In Criminal Case No. Q-95-64357, the information alleged: leave their house, they heard a commotion at the gate and on the roof of their house. Suddenly,
That on or about the 26th day of December 1995, in Quezon City, Philippines, the said accused, about 20 men in civilian attire, brandishing long firearms, climbed over the gate and descended
did then and there willfully, unlawfully and knowingly possess and/or use 11.14 grams of through an opening in the roof.[15]
Methamphetamine Hydrochloride (Shabu) a regulated drug, without the necessary license When accused-appellant demanded to be shown a search warrant, a piece of paper inside
and/or prescription therefor, in violation of said law. a folder was waved in front of him. As accused-appellant fumbled for his glasses, however, the
CONTRARY TO LAW.[2] paper was withdrawn and he had no chance to read it.[16]
In Criminal Case No. Q-95-64358, the information charged: Accused-appellant claimed that he was ordered to stay in one place of the house while the
That on or about the 26th day of December 1995, in Quezon City, Philippines, the said accused policemen conducted a search, forcibly opening cabinets and taking his bag containing money, a
not being authorized by law to possess or use any prohibited drug, did, then and there willfully, licensed .45 caliber firearm, jewelry, and canned goods.[17]
unlawfully and knowingly have in his possession and under his custody and control 1,254 grams The policemen left at around 12:30 a.m. of December 27, 1995, and, after putting
of Marijuana, a prohibited drug. handcuffs on accused-appellant, took him with them to the NARCOM on EDSA, Quezon City,
CONTRARY TO LAW.[3] where accused-appellant was detained.[18]
When arraigned on May 21, 1996, accused-appellant pleaded not guilty,[4] whereupon he Accused-appellants mother-in law, Soledad Arcano, corroborated his testimony. Arcano
was tried. testified that the policemen ransacked their house, ate their food, and took away canned goods
Three witnesses were presented by the prosecution: P/Insp. Sonia S. Ludovico, forensic and other valuables.[19]
chemist and chief of the Physical Science Branch of the Philippine National Police Crime After hearing, the trial court rendered its decision, the dispositive portion of which reads:
Laboratory, Senior Inspector Rodolfo Aguilar of the Narcotics Command, Camp Crame, Quezon WHEREFORE, judgment is hereby rendered:
City, and PO3 Rolando Duazo of Station 10, Kamuning, Quezon City, a field operative. The 1. In Criminal Case No. Q-95-64357, for violation of Sec. 16, Republic Act No. 6425, as amended,
prosecution evidence established the following: finding the accused ROBERTO SALANGUIT y KO guilty beyond reasonable doubt of the crime
On December 26, 1995, Sr. Insp. Aguilar applied for a warrant[5] in the Regional Trial Court, charged and he is hereby accordingly sentenced to suffer an indeterminate sentence with a
Branch 90, Dasmarias, Cavite, to search the residence of accused-appellant Robert Salanguit y Ko minimum of six (6) months of arresto mayor and a maximum of four (4) years and two (2)
on Binhagan St., Novaliches, Quezon City. He presented as his witness SPO1 Edmund Badua, who months of prision correccional; and,
testified that as a poseur-buyer, he was able to purchase 2.12 grams of shabu from accused- 2. In Criminal Case No. Q-95-64358, for violation of Sec. 8, Republic Act No. 6425, as amended,
appellant. The sale took place in accused-appellants room, and Badua saw that the shabu was finding the accused ROBERTO SALANGUIT y KO guilty beyond reasonable doubt of the crime
taken by accused-appellant from a cabinet inside his room. The application was granted, and a charged and he is hereby accordingly sentenced to suffer reclusion perpetua and to pay a fine of
search warrant was later issued by Presiding Judge Dolores L. Espaol. P700,000.00.
At about 10:30 p.m. of December 26, 1995, a group of about 10 policemen, along with one The accused shall further pay the costs of suit.
civilian informer, went to the residence of accused-appellant to serve the warrant.[6] The 11.14 grams of methamphetamine hydrochloride and the 1,254 grams of marijuana bricks
The police operatives knocked on accused-appellants door, but nobody opened it. They are hereby confiscated and condemned for disposition according to law. The evidence custodian
heard people inside the house, apparently panicking. The police operatives then forced the door of this Court is hereby directed to turn such substances over to the National Bureau of
open and entered the house.[7] Investigation pursuant to law.
After showing the search warrant to the occupants of the house, Lt. Cortes and his group SO ORDERED.[20]
started searching the house.[8] They found 12 small heat-sealed transparent plastic bags Hence this appeal. Accused-appellant contends that -
containing a white crystalline substance, a paper clip box also containing a white crystalline THE COURT A QUO GRAVELY ERRED IN DECLARING THE SEARCH WARRANT VALID
substance, and two bricks of dried leaves which appeared to be marijuana wrapped in THE COURT A QUO ERRED IN CONVICTING ACCUSED-APPELLANT FOR ILLEGAL POSSESSION
newsprint[9] having a total weight of approximately 1,255 grams.[10] A receipt of the items seized OF METHAMPHETAMINE HYDRO-CHLORIDE (SHABU)
was prepared, but the accused-appellant refused to sign it.[11] THE COURT A QUO GRAVELY ERRED IN CONVICTING ACCUSED-APPELLANT FOR VIOLATION
After the search, the police operatives took accused-appellant with them to Station 10, 8, R.A. NO. 6425
EDSA, Kamuning, Quezon City, along with the items they had seized.[12] THE COURT A QUO ERRED IN ADMITTING IN EVIDENCE THE TWO (2) BRICKS OF
MARIJUANA
93

THE COURT A QUO ERRED IN NOT FINDING THAT THE POLICEMEN USED EXCESSIVE FORCE Q - Being a member of the Intelligence and Operation Section, NMDU, NARCOM, do you
IN ENFORCING THE SEARCH WARRANT. remember if you were assigned into a monitoring or surveillance work?
Accused-appellant is contesting his conviction on three grounds. First, the admissibility of A - Yes, sir.
the shabu allegedly recovered from his residence as evidence against him on the ground that the Q - Of what particular assignment or area were you assigned for monitoring or surveillance?
warrant used in obtaining it was invalid. Second, the admissibility in evidence of the marijuana A - Its within the Quezon City area particularly a house without a number located at
allegedly seized from accused-appellant pursuant to the plain view doctrine. Third, the Binhagan St., San Jose, Quezon City, sir.
employment of unnecessary force by the police in the execution of the warrant. Q - Do you know the person who occupies the specific place?
First. Rule 126, 4 of the Revised Rules on Criminal Procedure [21] provides that a search A - Yes, sir, he is ROBERT SALANGUIT @ Robert.
warrant shall not issue except upon probable cause in connection with one specific offense to be Q - Are you familiar with that place?
determined personally by the judge after examination under oath or affirmation of the A - Yes, sir, as part of my surveillance, I was able to penetrate inside the area and established
complainant and the witnesses he may produce, and particularly describing the place to be contract with ROBERT SALANGUIT alias Robert through my friend who introduced me
searched and the things to be seized which may be anywhere in the Philippines. to the former.
In issuing a search warrant, judges must comply strictly with the requirements of the Q - In what particular occasion did you meet ROBERT SALANGUIT alias Robert?
Constitution and the Rules of Criminal Procedure. No presumption of regularity can be invoked in A - When I was introduced by my friend as a good buyer and drug pusher of shabu, sir.
aid of the process when an officer undertakes to justify its issuance. [22] Nothing can justify the Q - Were you able to buy at that time?
issuance of the search warrant unless all the legal requisites are fulfilled. A - Yes, sir.
In this case, the search warrant issued against accused-appellant reads: Q - How much if you can still remember the amount involved?
SEARCH WARRANT NO. 160 A - I was able to buy two point twelve (2.12) grams of shabu in the amount of Two Thousand
For: Violation of RA 6425 Seven Hundred Fifty (P2,750.00) pesos, sir.
SEARCH WARRANT Q - Having established contact with ROBERT SALANGUIT @ Robert, do you know where the
TO ANY PEACE OFFICER: stuff (shabu) were being kept?
GREETINGS: A - Yes, sir, inside a cabinet inside his room.
It appearing to the satisfaction of the undersigned after examining under oath SR. INSP. Q - How were you able to know the place where he kept the stuff?
RODOLFO V. AGUILAR, PNP and his witness SPO1 EDMUND M. BADUA, PNP that there is A - When I first bought the 2.12 grams of shabu from him, it was done inside his room and I
probable cause to believe that ROBERT SALANGUIT has in his possession and control in his saw that the shabu was taken by him inside his cabinet.
premises Binhagan St., San Jose, Quezon City as shown in Annex A, the properties to wit: Q - Do you know who is in control of the premises?
UNDETERMINED QUANTITY OF SHABU AND DRUG PARAPHERNALIA A - Yes, sir, it was ROBERT SALANGUIT @ Robert.
which should be seized and brought to the undersigned. Q - How sure are you, that the shabu that you bought from ROBERT SALANGUIT @ Robert is
You are hereby commanded to make an immediate search anytime of the day/night of the genuine shabu?
premises above-described and forthwith seize and take possession of the above-stated A - After I left the house of ROBERT SALANGUIT @ Robert, I proceeded back to our office and
properties and bring said properties to the undersigned to be dealt with as the law directs. reported the progress of my mission to our Chief and presented to him the 2.12 grams
GIVEN UNDER MY HAND this 26th day of December 1995 at Imus, Cavite, Philippines. of shabu I bought from the subject.Then afterwards, our Chief formally requested the
(SGD.) DOLORES L. ESPAOL Chief PNP Central Crime Laboratory Services, NPDC, for Technical Analysis which
Judge yielded positive result for shabu, a regulated drug as shown in the attached
Accused-appellant assails the validity of the warrant on three grounds: (1) that there was certification of PNP CLS result No. D-414-95 dated 19 Dec. 95.
no probable cause to search for drug paraphernalia; (2) that the search warrant was issued for Q - Do you have anything more to add or retract from your statement?
more than one specific offense; and (3) that the place to be searched was not described with A - Yes, sir, I was offered by him (ROBERT SALANGUIT @ Robert) that anything I wish to buy
sufficient particularity. bigger quantity of shabu, he is willing to transact to me on cash basis at his price of
Existence of Probable Cause One Thousand Seven Hundred Fifty (P1,750.00) pesos per gram.
The warrant authorized the seizure of undetermined quantity of shabu and drug Q - Are you willing to sign your statement freely and voluntarily?
paraphernalia. Evidence was presented showing probable cause of the existence of A - Yes, sir.[24]
methamphetamine hydrochloride orshabu. Accused-appellant contends, however, that the However, the fact that there was no probable cause to support the application for the
search warrant issued is void because no evidence was presented showing the existence of drug seizure of drug paraphernalia does not warrant the conclusion that the search warrant is
paraphernalia and the same should not have been ordered to be seized by the trial court.[23] void. This fact would be material only if drug paraphernalia was in fact seized by the police. The
The contention has no merit. To be sure, SPO1 Edmund Badua, the intelligence officer who fact is that none was taken by virtue of the search warrant issued. If at all, therefore, the search
acted as a poseur-buyer, did not testify in the proceedings for the issuance of a search warrant warrant is void only insofar as it authorized the seizure of drug paraphernalia, but it is valid as to
on anything about drug paraphernalia. He stated: the seizure of methamphetamine hydrochloride as to which evidence was presented showing
94

probable cause as to its existence. Thus, in Aday v. Superior Court,[25] the warrant properly Act of 1972 is a special law that deals specifically with dangerous drugs which are subsumed into
described two obscene books but improperly described other articles. It was held: prohibited and regulated drugs and defines and penalizes categories of offenses which are
Although the warrant was defective in the respects noted, it does not follow that it was closely related or which belong to the same class or species. Accordingly, one (1) search warrant
invalid as a whole. Such a conclusion would mean that the seizure of certain articles, even may thus be validly issued for the said violations of the Dangerous Drugs Act.[30]
though proper if viewed separately, must be condemned merely because the warrant was Similarly, in another case,[31] the search warrant was captioned: For Violation of P.D. No.
defective with respect to other articles. The invalid portions of the warrant are severable from 1866 (Illegal Possession of Firearms, etc.). The validity of the warrant was questioned on the
the authorization relating to the named books, which formed the principal basis of the charge of ground that it was issued without reference to any particular provision in P.D. No. 1866, which
obscenity. The search for and seizure of these books, if otherwise valid, were not rendered illegal punished several offenses. We held, however, that while illegal possession of firearms is
by the defects concerning other articles. . . . In so holding we do not mean to suggest that invalid penalized under 1 of P.D. No. 1866 and illegal possession of explosives is penalized under 3
portions of a warrant will be treated as severable under all circumstances. We recognize the thereof, the decree is a codification of the various laws on illegal possession of firearms,
danger that warrants might be obtained which are essentially general in character but as to ammunitions, and explosives which offenses are so related as to be subsumed within the
minor items meet the requirement of particularity, and that wholesale seizures might be made category of illegal possession of firearms, etc. under P.D. No. 1866. Thus, only one warrant was
under them, in the expectation that the seizure would in any event be upheld as to the property necessary to cover the violations under the various provisions of the said law.
specified. Such an abuse of the warrant procedure, of course, could not be tolerated. Particularity of the Place

It would be a drastic remedy indeed if a warrant, which was issued on probable cause and Accused-appellant contends that the search warrant failed to indicate the place to be
particularly describing the items to be seized on the basis thereof, is to be invalidated in searched with sufficient particularity.
toto because the judge erred in authorizing a search for other items not supported by the This contention is without merit. As the Solicitor General states:
evidence.[26] Accordingly, we hold that the first part of the search warrant, authorizing the search . . . While the address stated in the warrant is merely Binhagan St., San Jose, Quezon City, the
of accused-appellants house for an undetermined quantity of shabu, is valid, even though the trial court took note of the fact that the records of Search Warrant Case No. 160 contained
second part, with respect to the search for drug paraphernalia, is not. several documents which identified the premises to be searched, to wit: 1) the application for
Specificity of the Offense Charged search warrant which stated that the premises to be searched was located in between No. 7 and
Accused-appellant contends that the warrant was issued for more than one specific 11 at Binhagan Street, San Jose, Quezon City; 2) the deposition of witness which described the
offense because possession or use of methamphetamine hydrochloride and possession of drug premises as a house without a number located at Binhagan St., San Jose, Quezon City; and 3) the
paraphernalia are punished under two different provisions of R.A. No. 6425.[27] It will suffice to pencil sketch of the location of the premises to be searched. In fact, the police officers who
quote what this Court said in a similar case to dispose of this contention: raided appellants house under the leadership of Police Senior Inspector Rodolfo Aguilar could
While it is true that the caption of the search warrant states that it is in connection with not have been mistaken as Inspector Aguilar resides in the same neighborhood in Binhagan
Violation of R.A. 6425, otherwise known as the Dangerous Drugs Act of 1972, it is clearly recited where appellant lives and in fact Aguilars place is at the end of appellants place in Binhagan.
in the text thereof that There is probable cause to believe that Adolfo Olaes alias Debie and alias Moreover, the house raided by Aguilars team is undeniably appellants house and it was really
Baby of No. 628 Comia St., Filtration, Sta. Rita, Olongapo City, has in their session and control appellant who was the target. The raiding team even first ascertained through their informant
and custody of marijuana dried stalks/leaves/seeds/cigarettes and other regulated/prohibited that appellant was inside his residence before they actually started their operation.[32]
and exempt narcotics preparations which is the subject of the offense stated above. Although The rule is that a description of the place to be searched is sufficient if the officer with the
the specific section of the Dangerous Drugs Act is not pinpointed, there is no question at all of warrant can, with reasonable effort, ascertain and identify the place intended to be
the specific offense alleged to have been committed as a basis for the finding of probable searched.[33] For example, a search warrant authorized a search of Apartment Number 3 of a
cause. The search warrant also satisfies the requirement in the Bill of Rights of the particularity building at 83 Pleasant Street, Malborough, Massachusetts. As it turned out, there were five
of the description to be made of the place to be searched and the persons or things to be apartments in the basement and six apartments on both the ground and top floors and that
seized. [28] there was an Apartment Number 3 on each floor. However, the description was made
Indeed, in People v. Dichoso[29] the search warrant was also for Violation of R.A. 6425, determinate by a reference to the affidavit supporting the warrant that the apartment was
without specifying what provisions of the law were violated, and it authorized the search and occupied by the accused Morris Ferrante of 83 Pleasant Street, Malboro Mass.[34] In this case, the
seizure of dried marijuana leaves and methamphetamine hydrochloride (shabu) and sets of location of accused-appellants house being indicated by the evidence on record, there can be no
paraphernalias (sic). This Court, however, upheld the validity of the warrant: doubt that the warrant described the place to be searched with sufficient particularity.
Appellants contention that the search warrant in question was issued for more than (1) offense, In sum, we hold that with respect to the seizure of shabu from accused-appellants
hence, in violation of Section 3, Rule 126 of the Rules of Court, is unpersuasive. He engages in residence, Search Warrant No. 160 was properly issued, such warrant being founded on
semantic juggling by suggesting that since illegal possession of shabu, illegal possession of probable cause personally determined by the judge under oath or affirmation of the deposing
marijuana and illegal possession of paraphernalia are covered by different articles and sections witness and particularly describing the place to be searched and the things to be seized.
of the Dangerous Drugs Act of 1972, the search warrant is clearly for more than one (1) specific Second. The search warrant authorized the seizure of methamphetamine hydrochloride
offense. In short, following this theory, there should have been three (3) separate search or shabu but not marijuana. However, seizure of the latter drug is being justified on the ground
warrants, one for illegal possession of shabu, the second for illegal possession of marijuana and that the drug was seized within the plain view of the searching party. This is contested by
the third for illegal possession of paraphernalia. This argument is pedantic. The Dangerous Drugs accused-appellant.
95

Under the plain view doctrine, unlawful objects within the plain view of an officer who has No presumption of regularity may be invoked by an officer in aid of the process when he
the right to be in the position to have that view are subject to seizure and may be presented in undertakes to justify an encroachment of rights secured by the Constitution.[41] In this case, the
evidence.[35] For this doctrine to apply, there must be: (a) prior justification; (b) inadvertent marijuana allegedly found in the possession of accused-appellant was in the form of two bricks
discovery of the evidence; and (c) immediate apparent illegality of the evidence before the wrapped in newsprint. Not being in a transparent container, the contents wrapped in newsprint
police.[36] The question is whether these requisites were complied with by the authorities in could not have been readily discernible as marijuana. Nor was there mention of the time or
seizing the marijuana in this case. manner these items were discovered. Accordingly, for failure of the prosecution to prove that
Prior Justification and Discovery by Inadvertence the seizure of the marijuana without a warrant was conducted in accordance with the plain view
Because the location of the shabu was indicated in the warrant and thus known to the doctrine, we hold that the marijuana is inadmissible in evidence against accused-
police operatives, it is reasonable to assume that the police found the packets of appellant. However, the confiscation of the drug must be upheld.
the shabu first. Once the valid portion of the search warrant has been executed, the plain view Third. Accused-appellant claims that undue and unnecessary force was employed by the
doctrine can no longer provide any basis for admitting the other items subsequently found. As searching party in effecting the raid.
has been explained: Rule 126, 7 of the Revised Rules on Criminal Procedure[42] provides:
What the plain view cases have in common is that the police officer in each of them had a prior Right to break door or window to effect search. The officer, if refused admittance to the place of
justification for an intrusion in the course of which he came inadvertently across a piece of directed search after giving notice of his purpose and authority, may break open any outer or
evidence incriminating the accused. The doctrine serves to supplement the prior inner door or window of a house or any part of a house or anything therein to execute the
justification whether it be a warrant for another object, hot pursuit, search incident to lawful warrant or liberate himself or any person lawfully aiding him when unlawfully detained therein.
arrest, or some other legitimate reason for being present unconnected with a search directed Accused-appellants claim that the policemen had clambered up the roof of his house to
against the accused and permits the warrantless seizure. Of course, the extension of the original gain entry and had broken doors and windows in the process is unsupported by reliable and
justification is legitimate only where it is immediately apparent to the police that they have competent proof. No affidavit or sworn statement of disinterested persons, like the barangay
evidence before them; the plain view doctrine may not be used to extend a general exploratory officials or neighbors, has been presented by accused-appellant to attest to the truth of his
search from one object to another until something incriminating at last emerges.[37] claim.
The only other possible justification for an intrusion by the police is the conduct of a search In contrast, Aguilar and Duanos claim that they had to use some force in order to gain
pursuant to accused-appellants lawful arrest for possession of shabu. However, a search incident entry cannot be doubted. The occupants of the house, especially accused-appellant, refused to
to a lawful arrest is limited to the person of the one arrested and the premises within his open the door despite the fact that the searching party knocked on the door several
immediate control.[38] The rationale for permitting such a search is to prevent the person times. Furthermore, the agents saw the suspicious movements of the people inside the
arrested from obtaining a weapon to commit violence, or to reach for incriminatory evidence house. These circumstances justified the searching partys forcible entry into the house, founded
and destroy it. as it is on the apprehension that the execution of their mission would be frustrated unless they
The police failed to allege in this case the time when the marijuana was do so.
found, i.e., whether prior to, or contemporaneous with, the shabu subject of the warrant, or WHEREFORE, in Criminal Case No. Q-95-64357, the decision of the Regional Trial Court,
whether it was recovered on accused-appellants person or in an area within his immediate Branch 96, Quezon City, finding accused-appellant Roberto Salanguit y Ko guilty of possession of
control. Its recovery, therefore, presumably during the search conducted after the shabu had illegal drugs under 16 of R.A. No. 6425, otherwise known as the Dangerous Drugs Act, as
been recovered from the cabinet, as attested to by SPO1 Badua in his depostion, was invalid. amended, and sentencing him to suffer a prison term ranging from six (6) months of arresto
Apparent Illegality of the Evidence mayor, as minimum, and four (4) years and two (2) months of prision correccional, as maximum,
The marijuana bricks were wrapped in newsprint. There was no apparent illegality to and ordering the confiscation of 11.14 grams of methamphetamine hydrochloride is AFFIRMED.
justify their seizure. This case is similar to People. v. Musa[39] in which we declared inadmissible In Criminal Case No. Q-95-64358, the decision of the same court finding accused-appellant
the marijuana recovered by NARCOM agents because the said drugs were contained in a plastic Roberto Salanguit y Ko guilty of possession of prohibited drugs under 8 of R.A. No. 6425, as
bag which gave no indication of its contents. We explained: amended, and sentencing him to suffer the penalty of reclusion perpetua and to pay a fine
Moreover, when the NARCOM agents saw the plastic bag hanging in one corner of the kitchen, of P700,000.00 is hereby REVERSED and SET ASIDE and accused-appellant is ACQUITTED of the
they had no clue as to its contents. They had to ask the appellant what the bag contained. When crime charged. However, the confiscation of the 1,254 grams of marijuana, as well as the 11.14
the appellant refused to respond, they opened it and found the marijuana. Unlike Ker v. grams of methamphetamine hydrochloride, and its disposition as ordered by the trial court is
California, where the marijuana was visible to the police officers eyes, the NARCOM agents in AFFIRMED.
this case could not have discovered the inculpatory nature of the contents of the bag had they SO ORDERED.
not forcibly opened it. Even assuming then, that the NARCOM agents inadvertently came across
the plastic bag because it was within their plain view, what may be said to be the object in their
plain view was just the plastic bag and not the marijuana. The incriminating nature of the
contents of the plastic bag was not immediately apparent from the plain view of said object. It
cannot be claimed that the plastic bag clearly betrayed its contents, whether by its distinctive
configuration, is transparency, or otherwise, that its contents are obvious to an observer. [40]
96

PEOPLE VS AMINNUDIN impersonal record. But the trial judge sees all of this, discovering for himself the truant fact
The accused-appellant claimed his business was selling watches but he was nonetheless arrested, tried and amidst the falsities.
found guilty of illegally transporting marijuana. The trial court, disbelieving him, held it was high time to put The only exception we may make in this case is the trial court's conclusion that the accused-
him away and sentenced him to life imprisonment plus a fine of P20,000.00. 1 appellant was not really beaten up because he did not complain about it later nor did he submit
Idel Aminnudin was arrested on June 25, 1984, shortly after disembarking from the M/V Wilcon to a medical examination. That is hardly fair or realistic. It is possible Aminnudin never had that
9 at about 8:30 in the evening, in Iloilo City. The PC officers who were in fact waiting for him opportunity as he was at that time under detention by the PC authorities and in fact has never
simply accosted him, inspected his bag and finding what looked liked marijuana leaves took him been set free since he was arrested in 1984 and up to the present. No bail has been allowed for
to their headquarters for investigation. The two bundles of suspect articles were confiscated his release.
from him and later taken to the NBI laboratory for examination. When they were verified as There is one point that deserves closer examination, however, and it is Aminnudin's claim that he was
marijuana leaves, an information for violation of the Dangerous Drugs Act was filed against arrested and searched without warrant, making the marijuana allegedly found in his possession inadmissible
him. 2Later, the information was amended to include Farida Ali y Hassen, who had also been in evidence against him under the Bill of Rights. The decision did not even discuss this point. For his part, the
arrested with him that same evening and likewise investigated. 3 Both were arraigned and Solicitor General dismissed this after an all-too-short argument that the arrest of Aminnudin was valid
pleaded not guilty. 4 Subsequently, the fiscal filed a motion to dismiss the charge against Ali on because it came under Rule 113, Section 6(b) of the Rules of Court on warrantless arrests. This made the
the basis of a sworn statement of the arresting officers absolving her after a 'thorough search also valid as incidental to a lawful arrest.
It is not disputed, and in fact it is admitted by the PC officers who testified for the prosecution, that they had
investigation." 5 The motion was granted, and trial proceeded only against the accused-
no warrant when they arrested Aminnudin and seized the bag he was carrying. Their only justification was
appellant, who was eventually convicted . 6 the tip they had earlier received from a reliable and regular informer who reported to them that Aminnudin
According to the prosecution, the PC officers had earlier received a tip from one of their was arriving in Iloilo by boat with marijuana. Their testimony varies as to the time they received the tip, one
informers that the accused-appellant was on board a vessel bound for Iloilo City and was saying it was two days before the arrest, 20 another two weeks 21 and a third "weeks before June 25." 22 On
carrying marijuana. 7 He was Identified by name. 8 Acting on this tip, they waited for him in the this matter, we may prefer the declaration of the chief of the arresting team, Lt. Cipriano Querol, Jr., who
evening of June 25, 1984, and approached him as he descended from the gangplank after the testified as follows:
informer had pointed to him. 9 They detained him and inspected the bag he was carrying. It was Q You mentioned an intelligence report, you mean with respect to the coming of Idel Aminnudin on June 25,
found to contain three kilos of what were later analyzed as marijuana leaves by an NBI forensic 1984?
A Yes, sir.
examiner, 10 who testified that she conducted microscopic, chemical and chromatographic tests
Q When did you receive this intelligence report?
on them. On the basis of this finding, the corresponding charge was then filed against A Two days before June 25, 1984 and it was supported by reliable sources.
Aminnudin. Q Were you informed of the coming of the Wilcon 9 and the possible trafficking of marijuana leaves on that
In his defense, Aminnudin disclaimed the marijuana, averring that all he had in his bag was his date?
clothing consisting of a jacket, two shirts and two pairs of pants. 11 He alleged that he was A Yes, sir, two days before June 25, 1984 when we received this information from that particular informer,
arbitrarily arrested and immediately handcuffed. His bag was confiscated without a search prior to June 25, 1984 we have already reports of the particular operation which was being participated by
warrant. At the PC headquarters, he was manhandled to force him to admit he was carrying the Idel Aminnudin.
marijuana, the investigator hitting him with a piece of wood in the chest and arms even as he Q You said you received an intelligence report two days before June 25, 1984 with respect to the coming of
Wilcon 9?
parried the blows while he was still handcuffed. 12 He insisted he did not even know what
A Yes, sir.
marijuana looked like and that his business was selling watches and sometimes cigarettes. 13 He Q Did you receive any other report aside from this intelligence report?
also argued that the marijuana he was alleged to have been carrying was not properly Identified A Well, I have received also other reports but not pertaining to the coming of Wilcon 9. For instance, report
and could have been any of several bundles kept in the stock room of the PC headquarters. 14 of illegal gambling operation.
The trial court was unconvinced, noting from its own examination of the accused that he claimed COURT:
to have come to Iloilo City to sell watches but carried only two watches at the time, traveling Q Previous to that particular information which you said two days before June 25, 1984, did you also receive
from Jolo for that purpose and spending P107.00 for fare, not to mention his other daily report regarding the activities of Idel Aminnudin
expenses. 15 Aminnudin testified that he kept the two watches in a secret pocket below his belt A Previous to June 25, 1984 we received reports on the activities of Idel Aminnudin.
Q What were those activities?
but, strangely, they were not discovered when he was bodily searched by the arresting officers
A Purely marijuana trafficking.
nor were they damaged as a result of his manhandling. 16 He also said he sold one of the watches Q From whom did you get that information?
for P400.00 and gave away the other, although the watches belonged not to him but to his A It came to my hand which was written in a required sheet of information, maybe for security reason and
cousin, 17 to a friend whose full name he said did not even know. 18 The trial court also rejected we cannot Identify the person.
his allegations of maltreatment, observing that he had not sufficiently proved the injuries Q But you received it from your regular informer?
sustained by him. 19 A Yes, sir.
There is no justification to reverse these factual findings, considering that it was the trial judge ATTY. LLARIZA:
who had immediate access to the testimony of the witnesses and had the opportunity to weigh Q Previous to June 25, 1984, you were more or less sure that Idel Aminnudin is coming with drugs?
A Marijuana, sir.
their credibility on the stand. Nuances of tone or voice, meaningful pauses and hesitation, flush
Q And this information respecting Idel Aminnudin's coming to Iloilo with marijuana was received by you
of face and dart of eyes, which may reveal the truth or expose the lie, are not described in the many days before you received the intelligence report in writing?
A Not a report of the particular coming of Aminnudin but his activities.
97

Q You only knew that he was coming on June 25,1984 two days before? In the many cases where this Court has sustained the warrantless arrest of violators of the
A Yes, sir. Dangerous Drugs Act, it has always been shown that they were caught red-handed, as a result of
Q You mean that before June 23, 1984 you did not know that minnudin was coming? what are popularly called "buy-bust" operations of the narcotics agents. 25 Rule 113 was clearly
A Before June 23,1984, I, in my capacity, did not know that he was coming but on June 23, 1984 that was the
applicable because at the precise time of arrest the accused was in the act of selling the
time when I received the information that he was coming. Regarding the reports on his activities, we have
reports that he was already consummated the act of selling and shipping marijuana stuff. prohibited drug.
COURT: In the case at bar, the accused-appellant was not, at the moment of his arrest, committing a
Q And as a result of that report, you put him under surveillance? crime nor was it shown that he was about to do so or that he had just done so. What he was
A Yes, sir. doing was descending the gangplank of the M/V Wilcon 9 and there was no outward indication
Q In the intelligence report, only the name of Idel Aminnudin was mentioned? that called for his arrest. To all appearances, he was like any of the other passengers innocently
A Yes, sir. disembarking from the vessel. It was only when the informer pointed to him as the carrier of the
Q Are you sure of that? marijuana that he suddenly became suspect and so subject to apprehension. It was the furtive
A On the 23rd he will be coming with the woman.
finger that triggered his arrest. The Identification by the informer was the probable cause as
Q So that even before you received the official report on June 23, 1984, you had already gathered
information to the effect that Idel Aminnudin was coming to Iloilo on June 25, 1984? determined by the officers (and not a judge) that authorized them to pounce upon Aminnudin
A Only on the 23rd of June. and immediately arrest him.
Q You did not try to secure a search warrant for the seizure or search of the subject mentioned in your Now that we have succeeded in restoring democracy in our country after fourteen years of the
intelligence report? despised dictatorship, when any one could be picked up at will, detained without charges and
A No, more. punished without trial, we will have only ourselves to blame if that kind of arbitrariness is
Q Why not? allowed to return, to once more flaunt its disdain of the Constitution and the individual liberties
A Because we were very very sure that our operation will yield positive result. its Bill of Rights guarantees.
Q Is that your procedure that whenever it will yield positive result you do not need a search warrant
While this is not to say that the accused-appellant is innocent, for indeed his very own words
anymore?
A Search warrant is not necessary. 23 suggest that he is lying, that fact alone does not justify a finding that he is guilty. The
That last answer is a cavalier pronouncement, especially as it comes from a mere lieutenant of constitutional presumption is that he is innocent, and he will be so declared even if his defense is
the PC. The Supreme Court cannot countenance such a statement. This is still a government of weak as long as the prosecution is not strong enough to convict him.
laws and not of men. Without the evidence of the marijuana allegedly seized from Aminnudin, the case of the
The mandate of the Bill of Rights is clear: prosecution must fall. That evidence cannot be admitted, and should never have been
Sec. 2. The right of the people to be secure in their persons, houses, papers and effects considered by the trial court for the simple fact is that the marijuana was seized illegally. It is the
against unreasonable searches and seizures of whatever nature and for any purpose fruit of the poisonous tree, to use Justice Holmes' felicitous phrase. The search was not an
shall be inviolable, and no search warrant or warrant of arrest shall issue except upon incident of a lawful arrest because there was no warrant of arrest and the warrantless arrest did
probable cause to be determined personally by the judge after examination under not come under the exceptions allowed by the Rules of Court. Hence, the warrantless search was
oath or affirmation of the complainant and the witnesses he may produce, and also illegal and the evidence obtained thereby was inadmissible.
particularly describing the place to be searched and the persons or things to be seized. The Court strongly supports the campaign of the government against drug addiction and
In the case at bar, there was no warrant of arrest or search warrant issued by a judge after commends the efforts of our law-enforcement officers against those who would inflict this
personal determination by him of the existence of probable cause. Contrary to the averments of malediction upon our people, especially the susceptible youth. But as demanding as this
the government, the accused-appellant was not caught in flagrante nor was a crime about to be campaign may be, it cannot be more so than the compulsions of the Bill of Rights for the
committed or had just been committed to justify the warrantless arrest allowed under Rule 113 protection of the liberty of every individual in the realm, including the basest of criminals. The
of the Rules of Court. Even expediency could not be invoked to dispense with the obtention of Constitution covers with the mantle of its protection the innocent and the guilty alike against any
the warrant as in the case of Roldan v. Arca, 24 for example. Here it was held that vessels and manner of high- handedness from the authorities, however praiseworthy their intentions.
aircraft are subject to warrantless searches and seizures for violation of the customs law because Those who are supposed to enforce the law are not justified in disregarding the rights of the
these vehicles may be quickly moved out of the locality or jurisdiction before the warrant can be individual in the name of order. Order is too high a price for the loss of liberty. As Justice Holmes,
secured. again, said, "I think it a less evil that some criminals should escape than that the government
The present case presented no such urgency. From the conflicting declarations of the PC should play an ignoble part." It is simply not allowed in the free society to violate a law to
witnesses, it is clear that they had at least two days within which they could have obtained a enforce another, especially if the law violated is the Constitution itself.
warrant to arrest and search Aminnudin who was coming to Iloilo on the M/V Wilcon 9. His We find that with the exclusion of the illegally seized marijuana as evidence against the accused-appellant,
name was known. The vehicle was Identified. The date of its arrival was certain. And from the his guilt has not been proved beyond reasonable doubt and he must therefore be discharged on the
information they had received, they could have persuaded a judge that there was probable presumption that he is innocent.
cause, indeed, to justify the issuance of a warrant. Yet they did nothing. No effort was made to ACCORDINGLY, the decision of the trial court is REVERSED and the accused-appellant is ACQUITTED. It is so
comply with the law. The Bill of Rights was ignored altogether because the PC lieutenant who ordered.
was the head of the arresting team, had determined on his own authority that a "search warrant
was not necessary."
98

PEOPLE VS MARIACOS including a blue plastic bag, were already being carried away by two (2) women. He caught up
Before this Court is an appeal from the Decision1 of the Court of Appeals (CA) in CA-G.R. CR-HC with the women and introduced himself as a policeman. He told them that they were under
No. 02718, which affirmed the decision2 of the Regional Trial Court (RTC), Branch 29, San arrest, but one of the women got away.
Fernando City, La Union, in Criminal Case No. 7144, finding appellant Belen Mariacos guilty of PO2 Pallayoc brought the woman, who was later identified as herein accused-appellant Belen
violating Article II, Section 5 of Republic Act (R.A.) No. 9165, or the Comprehensive Dangerous Mariacos, and the bags to the police station. At the police station, the investigators contacted
Drugs Act of 2002. the Mayor of San Gabriel to witness the opening of the bags. When the Mayor arrived about
The facts of the case, as summarized by the CA, are as follows: fifteen (15) minutes later, the bags were opened and three (3) bricks of marijuana wrapped in
Accused-appellant Belen Mariacos was charged in an Information, dated November 7, 2005 of newspaper, two (2) round bundles of marijuana, and two (2) bricks of marijuana fruiting tops, all
violating Section 5, Article II of Republic Act [No.] 9165, allegedly committed as follows: wrapped in a newspaper, were recovered.
"That on or about the 27th day of October, 2005, in the Municipality of San Gabriel, Province of Thereafter, the investigators marked, inventoried and forwarded the confiscated marijuana to
La Union, Philippines, and within the jurisdiction of this Honorable Court, the above-named the crime laboratory for examination. The laboratory examination showed that the stuff found in
accused, did then and there willfully, unlawfully and feloniously transport, deliver 7,030.3, (sic) the bags all tested positive for marijuana, a dangerous drug.
grams of dried marijuana fruiting tops without the necessary permit or authority from the proper When it was accused-appellant’s turn to present evidence, she testified that:
government agency or office. On October 27, 2005, at around 7:00 in the morning, accused-appellant, together with Lani
CONTRARY TO LAW." Herbacio, was inside a passenger jeepney bound for the poblacion. While the jeepney was still at
When arraigned on December 13, 2005, accused-appellant pleaded not guilty. During the pre- the terminal waiting for passengers, one Bennie Lao-ang ("Lao-ang"), her neighbor, requested
trial, the following were stipulated upon: her to carry a few bags which had been loaded on top of the jeepney. At first, accused-appellant
"1. Accused admits that she is the same person identified in the information as Belen Mariacos; refused, but she was persuaded later when she was told that she would only be carrying the
2. That accused is a resident of Brgy. Lunoy, San Gabriel, La Union; bags. When they reached the poblacion, Lao-ang handed accused-appellant and her companion,
3. That at the time of the arrest of the accused, accused had just alighted from a passenger Lani Herbacio, the bags, and then Lao-ang suddenly ran away. A few moments later, PO2
jeepney;
Pallayoc was upon them, arresting them. Without explanation, they were brought to the police
4. That the marijuana allegedly taken from the possession of the accused contained in two (2)
bags were submitted for examination to the Crime Lab; station. When they were at the police station, Lani Herbacio disappeared. It was also at the
5. That per Chemistry Report No. D-109-2005, the alleged drug submitted for examination gave police station that accused-appellant discovered the true contents of the bags which she was
positive result for the presence of marijuana; asked to carry. She maintained that she was not the owner of the bags and that she did not
6. That the drugs allegedly obtained from the accused contained (sic) and submitted for know what were contained in the bags. At the police station (sic) she executed a Counter-
examination weighed 7,030.3 grams; Affidavit.3
7. The Prosecutor admits the existence of a counter-affidavit executed by the accused; and On January 31, 2007, the RTC promulgated a decision, the dispositive portion of which states:
8. The existence of the affidavits executed by the witnesses of the accused family (sic): Lyn WHEREFORE, the Court finds the accused Belen Mariacos GUILTY as charged and sentences here
Punasen, Mercedes Tila and Magdalena Carino."
(sic) to suffer the penalty of life imprisonment and to pay a fine of P500,000.00.
During the trial, the prosecution established the following evidence:
The 7,030.3 grams of marijuana are ordered confiscated and turned over to the Philippine Drug
On October 26, 2005, in the evening, the San Gabriel Police Station of San Gabriel, La Union,
Enforcement Agency for destruction in the presence of the Court personnel and media.
conducted a checkpoint near the police station at the poblacion to intercept a suspected
SO ORDERED.4
transportation of marijuana from Barangay Balbalayang, San Gabriel, La Union. The group at the
Appellant appealed her conviction to the CA. She argued that the trial court erred in considering
checkpoint was composed of PO2 Lunes B. Pallayoc ("PO2 Pallayoc"), the Chief of Police, and
the evidence of the prosecution despite its inadmissibility.5 She claimed that her right against an
other policemen. When the checkpoint did not yield any suspect or marijuana, the Chief of Police
unreasonable search was flagrantly violated by Police Officer (PO)2 Pallayoc when the latter
instructed PO2 Pallayoc to proceed to Barangay Balbalayang to conduct surveillance operation
searched the bag, assuming it was hers, without a search warrant and with no permission from
(sic).
her. She averred that PO2 Pallayoc’s purpose for apprehending her was to verify if the bag she
At dawn on October 27, 2005, in Barangay Balbalayang, PO2 Pallayoc met with a secret agent of
was carrying was the same one he had illegally searched earlier. Moreover, appellant contended
the Barangay Intelligence Network who informed him that a baggage of marijuana had been
that there was no probable cause for her arrest.6
loaded on a passenger jeepney that was about to leave for the poblacion. The agent mentioned
Further, appellant claimed that the prosecution failed to prove the corpus delicti of the
three (3) bags and one (1) blue plastic bag. Further, the agent described a backpack bag with an
crime.7 She alleged that the apprehending police officers violated Dangerous Drugs Board
"O.K." marking. PO2 Pallayoc then boarded the said jeepney and positioned himself on top
Regulation No. 3, Series of 1979, as amended by Board Regulation No. 2, Series of 1990, which
thereof. While the vehicle was in motion, he found the black backpack with an "O.K." marking
prescribes the procedure in the custody of seized prohibited and regulated drugs, instruments,
and peeked inside its contents. PO2 Pallayoc found bricks of marijuana wrapped in newspapers.
apparatuses, and articles. The said regulation directs the apprehending team having initial
He then asked the other passengers on top of the jeepney about the owner of the bag, but no
custody and control of the drugs and/or paraphernalia, immediately after seizure or confiscation,
one knew.
to have the same physically inventoried and photographed in the presence of appellant or her
When the jeepney reached the poblacion, PO2 Pallayoc alighted together with the other
representative, who shall be required to sign copies of the inventory. The failure to comply with
passengers. Unfortunately, he did not notice who took the black backpack from atop the
this directive, appellant claimed, casts a serious doubt on the identity of the items allegedly
jeepney. He only realized a few moments later that the said bag and three (3) other bags,
99

confiscated from her. She, likewise, averred that the prosecution failed to prove that the items seizures, providing, more or less, clear parameters in determining which are proper and which
allegedly confiscated were indeed prohibited drugs, and to establish the chain of custody over are not.1avvphi1
the same. Appellant’s main argument before the CA centered on the inadmissibility of the evidence used
On the other hand, the People, through the Office of the Solicitor General (OSG), argued that the against her. She claims that her constitutional right against unreasonable searches was flagrantly
warrantless arrest of appellant and the warrantless seizure of marijuana were valid and violated by the apprehending officer.
legal,8 justified as a search of a moving vehicle. It averred that PO2 Pallayoc had reasonable Thus, we must determine if the search was lawful. If it was, then there would have been
ground to believe that appellant had committed the crime of delivering dangerous drugs based probable cause for the warrantless arrest of appellant.
on reliable information from their agent, which was confirmed when he peeked into the bags Article III, Section 2 of the Philippine Constitution provides:
and smelled the distinctive odor of marijuana.9 The OSG also argued that appellant was now Section 2. The right of the people to be secure in their persons, houses, papers, and effects
estopped from questioning the illegality of her arrest since she voluntarily entered a plea of "not against unreasonable searches and seizures of whatever nature and for any purpose shall be
guilty" upon arraignment and participated in the trial and presented her evidence.10 The OSG inviolable, and no search warrant or warrant of arrest shall issue except upon probable cause to
brushed aside appellant’s argument that the bricks of marijuana were not photographed and be determined personally by the judge after examination under oath or affirmation of the
inventoried in her presence or that of her counsel immediately after confiscation, positing that complainant and the witnesses he may produce, and particularly describing the place to be
physical inventory may be done at the nearest police station or at the nearest office of the searched and the persons or things to be seized.
apprehending team, whichever was practicable.11 Law and jurisprudence have laid down the instances when a warrantless search is valid. These
In a Decision dated January 19, 2009, the CA dismissed appellant’s appeal and affirmed the RTC are:
decision in toto.12 It held that the prosecution had successfully proven that appellant carried 1. Warrantless search incidental to a lawful arrest recognized under Section 12 [now Section 13],
away from the jeepney a number of bags which, when inspected by the police, contained Rule 126 of the Rules of Court and by prevailing jurisprudence;
dangerous drugs. The CA ruled that appellant was caught in flagrante delicto of "carrying and 2. Seizure of evidence in "plain view," the elements of which are:
(a) a prior valid intrusion based on the valid warrantless arrest in which the police are
conveying" the bag that contained the illegal drugs, and thus held that appellant’s warrantless
legally present in the pursuit of their official duties;
arrest was valid. The appellate court ratiocinated: (b) the evidence was inadvertently discovered by the police who had the right to be
It must be stressed that PO2 Pallayoc had earlier ascertained the contents of the bags when he where they are;
was aboard the jeep. He saw the bricks of marijuana wrapped in newspaper. That said marijuana (c) the evidence must be immediately apparent[;] and;
was on board the jeepney to be delivered to a specified destination was already unlawful. PO2 (d) "plain view" justified mere seizure of evidence without further search.
Pallayoc needed only to see for himself to whom those bags belonged. So, when he saw accused- 3. Search of a moving vehicle. Highly regulated by the government, the vehicle's inherent mobility
appellant carrying the bags, PO2 Pallayoc was within his lawful duty to make a warrantless arrest reduces expectation of privacy especially when its transit in public thoroughfares furnishes a
of accused-appellant. highly reasonable suspicion amounting to probable cause that the occupant committed a criminal
activity;
xxxx
4. Consented warrantless search;
Firstly, this Court opines that the invocation of Section 2, Article III of the Constitution is 5. Customs search;
misplaced. At the time, when PO2 Pallayoc looked into the contents of the suspicious bags, there 6. Stop and Frisk; and
was no identified owner. He asked the other passengers atop the jeepney but no one knew who 7. Exigent and Emergency Circumstances.14
owned the bags. Thus, there could be no violation of the right when no one was entitled thereto Both the trial court and the CA anchored their respective decisions on the fact that the search
at that time. was conducted on a moving vehicle to justify the validity of the search.
Secondly, the facts of the case show the urgency of the situation. The local police has been trying Indeed, the search of a moving vehicle is one of the doctrinally accepted exceptions to the
to intercept the transport of the illegal drugs for more than a day, to no avail. Thus, when PO2 Constitutional mandate that no search or seizure shall be made except by virtue of a warrant
Pallayoc was tipped by the secret agent of the Barangay Intelligence Network, PO2 Pallayoc had issued by a judge after personally determining the existence of probable cause.15
no other recourse than to verify as promptly as possible the tip and check the contents of the In People v. Bagista,16 the Court said:
bags. The constitutional proscription against warrantless searches and seizures admits of certain
Thirdly, x x x the search was conducted in a moving vehicle. Time and again, a search of a moving exceptions. Aside from a search incident to a lawful arrest, a warrantless search had been upheld
vehicle has been justified on the ground that the mobility of motor vehicles makes it possible for in cases of a moving vehicle, and the seizure of evidence in plain view.
the vehicle to move out of the locality or jurisdiction in which the warrant must be sought. Thus, With regard to the search of moving vehicles, this had been justified on the ground that the
under the facts, PO2 Pallayoc could not be expected to secure a search warrant in order to check mobility of motor vehicles makes it possible for the vehicle to be searched to move out of the
the contents of the bags which were loaded on top of the moving jeepney. Otherwise, a search locality or jurisdiction in which the warrant must be sought.
warrant would have been of no use because the motor vehicle had already left the locality.13 This in no way, however, gives the police officers unlimited discretion to conduct warrantless
Appellant is now before this Court, appealing her conviction. searches of automobiles in the absence of probable cause. When a vehicle is stopped and
Once again, we are asked to determine the limits of the powers of the State’s agents to conduct subjected to an extensive search, such a warrantless search has been held to be valid only as
searches and seizures. Over the years, this Court had laid down the rules on searches and long as the officers conducting the search have reasonable or probable cause to believe before
100

the search that they will find the instrumentality or evidence pertaining to a crime, in the vehicle For this rule to apply, it is imperative that there be a prior valid arrest. Although, generally, a
to be searched. warrant is necessary for a valid arrest, the Rules of Court provides the exceptions therefor, to
It is well to remember that in the instances we have recognized as exceptions to the requirement wit:
of a judicial warrant, it is necessary that the officer effecting the arrest or seizure must have SEC. 5. Arrest without warrant; when lawful.—A peace officer or a private person may, without a
been impelled to do so because of probable cause. The essential requisite of probable cause warrant, arrest a person:
must be satisfied before a warrantless search and seizure can be lawfully conducted. 17 Without (a) When, in his presence, the person to be arrested has committed, is actually
probable cause, the articles seized cannot be admitted in evidence against the person arrested.18 committing, or is attempting to commit an offense;
Probable cause is defined as a reasonable ground of suspicion supported by circumstances (b) When an offense has just been committed and he has probable cause to believe
sufficiently strong in themselves to induce a cautious man to believe that the person accused is based on personal knowledge of facts or circumstances that the person to be arrested
guilty of the offense charged. It refers to the existence of such facts and circumstances that can has committed it; and
lead a reasonably discreet and prudent man to believe that an offense has been committed, and (c) When the person to be arrested is a prisoner who has escaped from a penal
that the items, articles or objects sought in connection with said offense or subject to seizure establishment or place where he is serving final judgment or is temporarily confined
and destruction by law are in the place to be searched.19 while his case is pending, or has escaped while being transferred from one
The grounds of suspicion are reasonable when, in the absence of actual belief of the arresting confinement to another.
officers, the suspicion that the person to be arrested is probably guilty of committing the offense In cases falling under paragraphs (a) and (b) above, the person arrested without a warrant shall
is based on actual facts, i.e., supported by circumstances sufficiently strong in themselves to be forthwith delivered to the nearest police station or jail and shall be proceeded against in
create the probable cause of guilt of the person to be arrested. A reasonable suspicion therefore accordance with section 7 of Rule 112.24
must be founded on probable cause, coupled with good faith on the part of the peace officers Be that as it may, we have held that a search substantially contemporaneous with an arrest can
making the arrest.20 precede the arrest if the police has probable cause to make the arrest at the outset of the
Over the years, the rules governing search and seizure have been steadily liberalized whenever a search.25
moving vehicle is the object of the search on the basis of practicality. This is so considering that Given that the search was valid, appellant’s arrest based on that search is also valid.
before a warrant could be obtained, the place, things and persons to be searched must be Article II, Section 5 of the Comprehensive Dangerous Drugs Act of 2002 states:
described to the satisfaction of the issuing judge – a requirement which borders on the SEC. 5 Sale, Trading, Administration, Dispensation, Delivery, Distribution and Transportation of
impossible in instances where moving vehicle is used to transport contraband from one place to Dangerous Drugs and/or Controlled Precursors and Essential Chemicals. – The penalty of life
another with impunity.21 imprisonment to death and a fine ranging from Five hundred thousand pesos (P500,000.00) to
This exception is easy to understand. A search warrant may readily be obtained when the search Ten million pesos (P10,000,000.00) shall be imposed upon any person, who, unless authorized by
is made in a store, dwelling house or other immobile structure. But it is impracticable to obtain a law, shall sell, trade, administer, dispense, deliver, give away to another, distribute, dispatch in
warrant when the search is conducted on a mobile ship, on an aircraft, or in other motor vehicles transit or transport any dangerous drug, including any and all species of opium poppy regardless
since they can quickly be moved out of the locality or jurisdiction where the warrant must be of the quantity and purity involved, or shall act as a broker in any of such transactions.
sought.22 The penalty of imprisonment ranging from twelve (12) years and one (1) day to twenty (20) years
Given the discussion above, it is readily apparent that the search in this case is valid. The vehicle and a fine ranging from One hundred thousand pesos (P100,000.00) to Five hundred thousand
that carried the contraband or prohibited drugs was about to leave. PO2 Pallayoc had to make a pesos (P500,000.00) shall be imposed upon any person who, unless authorized by law, shall sell,
quick decision and act fast. It would be unreasonable to require him to procure a warrant before trade, administer, dispense, deliver, give away to another, distribute, dispatch in transit or
conducting the search under the circumstances. Time was of the essence in this case. The transport any controlled precursor and essential chemical, or shall act as a broker in such
searching officer had no time to obtain a warrant. Indeed, he only had enough time to board the transactions.
vehicle before the same left for its destination. In her defense, appellant averred that the packages she was carrying did not belong to her but to
It is well to remember that on October 26, 2005, the night before appellant’s arrest, the police a neighbor who had asked her to carry the same for him. This contention, however, is of no
received information that marijuana was to be transported from Barangay Balbalayang, and had consequence.
set up a checkpoint around the area to intercept the suspects. At dawn of October 27, 2005, PO2 When an accused is charged with illegal possession or transportation of prohibited drugs, the
Pallayoc met the secret agent from the Barangay Intelligence Network, who informed him that a ownership thereof is immaterial. Consequently, proof of ownership of the confiscated marijuana
baggage of marijuana was loaded on a passenger jeepney about to leave for the poblacion. Thus, is not necessary.26
PO2 Pallayoc had probable cause to search the packages allegedly containing illegal drugs. Appellant’s alleged lack of knowledge does not constitute a valid defense. Lack of criminal intent
This Court has also, time and again, upheld as valid a warrantless search incident to a lawful and good faith are not exempting circumstances where the crime charged is malum prohibitum,
arrest. Thus, Section 13, Rule 126 of the Rules of Court provides: as in this case.27 Mere possession and/or delivery of a prohibited drug, without legal authority, is
SEC. 13. Search incident to lawful arrest.—A person lawfully arrested may be searched for punishable under the Dangerous Drugs Act.28
dangerous weapons or anything which may have been used or constitute proof in the Anti-narcotics laws, like anti-gambling laws, are regulatory statutes. They are rules of
commission of an offense without a search warrant.23 convenience designed to secure a more orderly regulation of the affairs of society, and their
101

violation gives rise to crimes mala prohibita. Laws defining crimes mala prohibita condemn Justice (DOJ), and any elected public official who shall be required to sign the copies of the
behavior directed not against particular individuals, but against public order.29 inventory and be given a copy thereof: Provided, that the physical inventory and photograph
Jurisprudence defines "transport" as "to carry or convey from one place to another."30 There is shall be conducted at the place where the search warrant is served; or at the nearest police
no definitive moment when an accused "transports" a prohibited drug. When the circumstances station or at the nearest office of the apprehending officer/team, whichever is practicable, in
establish the purpose of an accused to transport and the fact of transportation itself, there case of warrantless seizures; Provided, further, that non-compliance with these requirements
should be no question as to the perpetration of the criminal act.31 The fact that there is actual under justifiable grounds, as long as the integrity and the evidentiary value of the seized items
conveyance suffices to support a finding that the act of transporting was committed and it is are properly preserved by the apprehending officer/team, shall not render void and invalid such
immaterial whether or not the place of destination is reached.32 seizures of and custody over said items.
Moreover, appellant’s possession of the packages containing illegal drugs gave rise to the PO2 Pallayoc testified that after apprehending appellant, he immediately brought her to the
disputable presumption33 that she is the owner of the packages and their contents.34 Appellant police station. At the station, the police requested the Mayor to witness the opening of the bags
failed to rebut this presumption. Her uncorroborated claim of lack of knowledge that she had seized from appellant. When the Mayor arrived, he opened the bag in front of appellant and the
prohibited drug in her possession is insufficient. other police officers. The black bag yielded three bricks of marijuana wrapped in newspaper,
Appellant’s narration of facts deserves little credence. If it is true that Bennie Lao-ang merely while the plastic bag yielded two bundles of marijuana and two bricks of marijuana fruiting
asked her and her companion to carry some baggages, it is but logical to first ask what the tops.36 PO2 Pallayoc identified the bricks. He and PO3 Stanley Campit then marked the same.
packages contained and where these would be taken. Likewise, if, as appellant said, Lao-ang ran Then the seized items were brought to the PNP Crime Laboratory for examination.
away after they disembarked from the jeepney, appellant and her companion should have ran It is admitted that there were no photographs taken of the drugs seized, that appellant was not
after him to give him the bags he had left with them, and not to continue on their journey accompanied by counsel, and that no representative from the media and the DOJ were present.
without knowing where they were taking the bags. However, this Court has already previously held that non-compliance with Section 21 is not fatal
Next, appellant argues that the prosecution failed to prove the corpus delicti of the crime. In and will not render an accused’s arrest illegal, or make the items seized inadmissible. What is of
particular, she alleged that the apprehending police officers failed to follow the procedure in the utmost importance is the preservation of the integrity and evidentiary value of the seized
custody of seized prohibited and regulated drugs, instruments, apparatuses, and articles. items.37
In all prosecutions for violation of the Dangerous Drugs Act, the existence of all dangerous drugs Based on the testimony of PO2 Pallayoc, after appellant’s arrest, she was immediately brought to
is a sine qua non for conviction. The dangerous drug is the very corpus delicti of that crime.35 the police station where she stayed while waiting for the Mayor. It was the Mayor who opened
Thus, Section 21 of R.A. No. 9165 prescribes the procedure for custody and disposition of seized the packages, revealing the illegal drugs, which were thereafter marked and sent to the police
dangerous drugs, to wit: crime laboratory the following day. Contrary to appellant’s claim, the prosecution’s evidence
Section 21. Custody and Disposition of Confiscated, Seized, and/or Surrendered Dangerous Drugs, establishes the chain of custody from the time of appellant’s arrest until the prohibited drugs
Plant Sources of Dangerous Drugs, Controlled Precursors and Essential Chemicals, were tested at the police crime laboratory.
Instruments/Paraphernalia and/or Laboratory Equipment. – The PDEA shall take charge and have While it is true that the arresting officer failed to state explicitly the justifiable ground for non-
custody of all dangerous drugs, plant sources of dangerous drugs, controlled precursors and compliance with Section 21, this does not necessarily mean that appellant’s arrest was illegal or
essential chemicals, as well as instruments/paraphernalia and/or laboratory equipment so that the items seized are inadmissible. The justifiable ground will remain unknown because
confiscated, seized and/or surrendered, for proper disposition in the following manner: appellant did not question the custody and disposition of the items taken from her during the
(1) The apprehending team having initial custody and control of the drugs shall, immediately trial.38 Even assuming that the police officers failed to abide by Section 21, appellant should have
after seizure and confiscation, physically inventory and photograph the same in the presence of raised this issue before the trial court. She could have moved for the quashal of the information
the accused or the person/s from whom such items were confiscated and/or seized, or his/her at the first instance. But she did not. Hence, she is deemed to have waived any objection on the
representative or counsel, a representative from the media and the Department of Justice (DOJ), matter.
and any elected public official who shall be required to sign the copies of the inventory and be Further, the actions of the police officers, in relation to the procedural rules on the chain of
given a copy thereof. custody, enjoyed the presumption of regularity in the performance of official functions. Courts
The Implementing Rules and Regulations (IRR) of R.A. No. 9165 further provides: accord credence and full faith to the testimonies of police authorities, as they are presumed to
SECTION 21. Custody and Disposition of Confiscated, Seized and/or Surrendered Dangerous be performing their duties regularly, absent any convincing proof to the contrary.39
Drugs, Plant Sources of Dangerous Drugs, Controlled Precursors and Essential Chemicals, In sum, the prosecution successfully established appellant’s guilt. Thus, her conviction must be
Instruments/Paraphernalia and/or Laboratory Equipment. – The PDEA shall take charge and affirmed.
have custody of all dangerous drugs, plant sources of dangerous drugs, controlled precursors WHEREFORE, the foregoing premises considered, the appeal is DISMISSED. The Decision of the
and essential chemicals, as well as instruments/paraphernalia and/or laboratory equipment so Court of Appeals in CA-G.R. CR-HC No. 02718 is AFFIRMED.
confiscated, seized and/or surrendered, for proper disposition in the following manner: SO ORDERED.
(a) The apprehending officer/team having initial custody and control of the drugs shall,
immediately after seizure and confiscation, physically inventory and photograph the same in the
presence of the accused or the person/s from whom such items were confiscated and/or seized,
or his/her representative or counsel, a representative from the media and the Department of
102

PEOPLE VS ARUTA constitutional right against unreasonable search and seizure as well as their inadmissibility in
With the pervasive proliferation of illegal drugs and its pernicious effects on our society, our law evidence.
enforcers tend at times to overreach themselves in apprehending drug offenders to the extent of The said “Demurrer to Evidence” was, however, denied without the trial court ruling on the
failing to observe well-entrenched constitutional guarantees against illegal searches and arrests. alleged illegality of the search and seizure and the inadmissibility in evidence of the items seized
Consequently, drug offenders manage to evade the clutches of the law on mere technicalities. to avoid pre-judgment. Instead, the trial court continued to hear the case.
Accused-appellant Rosa Aruta y Menguin was arrested and charged with violating Section 4, In view of said denial, accused-appellant testified on her behalf. As expected, her version of the
Article II of Republic Act No. 6425 or the Dangerous Drugs Act. The information reads: incident differed from that of the prosecution. She claimed that immediately prior to her arrest,
That on or about the fourteenth (14th) day of December, 1988, in the City of Olongapo, she had just come from Choice Theater where she watched the movie “Balweg.” While about to
Philippines, and within the jurisdiction of this Honorable Court, the above-named accused, cross the road, an old woman asked her help in carrying a shoulder bag. In the middle of the
without being lawfully authorized, did then and there willfully, unlawfully and knowingly engage road, Lt. Abello and Lt. Domingo arrested her and asked her to go with them to the NARCOM
in transporting approximately eight (8) kilos and five hundred (500) grams of dried marijuana Office.
packed in plastic bag marked “Cash Katutak” placed in a traveling bag, which are prohibited During investigation at said office, she disclaimed any knowledge as to the identity of the woman
drugs. and averred that the old woman was nowhere to be found after she was arrested. Moreover,
Upon arraignment, she pleaded “not guilty.” After trial on the merits, the Regional Trial Court of she added that no search warrant was shown to her by the arresting officers.
Olongapo City convicted and sentenced her to suffer the penalty of life imprisonment and to pay After the prosecution made a formal offer of evidence, the defense filed a “Comment and/or
a fine of twenty thousand (P20,000.00) pesos. 1 Objection to Prosecution’s Formal Offer of Evidence” contesting the admissibility of the items
The prosecution substantially relied on the testimonies of P/Lt. Ernesto Abello, Officer-in-Charge seized as they were allegedly a product of an unreasonable search and seizure.
of the Narcotics Command (NARCOM) of Olongapo City and P/Lt. Jose Domingo. Based on their Not convinced with her version of the incident, the Regional Trial Court of Olongapo City
testimonies, the court a quo found the following: convicted accused-appellant of transporting eight (8) kilos and five hundred (500) grams of
On December 13, 1988, P/Lt. Abello was tipped off by his informant, known only as Benjie, that a marijuana from Baguio City to Olongapo City in violation of Section 4, Article 11 of R.A. No. 6425,
certain “Aling Rosa” would be arriving from Baguio City the following day, December 14, 1988, as amended, otherwise known as the Dangerous Drugs Act of 1972 and sentenced her to life
with a large volume of marijuana. Acting on said tip, P/Lt. Abello assembled a team composed of imprisonment and to pay a fine of twenty thousand (P20,000.00) pesos without subsidiary
P/Lt. Jose Domingo, Sgt. Angel Sudiacal, Sgt. Oscar Imperial, Sgt. Danilo Santiago and Sgt. Efren imprisonment in case of insolvency. 2
Quirubin. In this appeal, accused-appellant submits the following:
Said team proceeded to West Bajac-Bajac, Olongapo City at around 4:00 in the afternoon of 1. The trial court erred in holding that the NARCOM agents could not apply for a warrant for the
December 14, 1988 and deployed themselves near the Philippine National Bank (PNB) building search of a bus or a passenger who boarded a bus because one of the requirements for applying
along Rizal Avenue and the Caltex gasoline station. Dividing themselves into two groups, one a search warrant is that the place to be searched must be specifically designated and described.
group, made up of P/Lt. Abello, P/Lt. Domingo and the informant posted themselves near the 2. The trial court erred in holding or assuming that if a search warrant was applied for by the
PNB building while the other group waited near the Caltex gasoline station. NARCOM agents, still no court would issue a search warrant for the reason that the same would
While thus positioned, a Victory Liner Bus with body number 474 and the letters BGO printed on be considered a general search warrant which may be quashed.
its front and back bumpers stopped in front of the PNB building at around 6:30 in the evening of 3. The trial court erred in not finding that the warrantless search resulting to the arrest of
the same day from where two females and a male got off. It was at this stage that the informant accused-appellant violated the latter’s constitutional rights.
pointed out to the team “Aling Rosa” who was then carrying a traveling bag. 4. The trial court erred in not holding that although the defense of denial is weak yet the
Having ascertained that accused-appellant was “Aling Rosa,” the team approached her and evidence of the prosecution is even weaker.
introduced themselves as NARCOM agents. When P/Lt. Abello asked “Aling Rosa” about the These submissions are impressed with merit.
contents of her bag, the latter handed it to the former. In People v. Ramos, 3 this Court held that a search may be conducted by law enforcers only on
Upon inspection, the bag was found to contain dried marijuana leaves packed in a plastic bag the strength of a search warrant validly issued by a judge as provided in Article III, Section 2 of
marked “Cash Katutak.” The team confiscated the bag together with the Victory Liner bus ticket the Constitution which provides:
to which Lt. Domingo affixed his signature. Accused-appellant was then brought to the NARCOM Sec. 2. The right of the people to be secure in their persons, houses, papers, and effects against
office for investigation where a Receipt of Property Seized was prepared for the confiscated unreasonable searches and seizures of whatever nature and for any purpose shall be inviolable,
marijuana leaves. and no search warrant or warrant of arrest shall issue except upon probable cause to be
Upon examination of the seized marijuana specimen at the PC/INP Crime Laboratory, Camp determined personally by the judge after examination under oath or affirmation of the
Olivas, Pampanga, P/Maj. Marlene Salangad, a Forensic Chemist, prepared a Technical Report complainant and the witnesses he may produce, and particularly describing the place to be
stating that said specimen yielded positive results for marijuana, a prohibited drug. searched and the persons or things to be seized.
After the presentation of the testimonies of the arresting officers and of the above technical This constitutional guarantee is not a blanket prohibition against all searches and seizures as it
report, the prosecution rested its case. operates only against “unreasonable” searches and seizures. The plain import of the language of
Instead of presenting its evidence, the defense filed a “Demurrer to Evidence” alleging the the Constitution, which in one sentence prohibits unreasonable searches and seizures and at the
illegality of the search and seizure of the items thereby violating accused-appellant’s same time prescribes the requisites for a valid warrant, is that searches and seizures are
103

normally unreasonable unless authorized by a validly issued search warrant or warrant of arrest. article(s) or object(s) sought in connection with said offense or subject to seizure and destruction
Thus, the fundamental protection accorded by the search and seizure clause is that between by law is in the place to be searched. 12
person and police must stand the protective authority of a magistrate clothed with power to It ought to be emphasized that in determining probable cause, the average man weighs facts and
issue or refuse to issue search warrants or warrants of arrest. 4 circumstances without resorting to the calibrations of our rules of evidence of which his
Further, articles which are the product of unreasonable searches and seizures are inadmissible as knowledge is technically nil. Rather, he relies on the calculus of common sense which all
evidence pursuant to the doctrine pronounced in Stonehill v. Diokno. 5 This exclusionary rule was reasonable men have in abundance. The same quantum of evidence is required in determining
later enshrined in Article III, Section 3(2) of the Constitution, thus: probable cause relative to search. Before a search warrant can be issued, it must be shown by
Sec. 3(2). Any evidence obtained in violation of this or the preceding section shall be inadmissible substantial evidence that the items sought are in fact seizable by virtue of being connected with
in evidence for any purpose in any proceeding. criminal activity, and that the items will be found in the place to be searched. 13
From the foregoing, it can be said that the State cannot simply intrude indiscriminately into the In searches and seizures effected without a warrant, it is necessary for probable cause to be
houses, papers, effects, and most importantly, on the person of an individual. The constitutional present. Absent any probable cause, the article(s) seized could not be admitted and used as
provision guaranteed an impenetrable shield against unreasonable searches and seizures. As evidence against the person arrested. Probable cause, in these cases, must only be based on
such, it protects the privacy and sanctity of the person himself against unlawful arrests and other reasonable ground of suspicion or belief that a crime has been committed or is about to be
forms of restraint. 6 committed.
Therewithal, the right of a person to be secured against any unreasonable seizure of his body In our jurisprudence, there are instances where information has become a sufficient probable
and any deprivation of his liberty is a most basic and fundamental one. A statute, rule or cause to effect a warrantless search and seizure.
situation which allows exceptions to the requirement of a warrant of arrest or search warrant In People v. Tangliben, 14 acting on information supplied by informers, police officers conducted a
must perforce be strictly construed and their application limited only to cases specifically surveillance at the Victory Liner Terminal compound in San Fernando, Pampanga against persons
provided or allowed by law. To do otherwise is an infringement upon personal liberty and would who may commit misdemeanors and also on those who may be engaging in the traffic of
set back a right so basic and deserving of full protection and vindication yet often violated. 7 dangerous drugs. At 9:30 in the evening, the policemen noticed a person carrying a red traveling
The following cases are specifically provided or allowed by law: bag who was acting suspiciously. They confronted him and requested him to open his bag but
1. Warrantless search incidental to a lawful arrest recognized under Section 12, Rule 126 of the he refused. He acceded later on when the policemen identified themselves. Inside the bag were
Rules of Court 8 and by prevailing jurisprudence; marijuana leaves wrapped in a plastic wrapper. The police officers only knew of the activities of
2. Seizure of evidence in “plain view,” the elements of which are: Tangliben on the night of his arrest.
(a) a prior valid intrusion based on the valid warrantless arrest in which the police are legally In instant case, the apprehending officers already had prior knowledge from their informant
present in the pursuit of their official duties; regarding Aruta’s alleged activities. In Tangliben policemen were confronted with an on-the-spot
(b) the evidence was inadvertently discovered by the police who had the right to be where they tip. Moreover, the policemen knew that the Victory Liner compound is being used by drug
are; traffickers as their “business address”. More significantly, Tangliben was acting suspiciously. His
(c) the evidence must be immediately apparent, and actuations and surrounding circumstances led the policemen to reasonably suspect that
(d) “plain view” justified mere seizure of evidence without further search; Tangliben is committing a crime. In instant case, there is no single indication that Aruta was
3. Search of a moving vehicle. Highly regulated by the government, the vehicle’s inherent acting suspiciously.
mobility reduces expectation of privacy especially when its transit in public thoroughfares In People v. Malmstedt, 15 the Narcom agents received reports that vehicles coming from Sagada
furnishes a highly reasonable suspicion amounting to probable cause that the occupant were transporting marijuana. They likewise received information that a Caucasian coming from
committed a criminal activity; Sagada had prohibited drugs on his person. There was no reasonable time to obtain a search
4. Consented warrantless search; warrant, especially since the identity of the suspect could not be readily ascertained.
5. Customs search; 9 His actuations also aroused the suspicion of the officers conducting the operation. The Court
6. Stop and Frisk; 10and held that in light of such circumstances, to deprive the agents of the ability and facility to act
7. Exigent and Emergency Circumstances. 11 promptly, including a search without a warrant, would be to sanction impotence and
The above exceptions, however, should not become unbridled licenses for law enforcement ineffectiveness in law enforcement, to the detriment of society.
officers to trample upon the constitutionally guaranteed and more fundamental right of persons Note, however, the glaring differences of Malmstedt to the instant case. In present case, the
against unreasonable search and seizures. The essential requisite of probable cause must still be police officers had reasonable time within which to secure a search warrant. Second, Aruta’s
satisfied before a warrantless search and seizure can be lawfully conducted. identity was priorly ascertained. Third, Aruta was not acting suspiciously. Fourth, Malmstedt was
Although probable cause eludes exact and concrete definition, it generally signifies a reasonable searched aboard a moving vehicle, a legally accepted exception to the warrant requirement.
ground of suspicion supported by circumstances sufficiently strong in themselves to warrant a Aruta, on the other hand, was searched while about to cross a street.
cautious man to believe that the person accused is guilty of the offense with which he is charged. In People v. Bagista, 16 the NARCOM officers had probable cause to stop and search all vehicles
It likewise refers to the existence of such facts and circumstances which could lead a reasonably coming from the north to Acop, Tublay, Benguet in view of the confidential information they
discreet and prudent man to believe that an offense has been committed and that the item(s), received from their regular informant that a woman having the same appearance as that of
accused-appellant would be bringing marijuana from up north. They likewise had probable cause
104

to search accused-appellant’s belongings since she fitted the description given by the NARCOM In the instant case, the NARCOM agents were admittedly not armed with a warrant of arrest. To
informant. Since there was a valid warrantless search by the NARCOM agents, any evidence legitimize the warrantless search and seizure of accused-appellant’s bag, accused-appellant must
obtained in the course of said search is admissible against accused-appellant. Again, this case have been validly arrested under Section 5 of Rule 113 which provides inter alia:
differs from Aruta as this involves a search of a moving vehicle plus the fact that the police Sec. 5. Arrest without warrant; when lawful. — A peace officer or a private person may, without
officers erected a checkpoint. Both are exceptions to the requirements of a search warrant. a warrant, arrest a person:
In Manalili v. Court of Appeals and People, 17 the policemen conducted a surveillance in an area (a) When in his presence, the person to be arrested has committed, is actually committing, or is
of the Kalookan Cemetery based on information that drug addicts were roaming therein. Upon attempting to commit an offense;
reaching the place, they chanced upon a man in front of the cemetery who appeared to be xxx xxx xxx
“high” on drugs. He was observed to have reddish eyes and to be walking in a swaying manner. Accused-appellant Aruta cannot be said to be committing a crime. Neither was she about to
Moreover, he appeared to be trying to avoid the policemen. When approached and asked what commit one nor had she just committed a crime. Accused-appellant was merely crossing the
he was holding in his hands, he tried to resist. When he showed his wallet, it contained street and was not acting in any manner that would engender a reasonable ground for the
marijuana. The Court held that the policemen had sufficient reason to accost accused-appellant NARCOM agents to suspect and conclude that she was committing a crime. It was only when the
to determine if he was actually “high” on drugs due to his suspicious actuations, coupled with informant pointed to accused-appellant and identified her to the agents as the carrier of the
the fact that based on information, this area was a haven for drug addicts. marijuana that she was singled out as the suspect. The NARCOM agents would not have
In all the abovecited cases, there was information received which became the bases for apprehended accused-appellant were it not for the furtive finger of the informant because, as
conducting the warrantless search. Furthermore, additional factors and circumstances were clearly illustrated by the evidence on record, there was no reason whatsoever for them to
present which, when taken together with the information, constituted probable causes which suspect that accused-appellant was committing a crime, except for the pointing finger of the
justified the warrantless searches and seizures in each of the cases. informant. This the Court could neither sanction nor tolerate as it is a clear violation of the
In the instant case, the determination of the absence or existence of probable cause necessitates constitutional guarantee against unreasonable search and seizure. Neither was there any
a reexamination of the facts. The following have been established: (1) In the morning of semblance of any compliance with the rigid requirements of probable cause and warrantless
December 13, 1988, the law enforcement officers received information from an informant arrests.
named “Benjie” that a certain “Aling Rosa” would be leaving for Baguio City on December 14, Consequently, there was no legal basis for the NARCOM agents to effect a warrantless search of
1988 and would be back in the afternoon of the same day carrying with her a large volume of accused-appellant’s bag, there being no probable cause and the accused-appellant not having
marijuana; (2) At 6:30 in the evening of December 14, 1988, accused-appellant alighted from a been lawfully arrested. Stated otherwise, the arrest being incipiently illegal, it logically follows
Victory Liner Bus carrying a traveling bag even as the informant pointed her out to the law that the subsequent search was similarly illegal, it being not incidental to a lawful arrest. The
enforcement officers; (3) The law enforcement officers approached her and introduced constitutional guarantee against unreasonable search and seizure must perforce operate in favor
themselves as NARCOM agents; (4) When asked by Lt. Abello about the contents of her traveling of accused-appellant. As such, the articles seized could not be used as evidence against accused-
bag, she gave the same to him; (5) When they opened the same, they found dried marijuana appellant for these are “fruits of a poisoned tree” and, therefore, must be rejected, pursuant to
leaves; (6) Accused-appellant was then brought to the NARCOM office for investigation. Article III, Sec. 3(2) of the Constitution.
This case is similar to People v. Aminnudin where the police received information two days Emphasis is to be laid on the fact that the law requires that the search be incidental to a lawful
before the arrival of Aminnudin that the latter would be arriving from Iloilo on board the M/V arrest, in order that the search itself may likewise be considered legal. Therefore, it is beyond
Wilcon 9. His name was known, the vehicle was identified and the date of arrival was certain. cavil that a lawful arrest must precede the search of a person and his belongings. Where a search
From the information they had received, the police could have persuaded a judge that there was is first undertaken, and an arrest effected based on evidence produced by the search, both such
probable cause, indeed, to justify the issuance of a warrant. Instead of securing a warrant first, search and arrest would be unlawful, for being contrary to law. 18
they proceeded to apprehend Aminnudin. When the case was brought before this Court, the As previously discussed, the case in point is People v. Aminnudin 19 where, this Court observed
arrest was held to be illegal; hence any item seized from Aminnudin could not be used against that:
him. . . . accused-appellant was not, at the moment of his arrest, committing a crime nor was it shown
Another recent case is People v. Encinada where the police likewise received confidential that he was about to do so or that he had just done so. What he was doing was descending the
information the day before at 4:00 in the afternoon from their informant that Encinada would be gangplank of the M/V Wilcon 9 and there was no outward indication that called for his arrest. To
bringing in marijuana from Cebu City on board M/V Sweet Pearl at 7:00 in the morning of the all appearances, he was like any of the other passengers innocently disembarking from the
following day. This intelligence information regarding the culprit’s identity, the particular crime vessel. It was only when the informer pointed to him as the carrier of the marijuana that he
he allegedly committed and his exact whereabouts could have been a basis of probable cause for suddenly became suspect and so subject to apprehension. It was the furtive finger that triggered
the lawmen to secure a warrant. This Court held that in accordance with Administrative Circular his arrest. The identification by the informer was the probable cause as determined by the
No. 13 and Circular No. 19, series of 1987, the lawmen could have applied for a warrant even officers (and not a judge) that authorized them to pounce upon Aminnudin and immediately
after court hours. The failure or neglect to secure one cannot serve as an excuse for violating arrest him.
Encinada’s constitutional right. In the absence of probable cause to effect a valid and legal warrantless arrest, the search and
seizure of accused-appellant’s bag would also not be justified as seizure of evidence in “plain
view” under the second exception. The marijuana was obviously not immediately apparent as
105

shown by the fact that the NARCOM agents still had to request accused-appellant to open the [T]he Republic’s counsel avers that appellant voluntarily handed the chairs containing the package of
bag to ascertain its contents. marijuana to the arresting officer and thus effectively waived his right against the warrantless search. This he
Neither would the search and seizure of accused-appellant’s bag be justified as a search of a gleaned from Bolonia’s testimony.
Q: After Roel Encinada alighted from the motor tricycle, what happened next?
moving vehicle. There was no moving vehicle to speak of in the instant case as accused-appellant
A: I requested to him to see his chairs that he carried.
was apprehended several minutes after alighting from the Victory Liner bus. In fact, she was Q: Are you referring to the two plastic chairs?
accosted in the middle of the street and not while inside the vehicle. A: Yes, sir.
People v. Solayao, 20 applied the stop and frisk principle which has been adopted in Posadas Q: By the way, when Roel Encinada agreed to allow you to examine the two chairs that he carried, what did
v. Court of Appeals. 21 In said case, Solayao attempted to flee when he and his companions were you do next?
accosted by government agents. In the instant case, there was no observable manifestation that A: I examined the chairs and I noticed that something inside in between the two chairs.
could have aroused the suspicion of the NARCOM agents as to cause them to “stop and frisk” We are not convinced. While in principle we agree that consent will validate an otherwise illegal search, we
accused-appellant. To reiterate, accused-appellant was merely crossing the street when believe that appellant — based on the transcript quoted above — did not voluntarily consent to Bolonia’s
search of his belongings. Appellant’s silence should not be lightly taken as consent to such search. The implied
apprehended. Unlike in the abovementioned cases, accused-appellant never attempted to flee
acquiescence to the search, if there was any, could not have been more than mere passive conformity given
from the NARCOM agents when the latter identified themselves as such. Clearly, this is another under intimidating or coercive circumstances and is thus considered no consent at all within the purview of
indication of the paucity of probable cause that would sufficiently provoke a suspicion that the constitutional guarantee. Furthermore, considering that the search was conducted irregularly, i.e.,
accused-appellant was committing a crime. without a warrant, we cannot appreciate consent based merely on the presumption of regularity of the
The warrantless search and seizure could not likewise be categorized under exigent and performance of duty.” (Emphasis supplied)
emergency circumstances, as applied in People v. De Gracia. 22 In said case, there were Thus, accused-appellant’s lack of objection to the search is not tantamount to a waiver of her
intelligence reports that the building was being used as headquarters by the RAM during a coup constitutional rights or a voluntary submission to the warrantless search. As this Court held
d’ etat. A surveillance team was fired at by a group of armed men coming out of the building and in People v. Barros: 27
the occupants of said building refused to open the door despite repeated requests. There were . . . [T]he accused is not to be presumed to have waived the unlawful search conducted on the
large quantities of explosives and ammunitions inside the building. Nearby courts were closed occasion of his warrantless arrest “simply because he failed to object” —
and general chaos and disorder prevailed. The existing circumstances sufficiently showed that a . . . To constitute a waiver, it must appear first that the right exists; secondly, that the person
crime was being committed. In short, there was probable cause to effect a warrantless search of involved had knowledge, actual or constructive, of the existence of such right; and lastly, that
the building. The same could not be said in the instant case. said person had an actual intention to relinquish the right (Pasion Vda. de Garcia v. Locsin, 65
The only other exception that could possibly legitimize the warrantless search and seizure would Phil. 698). The fact that the accused failed to object to the entry into his house does not amount
be consent given by the accused-appellant to the warrantless search as to amount to a waiver of to a permission to make a search therein (Magoncia v. Palacio, 80 Phil. 770). As pointed out by
her constitutional right. The Solicitor General argues that accused-appellant voluntarily Justice Laurel in the case of Pasion Vda. de Garcia v. Locsin (supra):
submitted herself to search and inspection citing People v. Malasugui 23 where this Court ruled: xxx xxx xxx
When one voluntarily submits to a search or consents to have it made on his person or premises, . . . As the constitutional guaranty is not dependent upon any affirmative act of the citizen, the
he is precluded from complaining later thereof. (Cooley, Constitutional Limitations, 8th ed., [V]ol. courts do not place the citizen in the position of either contesting an officer’s authority by force,
I, p. 631.) The right to be secure from unreasonable search may, like every right, be waived and or waiving his constitutional rights; but instead they hold that a peaceful submission to a search
such waiver may be made either expressly or impliedly. or seizure is not a consent or an invitation thereto, but is merely a demonstration of regard for
In support of said argument, the Solicitor General cited the testimony of Lt. Abello, thus: the supremacy of the law. (Citation omitted).
Q When this informant by the name of alias Benjie pointed to Aling Rosa, what happened after that? We apply the rule that: “courts indulge every reasonable presumption against waiver of
A We followed her and introduced ourselves as NARCOM agents and confronted her with our informant and fundamental constitutional rights and that we do not presume acquiescence in the loss of
asked her what she was carrying and if we can see the bag she was carrying. fundamental rights.” 28(Emphasis supplied)
Q What was her reaction?
To repeat, to constitute a waiver, there should be an actual intention to relinquish the right. As
A She gave her bag to me.
Q So what happened after she gave the bag to you? clearly illustrated in People v. Omaweng, 29 where prosecution witness Joseph Layong testified
A I opened it and found out plastic bags of marijuana inside. 24 thus:
This Court cannot agree with the Solicitor General’s contention for the Malasugui case is inapplicable to the PROSECUTOR AYOCHOK:
instant case. In said case, there was probable cause for the warrantless arrest thereby making the Q — When you and David Fomocod saw the travelling bag, what did you do?
warrantless search effected immediately thereafter equally lawful. 25 On the contrary, the most essential A — When we saw that traveling bag, we asked the driver if we could see the contents.
element of probable cause, as expounded above in detail, is wanting in the instant case making the Q — And what did or what was the reply of the driver, if there was any?
warrantless arrest unjustified and illegal. Accordingly, the search which accompanied the warrantless arrest A — He said “you can see the contents but those are only clothings” (sic).
was likewise unjustified and illegal. Thus, all the articles seized from the accused-appellant could not be used Q — When he said that, what did you do?
as evidence against her. A — We asked him if we could open and see it.
Aside from the inapplicability of the abovecited case, the act of herein accused-appellant in handing over her Q — When you said that, what did he tell you?
bag to the NARCOM agents could not be construed as voluntary submission or an implied acquiescence to A — He said “you can see it”.
the unreasonable search. The instant case is similar to People v. Encinada, 26 where this Court held: Q — And when he said “you can see and open it,” what did you do?
106

A — When I went inside and opened the bag, I saw that it was not clothings (sic) that was contained in the 2. Granting that evidence obtained through a warrantless search becomes admissible upon
bag. failure to object thereto during the trial of the case, records show that accused-appellant filed a
Q — And when you saw that it was not clothings (sic), what did you do? Demurrer to Evidence and objected and opposed the prosecution’s Formal Offer of Evidence.
A — When I saw that the contents were not clothes, I took some of the contents and showed it to my
It is apropos to quote the case of People v. Barros, 33 which stated:
companion Fomocod and when Fomocod smelled it, he said it was marijuana. (Emphasis supplied)
It might be supposed that the non-admissibility of evidence secured through an invalid
In the above-mentioned case, accused was not subjected to any search which may be
warrantless arrest or a warrantless search and seizure may be waived by an accused person.
stigmatized as a violation of his Constitutional right against unreasonable searches and seizures.
The a priori argument is that the invalidity of an unjustified warrantless arrest, or an arrest
If one had been made, this Court would be the first to condemn it “as the protection of the
effected with a defective warrant of arrest may be waived by applying for and posting of bail for
citizen and the maintenance of his constitutional rights is one of the highest duties and privileges
provisional liberty, so as to estop an accused from questioning the legality or constitutionality of
of the Court.” He willingly gave prior consent to the search and voluntarily agreed to have it
his detention or the failure to accord him a preliminary investigation. We do not believe,
conducted on his vehicle and traveling bag, which is not the case with Aruta.
however, that waiver of the latter necessarily constitutes, or carries with it, waiver of the former
In an attempt to further justify the warrantless search, the Solicitor General next argues that the
— an argument that the Solicitor General appears to be making impliedly. Waiver of the non-
police officers would have encountered difficulty in securing a search warrant as it could be
admissibility of the “fruits” of an invalid warrantless arrest and of a warrantless search and
secured only if accused-appellant’s name was known, the vehicle identified and the date of its
seizure is not casually to be presumed, if the constitutional right against unlawful searches and
arrival certain, as in the Aminnudin case where the arresting officers had forty-eight hours within
seizures is to retain its vitality for the protection of our people. In the case at bar, defense counsel
which to act.
had expressly objected on constitutional grounds to the admission of the carton box and the four
This argument is untenable.
(4) kilos of marijuana when these were formally offered in evidence by the prosecution. We
Article IV, Section 3 of the Constitution provides:
consider that appellant’s objection to the admission of such evidence was made clearly and
. . . [N]o search warrant or warrant of arrest shall issue except upon probable cause to be
seasonably and that, under the circumstances, no intent to waive his rights under the premises
determined by the judge, or such other responsible officer as may be authorized by law, after
can be reasonably inferred from his conduct before or during the trial. (Emphasis supplied).
examination under oath or affirmation of the complainant and the witnesses he may produce,
In fine, there was really no excuse for the NARCOM agents not to procure a search warrant
and particularly describing the place to be searched and the persons or things to be seized.
considering that they had more than twenty-four hours to do so. Obviously, this is again an
(Emphasis supplied)
instance of seizure of the “fruit of the poisonous tree,” hence illegal and inadmissible
Search warrants to be valid must particularly describe the place to be searched and the persons
subsequently in evidence.
or things to be seized. The purpose of this rule is to limit the things to be seized to those and
The exclusion of such evidence is the only practical means of enforcing the constitutional
only those, particularly described in the warrant so as to leave the officers of the law with no
injunction against unreasonable searches and seizure. The non-exclusionary rule is contrary to
discretion regarding what articles they shall seize to the end that unreasonable searches and
the letter and spirit of the prohibition against unreasonable searches and seizures. 34
seizures may not be made. 30
While conceding that the officer making the unlawful search and seizure may be held criminally
Had the NARCOM agents only applied for a search warrant, they could have secured one without
and civilly liable, the Stonehill case observed that most jurisdictions have realized that the
too much difficulty, contrary to the assertions of the Solicitor General. The person intended to be
exclusionary rule is “the only practical means of enforcing the constitutional injunction” against
searched has been particularized and the thing to be seized specified. The time was also
abuse. This approach is based on the justification made by Judge Learned Hand that “only in case
sufficiently ascertained to be in the afternoon of December 14, 1988. “Aling Rosa” turned out to
the prosecution which itself controls the seizing officials, knows that it cannot profit by their
be accused-appellant and the thing to be seized was marijuana. The vehicle was identified to be
wrong, will the wrong be repressed.” 35
a Victory Liner bus. In fact, the NARCOM agents purposely positioned themselves near the spot
Unreasonable searches and seizures are the menace against which the constitutional guarantees
where Victory Liner buses normally unload their passengers. Assuming that the NARCOM agents
afford full protection. While the power to search and seize may at times be necessary to the
failed to particularize the vehicle, this would not in any way hinder them from securing a search
public welfare, still it may be exercised and the law enforced without transgressing the
warrant. The above particulars would have already sufficed. In any case, this Court has held that
constitutional rights of the citizens, for the enforcement of no statute is of sufficient importance
the police should particularly describe the place to be searched and the person or things to be
to justify indifference to the basic principles of government. 36
seized, wherever and whenever it is feasible. 31 (Emphasis supplied)
Those who are supposed to enforce the law are not justified in disregarding the rights of the
While it may be argued that by entering a plea during arraignment and by actively participating
individual in the name of order. Order is too high a price to pay for the loss of liberty. As Justice
in the trial, accused-appellant may be deemed to have waived objections to the illegality of the
Holmes declared: “I think it is less evil that some criminals escape than that the government
warrantless search and to the inadmissibility of the evidence obtained thereby, the same may
should play an ignoble part.” It is simply not allowed in free society to violate a law to enforce
not apply in the instant case for the following reasons:
another, especially if the law violated is the Constitution itself. 37
1. The waiver would only apply to objections pertaining to the illegality of the arrest as her plea
WHEREFORE, in view of the foregoing, the decision of the Regional Trial Court, Branch 73,
of “not guilty” and participation in the trial are indications of her voluntary submission to the
Olongapo City, is hereby REVERSED and SET ASIDE. For lack of evidence to establish her guilt
court’s jurisdiction. 32 The plea and active participation in the trial would not cure the illegality of
beyond reasonable doubt, accused-appellant ROSA ARUTA Y MENGUIN is hereby ACQUITTED
the search and transform the inadmissible evidence into objects of proof. The waiver simply
and ordered RELEASED from confinement unless she is being held for some other legal grounds.
does not extend this far.
No costs. SO ORDERED.
107

PEOPLE VS MALMSTEDT hashish found among the personal effects of accused and the same were brought to the PC
In an information dated 15 June 1989, accused-appellant Mikael Malmstedt (hereinafter referred Crime Laboratory for chemical analysis.
to as the accused) was charged before the Regional Trial Court (RTC) of La Trinidad, Benguet, In the chemistry report, it was established that the objects examined were hashish. a prohibited
Branch 10, in Criminal Case No. 89-CR-0663, for violation of Section 4, Art. II of Republic Act drug which is a derivative of marijuana. Thus, an information was filed against accused for
6425, as amended, otherwise known as the Dangerous Drugs Act of 1972, as amended. The violation of the Dangerous Drugs Act.
factual background of the case is as follows: During the arraignment, accused entered a plea of "not guilty." For his defense, he raised the
Accused Mikael Malmstedt, a Swedish national, entered the Philippines for the third time in issue of illegal search of his personal effects. He also claimed that the hashish was planted by the
December 1988 as a tourist. He had visited the country sometime in 1982 and 1985. NARCOM officers in his pouch bag and that the two (2) travelling bags were not owned by him,
In the evening of 7 May 1989, accused left for Baguio City. Upon his arrival thereat in the but were merely entrusted to him by an Australian couple whom he met in Sagada. He further
morning of the following day, he took a bus to Sagada and stayed in that place for two (2) days. claimed that the Australian couple intended to take the same bus with him but because there
At around 7:00 o'clock in the morning of 11 May 1989, accused went to the Nangonogan bus were no more seats available in said bus, they decided to take the next ride and asked accused
stop in Sagada to catch the first available trip to Baguio City. From Baguio City, accused planned to take charge of the bags, and that they would meet each other at the Dangwa Station.
to take a late afternoon trip to Angeles City, then proceed to Manila to catch his flight out of the Likewise, accused alleged that when the NARCOM officers demanded for his passport and other
country, scheduled on 13 May 1989. From Sagada, accused took a Skyline bus with body number Identification papers, he handed to one of the officers his pouch bag which was hanging on his
8005 and Plate number AVC 902.1 neck containing, among others, his passport, return ticket to Sweden and other papers. The
At about 8: 00 o'clock in the morning of that same day (11 May 1989), Captain Alen Vasco, the officer in turn handed it to his companion who brought the bag outside the bus. When said
Commanding Officer of the First Regional Command (NARCOM) stationed at Camp Dangwa, officer came back, he charged the accused that there was hashish in the bag. He was told to get
ordered his men to set up a temporary checkpoint at Kilometer 14, Acop, Tublay, Mountain off the bus and his picture was taken with the pouch bag placed around his neck. The trial court
Province, for the purpose of checking all vehicles coming from the Cordillera Region. The order did not give credence to accused's defense.
to establish a checkpoint in the said area was prompted by persistent reports that vehicles The claim of the accused that the hashish was planted by the NARCOM officers, was belied by his
coming from Sagada were transporting marijuana and other prohibited drugs. Moreover, failure to raise such defense at the earliest opportunity. When accused was investigated at the
information was received by the Commanding Officer of NARCOM, that same morning, that a Provincial Fiscal's Office, he did not inform the Fiscal or his lawyer that the hashish was planted
Caucasian coming from Sagada had in his possession prohibited drugs.2 by the NARCOM officers in his bag. It was only two (2) months after said investigation when he
The group composed of seven (7) NARCOM officers, in coordination with Tublay Police Station, told his lawyer about said claim, denying ownership of the two (2) travelling bags as well as
set up a checkpoint at the designated area at about 10:00 o'clock in the morning and inspected having hashish in his pouch bag.
all vehicles coming from the Cordillera Region. In a decision dated 12 October 1989, the trial court found accused guilty beyond reasonable
At about 1:30 o'clock in the afternoon, the bus where accused was riding was stopped. Sgt. Fider doubt for violation of the Dangerous Drugs Act, specifically Section 4, Art. II of RA 6425, as
and CIC Galutan boarded the bus and announced that they were members of the NARCOM and amended.3 The dispositive portion of the decision reads as follows:
that they would conduct an inspection. The two (2) NARCOM officers started their inspection WHEREFORE, finding the guilt of the accused Mikael Malmstedt established beyond
from the front going towards the rear of the bus. Accused who was the sole foreigner riding the reasonable doubt, this Court finds him GUILTY of violation of Section 4, Article 11 of
bus was seated at the rear thereof. Republic Act 6425, as amended, and hereby sentences him to suffer the penalty of life
During the inspection, CIC Galutan noticed a bulge on accused's waist. Suspecting the bulge on imprisonment and to pay a fine of Twenty Thousand Pesos (P20,000.00), with
accused's waist to be a gun, the officer asked for accused's passport and other identification subsidiary imprisonment in case of insolvency and to pay the costs.
papers. When accused failed to comply, the officer required him to bring out whatever it was Let the hashish subject of this case be turned over to the First Narcotics Regional Unit
that was bulging on his waist. The bulging object turned out to be a pouch bag and when at Camp Bado; Dangwa, La Trinidad Benguet for proper disposition under Section 20,
accused opened the same bag, as ordered, the officer noticed four (4) suspicious-looking objects Article IV of Republic Act 6425, as amended.
wrapped in brown packing tape, prompting the officer to open one of the wrapped objects. The SO ORDERED.4
wrapped objects turned out to contain hashish, a derivative of marijuana. Seeking the reversal of the decision of the trial court finding him guilty of the crime charged,
Thereafter, accused was invited outside the bus for questioning. But before he alighted from the accused argues that the search of his personal effects was illegal because it was made without a
bus, accused stopped to get two (2) travelling bags from the luggage carrier. search warrant and, therefore, the prohibited drugs which were discovered during the illegal
Upon stepping out of the bus, the officers got the bags and opened them. A teddy bear was search are not admissible as evidence against him.
found in each bag. Feeling the teddy bears, the officer noticed that there were bulges inside the The Constitution guarantees the right of the people to be secure in their persons, houses, papers
same which did not feel like foam stuffing. It was only after the officers had opened the bags that and effects against unreasonable searches and seizures.5 However, where the search is made
accused finally presented his passport. pursuant to a lawful arrest, there is no need to obtain a search warrant. A lawful arrest without a
Accused was then brought to the headquarters of the NARCOM at Camp Dangwa, La Trinidad, warrant may be made by a peace officer or a private person under the following circumstances.6
Benguet for further investigation. At the investigation room, the officers opened the teddy bears Sec. 5 Arrest without warrant; when lawful. –– A peace officer or a private person may,
and they were found to also contain hashish. Representative samples were taken from the without a warrant, arrest a person:
108

(a) When, in his presence, the person to be arrested has committed is actually for an innocent man, who has nothing to hide from the authorities, to readily present his
committing, or is attempting to commit an offense; identification papers when required to do so?
(b) When an offense has in fact just been committed, and he has personal knowledge The receipt of information by NARCOM that a Caucasian coming from Sagada had prohibited
of facts indicating that the person to be arrested has committed it; and drugs in his possession, plus the suspicious failure of the accused to produce his passport, taken
(c) When the person to be arrested is a prisoner who has escaped from a penal together as a whole, led the NARCOM officers to reasonably believe that the accused was trying
establishment or place where he is serving final judgment or temporarily confined to hide something illegal from the authorities. From these circumstances arose a probable
while his case is pending, or has escaped while being transferred from one cause which justified the warrantless search that was made on the personal effects of the
confinement to another. accused. In other words, the acts of the NARCOM officers in requiring the accused to open his
In cases falling under paragraphs (a) and (b) hereof, the person arrested without a pouch bag and in opening one of the wrapped objects found inside said bag (which was
warrant shall be forthwith delivered to the nearest police station or jail, and he shall be discovered to contain hashish) as well as the two (2) travelling bags containing two (2) teddy
proceeded against in accordance with Rule 112, Section 7. (6a 17a). bears with hashish stuffed inside them, were prompted by accused's own attempt to hide his
Accused was searched and arrested while transporting prohibited drugs (hashish). A crime was identity by refusing to present his passport, and by the information received by the NARCOM
actually being committed by the accused and he was caught in flagrante delicto. Thus, the search that a Caucasian coming from Sagada had prohibited drugs in his possession. To deprive the
made upon his personal effects falls squarely under paragraph (1) of the foregoing provisions of NARCOM agents of the ability and facility to act accordingly, including, to search even without
law, which allow a warrantless search incident to a lawful arrest.7 warrant, in the light of such circumstances, would be to sanction impotence and ineffectiveness
While it is true that the NARCOM officers were not armed with a search warrant when the in law enforcement, to the detriment of society.
search was made over the personal effects of accused, however, under the circumstances of the WHEREFORE, premises considered, the appealed judgment of conviction by the trial court is
case, there was sufficient probable cause for said officers to believe that accused was then and hereby AFFIRMED. Costs against the accused-appellant.
there committing a crime. SO ORDERED.
Probable cause has been defined as such facts and circumstances which could lead a reasonable,
discreet and prudent man to believe that an offense has been committed, and that the objects
sought in connection with the offense are in the place sought to be searched.8 The required
probable cause that will justify a warrantless search and seizure is not determined by any fixed
formula but is resolved according to the facts of each case.9
Warrantless search of the personal effects of an accused has been declared by this Court as valid,
because of existence of probable cause, where the smell of marijuana emanated from a plastic
bag owned by the accused,10or where the accused was acting suspiciously,11 and attempted to
flee.12
Aside from the persistent reports received by the NARCOM that vehicles coming from Sagada
were transporting marijuana and other prohibited drugs, their Commanding Officer also received
information that a Caucasian coming from Sagada on that particular day had prohibited drugs in
his possession. Said information was received by the Commanding Officer of NARCOM the very
same morning that accused came down by bus from Sagada on his way to Baguio City.
When NARCOM received the information, a few hours before the apprehension of herein
accused, that a Caucasian travelling from Sagada to Baguio City was carrying with him prohibited
drugs, there was no time to obtain a search warrant. In the Tangliben case,13 the police
authorities conducted a surveillance at the Victory Liner Terminal located at Bgy. San Nicolas,
San Fernando Pampanga, against persons engaged in the traffic of dangerous drugs, based on
information supplied by some informers. Accused Tangliben who was acting suspiciously and
pointed out by an informer was apprehended and searched by the police authorities. It was held
that when faced with on-the-spot information, the police officers had to act quickly and there
was no time to secure a search warrant.
It must be observed that, at first, the NARCOM officers merely conducted a routine check of the
bus (where accused was riding) and the passengers therein, and no extensive search was initially
made. It was only when one of the officers noticed a bulge on the waist of accused, during the
course of the inspection, that accused was required to present his passport. The failure of
accused to present his identification papers, when ordered to do so, only managed to arouse the
suspicion of the officer that accused was trying to hide his identity. For is it not a regular norm
109

PAPA VS MAGO examining the goods, and a writ of mandamus for the return of the goods and the trucks, as well
This is an original action for prohibition and certiorari, with preliminary injunction filed by as a judgment for actual, moral and exemplary damages in their favor.
Ricardo Papa, Chief of Police of Manila; Juan once Enrile, Commissioner of Customs; Pedro Pacis, On November 10, 1966, respondent Judge Hilarion Jarencio issued an order ex
Collector of Customs of the Port of Manila; and Martin Alagao, a patrolman of the Manila Police parte restraining the respondents in Civil Case No. 67496 — now petitioners in the instant case
Department, against Remedios Mago and Hon. Hilarion Jarencio, Presiding Judge of Branch 23 of before this Court — from opening the nine bales in question, and at the same time set the
the Court of First Instance of Manila, praying for the annulment of the order issued by hearing of the petition for preliminary injunction on November 16, 1966. However, when the
respondent Judge in Civil Case No. 67496 of the Court of First Instance of Manila under date of restraining order was received by herein petitioners, some bales had already been opened by
March 7, 1967, which authorized the release under bond of certain goods which were seized and the examiners of the Bureau of Customs in the presence of officials of the Manila Police
held by petitioners in connection with the enforcement of the Tariff and Customs Code, but Department, an assistant city fiscal and a representative of herein respondent Remedios Mago.
which were claimed by respondent Remedios Mago, and to prohibit respondent Judge from Under date of November 15, 1966, Remedios Mago filed an amended petition in Civil Case
further proceeding in any manner whatsoever in said Civil Case No. 67496. Pending the No. 67496, including as party defendants Collector of Customs Pedro Pacis of the Port of Manila
determination of this case this Court issued a writ of preliminary injunction restraining the and Lt. Martin Alagao of the Manila Police Department. Herein petitioners (defendants below)
respondent Judge from executing, enforcing and/or implementing the questioned order in Civil filed, on November 24, 1966, their "Answer with Opposition to the Issuance of a Writ of
Case No. 67496 and from proceeding with said case. Preliminary Injunction", denying the alleged illegality of the seizure and detention of the goods
Petitioner Martin Alagao, head of the counter-intelligence unit of the Manila Police and the trucks and of their other actuations, and alleging special and affirmative defenses, to
Department, acting upon a reliable information received on November 3, 1966 to the effect that wit: that the Court of First Instance of Manila had no jurisdiction to try the case; that the case fell
a certain shipment of personal effects, allegedly misdeclared and undervalued, would be within the exclusive jurisdiction of the Court of Tax Appeals; that, assuming that the court had
released the following day from the customs zone of the port of Manila and loaded on two jurisdiction over the case, the petition stated no cause of action in view of the failure of
trucks, and upon orders of petitioner Ricardo Papa, Chief of Police of Manila and a duly Remedios Mago to exhaust the administrative remedies provided for in the Tariff and Customs
deputized agent of the Bureau of Customs, conducted surveillance at gate No. 1 of the customs Code; that the Bureau of Customs had not lost jurisdiction over the goods because the full duties
zone. When the trucks left gate No. 1 at about 4:30 in the afternoon of November 4, 1966, and charges thereon had not been paid; that the members of the Manila Police Department had
elements of the counter-intelligence unit went after the trucks and intercepted them at the the power to make the seizure; that the seizure was not unreasonable; and the persons
Agrifina Circle, Ermita, Manila. The load of the two trucks consisting of nine bales of goods, and deputized under Section 2203 (c) of the Tariff and Customs Code could effect search, seizures
the two trucks, were seized on instructions of the Chief of Police. Upon investigation, a person and arrests in inland places in connection with the enforcement of the said Code. In opposing the
claimed ownership of the goods and showed to the policemen a "Statement and Receipts of issuance of the writ of preliminary injunction, herein petitioners averred in the court below that
Duties Collected in Informal Entry No. 147-5501", issued by the Bureau of Customs in the name the writ could not be granted for the reason that Remedios Mago was not entitled to the main
of a certain Bienvenido Naguit. reliefs she prayed for; that the release of the goods, which were subject to seizure proceedings
Claiming to have been prejudiced by the seizure and detention of the two trucks and their under the Tariff and Customs Code, would deprive the Bureau of Customs of the authority to
cargo, Remedios Mago and Valentin B. Lanopa filed with the Court of First Instance of Manila a forfeit them; and that Remedios Mago and Valentin Lanopa would not suffer irreparable injury.
petition "for mandamus with restraining order or preliminary injunction, docketed as Civil Case Herein petitioners prayed the court below for the lifting of the restraining order, for the denial of
No. 67496, alleging, among others, that Remedios Mago was the owner of the goods seized, the issuance of the writ of preliminary injunction, and for the dismissal of the case.
having purchased them from the Sta. Monica Grocery in San Fernando, Pampanga; that she hired At the hearing on December 9, 1966, the lower Court, with the conformity of the parties,
the trucks owned by Valentin Lanopa to transport, the goods from said place to her residence at ordered that an inventory of the goods be made by its clerk of court in the presence of the
1657 Laon Laan St., Sampaloc, Manila; that the goods were seized by members of the Manila representatives of the claimant of the goods, the Bureau of Customs, and the Anti-Smuggling
Police Department without search warrant issued by a competent court; that anila Chief of Center of the Manila Police Department. On December 13, 1966, the above-named persons filed
Police Ricardo Papa denied the request of counsel for Remedios Mago that the bales be not a "Compliance" itemizing the contents of the nine bales.
opened and the goods contained therein be not examined; that then Customs Commissioner Herein respondent Remedios Mago, on December 23, 1966, filed an ex parte motion to
Jacinto Gavino had illegally assigned appraisers to examine the goods because the goods were release the goods, alleging that since the inventory of the goods seized did not show any article
no longer under the control and supervision of the Commissioner of Customs; that the goods, of prohibited importation, the same should be released as per agreement of the patties upon her
even assuming them to have been misdeclared and, undervalued, were not subject to seizure posting of the appropriate bond that may be determined by the court. Herein petitioners filed
under Section 2531 of the Tariff and Customs Code because Remedios Mago had bought them their opposition to the motion, alleging that the court had no jurisdiction to order the release of
from another person without knowledge that they were imported illegally; that the bales had the goods in view of the fact that the court had no jurisdiction over the case, and that most of
not yet been opened, although Chief of Police Papa had arranged with the Commissioner of the goods, as shown in the inventory, were not declared and were, therefore, subject to
Customs regarding the disposition of the goods, and that unless restrained their constitutional forfeiture. A supplemental opposition was filed by herein petitioners on January 19, 1967,
rights would be violated and they would truly suffer irreparable injury. Hence, Remedios Mago alleging that on January 12, 1967 seizure proceedings against the goods had been instituted by
and Valentin Lanopa prayed for the issuance of a restraining order, ex parte, enjoining the the Collector of Customs of the Port of Manila, and the determination of all questions affecting
above-named police and customs authorities, or their agents, from opening the bales and the disposal of property proceeded against in seizure and forfeiture proceedings should thereby
be left to the Collector of Customs. On January 30, 1967, herein petitioners filed a manifestation
110

that the estimated duties, taxes and other charges due on the goods amounted to P95,772.00. question were imported from Hongkong, as shown in the "Statement and Receipts of Duties
On February 2, 1967, herein respondent Remedios Mago filed an urgent manifestation and Collected on Informal Entry". 2 As long as the importation has not been terminated the imported
reiteration of the motion for the release under bond of the goods. goods remain under the jurisdiction of the Bureau of customs. Importation is deemed
On March 7, 1967, the respondent Judge issued an order releasing the goods to herein terminated only upon the payment of the duties, taxes and other charges upon the articles, or
respondent Remedios Mago upon her filing of a bond in the amount of P40,000.00, and on secured to be paid, at the port of entry and the legal permit for withdrawal shall have been
March 13, 1967, said respondent filed the corresponding bond. granted. 3 The payment of the duties, taxes, fees and other charges must be in full. 4
On March 13, 1967, herein petitioner Ricardo Papa, on his own behalf, filed a motion for The record shows, by comparing the articles and duties stated in the aforesaid "Statement
reconsideration of the order of the court releasing the goods under bond, upon the ground that and Receipts of Duties Collected on Informal Entry" with the manifestation of the Office of the
the Manila Police Department had been directed by the Collector of Customs of the Port of Solicitor General 5 wherein it is stated that the estimated duties, taxes and other charges on the
Manila to hold the goods pending termination of the seizure proceedings. goods subject of this case amounted to P95,772.00 as evidenced by the report of the appraiser
Without waiting for the court's action on the motion for reconsideration, and alleging that of the Bureau of Customs, that the duties, taxes and other charges had not been paid in full.
they had no plain, speedy and adequate remedy in the ordinary course of law, herein petitioners Furthermore, a comparison of the goods on which duties had been assessed, as shown in the
filed the present action for prohibition and certiorari with preliminary injunction before this "Statement and Receipts of Duties Collected on Informal Entry" and the "compliance" itemizing
Court. In their petition petitioners alleged, among others, that the respondent Judge acted the articles found in the bales upon examination and inventory, 6 shows that the quantity of the
without jurisdiction in ordering the release to respondent Remedios Mago of the disputed goods was underdeclared, presumably to avoid the payment of duties thereon. For example,
goods, for the following reasons: (1) the Court of First Instance of Manila, presided by Annex B (the statement and receipts of duties collected) states that there were 40 pieces of
respondent Judge, had no jurisdiction over the case; (2) respondent Remedios Mago had no ladies' sweaters, whereas Annex H (the inventory contained in the "compliance") states that in
cause of action in Civil Case No. 67496 of the Court of First Instance of Manila due to her failure bale No. 1 alone there were 42 dozens and 1 piece of ladies' sweaters of assorted colors; in
to exhaust all administrative remedies before invoking judicial intervention; (3) the Government Annex B, only 100 pieces of watch bands were assessed, but in Annex H, there were in bale No.
was not estopped by the negligent and/or illegal acts of its agent in not collecting the correct 2, 209 dozens and 5 pieces of men's metal watch bands (white) and 120 dozens of men's metal
taxes; and (4) the bond fixed by respondent Judge for the release of the goods was grossly watch band (gold color), and in bale No. 7, 320 dozens of men's metal watch bands (gold color);
insufficient. in Annex B, 20 dozens only of men's handkerchief were declared, but in Annex H it appears that
In due time, the respondents filed their answer to the petition for prohibition there were 224 dozens of said goods in bale No. 2, 120 dozens in bale No. 6, 380 dozens in bale
and certiorari in this case. In their answer, respondents alleged, among others: (1) that it was No. 7, 220 dozens in bale No. 8, and another 200 dozens in bale No. 9. The articles contained in
within the jurisdiction of the lower court presided by respondent Judge to hear and decide Civil the nine bales in question, were, therefore, subject to forfeiture under Section 2530, pars. e and
Case No. 67496 and to issue the questioned order of March 7, 1967, because said Civil Case No. m, (1), (3), (4), and (5) of the Tariff and Customs Code. And this Court has held that merchandise,
67496 was instituted long before seizure, and identification proceedings against the nine bales of the importation of which is effected contrary to law, is subject to forfeiture, 7 and that goods
goods in question were instituted by the Collector of Customs; (2) that petitioners could no released contrary to law are subject to seizure and forfeiture. 8
longer go after the goods in question after the corresponding duties and taxes had been paid Even if it be granted, arguendo, that after the goods in question had been brought out of
and said goods had left the customs premises and were no longer within the control of the the customs area the Bureau of Customs had lost jurisdiction over the same, nevertheless, when
Bureau of Customs; (3) that respondent Remedios Mago was purchaser in good faith of the said goods were intercepted at the Agrifina Circle on November 4, 1966 by members of the
goods in question so that those goods can not be the subject of seizure and forfeiture Manila Police Department, acting under directions and orders of their Chief, Ricardo C. Papa,
proceedings; (4) that the seizure of the goods was affected by members of the Manila Police who had been formally deputized by the Commissioner of Customs, 9 the Bureau of Customs had
Department at a place outside control of jurisdiction of the Bureau of Customs and affected regained jurisdiction and custody of the goods. Section 1206 of the Tariff and Customs Code
without any search warrant or a warrant of seizure and detention; (5) that the warrant of seizure imposes upon the Collector of Customs the duty to hold possession of all imported articles upon
and detention subsequently issued by the Collector of Customs is illegal and unconstitutional, it which duties, taxes, and other charges have not been paid or secured to be paid, and to dispose
not being issued by a judge; (6) that the seizing officers have no authority to seize the goods in of the same according to law. The goods in question, therefore, were under the custody and at
question because they are not articles of prohibited importation; (7) that petitioners are the disposal of the Bureau of Customs at the time the petition for mandamus, docketed as Civil
estopped to institute the present action because they had agreed before the respondent Judge Case No. 67496, was filed in the Court of First Instance of Manila on November 9, 1966. The
that they would not interpose any objection to the release of the goods under bond to answer Court of First Instance of Manila, therefore, could not exercise jurisdiction over said goods even
for whatever duties and taxes the said goods may still be liable; and (8) that the bond for the if the warrant of seizure and detention of the goods for the purposes of the seizure and
release of the goods was sufficient. forfeiture proceedings had not yet been issued by the Collector of Customs.
The principal issue in the instant case is whether or not, the respondent Judge had acted The ruling in the case of "Alberto de Joya, et al. v. Hon. Gregorio Lantin, et al.," G.R. No. L-
with jurisdiction in issuing the order of March 7, 1967 releasing the goods in question. 24037, decided by this Court on April 27, 1967, is squarely applicable to the instant case. In
The Bureau of Customs has the duties, powers and jurisdiction, among others, (1) to assess the De Joya case, it appears that Francindy Commercial of Manila bought from Ernerose
and collect all lawful revenues from imported articles, and all other dues, fees, charges, fines and Commercial of Cebu City 90 bales of assorted textiles and rags, valued at P117,731.00, which had
penalties, accruing under the tariff and customs laws; (2) to prevent and suppress smuggling and been imported and entered thru the port of Cebu. Ernerose Commercial shipped the goods to
other frauds upon the customs; and (3) to enforce tariff and customs laws. 1 The goods in Manila on board an inter-island vessel. When the goods where about to leave the customs
111

premises in Manila, on October 6, 1964, the customs authorities held them for further frauds on customs; and to enforce tariff and customs laws (Sec. 602, Republic Act
verification, and upon examination the goods were found to be different from the declaration in 1957).
the cargo manifest of the carrying vessel. Francindy Commercial subsequently demanded from The goods in question are imported articles entered at the Port of Cebu. Should
the customs authorities the release of the goods, asserting that it is a purchaser in good faith of they be found to have been released irregularly from Customs custody in Cebu City,
those goods; that a local purchaser was involved so the Bureau of Customs had no right to they are subject to seizure and forfeiture, the proceedings for which comes within the
examine the goods; and that the goods came from a coastwise port. On October 26, 1964, jurisdiction of the Bureau of Customs pursuant to Republic Act 1937.
Francindy Commercial filed in the Court of First Instance of Manila a petition Said proceeding should be followed; the owner of the goods may set up defenses
for mandamus against the Commissioner of Customs and the Collector of Customs of the port of therein (Pacis v. Averia, L-22526, Nov. 20, 1966.) From the decision of the
Manila to compel said customs authorities to release the goods. Commissioner of Customs appeal lies to the Court of Tax Appeals, as provided in Sec.
Francindy Commercial alleged in its petition for mandamus that the Bureau of Customs had 2402 of Republic Act 1937 and Sec. 11 of Republic Act, 1125. To permit recourse to the
no jurisdiction over the goods because the same were not imported to the port of Manila; that it Court of First Instance in cases of seizure of imported goods would in effect render
was not liable for duties and taxes because the transaction was not an original importation; that ineffective the power of the Customs authorities under the Tariff and Customs Code
the goods were not in the hands of the importer nor subject to importer's control, nor were the and deprive the Court of Tax Appeals of one of its exclusive appellate jurisdictions. As
goods imported contrary to law with its (Francindy Commercial's) knowledge; and that the this Court has ruled in Pacis v. Averia,supra, Republic Acts 1937 and 1125 vest
importation had been terminated. On November 12, 1964, the Collector of Customs of Manila jurisdiction over seizure and forfeiture proceedings exclusively upon the Bureau of
issued a warrant of seizure and identification against the goods. On December 3, 1964, the Customs and the Court of Tax Appeals. Such law being special in nature, while the
Commissioner of Customs and the Collector of Customs, as respondents in the mandamus case, Judiciary Act defining the jurisdiction of Courts of First Instance is a general legislation,
filed a motion to dismiss the petition on the grounds of lack of jurisdiction, lack of cause of not to mention that the former are later enactments, the Court of First Instance should
action, and in view of the pending seizure and forfeiture proceedings. The Court of First Instance yield to the jurisdiction of the Customs authorities.
held resolution on the motion to dismiss in abeyance pending decision on the merits. On It is the settled rule, therefore, that the Bureau of Customs acquires exclusive jurisdiction
December 14, 1964, the Court of First Instance of Manila issued a preventive and mandatory over imported goods, for the purposes of enforcement of the customs laws, from the moment
injunction, on prayer by Francindy Commercial, upon a bond of P20,000.00. The Commissioner the goods are actually in its possession or control, even if no warrant of seizure or detention had
of Customs and the Collector of Customs sought the lifting of the preliminary and mandatory previously been issued by the Collector of Customs in connection with seizure and forfeiture
injunction, and the resolution of their motion to dismiss. The Court of First Instance of Manila, proceedings. In the present case, the Bureau of Customs actually seized the goods in question on
however, on January 12, 1965, ordered them to comply with the preliminary and mandatory November 4, 1966, and so from that date the Bureau of Customs acquired jurisdiction over the
injunction, upon the filing by Francindy Commercial of an additional bond of P50,000.00. Said goods for the purposes of the enforcement of the tariff and customs laws, to the exclusion of the
customs authorities thereupon filed with this Court, on January 14, 1965, a petition regular courts. Much less then would the Court of First Instance of Manila have jurisdiction over
for certiorari and prohibition with preliminary injunction. In resolving the question raised in that the goods in question after the Collector of Customs had issued the warrant of seizure and
case, this Court held: detention on January 12, 1967. 10 And so, it cannot be said, as respondents contend, that the
This petition raises two related issues: first, has the Customs bureau jurisdiction issuance of said warrant was only an attempt to divest the respondent Judge of jurisdiction over
to seize the goods and institute forfeiture proceedings against them? and (2) has the the subject matter of the case. The court presided by respondent Judge did not acquire
Court of First Instance jurisdiction to entertain the petition for mandamus to compel jurisdiction over the goods in question when the petition for mandamus was filed before it, and
the Customs authorities to release the goods? so there was no need of divesting it of jurisdiction. Not having acquired jurisdiction over the
Francindy Commercial contends that since the petition in the Court of first goods, it follows that the Court of First Instance of Manila had no jurisdiction to issue the
Instance was filed (on October 26, 1964) ahead of the issuance of the Customs warrant questioned order of March 7, 1967 releasing said goods.
of seizure and forfeiture (on November 12, 1964),the Customs bureau should yield the Respondents also aver that petitioner Martin Alagao, an officer of the Manila Police
jurisdiction of the said court. Department, could not seize the goods in question without a search warrant. This contention
The record shows, however, that the goods in question were actually seized on cannot be sustained. The Chief of the Manila Police Department, Ricardo G. Papa, having been
October 6, 1964, i.e., before Francindy Commercial sued in court. The purpose of the deputized in writing by the Commissioner of Customs, could, for the purposes of the
seizure by the Customs bureau was to verify whether or not Custom duties and taxes enforcement of the customs and tariff laws, effect searches, seizures, and arrests,11 and it was
were paid for their importation. Hence, on December 23, 1964, Customs released 22 his duty to make seizure, among others, of any cargo, articles or other movable property when
bales thereof, for the same were found to have been released regularly from the Cebu the same may be subject to forfeiture or liable for any fine imposed under customs and tariff
Port (Petition Annex "L"). As to goods imported illegally or released irregularly from laws. 12 He could lawfully open and examine any box, trunk, envelope or other container
Customs custody, these are subject to seizure under Section 2530 m. of the Tariff and wherever found when he had reasonable cause to suspect the presence therein of dutiable
Customs Code (RA 1957). articles introduced into the Philippines contrary to law; and likewise to stop, search and examine
The Bureau of Customs has jurisdiction and power, among others to collect any vehicle, beast or person reasonably suspected of holding or conveying such article as
revenues from imported articles, fines and penalties and suppress smuggling and other aforesaid. 13 It cannot be doubted, therefore, that petitioner Ricardo G. Papa, Chief of Police of
Manila, could lawfully effect the search and seizure of the goods in question. The Tariff and
112

Customs Code authorizes him to demand assistance of any police officer to effect said search original petition, and amended petition, in the court below Remedios Mago and Valentin Lanopa
and seizure, and the latter has the legal duty to render said assistance. 14 This was what did not even allege that there was a search. 18All that they complained of was,
happened precisely in the case of Lt. Martin Alagao who, with his unit, made the search and That while the trucks were on their way, they were intercepted without any
seizure of the two trucks loaded with the nine bales of goods in question at the Agrifina Circle. search warrant near the Agrifina Circle and taken to the Manila Police Department,
He was given authority by the Chief of Police to make the interception of the cargo. 15 where they were detained.
Petitioner Martin Alagao and his companion policemen had authority to effect the seizure But even if there was a search, there is still authority to the effect that no search warrant
without any search warrant issued by a competent court. The Tariff and Customs Code does not would be needed under the circumstances obtaining in the instant case. Thus, it has been held
require said warrant in the instant case. The Code authorizes persons having police authority that:
under Section 2203 of the Tariff and Customs Code to enter, pass through or search any land, The guaranty of freedom from unreasonable searches and seizures is construed
inclosure, warehouse, store or building, not being a dwelling house; and also to inspect, search as recognizing a necessary difference between a search of a dwelling house or other
and examine any vessel or aircraft and any trunk, package, or envelope or any person on board, structure in respect of which a search warrant may readily be obtained and a search of
or to stop and search and examine any vehicle, beast or person suspected of holding or a ship, motorboat, wagon, or automobile for contraband goods, where it is not
conveying any dutiable or prohibited article introduced into the Philippines contrary to law, practicable to secure a warrant because the vehicle can be quickly moved out of the
without mentioning the need of a search warrant in said cases. 16 But in the search of a dwelling locality or jurisdiction in which the warrant must be sought. (47 Am. Jur., pp. 513-514,
house, the Code provides that said "dwelling house may be entered and searched only upon citing Carroll v. United States, 267 U.S. 132, 69 L. ed., 543, 45 S. Ct., 280, 39 A.L.R., 790;
warrant issued by a judge or justice of the peace. . . ."17 It is our considered view, therefor, that People v. Case, 320 Mich., 379, 190 N.W., 389, 27 A.L.R., 686.)
except in the case of the search of a dwelling house, persons exercising police authority under In the case of People v. Case (320 Mich., 379, 190 N.W., 389, 27 A.L.R., 686), the question
the customs law may effect search and seizure without a search warrant in the enforcement of raised by defendant's counsel was whether an automobile truck or an automobile could be
customs laws. searched without search warrant or other process and the goods therein seized used afterwards
Our conclusion finds support in the case of Carroll v. United States, 39 A.L.R., 790, 799, as evidence in a trial for violation of the prohibition laws of the State. Same counsel contended
wherein the court, considering a legal provision similar to Section 2211 of the Philippine Tariff the negative, urging the constitutional provision forbidding unreasonable searches and seizures.
and Customs Code, said as follows: The Court said:
Thus contemporaneously with the adoption of the 4th Amendment, we find in . . . Neither our state nor the Federal Constitution directly prohibits search and
the first Congress, and in the following second and fourth Congresses, a difference seizure without a warrant, as is sometimes asserted. Only "unreasonable" search and
made as to the necessity for a search warrant between goods subject to forfeiture, seizure is forbidden. . . .
when concealed in a dwelling house of similar place, and like goods in course of . . . The question whether a seizure or a search is unreasonable in the language of
transportation and concealed in a movable vessel, where readily they could be put out the Constitution is a judicial and not a legislative question; but in determining whether
of reach of a search warrant. . . . a seizure is or is not unreasonable, all of the circumstances under which it is made
Again, by the 2d section of the Act of March 3, 1815 (3 Stat. at L.231, 232, chap. must be looked to.
94), it was made lawful for customs officers not only to board and search vessels within The automobile is a swift and powerful vehicle of recent development, which has
their own and adjoining districts, but also to stop, search and examine any vehicle, multiplied by quantity production and taken possession of our highways in battalions
beast or person on which or whom they should suspect there was merchandise which until the slower, animal-drawn vehicles, with their easily noted individuality, are rare.
was subject to duty, or had been introduced into the United States in any manner Constructed as covered vehicles to standard form in immense quantities, and with a
contrary to law, whether by the person in charge of the vehicle or beast or otherwise, capacity for speed rivaling express trains, they furnish for successful commission of
and if they should find any goods, wares, or merchandise thereon, which they had crime a disguising means of silent approach and swift escape unknown in the history of
probably cause to believe had been so unlawfully brought into the country, to seize the world before their advent. The question of their police control and reasonable
and secure the same, and the vehicle or beast as well, for trial and forfeiture. This Act search on highways or other public places is a serious question far deeper and broader
was renewed April 27, 1816 (3 Sta. at L. 315, chap. 100), for a year and expired. The Act than their use in so-called "bootleging" or "rum running," which is itself is no small
of February 28, 1865, revived § 2 of the Act of 1815, above described, chap. 67, 13 matter. While a possession in the sense of private ownership, they are but a vehicle
Stat. at L. 441. The substance of this section was re-enacted in the 3d section of the Act constructed for travel and transportation on highways. Their active use is not in homes
of July 18, 1866, chap. 201, 14 Stat. at L. 178, and was thereafter embodied in the or on private premises, the privacy of which the law especially guards from search and
Revised Statutes as § 3061, Comp. Stat. § 5763, 2 Fed. Stat. Anno. 2d ed. p. 1161. seizure without process. The baffling extent to which they are successfully utilized to
Neither § 3061 nor any of its earlier counterparts has ever been attacked as facilitate commission of crime of all degrees, from those against morality, chastity, and
unconstitutional. Indeed, that section was referred to and treated as operative by this decency, to robbery, rape, burglary, and murder, is a matter of common knowledge.
court in Von Cotzhausen v. Nazro, 107 U.S. 215, 219, 27 L. ed. 540, 541, 2 Sup. Ct. Rep. Upon that problem a condition, and not a theory, confronts proper administration of
503. . . . our criminal laws. Whether search of and seizure from an automobile upon a highway
In the instant case, we note that petitioner Martin Alagao and his companion policemen or other public place without a search warrant is unreasonable is in its final analysis to
did not have to make any search before they seized the two trucks and their cargo. In their
113

be determined as a judicial question in view of all the circumstances under which it is Tia was introduced to his co-accused Lim Cheng Huat (hereinafter referred to as Lim) by another
made. confidential agent named George on August 3, 1987. Lim expressed a desire to hire a male
Having declared that the seizure by the members of the Manila Police Department of the travelling companion for his business nips abroad. Tia offered his services and was hired.
goods in question was in accordance with law and by that seizure the Bureau of Customs had Lim and Tia met anew on several occasions to make arrangements for a trip to China. In the
acquired jurisdiction over the goods for the purpose of the enforcement of the customs and course of those meetings, Tia was introduced to Peter Lo (hereinafter referred to as appellant),
tariff laws, to the exclusion of the Court of First Instance of Manila, We have thus resolved the whom Tia found out to be the person he was to accompany to China in lieu of Lim.
principal and decisive issue in the present case. We do not consider it necessary, for the As a "deep penetration agent," Tia regularly submitted reports of his undercover activities on the
purposes of this decision, to discuss the incidental issues raised by the parties in their pleadings. suspected criminal syndicate. Meanwhile, the officer-in-charge of OPLAN SHARON 887, Captain
WHEREFORE, judgment is hereby rendered, as follows: Luisito Palmera, filed with his superiors the reports submitted to him, and officially informed the
(a) Granting the writ of certiorari and prohibition prayed for by petitioners; Dangerous Drugs Board of Tia's activities.
(b) Declaring null and void, for having been issued without jurisdiction, the order of On October 4, 1987, appellant and Tia left for Hongkong on board a Philippine Airlines flight.
respondent Judge Hilarion U. Jarencio, dated March 7, 1967, in Civil Code No. 67496 of the Court Before they departed, Tia was able to telephone Captain Palmera to inform him of their
of First Instance of Manila; expected date of return to the Philippines as declared in his round-trip plane ticket-October 6,
(c) Declaring permanent the preliminary injunction issued by this Court on March 31, 1967 1987 at two o'clock in the afternoon.
restraining respondent Judge from executing, enforcing and/or implementing his order of March The day after they arrived in Hongkong, Tia and appellant boarded a train bound for Guangzhou,
7, 1967 in Civil Case No. 67496 of the Court of First Instance of Manila, and from proceeding in in the People's Republic of China. Upon arriving there, they checked in at a hotel, and rested for
any manner in said case; a few hours. The pair thereafter went to a local store where appellant purchased six (6) tin cans
(d) Ordering the dismissal of Civil Case No. 67496 of the Court of First Instance of Manila; of tea. Tia saw the paper tea bags when the cans were opened for examination during the
and1äwphï1.ñët purchase. Afterwards, they returned to the hotel. Appellant kept the cans of tea in his hotel
(e) Ordering the private respondent, Remedios Mago, to pay the costs. room. That evening, Tia went to appellant's room to talk to him. Upon entering, he saw two
It is so ordered other men with appellant. One was fixing the tea bags, while the other was burning substance on
PEOPLE VS LO HO WING a piece of aluminum foil using a cigarette lighter. Appellant joined the second man and sniffed
This case involves the unlawful transport of metamphetamine, a regulated drug under Republic the smoke emitted by the burning substance. Tia asked the latter what they would be bringing
Act No. 6425, as amended. One of its derivatives is metamphetamine hydrochloride, notoriously back to the Philippines. He was informed that their cargo consisted of Chinese drugs. Tia stayed
known in street parlance as "shabu" or "poor man's cocaine." in the room for about twenty minutes before going back to his room to sleep.
Appellant Peter Lo, together with co-accused Lim Cheng Huat alias Antonio Lim and Reynaldo The next day, October 6,1987, the two returned to Manila via a China Airlines flight. Appellant
Tia, were charged with a violation of Section 15, Article III of the aforementioned statute had with him his red traveling bag with wheels. Before departing from Guangzhou however,
otherwise known as the Dangerous Drugs Act of 1972, before Branch 114 of the Regional Trial customs examiners inspected their luggage. The tin cans of tea were brought out from the
Court of Pasay City. Only appellant and co-accused Lim Cheng Huat were convicted. They were traveling bag of appellant. The contents of the cans were not closely examined, and appellant
sentenced to suffer life imprisonment, to pay a fine of P25,000.00 each, and to pay the costs. was cleared along with Tia.
Their co-accused Reynaldo Tia was discharged as a state witness. The pertinent portion of the The plane landed at the Ninoy Aquino International Airport (NAIA), then named Manila
information reads as follows: International Airport, on schedule. Lim met the newly-arrived pair at the arrival area. Lim talked
That on or about the 6th day of October, 1987, in Pasay City, Metro Manila, and within to appellant, while Tia, upon being instructed, looked after their luggage. After Lim and appellant
the jurisdiction of this Honorable Court, the above-named accused, conspiring and finished their conversation, the latter hailed a taxicab. Appellant and Tia boarded the taxicab
confederating together and mutually helping one another, without authority of law, after putting their luggage inside the back compartment of the vehicle. Lim followed in another
did then and there willfully, unlawfully and feloniously deliver, dispatch or transport 56 taxi cab.
teabags of Metamphetamine, a regulated drug. Meanwhile, a team composed of six operatives headed by Captain Palmera was formed to act on
Contrary to law. 1 the tip given by Tia. On the expected date of arrival, the team proceeded to the NAIA. Captain
The antecedent facts of the case as found by the trial court are as follows: Palmera notified the Narcotics Command (NARCOM) Detachment at the airport for coordination.
In July 1987, the Special Operations Group, a unit of the Criminal Investigation Service (CIS) of After a briefing, the operatives were ordered to take strategic positions around the arrival area.
the Philippine Constabulary (PC), received a tip from one of its informers about an organized Two operatives stationed just outside the arrival area were the first ones to spot the suspects
group engaged in the importation of illegal drugs, smuggling of contraband goods, and emerging therefrom. Word was passed on to the other members of the team that the suspects
gunrunning. After an evaluation of the information thus received, a project codenamed "OPLAN were in sight. Appellant was pulling along his red traveling bag while Tia was carrying a shoulder
SHARON 887" was created in order to bust the suspected syndicate. bag. The operatives also spotted Lim meeting their quarry.
As part of the operations, the recruitment of confidential men and "deep penetration agents' Upon seeing appellant and Tia leave the airport, the operatives who first spotted them followed
was carried out to infiltrate the crime syndicate. One of those recruited was the discharged them. Along Imelda Avenue, the car of the operatives overtook the taxicab ridden by appellant
accused, Reynaldo Tia (hereinafter referred to as Tia). and Tia and cut into its path forcing the taxi driver to stop his vehicle. Meanwhile, the other
taxicab carrying Lim sped away in an attempt to escape. The operatives disembarked from their
114

car, approached the taxicab, and asked the driver to open the baggage compartment. Three Sandiganbayan, 3 these are: [1] a search incidental to an arrest, [2] a search of a moving vehicle,
pieces of luggage were retrieved from the back compartment of the vehicle. The operatives and [3] seizure of evidence in plain view (emphasis supplied). The circumstances of the case
requested from the suspects permission to search their luggage. A tin can of tea was taken out of clearly show that the search in question was made as regards a moving vehicle. Therefore, a
the red traveling bag owned by appellant. Sgt. Roberto Cayabyab, one of the operatives, pried valid warrant was not necessary to effect the search on appellant and his co-accused.
the lid open, pulled out a paper tea bag from the can and pressed it in the middle to feel its In this connection, We cite with approval the averment of the Solicitor General, as contained in
contents. Some crystalline white powder resembling crushed alum came out of the bag. The the appellee's brief, that the rules governing search and seizure have over the years been
sergeant then opened the tea bag and examined its contents more closely. Suspecting the steadily liberalized whenever a moving vehicle is the object of the search on the basis of
crystalline powder to be a dangerous drug, he had the three traveling bags opened for practicality. This is so considering that before a warrant could be obtained, the place, things and
inspection. From the red traveling bag, a total of six (6) tin cans were found, including the one persons to be searched must be described to the satisfaction of the issuing judge—a
previously opened. Nothing else of consequence was recovered from the other bags. Tia and requirement which borders on the impossible in the case of smuggling effected by the use of a
appellant were taken to the CIS Headquarters in Quezon City for questioning. moving vehicle that can transport contraband from one place to another with impunity. 4
Meanwhile, the second taxicab was eventually overtaken by two other operatives on Retiro We might add that a warrantless search of a moving vehicle is justified on the ground that "it is
Street, Quezon City. Lim was likewise apprehended and brought to the CIS Headquarters for not practicable to secure a warrant because the vehicle can be quickly moved out of the locality
interrogation. or jurisdiction in which the warrant must be sought." 5
During the investigation of the case, the six tin cans recovered from the traveling bag of In the instant case, it was firmly established from the factual findings of the trial court that the
appellant were opened and examined. They contained a total of fifty-six (56) paper tea bags with authorities had reasonable ground to believe that appellant would attempt to bring in
white crystalline powder inside instead of tea leaves. contraband and transport it within the country. The belief was based on intelligence reports
The tea bag opened by Sgt. Cayabyab during the search and seizure was sent to the PC-INP Crime gathered from surveillance activities on the suspected syndicate, of which appellant was touted
Laboratory for preliminary examination. Tests conducted on a sample of the crystalline powder to be a member. Aside from this, they were also certain as to the expected date and time of
inside the tea bag yielded a positive result that the specimen submitted was metamphetamine. arrival of the accused from China. But such knowledge was clearly insufficient to enable them to
Samples from each of the fifty-six (56) tea bags were similarly tested. The tests were also positive fulfill the requirements for the issuance of a search warrant. Still and all, the important thing is
for metamphetamine. Hence, the three suspects were indicted. that there was probable cause to conduct the warrantless search, which must still be present in
In rendering a judgment of conviction, the trial court gave full credence to the testimonies of the such a case.
government anti-narcotics operatives, to whom the said court applied the well-settled The second assignment of error is likewise lacking in merit. Appellant was charged and convicted
presumption of regularity in the performance of official duties. under Section 15, Article III of Republic Act No. 6425, as amended, which reads:
Appellant now assigns three errors alleged to have been committed by the trial court, namely: The penalty of life imprisonment to death and a fine ranging from twenty thousand to
I. thirty thousand pesos shall be imposed upon any person who, unless authorized by
THE TRIAL COURT ERRED IN NOT DECLARING THE SEARCH AND SEIZURE ON THE law, shall sell, dispose, deliver, transport or distribute any regulated drug (emphasis
ACCUSED AS ILLEGAL. supplied).
II. The information charged the accused of delivering, transporting or dispatching fifty-six (56) tea
THE TRIAL COURT ERRED IN FINDING THE ACCUSED GUILTY OF DELIVERING, bags containing metamphetamine, a regulated drug. The conjunction "or' was used, thereby
DISPATCHING OR TRANSPORTING METAMPHETAMINE, A REGULATED DRUG. implying that the accused were being charged of the three specified acts in the alternative.
III. Appellant argues that he cannot be convicted of "delivery" because the term connotes a source
THE TRIAL COURT ERRED IN DISCHARGING REYNALDO TIA TO TESTIFY FOR THE and a recipient, the latter being absent under the facts of the case. It is also argued that
PROSECUTION. 2 "dispatching' cannot apply either since appellant never sent off or disposed of drugs. As for
We affirm. "transporting," appellant contends that he cannot also be held liable therefor because the act of
Anent the first assignment of error, appellant contends that the warrantless search and seizure transporting necessarily requires a point of destination, which again is non- existent under the
made against the accused is illegal for being violative of Section 2, Article III of the Constitution. given facts.
He reasons that the PC-CIS officers concerned could very well have procured a search warrant The contentions are futile attempts to strain the meaning of the operative acts of which
since they had been informed of the date and time of a arrival of the accused at the NAIA well appellant and his co-accused were charged in relation to the facts of the case. There is no doubt
ahead of time, specifically two (2) days in advance. The fact that the search and seizure in that law enforcers caught appellant and his co-accused in flagrante delicto of transporting a
question were made on a moving vehicle, appellant argues, does not automatically make the prohibited drug. The term "transport" is defined as "to carry or convey from one place to
warrantless search herein fall within the coverage of the well-known exception to the rule of the another." 6 The operative words in the definition are "to carry or convey." The fact that there is
necessity of a valid warrant to effect a search because, as aforementioned, the anti-narcotics actual conveyance suffices to support a finding that the act of transporting was committed. It is
agents had both time and opportunity to secure a search warrant. immaterial whether or not the place of destination is reached. Furthermore, the argument of
The contentions are without merit. As correctly averred by appellee, that search and seizure appellant gives rise to the illogical conclusion that he and his co- accused did not intend to bring
must be supported by a valid warrant is not an absolute rule. There are at least three (3) well- the metamphetamine anywhere, i.e.they had no place of destination.
recognized exceptions thereto. As set forth in the case of Manipon, Jr. vs.
115

The situation in the instant case is one where the transport of a prohibited drug was interrupted
by the search and arrest of the accused. Interruption necessarily infers that an act had already
been commenced. Otherwise, there would be nothing to interrupt.
Therefore, considering the foregoing, since the information included the acts of delivery,
dispatch or transport, proof beyond reasonable doubt of the commission of any of the acts so
included is sufficient for conviction under Section 15, Article III of Republic Act No. 6425, as
amended.
Moreover, the act of transporting a prohibited drug is a malum prohibitum because it is punished
as an offense under a special law. It is a wrong because it is prohibited by law. Without the law
punishing the act, it cannot be considered a wrong. As such, the mere commission of said act is
what constitutes the offense punished and suffices to validly charge and convict an individual
caught committing the act so punished, regardless of criminal intent. 7
As to the third assigned error, appellant contests the discharge of accused Reynaldo Tia to testify
for the prosecution on the ground that there was no necessity for the same. Appellant argues
that deep penetration agents such as Tia "have to take risks and accept the consequences of
their actions." 8 The argument is devoid of merit. The discharge of accused Tia was based on
Section 9, Rule 119 of the Rules of Court, which reads in part:
Sec. 9. Discharge of the accused to be state witness. — When two or more persons are
jointly charged with the commission of any offense, upon motion of the prosecution
before resting its case, the court may directone or more of the accused to be
discharged with their consent so that they may be witnesses for the state . . .
(emphasis supplied).
As correctly pointed out by the Solicitor General, the discharge of an accused is left to the sound
discretion of the lower court.1âwphi1 The trial court has the exclusive responsibility to see that
the conditions prescribed by the rule exist.9 In the instant case, appellant does not allege that
any of the conditions for the discharge had not been met by the prosecution. Therefore, the
discharge, as ordered by the trial court, stands.
Finally, appellant alleges that the testimony of Sgt. Roberto Cayabyab regarding the facts
surrounding the commission of the offense proves that the discharge of accused Tia is
unnecessary. The allegation is baseless. Appellant himself admits that the sergeant's testimony
corroborates the testimony of the discharged accused. The fact of corroboration of the
testimonies bolsters the validity of the questioned discharge precisely because paragraph (a) of
the aforequoted rule on discharge requires that the testimony be substantially corroborated in
its material points. The corroborative testimony of the PC-CIS operative does not debunk the
claim of the prosecution that there is absolute necessity for the testimony of accused Tia.
WHEREFORE, the decision appealed from is hereby AFFIRMED in toto and the appeal is thereby
DISMISSED. No costs.
SO ORDERED.
116

PEOPLE VS MARTI that dried marijuana leaves were neatly stocked underneath the cigars (tsn, p. 39,
This is an appeal from a decision * rendered by the Special Criminal Court of Manila (Regional October 6, 1987).
Trial Court, Branch XLIX) convicting accused-appellant of violation of Section 21 (b), Article IV in The NBI agents made an inventory and took charge of the box and of the contents
relation to Section 4, Article 11 and Section 2 (e) (i), Article 1 of Republic Act 6425, as amended, thereof, after signing a "Receipt" acknowledging custody of the said effects (tsn, pp. 2-
otherwise known as the Dangerous Drugs Act. 3, October 7, 1987).
The facts as summarized in the brief of the prosecution are as follows: Thereupon, the NBI agents tried to locate appellant but to no avail. Appellant's stated address in
On August 14, 1987, between 10:00 and 11:00 a.m., the appellant and his common-law his passport being the Manila Central Post Office, the agents requested assistance from the
wife, Shirley Reyes, went to the booth of the "Manila Packing and Export Forwarders" latter's Chief Security. On August 27, 1987, appellant, while claiming his mail at the Central Post
in the Pistang Pilipino Complex, Ermita, Manila, carrying with them four (4) gift Office, was invited by the NBI to shed light on the attempted shipment of the seized dried leaves.
wrapped packages. Anita Reyes (the proprietress and no relation to Shirley Reyes) On the same day the Narcotics Section of the NBI submitted the dried leaves to the Forensic
attended to them. The appellant informed Anita Reyes that he was sending the Chemistry Section for laboratory examination. It turned out that the dried leaves were marijuana
packages to a friend in Zurich, Switzerland. Appellant filled up the contract necessary flowering tops as certified by the forensic chemist. (Appellee's Brief, pp. 9-11, Rollo, pp. 132-
for the transaction, writing therein his name, passport number, the date of shipment 134).
and the name and address of the consignee, namely, "WALTER FIERZ, Mattacketr II, Thereafter, an Information was filed against appellant for violation of RA 6425, otherwise known
8052 Zurich, Switzerland" (Decision, p. 6) as the Dangerous Drugs Act.
Anita Reyes then asked the appellant if she could examine and inspect the packages. After trial, the court a quo rendered the assailed decision.
Appellant, however, refused, assuring her that the packages simply contained books, In this appeal, accused/appellant assigns the following errors, to wit:
cigars, and gloves and were gifts to his friend in Zurich. In view of appellant's THE LOWER COURT ERRED IN ADMITTING IN EVIDENCE THE ILLEGALLY SEARCHED AND
representation, Anita Reyes no longer insisted on inspecting the packages. The four (4) SEIZED OBJECTS CONTAINED IN THE FOUR PARCELS.
packages were then placed inside a brown corrugated box one by two feet in size (1' x THE LOWER COURT ERRED IN CONVICTING APPELLANT DESPITE THE UNDISPUTED FACT
2'). Styro-foam was placed at the bottom and on top of the packages before the box THAT HIS RIGHTS UNDER THE CONSTITUTION WHILE UNDER CUSTODIAL PROCEEDINGS
was sealed with masking tape, thus making the box ready for shipment (Decision, p. 8). WERE NOT OBSERVED.
Before delivery of appellant's box to the Bureau of Customs and/or Bureau of THE LOWER COURT ERRED IN NOT GIVING CREDENCE TO THE EXPLANATION OF THE
Posts, Mr. Job Reyes (proprietor) and husband of Anita (Reyes), following standard APPELLANT ON HOW THE FOUR PARCELS CAME INTO HIS POSSESSION (Appellant's
operating procedure, opened the boxes for final inspection. When he opened Brief, p. 1; Rollo, p. 55)
appellant's box, a peculiar odor emitted therefrom. His curiousity aroused, he squeezed 1. Appellant contends that the evidence subject of the imputed offense had been obtained in
one of the bundles allegedly containing gloves and felt dried leaves inside. Opening one violation of his constitutional rights against unreasonable search and seizure and privacy of
of the bundles, he pulled out a cellophane wrapper protruding from the opening of one communication (Sec. 2 and 3, Art. III, Constitution) and therefore argues that the same should be
of the gloves. He made an opening on one of the cellophane wrappers and took several held inadmissible in evidence (Sec. 3 (2), Art. III).
grams of the contents thereof(tsn, pp. 29-30, October 6, 1987; Emphasis supplied). Sections 2 and 3, Article III of the Constitution provide:
Job Reyes forthwith prepared a letter reporting the shipment to the NBI and Sec. 2. The right of the people to be secure in their persons, houses, papers and effects
requesting a laboratory examination of the samples he extracted from the cellophane against unreasonable searches and seizures of whatever nature and for any purpose
wrapper (tsn, pp. 5-6, October 6, 1987). shall be inviolable, and no search warrant or warrant of arrest shall issue except upon
He brought the letter and a sample of appellant's shipment to the Narcotics Section of probable cause to be determined personally by the judge after examination under oath
the National Bureau of Investigation (NBI), at about 1:30 o'clock in the afternoon of or affirmation of the complainant and the witnesses he may produce, and particularly
that date, i.e., August 14, 1987. He was interviewed by the Chief of Narcotics Section. describing the place to be searched and the persons or things to be seized.
Job Reyes informed the NBI that the rest of the shipment was still in his office. Sec. 3. (1) The privacy of communication and correspondence shall be inviolable except
Therefore, Job Reyes and three (3) NBI agents, and a photographer, went to the Reyes' upon lawful order of the court, or when public safety or order requires otherwise as
office at Ermita, Manila (tsn, p. 30, October 6, 1987). prescribed by law.
Job Reyes brought out the box in which appellant's packages were placed and, in the (2) Any evidence obtained in violation of this or the preceding section shall be
presence of the NBI agents, opened the top flaps, removed the styro-foam and took out inadmissible for any purpose in any proceeding.
the cellophane wrappers from inside the gloves. Dried marijuana leaves were found to Our present constitutional provision on the guarantee against unreasonable search and seizure
have been contained inside the cellophane wrappers (tsn, p. 38, October 6, 1987; had its origin in the 1935 Charter which, worded as follows:
Emphasis supplied). The right of the people to be secure in their persons, houses, papers and effects
The package which allegedly contained books was likewise opened by Job Reyes. He against unreasonable searches and seizures shall not be violated, and no warrants shall
discovered that the package contained bricks or cake-like dried marijuana leaves. The issue but upon probable cause, to be determined by the judge after examination under
package which allegedly contained tabacalera cigars was also opened. It turned out oath or affirmation of the complainant and the witnesses he may produce, and
117

particularly describing the place to be searched, and the persons or things to be seized. The above ruling was reiterated in State v. Bryan (457 P.2d 661 [1968]) where a parking
(Sec. 1 [3], Article III) attendant who searched the automobile to ascertain the owner thereof found marijuana
was in turn derived almost verbatim from the Fourth Amendment ** to the United States instead, without the knowledge and participation of police authorities, was declared admissible
Constitution. As such, the Court may turn to the pronouncements of the United States Federal in prosecution for illegal possession of narcotics.
Supreme Court and State Appellate Courts which are considered doctrinal in this jurisdiction. And again in the 1969 case of Walker v. State (429 S.W.2d 121), it was held that the search and
Thus, following the exclusionary rule laid down in Mapp v. Ohio by the US Federal Supreme seizure clauses are restraints upon the government and its agents, not upon private individuals
Court (367 US 643, 81 S.Ct. 1684, 6 L.Ed. 1081 [1961]), this Court, in Stonehill v. Diokno (20 SCRA (citing People v. Potter, 240 Cal. App.2d 621, 49 Cap. Rptr, 892 (1966); State v. Brown, Mo., 391
383 [1967]), declared as inadmissible any evidence obtained by virtue of a defective search and S.W.2d 903 (1965); State v. Olsen, Or., 317 P.2d 938 (1957).
seizure warrant, abandoning in the process the ruling earlier adopted in Moncado v. People's Likewise appropos is the case of Bernas v. US (373 F.2d 517 (1967). The Court there said:
Court (80 Phil. 1 [1948]) wherein the admissibility of evidence was not affected by the illegality of The search of which appellant complains, however, was made by a private citizen —
its seizure. The 1973 Charter (Sec. 4 [2], Art. IV) constitutionalized the Stonehill ruling and is the owner of a motel in which appellant stayed overnight and in which he left behind a
carried over up to the present with the advent of the 1987 Constitution. travel case containing the evidence***complained of. The search was made on the
In a number of cases, the Court strictly adhered to the exclusionary rule and has struck down the motel owner's own initiative. Because of it, he became suspicious, called the local
admissibility of evidence obtained in violation of the constitutional safeguard against police, informed them of the bag's contents, and made it available to the authorities.
unreasonable searches and seizures. (Bache & Co., (Phil.), Inc., v. Ruiz, 37 SCRA 823 [1971]; Lim v. The fourth amendment and the case law applying it do not require exclusion of
Ponce de Leon, 66 SCRA 299 [1975]; People v. Burgos, 144 SCRA 1 [1986]; Roan v. Gonzales, 145 evidence obtained through a search by a private citizen. Rather, the amendment only
SCRA 687 [1987]; See also Salazar v. Hon. Achacoso, et al., GR No. 81510, March 14, 1990). proscribes governmental action."
It must be noted, however, that in all those cases adverted to, the evidence so obtained were The contraband in the case at bar having come into possession of the Government without the
invariably procured by the State acting through the medium of its law enforcers or other latter transgressing appellant's rights against unreasonable search and seizure, the Court sees no
authorized government agencies. cogent reason why the same should not be admitted against him in the prosecution of the
On the other hand, the case at bar assumes a peculiar character since the evidence sought to be offense charged.
excluded was primarily discovered and obtained by a private person, acting in a private capacity Appellant, however, would like this court to believe that NBI agents made an illegal search and
and without the intervention and participation of State authorities. Under the circumstances, seizure of the evidence later on used in prosecuting the case which resulted in his conviction.
can accused/appellant validly claim that his constitutional right against unreasonable searches The postulate advanced by accused/appellant needs to be clarified in two days. In both
and seizure has been violated? Stated otherwise, may an act of a private individual, allegedly in instances, the argument stands to fall on its own weight, or the lack of it.
violation of appellant's constitutional rights, be invoked against the State? First, the factual considerations of the case at bar readily foreclose the proposition that NBI
We hold in the negative. In the absence of governmental interference, the liberties guaranteed agents conducted an illegal search and seizure of the prohibited merchandise. Records of the
by the Constitution cannot be invoked against the State. case clearly indicate that it was Mr. Job Reyes, the proprietor of the forwarding agency, who
As this Court held in Villanueva v. Querubin (48 SCRA 345 [1972]: made search/inspection of the packages. Said inspection was reasonable and a standard
1. This constitutional right (against unreasonable search and seizure) refers to the operating procedure on the part of Mr. Reyes as a precautionary measure before delivery of
immunity of one's person, whether citizen or alien, from interference by government, packages to the Bureau of Customs or the Bureau of Posts (TSN, October 6 & 7, 1987, pp. 15-18;
included in which is his residence, his papers, and other possessions. . . . pp. 7-8; Original Records, pp. 119-122; 167-168).
. . . There the state, however powerful, does not as such have the access except under It will be recalled that after Reyes opened the box containing the illicit cargo, he took samples of
the circumstances above noted, for in the traditional formulation, his house, however the same to the NBI and later summoned the agents to his place of business. Thereafter, he
humble, is his castle. Thus is outlawed any unwarranted intrusion by government, opened the parcel containing the rest of the shipment and entrusted the care and custody
which is called upon to refrain from any invasion of his dwelling and to respect the thereof to the NBI agents. Clearly, the NBI agents made no search and seizure, much less an
privacies of his life. . . . (Cf. Schermerber v. California, 384 US 757 [1966] and Boyd v. illegal one, contrary to the postulate of accused/appellant.
United States, 116 US 616 [1886]; Emphasis supplied). Second, the mere presence of the NBI agents did not convert the reasonable search effected by
In Burdeau v. McDowell (256 US 465 (1921), 41 S Ct. 547; 65 L.Ed. 1048), the Court there in Reyes into a warrantless search and seizure proscribed by the Constitution. Merely to observe
construing the right against unreasonable searches and seizures declared that: and look at that which is in plain sight is not a search. Having observed that which is open, where
(t)he Fourth Amendment gives protection against unlawful searches and seizures, and no trespass has been committed in aid thereof, is not search (Chadwick v. State, 429 SW2d 135).
as shown in previous cases, its protection applies to governmental action. Its origin and Where the contraband articles are identified without a trespass on the part of the arresting
history clearly show that it was intended as a restraint upon the activities of sovereign officer, there is not the search that is prohibited by the constitution (US v. Lee 274 US 559, 71
authority, and was not intended to be a limitation upon other than governmental L.Ed. 1202 [1927]; Ker v. State of California 374 US 23, 10 L.Ed.2d. 726 [1963]; Moore v. State,
agencies; as against such authority it was the purpose of the Fourth Amendment to 429 SW2d 122 [1968]).
secure the citizen in the right of unmolested occupation of his dwelling and the In Gandy v. Watkins (237 F. Supp. 266 [1964]), it was likewise held that where the property was
possession of his property, subject to the right of seizure by process duly served. taken into custody of the police at the specific request of the manager and where the search was
118

initially made by the owner there is no unreasonable search and seizure within the constitutional Similarly, the admissibility of the evidence procured by an individual effected through private
meaning of the term. seizure equally applies, in pari passu, to the alleged violation, non-governmental as it is, of
That the Bill of Rights embodied in the Constitution is not meant to be invoked against acts of appellant's constitutional rights to privacy and communication.
private individuals finds support in the deliberations of the Constitutional Commission. True, the 2. In his second assignment of error, appellant contends that the lower court erred in convicting
liberties guaranteed by the fundamental law of the land must always be subject to protection. him despite the undisputed fact that his rights under the constitution while under custodial
But protection against whom? Commissioner Bernas in his sponsorship speech in the Bill of investigation were not observed.
Rights answers the query which he himself posed, as follows: Again, the contention is without merit, We have carefully examined the records of the case and
First, the general reflections. The protection of fundamental liberties in the essence of found nothing to indicate, as an "undisputed fact", that appellant was not informed of his
constitutional democracy. Protection against whom? Protection against the state. The constitutional rights or that he gave statements without the assistance of counsel. The law
Bill of Rights governs the relationship between the individual and the state. Its concern enforcers testified that accused/appellant was informed of his constitutional rights. It is
is not the relation between individuals, between a private individual and other presumed that they have regularly performed their duties (See. 5(m), Rule 131) and their
individuals. What the Bill of Rights does is to declare some forbidden zones in the testimonies should be given full faith and credence, there being no evidence to the contrary.
private sphere inaccessible to any power holder. (Sponsorship Speech of Commissioner What is clear from the records, on the other hand, is that appellant refused to give any written
Bernas , Record of the Constitutional Commission, Vol. 1, p. 674; July 17, 1986; statement while under investigation as testified by Atty. Lastimoso of the NBI, Thus:
Emphasis supplied) Fiscal Formoso:
The constitutional proscription against unlawful searches and seizures therefore applies as a You said that you investigated Mr. and Mrs. Job Reyes. What about the accused here,
restraint directed only against the government and its agencies tasked with the enforcement of did you investigate the accused together with the girl?
the law. Thus, it could only be invoked against the State to whom the restraint against arbitrary WITNESS:
and unreasonable exercise of power is imposed. Yes, we have interviewed the accused together with the girl but the accused availed of
If the search is made upon the request of law enforcers, a warrant must generally be first his constitutional right not to give any written statement, sir. (TSN, October 8, 1987, p.
secured if it is to pass the test of constitutionality. However, if the search is made at the behest 62; Original Records, p. 240)
or initiative of the proprietor of a private establishment for its own and private purposes, as in The above testimony of the witness for the prosecution was not contradicted by the defense on
the case at bar, and without the intervention of police authorities, the right against cross-examination. As borne out by the records, neither was there any proof by the defense that
unreasonable search and seizure cannot be invoked for only the act of private individual, not the appellant gave uncounselled confession while being investigated. What is more, we
law enforcers, is involved. In sum, the protection against unreasonable searches and seizures have examined the assailed judgment of the trial court and nowhere is there any reference made
cannot be extended to acts committed by private individuals so as to bring it within the ambit of to the testimony of appellant while under custodial investigation which was utilized in the
alleged unlawful intrusion by the government. finding of conviction. Appellant's second assignment of error is therefore misplaced.
Appellant argues, however, that since the provisions of the 1935 Constitution has been modified 3. Coming now to appellant's third assignment of error, appellant would like us to believe that he
by the present phraseology found in the 1987 Charter, expressly declaring as inadmissible any was not the owner of the packages which contained prohibited drugs but rather a certain
evidence obtained in violation of the constitutional prohibition against illegal search and seizure, Michael, a German national, whom appellant met in a pub along Ermita, Manila: that in the
it matters not whether the evidence was procured by police authorities or private individuals course of their 30-minute conversation, Michael requested him to ship the packages and gave
(Appellant's Brief, p. 8, Rollo, p. 62). him P2,000.00 for the cost of the shipment since the German national was about to leave the
The argument is untenable. For one thing, the constitution, in laying down the principles of the country the next day (October 15, 1987, TSN, pp. 2-10).
government and fundamental liberties of the people, does not govern relationships between Rather than give the appearance of veracity, we find appellant's disclaimer as incredulous, self-
individuals. Moreover, it must be emphasized that the modifications introduced in the 1987 serving and contrary to human experience. It can easily be fabricated. An acquaintance with a
Constitution (re: Sec. 2, Art. III) relate to the issuance of either a search warrant or warrant of complete stranger struck in half an hour could not have pushed a man to entrust the shipment of
arrest vis-a-vis the responsibility of the judge in the issuance thereof (SeeSoliven v. Makasiar, 167 four (4) parcels and shell out P2,000.00 for the purpose and for appellant to readily accede to
SCRA 393 [1988]; Circular No. 13 [October 1, 1985] and Circular No. 12 [June 30, 1987]. The comply with the undertaking without first ascertaining its contents. As stated by the trial court,
modifications introduced deviate in no manner as to whom the restriction or inhibition against "(a) person would not simply entrust contraband and of considerable value at that as the
unreasonable search and seizure is directed against. The restraint stayed with the State and did marijuana flowering tops, and the cash amount of P2,000.00 to a complete stranger like the
not shift to anyone else. Accused. The Accused, on the other hand, would not simply accept such undertaking to take
Corolarilly, alleged violations against unreasonable search and seizure may only be invoked custody of the packages and ship the same from a complete stranger on his mere say-so"
against the State by an individual unjustly traduced by the exercise of sovereign authority. To (Decision, p. 19, Rollo, p. 91). As to why he readily agreed to do the errand, appellant failed to
agree with appellant that an act of a private individual in violation of the Bill of Rights should also explain. Denials, if unsubstantiated by clear and convincing evidence, are negative self-serving
be construed as an act of the State would result in serious legal complications and an absurd evidence which deserve no weight in law and cannot be given greater evidentiary weight than
interpretation of the constitution. the testimony of credible witnesses who testify on affirmative matters (People v. Esquillo, 171
SCRA 571 [1989]; People vs. Sariol, 174 SCRA 237 [1989]).
119

Appellant's bare denial is even made more suspect considering that, as per records of the consent, he was bodily searched, but no jewelry was found. He was then escorted by two (2)
Interpol, he was previously convicted of possession of hashish by the Kleve Court in the Federal security agents back to the economy section to get his baggage. The accused took a Samsonite
Republic of Germany on January 1, 1982 and that the consignee of the frustrated shipment, suitcase and brought this back to the cabin. When requested by the security, the accused
Walter Fierz, also a Swiss national, was likewise convicted for drug abuse and is just about an opened the suitcase, revealing a brown bag and small plastic packs containing white crystalline
hour's drive from appellant's residence in Zurich, Switzerland (TSN, October 8, 1987, p. 66; substance. Suspecting the substance to be shabu, the security personnel immediately reported
Original Records, p. 244; Decision, p. 21; Rollo, p. 93). the matter to the ship captain and took pictures of the accused beside the suitcase and its
Evidence to be believed, must not only proceed from the mouth of a credible witness, but it contents. They also called the Philippine Coast Guard for assistance.[5] At about 6:00 a.m., Lt.
must be credible in itself such as the common experience and observation of mankind can Robert Patrimonio, YN Aurelio Estoque, CD2 Phoudinie Lantao and RM3 Merchardo De Guzman
approve as probable under the circumstances (People v. Alto, 26 SCRA 342 [1968], citing Daggers of the Philippine Coast Guard arrived and took custody of the accused and the seized items--the
v. Van Dyke, 37 N.J. Eg. 130; see also People v. Sarda, 172 SCRA 651 [1989]; People v. Sunga, 123 Samsonite suitcase, a brown bag[6] and eight (8) small plastic packs of white crystalline
SCRA 327 [1983]); Castañares v. CA, 92 SCRA 567 [1979]). As records further show, appellant did substance.[7] When asked about the contraband articles, the accused explained that he was just
not even bother to ask Michael's full name, his complete address or passport number. requested by a certain Alican Alex Macapudi to bring the suitcase to the latters brother in Iligan
Furthermore, if indeed, the German national was the owner of the merchandise, appellant City.[8] The accused and the seized items were later turned over by the coast guard to the
should have so indicated in the contract of shipment (Exh. "B", Original Records, p. 40). On the Presidential Anti-Organized Crime Task Force (PAOCTF).Chief Inspector Graciano Mijares and his
contrary, appellant signed the contract as the owner and shipper thereof giving more weight to men brought the accused to the PAOCTF Headquarters,[9] while the packs of white crystalline
the presumption that things which a person possesses, or exercises acts of ownership over, are substance were sent to the NBI Regional Office in Cagayan de Oro City for laboratory
owned by him (Sec. 5 [j], Rule 131). At this point, appellant is therefore estopped to claim examination. NBI Forensic Chemist Nicanor Cruz later confirmed the substance to be
otherwise. methamphetamine hydrochloride, commonly known as shabu, weighing 399.3266 grams.[10]
Premises considered, we see no error committed by the trial court in rendering the assailed The accused testified and proffered his own version. On March 11, 1999, at about 10:00
judgment. p.m., he was in Quiapo, Manila where he met Alican Alex Macapudi, a neighbor who has a store
WHEREFORE, the judgment of conviction finding appellant guilty beyond reasonable doubt of in Marawi City. He was requested by Macapudi to bring a Samsonite suitcase containing
the crime charged is hereby AFFIRMED. No costs. sunglasses and watches to Iligan City, and to give it to Macapudis brother at the Iligan port. He
SO ORDERED. boarded the M/V Super Ferry 5 on the same night, carrying a big luggage full of clothes, a small
PEOPLE VS BONGCARAWAN luggage or maleta containing the sunglasses and brushes he bought from Manila, and the
his is an appeal from the Decision[1] dated December 27, 1999 of the Regional Trial Court of Samsonite suitcase of Macapudi.[11] He stayed at cabin no. 106. At about 4:00 a.m of March 13,
Iligan City, Branch 06, in Criminal Case No. 06-7542, finding accused Basher Bongcarawan y 1999, as the vessel was about to dock at the Iligan port, he took his baggage and positioned
Macarambon guilty beyond reasonable doubt of violation of Section 16, Article III of Republic Act himself at the economy section to be able to disembark ahead of the other passengers. There,
No. 6425[2] as amended, and sentencing him to suffer the penalty ofreclusion perpetua, and to he met a friend, Ansari Ambor. While they were conversing, five (5) members of the vessel
pay a fine of Five Hundred Thousand Pesos (P500,000.00) without subsidiary imprisonment in security force and a woman whom he recognized as his co-passenger at cabin no. 106 came and
case of insolvency. told him that he was suspected of stealing jewelry. He voluntarily went with the group back to
Accused Basher Bongcarawan y Macarambon was charged in an Information which reads, cabin no. 106 where he was frisked. Subsequently, he was asked to get his baggage, so he went
thus: back to the economy section and took the big luggage and Macapudis Samsonite suitcase. He left
That on or about March 13, 1999, in the City of Iligan, Philippines, and within the jurisdiction of the small maleta containing sunglasses and brushes for fear that they would be confiscated by
this Honorable Court, the said accused, without authority of law, did then and there wilfully, the security personnel. When requested, he voluntarily opened the big luggage, but refused to
unlawfully and feloniously have in his possession, custody and control eight (8) packs of do the same to the Samsonite suitcase which he claimed was not his and had a secret
Methamphetamine Hydrochloride, a regulated drug commonly known as Shabu, weighing combination lock. The security personnel forcibly opened the suitcase and found packs of white
approximately 400 grams, without the corresponding license or prescription. crystalline substance inside which they suspected to be shabu. They took pictures of him with
Contrary to and in violation of Section 16, Article III of RA 6425, otherwise known as the the merchandise, and asked him to sign a turn over receipt which was later given to the
Dangerous Drugs Act of 1972, as amended by RA 7659.[3] Philippine Coast Guard, then to the PAOCTF.[12]
During the arraignment, the accused pleaded not guilty. Trial ensued. On December 27, 1999, the trial court rendered judgment, the dispositive portion of which
Evidence for the prosecution shows that on March 11, 1999, an interisland passenger ship, reads:
M/V Super Ferry 5, sailed from Manila to Iligan City. At about 3:00 a.m. on March 13, 1999, the WHEREFORE, the court finds the accused Basher Bongcarawan y Macarambon GUILTY beyond
vessel was about to dock at the port of Iligan City when its security officer, Mark Diesmo, reasonable doubt as principal of the offense of violation of Section 16, Art. III, R.A. No. 6425 as
received a complaint from passenger Lorena Canoy about her missing jewelry. Canoy suspected amended by R.A. No. 7659 and hereby imposes upon him the penalty of RECLUSION PERPETUA
one of her co-passengers at cabin no. 106 as the culprit. Diesmo and four (4) other members of and a fine of FIVE HUNDRED THOUSAND (P500,000.00) PESOS, without subsidiary imprisonment
the vessel security force accompanied Canoy to search for the suspect whom they later found at in case of insolvency.
the economy section.[4] The suspect was identified as the accused, Basher Bongcarawan. The
accused was informed of the complaint and was invited to go back to cabin no. 106. With his
120

Having been under preventive imprisonment since March 13, 1999 until the present, the period submits that without knowledge or intent to possess the dangerous drug, he cannot be
of such preventive detention shall be credited in full in favor of the accused in the service of his convicted of the crime charged.[21]
sentence. We are not persuaded.
The 399.3266 grams of methamphetamine hydrochloride or shabu is hereby ordered delivered In a prosecution for illegal possession of dangerous drugs, the following facts must be
to the National Bureau of Investigation for proper disposition. proven beyond reasonable doubt, viz: (1) that the accused is in possession of the object
SO ORDERED.[13] identified as a prohibited or a regulated drug; (2) that such possession is not authorized by law;
Hence, this appeal where the accused raises the following assignment of errors: and (3) that the accused freely and consciously possessed the said drug.[22] The first two
I. elements were sufficiently proven in this case, and were in fact undisputed. We are left with the
THE COURT A QUO ERRED IN SO HOLDING THAT THE DRUG CONFISCATED IS ADMISSIBLE IN third.
EVIDENCE AGAINST THE ACCUSED/APPELLANT. As early as 1910 in the case of United States v. Tan Misa,[23] this Court has ruled that to
II. warrant conviction, the possession of dangerous drugs must be with knowledge of the accused,
THE COURT A QUO ERRED IN SO HOLDING THAT THE APPELLANT OWNED THE CONFISCATED or that animus possidendi existed together with the possession or control of such articles.[24] It
EVIDENCE AND THEREFORE ADMISSIBLE IN EVIDENCE AGAINST HIM.[14] has been ruled, however, that possession of dangerous drugs constitutes prima facie evidence of
On the first assignment of error, the accused-appellant contends that the Samsonite knowledge or animus possidendi sufficient to convict an accused in the absence of a satisfactory
suitcase containing the methamphetamine hydrochloride or shabu was forcibly opened and explanation of such possession.[25] Hence, the burden of evidence is shifted to the accused to
searched without his consent, and hence, in violation of his constitutional right against explain the absence of knowledge or animus possidendi.[26]
unreasonable search and seizure. Any evidence acquired pursuant to such unlawful search and In this respect, the accused-appellant has utterly failed. His testimony, uncorroborated,
seizure, he claims, is inadmissible in evidence against him. He also contends that People v. self-serving and incredulous, was not given credence by the trial court. We find no reason to
Marti[15] is not applicable in this case because a vessel security personnel is deemed to perform disagree. Well-settled is the rule that in the absence of palpable error or grave abuse of
the duties of a policeman. discretion on the part of the trial judge, the trial courts evaluation of the credibility of witnesses
The contentions are devoid of merit. will not be disturbed on appeal.[27] Moreover, evidence must be credible in itself to deserve
The right against unreasonable search and seizure is a fundamental right protected by the credence and weight in law. In this case, the accused-appellant admits that when he was asked
Constitution.[16] Evidence acquired in violation of this right shall be inadmissible for any purpose to get his baggage, he knew it would be inspected. [28] Why he got the Samsonite suitcase
in any proceeding.[17] Whenever this right is challenged, an individual may choose between allegedly not owned by him and which had a combination lock known only to the owner remains
invoking the constitutional protection or waiving his right by giving consent to the search and unclear. He also claims that he did not present his small maleta for inspection for fear that its
seizure. It should be stressed, however, that protection is against transgression committed by contents consisting of expensive sunglasses and brushes would be confiscated,[29] but he brought
the government or its agent. As held by this Court in the case of People v. Marti,[18] [i]n the the Samsonite suitcase which is not his and also contained expensive sunglasses, and even
absence of governmental interference, liberties guaranteed by the Constitution cannot be watches.[30]
invoked against the State.[19] The constitutional proscription against unlawful searches and The things in possession of a person are presumed by law to be owned by him. [31] To
seizures applies as a restraint directed only against the government and its agencies tasked with overcome this presumption, it is necessary to present clear and convincing evidence to the
the enforcement of the law. Thus, it could only be invoked against the State to whom the contrary. In this case, the accused points to a certain Alican Alex Macapudi as the owner of the
restraint against arbitrary and unreasonable exercise of power is imposed.[20] contraband, but presented no evidence to support his claim. As aptly observed by the trial judge:
In the case before us, the baggage of the accused-appellant was searched by the vessel First, who is Alex Macap[u]di aka Ali[c]an Macap[u]di? Does he really exist or simply a figment of
security personnel. It was only after they found shabu inside the suitcase that they called the the imagination? He says that Alex Macap[u]di is a friend and a fellow businessman who has a
Philippine Coast Guard for assistance. The search and seizure of the suitcase and the contraband stall selling sunglasses in Marawi City. But no witnesses were presented to prove that there is
items was therefore carried out without government intervention, and hence, the constitutional such a living, breathing, flesh and blood person named Alex Macap[u]di who entrusted the
protection against unreasonable search and seizure does not apply. Samsonite to the accused. Surely, if he does exist, he has friends, fellow businessmen and
There is no merit in the contention of the accused-appellant that the search and seizure acquaintances who could testify and support the claim of the accused.[32]
performed by the vessel security personnel should be considered as one conducted by the police Mere denial of ownership will not suffice especially if, as in the case at bar, it is the keystone of
authorities for like the latter, the former are armed and tasked to maintain peace and order. The the defense of the accused-appellant. Stories can easily be fabricated. It will take more than
vessel security officer in the case at bar is a private employee and does not discharge any bare-bone allegations to convince this Court that a courier of dangerous drugs is not its owner
governmental function. In contrast, police officers are agents of the state tasked with the and has no knowledge or intent to possess the same.
sovereign function of enforcement of the law. Historically and until now, it is against them and WHEREFORE, the decision of the Regional Trial Court of Iligan City, Branch 06, in Criminal
other agents of the state that the protection against unreasonable searches and seizures may be Case No. 06-7542, convicting accused-appellant Basher Bongcarawan of violation of Section 16,
invoked. Article III of Republic Act No. 6425, as amended, and sentencing him to suffer the penalty
On the second assignment of error, the accused-appellant contends that he is not the of Reclusion Perpetua and to pay a fine of Five Hundred Thousand Pesos (P500,000.00) without
owner of the Samsonite suitcase and he had no knowledge that the same contained shabu. He subsidiary imprisonment in case of insolvency, is AFFIRMED.
Costs against the accused-appellant. SO ORDERED.

Potrebbero piacerti anche